Download as pdf or txt
Download as pdf or txt
You are on page 1of 691

USMLE STEP 1 REVIEW

STUDY GUIDE
2016 EDITION

~
I

. DOCTORS IN TRAINING®
BEITER DOCTORS. BETTER WORLD.

BRIAN jENKINS, MD
TABLE OF CONTENTS
FOUNDATIONS
1. ~mbl'yolosy__ _
2. Teratogenesis
3. ~~lfBiology: (;)fgan~IJes ·
4. Cell Biology: Cytoskeleton
5. c~r Biology: Plasfl1a Membrciil~ ":f
6. Cellular Suffering and Death
7. lijfl~m~a~ion5 "'" '' _ ..
8. Extracellular Environment 20
9. tellul~r Adaptations 22
II NEURO
1. hf~vou~_~ys!ernfJasics 2_5
2. CNS Embryology 28
3. (t~l'!~i< ana-·B(aiilNascuJature·g '"""'''-'~'''"7:&277~-~~---· 3~

4. Vascular Events 36
5. Hypothalamus and limbic System 3S
6. Cranial Nerves part 1 41
7. ~l"~!!!afl'lerye_!_Pal"f~ ' 44
8. Brainstem and Spinal Cord 47
9. ~f!!nsferri' tesioOs -- 52
10. CSF and ICP
11. He-adache
12. Brain Tumors
13. PeUrium arid De~_entia
14. Seizures
~ 15. $pinal Cord Lesions
I
16. Movement
17. J!~sarGanglia
18. Skeletal Muscle
19. 'l,Jpp~ tx!r,~ffiiJyNerves pa!'~- ~ -
20. Upper Extremity Nerves part 2
21. :rower Extremity
22. Sensation
23. Eye _
24. Ear
25. $[~ep
26. Anesthetics
Ill PHARM BASICS
1. Pai'11sympat~tlt Activation ·. --~~ 1:1.1

2. Parasympathetic Inhibition 114


3. Sympathetic Activation 116.
4. Sympathetic Inhibition 118
5. fiJeurofransinissJon 120
6. G Protein Second Messengers 123
7. ~parmacokinetiCs 126
8. Pharmacodynamics 131
9. QI"Qg MiH:al:)oli~l!l=· ~ 134
10. Drug Elimination 136
11. t>rug
sidel:tt~as 140
12. Antidotes 144

[i]
TABLE OF CONTENTS
IV CARDIOVASCULA R
1. EmbryoiC)gy 149
2. Congenital Heart Defects 152
3. cardi~c qyJ;:I!utt ··· 158
4. Heart Failure 160
5. Treatmeiifol~(:fi F 162
6. Edema and Shock 164
7. Cardiac Cycle 167
8. Diastolic Heart Murmurs 171
9. §y stolic He<!f.~Mllrmurs 174
10. Eiectrophysiology · 177
11. ~ntiarrhythmics- . 1i9
12. Electrocardiography 182
13. ~rrhytlimfas ~ f84
14. Regulation of BP 189
15. Hypertension ··-· "--~·-·"·~~-~-··---=~~<,L<''Y YOO~ 191
~····-~,=~"' 193
16. Antihypertensives
17. Attierosclerosrs-- 196
18. Antianginal Therapy and lipid-lowering Agents 199
19. Myocar_gj~Jnt~rftlc>n ea((!!~,·· · 201
20. Myocardial Infarction part 2 204
21. carafoiny~p~iil~ijles andEndq~aroftis ==··""·'·'··---~--"""-"~- 206
22. Other Cardiac Pathology 208
23. Vasculitis:-=: _;:~ 21f
24. Other Vascular Diseases 213
v RHEUM & DERM
1. ......... ·- -• ·--~···· ~·· ·····--~-·-·-·· '''"'~-~--~~~.~- '217
2. Bone Disorders 219
3. ~usculosk~J~~CIIflljuries ~· . 7 _ 222
4. Osteoarthritis and Rheumatoid Arthritis 225
5. f,)~her Jypei$)Jl-rtljritis • 227
6. Systemic Disorders 229
7. :Q.ermatology pi:trt 1 232
8. Dermatology part 2 235
VI REPRODUCTION
1. · B~J>roductive AnatC)mY - 239
2. Genital Embryology 241
3. Androgens~ ,,. _: 243
4. Testicular Pathology 245
5. penis and Prostate PathologY,- 247,
6 . Female Reproductive Cycle - --- 250
7. f!l~hstr:uatiorf_~na ~enopau§~\ -; - = • -··-·~.-
8. Vulva, Vagina and Cervix
'"'"''"''·''····-·' 256
25~
9. uterine Pathology 258
10. Ovarian Pathology 260
11. Ovarian_Neo'p(CI~Ill.s 262
12. Pregnancy part 1 265
13. Pregnancy R'art2 ..... 267
14. Chromosomal Disorders 269
15. ~~!letlc i:>ls()rders: AD and ! ri_iiucleotide Repeats 211
16. Genetic Disorders: AR and X- linked 275
17. ~f-e~st .279

r ii 1
TABLE OF CONTENTS
VII MIC RO
1. 28S
2.
3.
4. 291
5. 293
6. 295
7. ,e=_~d&=·~~~-~·- ~7
8. 299
9. 301:
303
,_$'05
307
30~
311
···----"'···~-~,~~._.._~---'~··=314
317

322
. ;3~5
327
~ 329
331
: ;333'
336
338
341
~=·····~~...c'·"''=--~~~-·343
345
348
351

VIII IMMUNO LOGY


1. tmmune=§ystem Basic~ . ~ :~55
2. Antigen Presentation 357
3. TCells 359
4. B Cells and Antibodies 361
s. [i!l'inunization ~i!fA~f§~~tiboaie~ .~ .-:;363
6. Hypersensitivity and Complement 366
7. tv'f~r~piJ.~ge~.~ . ~ian1JI()0f!~.~)''D~~Cy!Q!<1nes ···-·· =··~~--.~~3..§~
8. Immunosuppressants 370
9. Immunodeficiencies
~ '---~~ ,,_
'37~

IX Gl
1. QrophafYnx
2. Esophagus 383
3. §.tomaC:n" . · ; 3~~
4. Upper Gl Pathology 388
5. Duod~-m..ll'n. •• .... . . 390
~ _ , . -. ,::._ .

6. Enterocytes and Absorption 392


7. O:iseas~j_of th~Smalll.rite~tine 894
8. 396
9. 398
10. 400
11. .c+Lt..-m-~-~-=~-""'·'--- AJ:>2
12. Liver Basics 404
13. Al£9.~o1Jf.. Ljy~x [)is~~s~ and Ci(r_hosi$ 406
14. Liver Pathology 408
15. f@atifi! ~·-· 410
16. Biliary Tract 414

r iii l
TABLE OF CONTENTS
X PULM ONARY
1. Anatomy and Physfolggy 419
2. Lung Volumes and Pulmonary Circulation 421
3. OxY_B~n 'ano_-Hemoglgbin_ ··=- ·- ·~''''''~~ '''"'~=·g' '"'~' 424
4. Oxygenation 426
5. Verftilation and Perfusion 428
6. Extreme Environments 430
7. PE, QVT allaPij~umothora~~ 433
8. COPD and Asthma 435
9. PITrm§~arY''I\f1.eatcations ··
10. Restrictive Lung Disease
11. Lung"Cal!9er
12. Lung Infections
XI MED ICINE IN SOCI ET Y
1. Re~ec:~rch Stu'Oy Designs 449
2. Diagnostic Test Evaluation 451
3. lnteri)retation of Study Data 454
4. Study Validity 457
5. StatlsticaTOlstribution andError ~6_0
6. Biostatistics 462
7. Disease P£eventio.n and public Health 464
8. Healthcare System 468
9. Ethics "'· 414
XII BI OCH EM
1. oNABasics 4Z2
2. DNA Replication, Mutation and Repair 483
3. 'RNA '~ .,- 486
4. Protein Synthesis 490
5. Inheritance 4~2
6. Genetic lab Techniques 495
7. Glycolysis _ 498
8. Gluconeogenesis 503
9. ~~y~oieit :. · ·-- -~.5.06

10. Energy Metabolism 509


11. J:IMJL~hunfand()th~! Sugar{ · 513
12. Starvation and Malnutrition 517
13. ,l..ieJ_9~ ··=~ .-~~ ~--~~-=---~----,.._,,~S20
14. Amino Acids and Nitrogen 523
15. ~mino AcJi:l'])isorders · 525
527
529
531
XIII PSYC H
1. 'Psy~hosis · 535
2. Antipsychotics 537
3. slp 9ra·r: nisor aer 540
4. Depression 542
5. A:ntid~pressants - 545'
6. Anxiety and Somatoform Disorders 547
7. EatinK Dis()crders 549
8. ChildPsych -· .. 551
9. Ego Defense Mechanisms and Personal.i~y Disorders 553
10. Alcohol Abuse -· · · · · · ·· .. · . · · ·· · · ·
555
11. ?ubst~nc~..Abuse 558

r iv l
TABLE OF CONTENTS
XIV RENAL
1. Renal Embryology a'li'f.f\natofTii ' 563
2. Glomerular Physiology 565
3. f:J"ephro':!:.PIWsiolo~yL -~··- ••==~·=,···x ·•· ••~·~--·~=·· ~·--~·~~=--·-~==·~~· 567

4. Diuretics 572
5. £1e<;tro{x!c;:J?1~9r~~r~: -~. . •a*"*"'"'--"~"'"'-·~·-,...."~·~~~""'"'''"" _;?7-1
6. Acid-Base Disorders 576
7. f\!ephritif :··· sz!f
8. Nephrosis 580
9. :Q~JiaLE~iliJt~ ~
10. Other Urinary Pathology

XV PEDIATRICS

XVI ENDOCRINE
1. ~!idocrin_e~Over..,iew { _59~
2. Pituitary 601
3. ~J:;I~ehal St~]~id Syri!!Jj sis 605
4. Cushing Syndrome 608
5. Ryeera d~!ero!:lism and _AdrE!l!~t lnsuf:fis!~~cy · 610
6. Adrenal Medulla 612
7. Hypothyroidism ~1_31:
8. Hyperthyroidism 616
9. [lWro-id. ~~~~cer 618
10. Diabetes 620
11. piahetic EmergencE~! · o22
12. Diabetes Drugs 624
13. Qlfesity ·:: • 621:
14. Calcium Metabolism 629
XV II hEME
1. ~(qJtingf~E!9rs
2. Anticoagulant Drugs
3. RB·c ·easlcs__: ~ ·
4. Erythropoiesis
5. fV1i£IOCJI!i£f\.ri~lllJa · ...
6. Macrocytic and Normocytic Anemia
7. Hemol)'tLc Anemia
8. Platelet Function
9. el~!~t~t.Qj5Qr~ers~:
10. Lymphoma
11. Leul<emias
12. Multiple Myeloma
XVIII ONCOLOGY
1. c·ancer Basics
~-- -··« -- ~-
663
2. Genetics of Cancer 665
3. C'incer RiskF'actors 66.8
4. Cancer Screening and Prevention 670
5. ~.ancer Dr~s-part1-·:· 612.
6. Cancer Drugs part 2 674

rv 1
NOTES

KEY TO AB BREVI ATI ONS


This Study Guid e contains page references to several medical educational resources. Below is a key to
the abbreviations that accompany t he va rious page number references appearing in this text.

Le, T, Bhushan , V, et al. First Aid for the USMLE Step 1 2016. New York, NY:
FA16
McGraw-Hill; 2016.

Le, T, Bhushan, V, et al. First Aid for the USMLE Step 1 2015. New Yo rk, NY:
FA15
McGraw-Hill; 2015 .

Jenkins, B, et al. Step-Up to USMLE Step 1 2015. Philadelphia, PA: Lippincott


su Williams & Wilkins; 2014.

Hall , JE. Guyton and Hall Textbook of Medical Physiology. 12th ed. Phil adelphia,
Phys
PA: Saund ers Elsevier; 2011.

Kumar, V, Abbas , AK, et al. Robbins and Cotran Pathologic Basis of Disease. 9th
R
ed. Philadelphia, PA: Saunders Elsevier; 2015.

Moore, KL, Dailey, AF, & Agur, AMR. Clinically Oriented Anatomy. 7th ed .
COA
Philad elphia , PA: Lippincott Williams & Wilkins; 2014.

Longo, DL, Fauci, AS, Ka spe r, DL, Hauser, SL, Jameson, JL, Loscalzo, J, eds.
H Harrison's Principles of Internal Medicine. 18th ed. New York, NY: M cGraw-Hill ;
2012.

Brunton, LL, Chabner, BA, & Knoll man, BC, eds. Goodman &Gilman's The
GG Pharmacological Basis of Therapeutics. 12th ed. New York, NY: McGraw-Hill;
2011.

r , ,; l
NOTES

COURSE LECTU RERS


• Brian Jenkins, M.D.
• Michael Mcinnis, M.D.
• Chris Lewis, M.D.
• Mohit Joshipura, M.D.
• Mary Beth Cox, M.D.
• Hampton Richards, M.D.
• Anji Selzer, M.D.
• Jennifer Shuford , M.D.

COURSE VIEWING OPTIONS


Part 2 consists of 244 videos. Examples of 15, 20, 24, 27, and 30-day plans are provided below for
maximum flexibility to meet your personal study needs.
, - -- -····-- · · · ·· · - -·········-·········· · -··········- ····- - ·· - -, - -·········-- · ······- -··········- -·······

No. of Videos/Day Video Runtime/Day*


f----······--·····--·-······--f···--······-·······-·--·· ,.-----··········- --·-····- -··········-···
15-day plan 16 Approx. 5.5 hrs

20-day plan 12 Approx. 4 hrs

24-day plan 10 Approx. 3.5 hrs


!-···-- -········-- ······- - · + - · · - - · -- - ······-- ... - - · · · ···········- · · ········--·······
27-day plan 9 Approx. 3 hrs

30-day plan 8 Approx. 2.5 hrs


1...... ---·········--·············--····.L.-········- ·········· -······--····-~··· ·······--······-·········--···········

*Does not include study breaks or tim e spent annotating and answering questions.

r vii l
RECOMMENDED COURSE ORDER
Although you have the fl exibility to view the videos in any order, we strongl y recommend that yo u watch
the videos in the order in which your personalized dashboard presents them regardless of how many videos
you view in a day.

1A Fou:r\:dations 1 - Embryology ·
'18 Fo tions 2 - Teratogenesis
~K 'Fo 'ons 3- Biology:
1:o Fo't'J'~Eatlons 4- G~!Lstdlogy;
1E Fo(Ihdations's- Cei(Siology: Pt~ma Membrane
:1F ations 6 -Cellular Suffering and Death
>
1 5· ·ons T'- matio
1H Foun )3tions 8 -Extracellular onment
11 FoLtridations 9 - Cellular Adaptations
1J Neuro 1- Nervous'
£""""' ._.
<
System
->>•" •"'"····· .
Baslcs
. 0

1K Neu.rg 2 - CNS Embryology


1L
1M Neurb4: VascularEvents
J.N NeL!~~bS
i. ~~
- Hypqthatanius and
. • . .•
Li~bic System -c0 .12e~\---~C0·"""""~--
--== 1

2A Neuro 6 - Cranial Nerves part 1


2B Neuro 7 - Cranial Nerves part 2
2C Neuro 8 - Brainstem and Spinal Cord
20 Neuro 9 - Brain stem Lesion s
2E Neuro 10 - CSF and ICP
2F Neuro 11- Headache
2G Neuro 12 - Brain Tumors
2 H Neuro 13 - Delirium and Dementia
21 Pharm Basics 1 - Parasymp athetic Activation
2J Pharm Basics 2 - Parasympathetic Inhibition
2K Pharm Basics 3 - Sympathetic Activation
2L Pharm Basics 4 - Sympathetic Inhibition
2M Pharm Basics 5 - Neurotransmission

3D
3E
3F
-GG
l
.3 H Card1ovascular l ~. [mbryology,
Ca_r!iiovascular
, .. .. . .
2·1c·ongenital
,___ ..

Card~ovascular 3 :cardiac Out


GK · Cardi<)vascularA .c'"bteart Failure·3 ,

3L car8Ievascular~~~~freatmer1t of0HF
3.M Cardla_'l(~scular 6 '.. ~dema and sHock

r viii l
RECOMMENDED COURSE ORDER
4A Cardiovascular 7 - Cardiac Cycle
4B Cardiovascular 8 - Diastolic Heart Murmurs
4C Cardiovascular 9 - Systo lic Heart Murmurs
4D Cardiovascular 10 - Electrophysiology
4E Cardiovascular 11 - Antiarrhythm ics
4F Cardiovascular 12 - El ectrocardiography
4G Cardiovascular 13 -Arrhythmias
4H Rheum & Derm 1 - Bone Formation and Bone Tumors
41 Rheum & Derm 2 - Bone Disorders
4J Rheum & Derm 3 - Musculoskeletal Injuries
4K Rheum & Derm 4 - Osteoarthritis and Rheumatoid Arthritis
4L Rheum & Derm 5- Other Types of Arthritis
4M

ction 1 - Reproducti,~e Anatom~ .


uctiqh 2 - Genital Erribryol9gy

Repro~uctioh12 '!' . Pregnancy part:1


f$~QrO~IJ~C~~l~~'"':_.:..=:.~>hc~~l~.t:::.':!!.Jtc.~.....;.ii',~-2atc~~··;""--...i$···--'2!liii..........~>-..~···Cb~,,,,
6A Reproduction 14 - Chromosoma l Disorders
6B Reproduction 15 - Genetic Disorders: AD and Trinucleotide Repeats
6C Reproduction 16 - Genetic Disorders: AR and X-Linked
6D Reproduction 17- Breast
6E Micro 1 - Bacterial Basics
6F Micro 2 - Bacteria l Toxins
6G Micro 3 - Staphylococcus
6H Micro 4 - Streptococcus
61 Micro 5 - Other Gram-Positive Bacteria
6J Micro 6 - Gram -Negative Bacteria part 1
6K Micro 7 - Gram-Negative Bacteria part 2
6L Micro 8- Spirochetes and Zoonotics
~ 6M Micro 9 - Mycobacteria
6N Micro 10 - Nonstaining Bacteria

r ix l
RECOMMENDED COURSE ORDER
7A lmm~nology 1- lmmll'ne SystemBasics
78 lm ology 2 -Antigen Presentation
7C ology 3 lis
ilD ells and Antib~odies
7E lm~ui:lology 5 - Immunization ~nd Autoantibodies
7F . Immunology 6 .:. Hypersensitivity and Complement
f(G lnr logy 7 - M<3crophages Cytoki
;7H logy 8 ° lmcfuunosuppr
71
'
'lJ Neuro 14 - Seizure$
7K Neu~o15 - SpinaiCord Lesion
'? L Neurg16 - Move~nt
7M Neuro 17- Basal Ganglia
"" -- ~-"'--'-- ~i;n!i

SA Neuro 18 - Skeleta l Muscle


8B Neuro 19- Upper Extremity Nerves part 1
8C Neuro 20 - Upper Extremity Nerves part 2
80 Neuro 21 - Lower Extremity
BE Neuro 22 - Sensation
SF Neuro 23 - Eye
8G Neuro 24 - Ear
8H Neuro 25- Sleep
81 Neuro 26 - Anesthetics
8J Gl 1 ~ Oropharynx
8K Gl 2 ~ Esophagus
8L Gl 3 ~ Stomach
8M Gl 4 ~ Upper Gl Pathology
8N Gl 5 - Duodenum
80 Gl 6- Enterocytes and Absorption
8P ..... Gl 7 ~ Diseases of the Small Intestine
~ ···-··------~ -···-· -·- - - - -~.-.- ~.- --~

9A ·GIB ;:Large lntes§nepart 1


98 Gl9 :=;:Large Intestine part 2
~c
9-D
9E ~

9F Gl 13 - Alcoholic Liver Disease and Cirrhosis


9G Gl Liver Path
9H G
..
91

9M t>ulmqnary 4 - Oxygenation
9N ·pu!rnqJ:ary 5 - Ven~lation and Perf,Ysion

rx 1
RECOMMENDED COURSE ORDER
10A Pulmonary 6 - Extrem e Environments
108 Pulmonary 7 - PE, DVT and Pneumothora x
10C Pulmonary 8 - COPD and Asthma
10D Pulmonary 9 - Pulmonary Medications
10E Pulmonary 10 - Restrictive Lung Disease
10F Pulmonary 11 - Lung Cancer
10G Pulmonary 12 - Lung Infections
10H Cardiovascular 14 - Regulation of BP
101 Cardiovascular 15 - Hypertension
10J Cardiovascular 16 -Anti hypertensives
10K Cardiovascular 17- Atherosclerosis
10L Cardiovascular 18- Antianginal Therapy and Lipid -Lowering Agents
10M Cardiovascular 19- Myocardial Infarction part 1

11C Cardlo~ascular 23 - Vas~ul ibs


~1 ardiovascular 24 ::' Other Vascular Di$e-Bses
1.1 ediGine in Society 1 - Research Study Designs
Medk;ine in Society: 2 - Diagnosti~,Test~va
;.::_',X- .<, ,. _.,-._, -- __. = -> ._..
"";-~" /'"-
luati6n ~

~ Medicine in,SoCiety,,3- lnterpret;:rtlon of>StudyData


tl 1 edidne in $ocietV'tt"- Stucdy Vaildity

:1 1 edic;ine in Society6 -: Brdstatistics , , ,....


,edid1ne
%:''"-
in Societ/7 - Dis~ase ~ieventlon
~ T- --- -

... •,Medic4rje in §pciety:B - H~althcC}.r;~ System


Medicine in'S~ciet~;9 - Etfifcs ....'-~"----""---~-------"--.EG~~----'-_.
2
!11M
12A Micro 11 - Penicillins
128 M icro 12 - Cephalosporins
12C Micro 13- Other Cell Wall Inhibitors
12D Micro 14 - Protei n Synthesis Inhibitors
12E Micro 15 - Other Antibiotics
12F Micro 16 - Micro by Systems part 1
12G Micro 17- Micro by Systems part 2
12H Micro 18 - Micro by Systems part 3
121 Biochem 1 - DNA Basics
12J Biochem 2- DNA Replication , Mutation and Repair
12K Biochem 3- RN A
12L Biochem 4- Protein Synthesis
12M Biochem 5- Inheritance
12N Biochem 6- Genetic Lab Techniques

f xi l
RECOMMENDED COURSE ORDER
13A Psyd'i1 - Psycho$i;S
138 Psycl::l2 - Antip
f;l3C Psy§f1' 3 - Bipol sorder
&3 D Psych4 - Depr~ssio_n
'13E Psych5 - Antidepcessants
:13F Ps -
[3(;' p

131
13J
\

14A Renal 1 - Renal Embryology and Anatomy


148 Renal 2 - Glomerular Physiology
14C Renal 3 - Nephron Physiology
14D Renal 4 - Diuretics
14E Renal 5 - Electrolyte Disorders
14F Renal 6- Acid-Base Disorders
14G Renal 7 - Nephritis
14H Renal 8 - Nephrosis
141 Renal 9 - Renal Failure
14J Renal 10 - Other Urinary Pathology
14K Pediatrics - Pediatric Review
14L Biochem 7 - Glycolysis
14M Biochem 8 - Gluconeogenesis
14N Biochem 9 - Glycogen

15D Biochem
,< -¥0"
13_- Lipids
-

;15E
rtSF Biothem 15 - Amino Acid D

ISJ
15K
15L En e 3-
15M EndO"crine 4 - Cushing Syndrome ·
;1 5N Endocrine 5 - Hytyr:;[aldosteroni -
150 6 - Adrenal Medu ·
&,,;_;=ci-~

r xii l
RECOIUIENDEDCOURSEORDER
16A Endocrine 7 - Hypothyroidism
168 Endocrine 8 - Hyperthyroidism
16C Endocrine 9 -Thyroid Cancer
16D Endocrine 10- Diabetes
16E Endocrine 11 - Diabetic Emergencies
16F Endocrine 12 - Diabetes Drugs
16G Endocrine 13 - Obesity
16H Endocrine 14- Ca lcium Metabolism
161 Heme 1 - Clotting Factors
16J Heme 2 - Anticoagulant Drugs
16K Heme 3 - RBC Basics
16L Heme 4 - Erythropoiesis
16M Heme 5 - Microcytic Anemia

eme
4
eme 9 - Platelet-DisordE;rs
emE;:iO - Ly:mphorna
~

-17D-:Heme 11 - Leukemias
'17L:H;m~~ 12 - Multiple~Myeloma
-17F : Micro-~19- Vi~al Basics
n 7G-_MicrcY20 -15NA Virl!ses part 1
-17H::Micro21 - [)!JA Viruses part 2

18A
18B Micro 27 - Mycology part 2
18C Micro 28- Antifungals
18D Micro 29 - Protozoa
18E Micro 30 - Helminths and Ectoparasites
18F Onco logy 1 - Cancer Basics
18G Onco logy 2 - Genetics of Cancer
18H Oncology 3- Cancer Risk Factors
181 Oncology 4- Cancer Screening and Prevention
18J Oncology 5- Cancer Drugs part 1
18K Oncology 6- Cancer Drugs part 2

r xiii l
FROM EMBRYOLOGYTO

FOUNDATIONS THE FUNDAMENTALS


OF CELL BIOLOGY
'
INFLAMMATION, AND
l Embryology
APOPTOSlS, THIS
SECTION REVIEWS
2 Teratogenes is
SOME OF THE<BASIC
SCIENCE THAT FORMS
3 Cell Biology Organelles
THE FOUNDATIONAL
UND~RPINNJNGS OF
4 Cell Biology Cytoskeleton
MUCH. . OF OUR MEDICAL
KNOWLEDGE
5 Cell Biology Plasma Membrane

6 Cellular Suffering and Death

7 Inflammation

8 Extracellular Environment
/.---.......
9 Cellular Adaptations
rn
s:
EMBRYOLOGY OJ
:::0
-<
0
Embryonic development (FA15 p558) (FA16 p562) r---
0
Placental development (FA15 p562) (FA16 p566) Cl
Gastrulation (FA15 p558) (FA16 p562) -<
Neurulation (FA15 p448) (FA16 p450)
Neural crest derivatives (FA15 p559) (FA16 p563) (SU p23-24)
Germ layer derivatives (FA15 p559) (FA16 p563)
Organogenesis (FA15 p558) (FA16 p562)

1. What tissues and cells are derived from ectoderm, mesoderm and endoderm?

Morula

• Placenta
• Embryonic membranes

[3 J
>-
(__:) 2. What neural crest derivatives are found in each of the following adult structures?
0_ j
0
>-
0:::
a:J Peripheral nervous system
2:
LLJ

Ear

Eye

Mouth

Thyroid

Heart

Digestive system

Adrenal gland

Skin

End of Sess1o n Qu1 z


3. What is the relationship between the notochord, the neural crest, the neural plate,
and the neural tube?

4. What is the embryologic origin of each of the following adult structures?


• Anterior pituitary
• Posterior pituitary
• Sclera
• Lens
• Retina
• Mammary glands
• Parotid glands
• ..Sweat glands

5. At what developmental age does each of the following events typically take place?
• Four-chamber heart begins to develop
• Genitalia begin to take on visible sex-specific characteristics
• Fetal movement begins
• Limb buds begin to form

r4 1
-j
rn
TERATOGENESIS :;:o
~
0
GG: Chapters 23, 66 Gl
rn
Teratogenic drugs (FA15 p560) (FA16 p564) (SU p397) (GG p1845) z
rn
Other teratogens (FA15 p560) (FA16 p564) Vl
Vl
Fetal alcohol syndrome (FA15 p561) (FA16 p565) (GG p641)
Homeobox (HOX) genes (FA15 p558) (FA16 p562)
Genes of embryogenesis (FA15 p558) (FA16 p562)

1. Teratogenic drugs
• ACE inhibitors and ARBs - _ _ _ _ __ __ _ _ _ _ _ _ _ __
• Aminoglycosides - _ __ _ _ __ __ __ __ __ _ _ _ __
• Fluoroquin olones - _ _ _ _ _ __ _ _ __ _ _ _ _ _ _ __
• Tetracyclines- _ _ _ _ _ _ __ __ __ __ _ __ _ _ __
• Cyclophosphamide - ear/facial anomalies, limb hypoplasia, absence of digits
• Methotrexate - neural tube defects, abortion
• Carbamazepine - _ _ _ _ _ _ _ _ _ _ _ _ _ _ _ _ _ _ __
• Valproic acid - _ _ _ _ _ _ _ _ _ _ _ _ _ _ _ _ _ _ __
• Phenytoin - fetal hydantoin syndrome
• Lithium -
• Statins - CNS and limb abnormalities
• Warfarin - bone/ca rtilage defects, nasal hypoplasia
• lsotretinoin - _ __ _ _ __ _ _ _ _ _ _ _ _ _ _ _ _ _ __
• Diethylstilbestrol (DES) - clear cell vaginal adenocarcinoma
• Thalidomide - phocomelia

2. Other teratogens
• _ _ _ _ _ _ _ _ _ _ __ _ _ __ _ - CNS malformations (early exposure), malignancy (late
exposure)
• Excess - spontaneous abortion, microcephaly, cardiac
defects; can interfere with neural crest cell migration or HOX gene expression
• rvlaternal - cardiac defects, neural tube defects, caudal
regression syndrome
• rvlaternal _ __ _ __ _ _ _ _ _ _ _ - congenital hypothyroidism (cretinism)
• _ _ _ _ _ _ _ _ _ _ _ _ _ _ _ _ - placental abruption , low birth weight, IUGR
• _ _ __ _ _ _ _ _ _ _ _ __ ___ -placental abruption, low birth weight, IUGR
• _ _ __ _ __ _ _ __ _ _ _ __ _ - fetal alcohol syndrome (see below)

3. Fetal alcohol syndrome


• Facial abnormalities (narrow eye openings, wide-set eyes, smooth philtrum, thin upper lip, cleft
palate)

rvlicrocephaly
Holoprosencephaly

[ 5]
U)
U)
4. Homeobox (HOX) genes
w
z
w
• Blueprint for skeletal morphology
(_:J
0 • Code for transcription regulators
~ • Mutation in Homeobox HOXD-13 ~ synpolydactyly (extra fused digit between 3rd and 4th fingers)
er::
LLJ
f- • Retinoic acid alters HOX gene expression

EndofSe
5. .Which antibiotics are potentially teratogenic?

6. A pregnant woman ·is diagnosed with a deep venous thrombosis of the left leg. Which
anticoagulant is contraindicated during pregnancy? Which anticoagulant is preferred?

Examination of a fetus from a spontaneous abortion at ·14 weeks gestation reveals


a vestigic:d pair of legs that arises from the torso just below the arms. A mutation in
whatfamily of genes is the most likely cause ofthis malformation?

RAP! O-F! RE FACTS


Most common cause of neural tube defects

Most common preventable cause of congenital


malformations in the U.S.

[ 6]
(}
m
CELL BIOLOGY: ORGANELLES r
r
OJ
Phys: Chapter 2 0
r
R: Chapter 3 0
Cl
Nucleus (Phys p17) ~
Regulation of the cell cycle (FA15 p72) (FA16 p58) 0
:::0
Endoplasmic reticulum (FA15 p72) (FA16 p58) Cl
)>
Trafficking proteins (FA15 p73) (FA16 p59) z
Golgi apparatus (FA15 p73) (FA16 p59) m
r
/-cell disease (FA15 p73) (FA16 p59) r
m
Chaperones and heat shock proteins (FA15 p71) (FA16 p57) Vl

Protein degradation (FA15 p73) (FA16 p59)


Sarcoplasmic reticulum (Phys p73)
Mitochondria (Phys p16)
Peroxisomes (FA15 p73) (FA16 p59) (Phys p15)

1. Nuclear localization signals


• Amino acid sequences: 4-8 amino acids; rich in proline, arginine, and lysine
• Essential component of proteins bound for or residing in the nucleus (e.g., histones)
• Nuclear pores recognize these signals and transport proteins into the nucleus via ATPase
• A mutation in a single amino acid may prevent nuclear transport

2. Cell cycle basics


• Cyclins + cyclin-dependent kinases (CDK) phosphorylate target proteins to drive the cell cycle
• All cyclins are degraded by when their cell
cycle-specific job is complete
• p21, p27, and p57 bind to and inactivate cyclin-CDK complexes (p53 controls the transcription of p21)

G 1 -t S
• Cyclin D binds/activates CDK4 -t phosphorylation of Rb protein -t Rb protein is released from
transcription factor E2F -t with E2F unbound, the cell is free to transcribe/synthesize components
needed for progression through the S phase (cyclin E, DNA polymerase, thymidine kinase,
dihydrofolate reductase)
• Cyclin E binds/activates CDK2 -t the cell is allowed to progress into S phase

G2 -t M
• Cyclin A - CDK2 complex -t mitotic prophase
• Cyclin B - CDK1 complex is activated by cdc25 -t breakdown of _ __ _ _ _ _ _ _ _ _ __
and initiation of mitosis

[7J
(.()
w
_j
3. 1-cell disease
_j
w • Deficiency in mannose phosphorylation
z
<(
l'J
• No _ _ _ __ __ _ _ __ _ __ __ __ to target lysosoma l proteins --7 secretion out
cr:
0 of ce ll instead of into lysosomes
;>.: • Death by age 8
l'J
0
_j • Corneal clouding, coarse facies, hepatosplenomegaly, skeletal abnorma lities, restricted joint
0
OJ
movement
_j
_j • May have intellectual disability
w
u

4. Chaperones and heat shock proteins


• Chaperones assist in the proper folding and transport of polypeptides in the ER, Golgi, etc.
• Some chaperones are synthesized constantly and are involved in normal intracellular protein
trafficking.
• Other chaperones are induced by stressors such as _ _ __ _ _ __ _ __ _ _ _ _ _ __
(heat shock proteins hsp70 and hsp90). Th ese chaperones "rescue" shock-stressed proteins from
misfolding.
If the folding process is not successful , the chaperones facilitate degradation of the damaged
protein.
- This degradative process often involves ubiquitin (also a heat shock protein), which is added to
the abnormal protein and marks it for degradation.

5. Peroxisomes
• ~-oxidation of very long chain fatty acids and branched chain fatty acids
• Synthesis of _ _ __ _ _ __ _ _ _ _ _ ___ (important phospholipids found
in _ _ _ _ _ _ _ _ _ _ __ _ _ __
• Oxidases and catalase for metabolizing ethanol (and other toxic substances)

r a1
n
fTl

''
Quiz OJ
0
r-
6. What must be present on a protein in order for that protein to gain entry into the 0
G>
nucleus? ;-<:
0
:;:o
G>
)>
z:
fTl

'
'rn
7. Which cyclin"CDK complexes assist in the progression fr:om G1 phase to S phase? U1

8. Which cyclin-CDK complexes assist inthe progression from G2 phase to


M phase?

9. What molecule.does the Golgi apparatllsadd to proteins in order to directthe


proteins to the lysosomes?

10. What are the different methods that a cell uses to break down proteins?
z
0
f-
UJ
_j
CELL BIOLOGY: CYTOSKELETON
UJ
~ Phys: Chapter 2
U)
0 Microfilaments (FA15 p74) (FA16 p60)
f-
>- Intermediate filaments (FA15 p74) (FA16 p60)
u
;;...: Microtubules (FA15 p74) (FA16 p60)
\CJ Cilia and flagella (FA15 p74) (FA16 p60)
0
_j Primary ciliary dyskinesia (FA15 p74) (FA16 p60) (SU p61)
0
co
_j
_j
UJ 1. Intermediate filament structures
u
Intermediate filaments Structural component of: Stain used to identify...

Cytokeratin Epithelial cells (keratin


in desmosomes and
hemidesmosomes)

Desmin Muscle cells (smooth,


skeletal, cardiac)

Glial fibrilla ry acid proteins Astrocytes, Schwann cells,


(GFAP) other neuroglia

Neurofilaments Axons within neurons


(L, M, H molecular weight)

Nuclear lamins (A, B, C) Nuclear envelope and DNA


within

Vimentin Connective tissue


• Supports cellular (fibroblasts, leukocytes,
membranes endothelium)
• Keeps certain organelles
fixed in cytoplasm

r 10 1
n
m
''
nd Sesslon IZ
0
OJ

2. What drugs act on microtubules? '0


C)
;-<.
n
-<
___,
0
(J)
:;><;
m
'
m
___,
0
z

3. What intermediate filaments are found in the following tissue types and cellular
structures?
• Connective tissue
• Muscle tissue
• Epithelial tissue
• Axons

4. What are the defects se~n in Kartagenersyndrome?

r II 1
UJ
z
<1:
0::
CELL BIOLOGY: PLASMA MEMBRANE
m
~ Phys: Chapters 2, 4
UJ
~ R: Chapter 2
<1:
~ Composition of the plasma membrane (FA15 p75) (FA16 p61)
(/)
Membrane-bound receptors
<1:
_j
Q_
Enzyme terminology (FA15 p96) (FA16 p84)
Sodium-potassium ATPase (FA15 p75) (FA16 p61)
:>-'
l'J Receptor-mediated endocytosis (FA15 p73) (FA16 p59)
0 Arachidonic acid (FA15 p444) (FA16 p446) (SU p243)
_j

0
m
_j
_j

tj 1. Tyrosine kinase receptors


• Transmembrane receptors that bind an extracellular ligand then intracellularly transfer a
phosphate group from ATP to selected tyrosine side chains on specific cellular proteins
(__________________________~
• The first step in the signaling cascade is _ _ _ _ _ _ _ _ _ _ _ _ _ __ _ _ __ __
• PDGF and other growth factor receptors: single-pass transmembrane protein
• Insulin and IGF-1 receptors:
Two a subunits (bound by disu lfide bonds) - bind extracellular ligand
Two ~subunits - tyrosine kinase activity

2. Enzyme terminology
Adds a phosphate group using ATP

Removes a phosphate group

Adds inorganic phosphate (without using ATP)

Adds carboxyl group (COOH); requires biotin


as a cofactor

Removes a carboxyl group

Adds a hydroxyl group (OH)

Removes a hydroxyl group

Oxidizes a substrate using an electron acceptor


(NAD+ or NADP+)

[ 12 J
3. Receptor-mediated endocytosis n
m
''
OJ
Clathrin 0
0'
C)
:-<
-o

cargo receptor
')>
(J)
:s
)>

:s
m
:s
OJ
Al
)>
z
m

A) NormallDl receptor
LDL binding site
LDL
LDL receptor protein A mutant LDL
receptor lacks the
coated-pit binding
clathrin site but retains a
functioning LDL-
CYTOSOl binding site. As a
result, cells with
mutant receptors
B) Mutant lDl receptor are able to bind
LDL normally but
are unable to ingest
it Individuals with
this mutation have a
higher risk of dying
LDL receptor protein with prematurely from a
· .,..abnormal coated pitj:)indiog sit~:! myocardial infarction.
CYTOSOl

r 13 1
L1.J
:z: 4. List the steps outlining the derivatives of arachidonic acid.
<t
0:::
en
~
L1.J
~
E
<t
~
<J)
~ D
<t
_.J
o_
;:,..: F
l'J
0_.J
0
en
l
_.J
G

~~
_J
L1.J
u

A N.

B.

c.
D.
r
c. 0.

F. p

G.

H.

I. Q. _____________________________

J. --······-····------------------- R. -----------------------------
K.

L.

M. _____________________________
()
rn
.-
.-
En d of Sess ion Qu iz CD
0
5. Which arachidonic acid product causes each of the following effects? .-
0
C)
• Increased bronchial tone ;-<:
""D
• Decreased bronchial tone
')>
• Increased platelet aggregation Vl

• Decreased platelet aggregation


:s
)>

• Increased uterine tone :s


rn
• Decreased.uterine tone :s
OJ
;:o
• Increased vascular tone )>
z
• Decreased vascular tone rn

6. What are the two most abundant substances in plasma membranes?

7. What drugs act on the arachidonic acid product pathway? What enzymes do they
affect?

r 1s 1
:r
~
u.J
0
CELLULAR SUFFERING AND DEATH
0 R: Chapter 1
z
<l: Apoptosis (FA15 p222) (FA16 p216)
<-.:)
z - Intrinsic pathway
IX - Extrinsic pathway
u.J
l..J.... Necrosis (FA15 p223) (FA16 p217)
l..J....
=:J Reversible vs. irreversible injury (FA15 p224) (FA16 p218)
l/)
Mechanisms of cell injury (FA15 p224) (FA16 p218)
IX
<l: Defects in membrane permeability
_j

=:J Red vs. pale infarcts (FA15 p224) (FA16 p219)


_j
_j
u.J
u 1. Intrinsic pathway of apoptosis
• Bcl-2 is the major anti-apoptotic regulator of mitochondrial permeability
• DNA damage or apoptotic signal activates BAX (pro-apoptotic)
• BAX creates channels in mitochondrial membrane
• Cytochrome c moves from the mitochondria into the cytosol and activates caspases
• Caspases are primary initiator of apoptosis

2. Extrinsic pathways of apoptosis


• Death receptor
TNF receptor and Fas receptor (FasR) are located on cell membrane
TN Fa and Fas ligand (FasL) activate the receptors
Activated receptors will lead to activation of caspases
• Cytotoxic T lymphocyte (CTL)
Cytotoxic T cells recognize foreign or infected cell
CTLs release perforin and granzyme B
Perforin punches holes in membrane
Granzyme B enters and activates caspases

Intrinsic Extrinsic

TN Fa

Mitochondria

Killer T cell

Cellular breakdown

I i I) l
3. What cellular by-products might you detect in the serum when the following cell types are injured? n
m

Cardiac myocytes
'
c'
'
)>
Skeletal myocytes ;;o
V1
c
Hepatocytes -n
-n
m
;;o
Salivary gland cells z
Gl
Pancreatic exocrine cells )>
z
0
RBCs 0
m
~
I

4. Mechanisms of cell injury


• ATP depletion: due to decreased oxygen/nutrients and toxins (cyanide)
• Mitochondrial damage: impairs ATP production and can induce apoptosis
• Influx of calcium: increases mitochondrial permeability and can activate phospholipases, proteases,
endonucleases and ATPases
• Accumulation of oxygen-derived free radicals: cell damage through membrane lipid peroxidation,
protein modification and DNA breakage. Multiple causes like radiation exposure, metabolism of
drugs, redox reaction, nitric oxide, transition meta ls, leukocyte oxidative burst, iron overdose and
reperfusion injury.

End of Session Outz


5. What histologic features are seen in apoptotic liver cells?

6. What substances do cytotoxic T cells and NK cells use to induce apoptosis in the cells
infected with a virus?

7. What highly damaging events can cause irreversible cell injury?

8. What cellular enzymes are responsible for handling oxygen free radicals?

r 17 1
INFLAMMATION
R: Chapter 2

Acute vs. chronic inflammation (FA15 p225) (FA16 p219)


Neutrophil extravasation (FA15 p227) (FA16 p221)
Transudate vs. exudate (FA15 p230) (FA16 p224)
Granulomatous diseases (FA15 p230) (FA16 p223)
Ery throcyte sedimentation rate (ESR) (FA15 p230) (FA16 p224)
C-reactive protein (CRP) (FA15 p205) (FA16 p198) (R p99)

1. Acute inflammation
• Release of inflammatory mediators

• Increased vascu lar permeability~ fluid exudation

Serotonin
Bradykinin
• Fibrosis
Fibroblasts lay down - - - -- -- - - -- -- - -
(requ ires - - -- -- - - -- -
• Tissue remodeling by metalloproteinases (which contain _____________

2. Chronic inflammation
• Mediated by _____ _ _ _ _ _ _ _ _ __
• Granuloma: clusters of macrophages, lymphocytes and fibroblasts
• Macrophages fuse plasma membranes to form multinucleate giant cells

3. Neutrophil extravasation
• Rolling: neutrophils roll along vascular endothelium
• Tight binding: ICAM-1 (on endothelial cell) and LFA-1 (on leukocyte)
• Diapedesis: leukocyte moves between endothelial cells with help from PECAM-1
• Migration: leukocyte travels through the interstitium

r 18 1
,z
En SessJ QuTz '
)>
s
s
4. Describe the process of leukocyte extravasation. ~
0
z

5. What cytokine is particularly important in the formation of granulomas?

6. Whafcell type plays a role in .inflammation by generating fibrinogen and CRP?

7. cell is mostresponsible for the acute phase of inflammation?

r 19 1
r-
z
LLJ
::?:
z
EXTRACELLULIRERnnRONMENT
0 R: Chapter 3
a:
>
z
Collagen (FA15 p75) (FA16 p61) (SU p294)
LLJ Collagen synthesis (FA15 p76) (FA16 p62) (SU p294-295)
a: Osteogenesis imperfecta (FA15 p76) (FA16 p63) (SU p294)
<(
_j
Ehlers-Danlos syndrome (FA15 p77) (FA16 p63) (SU p294)
::::J
_j A/port syndrome (FA15 p541) (FA16 p547) (SU p294)
_j
LLJ Elastin (FA15 p77) (FA16 p64) (SU p296)
u
<( Angiogenesis
a: Cutaneous wound healing (FA15 p229) (FA16 p223)
r-
x
LLJ

1. Cutaneous wound healing


0 -3 hours Hemorrhage and clotting
12-24 hours Acute inflammation (PMNs)
1-3 days • Macrophage infiltration
• Granulation tissue (fibroblasts and vascular endothelial cells)
• Epithelialization
Weeks-months Collagen production (type Ill then type I)

f 20 l
rn
X
---1

En d Sess 1on u1z :;o


)>
n
rn
,-
What findings are associated with Ehlers-Danlos syndrome? ,-
c
,-
)>
:;o
rn
z
<
:;o
0
z
s:
rn
z
---1

· 3. What are the diffen:mt types of collagen, and where can they be found?

4. Which amino adds are found in large concentrations in collagen? In elastin?

5. What is the role of vitamin in collagen production?

RA PID - FIRE FACTS


Hyperflexible joints, arachnodactyly, aortic
dissection, lens dislocation

Hereditary nephritis, cataracts, sensorineura l


hearing loss

I 2.1 1
VJ
z
0 CELLULAR ADAPTATIONS
s
o_
<(
R: Chapters 1, 3

0 Hypertrophy vs. hyperplasia (FA15 p233) (FA16 p226) (R p34)


<(
Stem cells (R p26-27)
0:::
<( Metaplasia (FA15 p233) (FA16 p226)
_j

:::J Atrophy (FA15 p225) (FA16 p226) (R p36-37)


_j
_j Cellular aging (R p66-67)
UJ
u

of.Sess ton . . .Outz


1. Does the compensatory growth of muscle fibers occur primarily as a result of hyperplasia
or hypertrophy?

2. Does myometrial growth in pregnancy occur primarily as a result of hyperplasia or


hypertrophy?

What can happen to the cells of the lower esophagus in response to chronic acid
reflux?

4. . What is actually occurring at the cellular level during atrophy?

5. What is a lipofuscin granule?

r n:
NEUROANATOMY AND

NEUROLOGY EUROSCIENCEARE
QUITE<IMPOSING FOR
LOT OF STUDENTS...
1 Nervous System Bas1cs
AND ALSC} FREOUEN
2 CNS Embryology TESTED. TO MAKE IT
MORE MANAGEABL
3 Cortex and Brain Vasculature
WE HAVE DIVID ED
4 Vascular Even t s NEUROLOGY TOPICS
5 Hypothalamus and L1mbic System INTO TWO DISTINCT
GRO OF
6 Cran1al Nerves part 1 WITHIN PART2 WE
7 Cran1al Nerves part 2 STRONGLY ENCOU
YOU TO FOLLOW
8 Brainstem and Spinal Cord
RECOMMENDED COURSE
9 Bramstem Les1ons ORDER AS NED ON
10 CSF and ICP YOUR DASHBOARD.

11 Headache
12 Bram Tumors
13 Del ir1u m and Dementia
14 Sei zures
15 Spinal Cord Lesions
16 Movement
17 Basal Gangl1a
18 Skeleta l Muscle
19 Upper Extremity Nerves part 1
20 Upper Extremity Nerves part 2
21 Lower Extrem1ty
22 Sen sation
23 Eye
24 Ear
25 Sleep
26 Anesthetics
NERVOUS SYSTEM BASICS
R: Chapter 28
GG: Chapter 14
Phys: Chapter 45
Cells of the nervous system (FA15 p451-452) (FA16 p453-454)
Blood-brain barrier (FA15 p453) (FA16 p455) (SU p27) z
m
Neurotransmitters (FA15 p453) (FA16 p455) (GG p376) (Phys p550) ;;u
<
0
c
Ul

Warm -Up Rev1ew Ul


-<
Ul
--1
1. What are the three germ layers that are derived from the epiblast? m
s:
co
)>
Ul
n
Ul

Which amino acids are modified in the Golgi apparatus? (FA15 p73) (FA16p59)

3. Failure ofwhat process results in 1-cell disease? (FA15 p73)(FA16 p59)

4. Dopamine
• Schizophrenia: increased
• Depression: decreased
• Parkinson disease: decreased

5. What are the four major dopaminergic pathways, and what is the result of blocking these
pathways?
Major pathway Result of blocking

Mesocortical pathway

Mesolimbic pathway

Nigrostriatal pathway

Tuberoinfundibular
pathway

r 2s 1
6. What disorder is thought to arise from reduced norepinephrine activity? Increased norepinephrine
activity?

Vl
u
Vl
7. Serotonin
<(
co • Anxiety: decreased
:?:
w • Depression: decreased
f-
Vl
>- • Produced in Raphe nucleus
Vl
Vl
::J
0
>
a::
w 8. What disease is associated with the degeneration of the basal nucleus of Meynert and less CNS
z
acetylcholine?

9. GABA
• Primary inhibitory neurotransmitter of the CNS
• Produced from glutamate and vitamin 86
• Anxiety: decreased
• Huntington disease: decreased
• Produced in nucleus accumbens

10. Reticular activating system


• Reticu lar formation : mediates alertness
• Locus ceruleus: produces norepinephrine
• Raphe nuclei: produce serotonin

r 26 1
End of S 1o n IZ
11. Which nervous system cell matches each of the following descriptions?
• Look)!.~e fried eggs under hist?logic staining
• Form multinucleated giant cells in the C:NS when infected with z:
rn
• Myelinates multiple C:NS axons AJ
<
• Myelinates one PNS axon 0
c
Ul
• Damaged in Guillain-Barre syndrome
Ul
• Damaged in multiple sclerosis -<
Ul
~
• Macro phages of the C:NS rn
• Cells of the blood-brain barrier
:s:
CD
)>
Ul
n
Ul

In which neurological diseases is acetylcholine altered?

13. What is the main inhibitory neurotransmitter of the CNS? In which diseases are levels
altered?

14. What are the components ()f the blood-brain barrier?

rn 1
CNS EMBRYOLOGY
R: Chapter 28
Brain development (FA15 p448) (FA16 p450) (SU p23)
Neural tube defects (FA15 p449) (FA16 p451) (SU p24) (R p1256)
Forebrain anomalies (FA15 p449) (FA16 p451) (SU p24)
>- Chiari malformations (FA15 p449) (FA16 p451) (SU p24)
l'J
0_ J Dandy-Walker syndrome (FA15 p449) (FA16 p451) (SU p24)
0 Pharyngeal apparatus (FA15 p564) (FA16 p568)
>-
a:
o:J - Pharyngeal cleft derivatives (FA15 p564) (FA16 p568)
:2: - Pharyngeal pouch derivatives (FA15 p566) (FA16 p570)
w
Ul - Pharyngeal arch derivatives (FA15 p565) (FA16 p569) (SU p226-227)
z
u

Up Review
1. Which organelle becomes hypertrophied in hepatocytes with chronic phenobarbital use?
(R p12)

2. Which enzyme mitigates the aging effects of cellulardivision by maintaining chromosomal


length? {R r67J

3. crest derivatives in each of the following adult structures? (FA15 p5S9)

Adrenal

r 2s 1
4. Brain development

~TeleO<'Ph•lon -o Cemb,.lheml>phem•

Forebrain~
(pro~n<eph•lonl Dlen<eph•lon s/ .;, ' ( "
.~·""\,.---Thalamus n
z

-~' ~
(/)
m
:s:
co
Midbrain Mesencephalon • Midi"''" ;;o
(mesencephalon) ·~ -<
0
,--
0
G)
~~Cerebellum -<
. - - - - · Metencephalon - - - ~. .
Hindbrain - - ---1 -. Pons
(rhombencephalon)

5. Neural tube defects

Spina bifida occulta Meningocele Myelomeningocele

r 29 1
6. Pharyngeal arch derivatives

Arch Cartilage Muscle Nerve

>-
t.:J
0
_j

0
>-
a:
OJ
:?:
w
(/) 1
z
u

4&6

f 30 l
d of Sess1 n Quiz
7. What abnormalities are often found with a Chiari malformation?

n
:z
Vl
rn
s:
OJ
;;o
8. What amniotic fluid lab abnormality might point you to a diagnosis of anencephaly? -<
0
,--
0
G>
-<

9. From which pharyngeal pouch is.each of the following structures derived?


• Middle ear and eustachian tubes
• Superior parathyroids
• Inferior parathyroids
• Epithelialliningof the palatine tonsil
• Thymus

r 3' 1
CORTEX AND BRAIN VASCULATURE
Phys: Chapter 57
COA: Chapter 7
Cerebral cortex (FA15 p460) (FA16 p462) (SU p31)
Homunculus (FA15 p463) (FA16 p465)
LLJ Blood supply to the brain (FA15 p462) (FA16 p466) (SU p28)
CY Circle of Willis (FA15 p462) (FA16 p466) (SU p28)
=:J
Arterial blood supply to the cortex (FA15 p462) (FA16 p466) (SU p28)
~
_j Aphasias (FA15 p460) (FA16 p463)
=:J
u Parietal cortex lesions (FA15 p461) (FA16 p464) (SU p31)
(J)

:;;
z
<(
CY
m
0
Warm - Up Rev1ew
z:
<( 1. Which types of proteins are responsible for fostering the progression through the cell
X
LLJ
f-
cycle?
er:
0
u

2. Which cytokine is particularly important in maintaining granulomas? (FA15 p230)


(FA16 p223)

3. Which type of collagen is typically deficient in osteogenesis imperfecta type I?


(FA15 p75-77) (FA16 p61-63)

r 32 1
4. Cerebral cortex

Central sulcus
Premotor cortex Primary sensory area

n
0
:::0
-l
rn
X
)>
Arcuate fasciculus z
0
CJJ
Primary auditory cortex :::0
)>
z
Principal visual cortex
<
)>
(J)
n
c
,-
Broca's area
~
c
:::0
rn

5. Homunculus

Sensory area Motor area

r 33 1
6. Label the following diagram of the Circle of Willis:

LU
0::
::::J
f-
<(
_j

::::J
u
c
<Jl
<(
>
z
<C
0::
D
CD
0
:z:
<(
X
LU
f-
0::
F
0
u

H G

B.

c.
D.

E.

F.

G.

H.

I.

J.
K. r-.......

L.

I 34 l
7. Dysprosody
• Expressive dysprosody
Inability to express emotion or inflection in speech
Nondominant cortical lesion corresponding to Broca's area
• Receptive dysprosody
Inability to comprehend emotion or inflection in speech n
0
Nondominant cortical lesion corresponding to Wernicke's area :AJ
---1
m
><
)>
z
0
8. Parietal cortex lesions OJ
;;u
)>
• Gerstmann syndrome - lesion of the dominant angular gyrus, resulting in: z
_ _ _ _ _ _ _ _ _ _ __ _ _ _ - inability to write <
)>
(/)
_ _ _ _ _ _ _ _ _ _ _ _ _ __ - inability to do mathematical calculations n
c::
Right-left disorientation r-
~
_ __ _ _ _ _ _ _ _ _ _ _ _ _ - inability to distinguish fingers c::
;;u
• m

Lesion of the nondominant angular gyrus


Results in neglect of the body or surroundings contralateral to the lesion

En d of Sess ion Qu iz
A lesion to Which area of the brain is.responsible for each of the following clinical scenarios?
• Hemispatial neglect syndrome
• Poor repetition
• Poor comprehension
• Poor verbal expression
• Personality changes and disinhibition
• Dysarthria
• Agraphia and acalculia

10. Which arteries are chiefly responsible for supplying blood to the brainstem?

11. What artery is damaged in each of the following presentations?


• Broca's or Wernicke's aphasia
• Unilateral lower extremity sensory and/or motor loss
• Unilateral sensory and/or motor loss in the face and arm

r 35 1
VASCULAR EVENTS
R: Chapter 28
H: Chapters 370, 378
Aneurysms (FA15 p465) (FA16 p468) (SU p79)
Subarachnoid hemorrhage (FA15 p466) (FA16 p469) (SU p29-30)
[/)
Epidural hematoma (FA15 p466) (FA16 p469) (SU p29-30)
f-
z Subdural hematoma (FA15 p466) (FA16 p469) (SU p29-30) (H p3379)
UJ
>
UJ
Parenchymal hemorrhage (FA15 p466) (FA16 p469) (H p3294)
a: Intraventricular hemorrhage
<t
_j
Ischemic stroke (FA15 p467) (FA16 p470) (SU p29)
::J Watershed areas (FA15 p462) (FA16 p466)
u
[/)
<t
>

Warm-U p Rev1ew
1. What regulates the progression of the cell cycle from the G1 phase to the S phase?
(FA15 p72) (FA16 p58)

2. Which arachidonic acid product has actions that oppose that of prostacyclin?
(FA15 p444) (FA16 p446)

3. ln.what disorder is there an abnormal breakdown of elastin? (FA15 p77) (FA16 p64)
(SU p296)

4. Intraventricular hemorrhage in the newborn


• Hemorrhage into the ventricular system
• Most common in premature/very low birth weight infants(< 32 weeks, < 1500 g) within the first 72
hours of life
• Originates from the germinal matrix in the subependymal, subventricu lar zone that gives rise to
neurons and glia during development
• All infants born younger than 30 -32 wks gestational age should receive a screening US to detect

r 36 1
End of Session Quiz
5. What is the most common site of a berry aneurysm? What diseases are often
associated with berry aneurysms?

:;;
Vl
n
c
,--
)>
;:o
rn
6. An 85-year-old man with Alzheimer disease falls at horlle and presents three days <
rn
z
later with severe headache and vomiting. What is the most likely diagnosis, and which ---1
Vl
structures were damaged?

7. A healthy 52-year-old man presents to the ER with stroke symptoms that began two
hours ago. What treatment should be considered immediately? What study must be
ordered immediately?

RAPID - FIRE FACTS


"W orst headache of my life"

Lucid interval fo llowing head trauma

Blood y CSF on LP

Most co mmon cause of subdura l hematoma

Most co mm on cause of epidural hemat o ma


HYPOTHALAMUS AND LIMBIC SYSTEM
Phys: Chapter 58
Hypothalamus (FA15 p454) (FA16 p456) (SU p39)
Melatonin (SU p435)
Limbic system (FA15 p456) (FA16 p458) (SU p38)
::?: Amygdala
UJ
1- KWver-Bucy syndrome (FA15 p461) (FA16 p464) (SU p31)
l/)
>- Other limbic system lesions (FA15 p461) (FA16 p464) (SU p31)
(./)

u
CD
::?:
__j

0
Warm -Up Rev1ew
z
<( 1. Which organelle and cytochrome are particularly important in intrinsic apoptosis?
(./)
::::J (FA15 p222) (FA16 p216)
::?:
<(
__j
<(
I
1-
0
o__
>-
I

2. What is the embryologic origin of the tissue just proximal to the pectinate line? What
is the origin of the tissue just distal to the pectinate line?

3. Excessive amounts of which vitamin should be avoided during pregnancy? (FA15 p560)
(FA16 p564) (SU p312)
4. Hypothalamic nuclei
NUCLEUS FUNCTIONS NOTES
ANTERIOR HYPOTHALAMUS

Anterior Thermoregulation (cooling) Damage causes


I
-<
-u
Suprachiasmatic Circadian rhythms "Master clock" 0___,
::r:
)>

Preoptic area Secretes ')>


s:
c
(J)
)>
Supraoptic Secretes Damage causes z
0
r::::
Secretes s:
Paraventricular Secretes CRH co
n
Secretes TRH (J)
-<
TUBERAL HYPOTHALAMUS (J)
___,
rn
Secretes GHRH s:
Secretes dopamine
Arcuate
GnRH secretion
Regulates appetite
Regulates hunger Damage leads to
Lateral
Inhibited by

Regulates satiety Damage leads to


Ventromedial
Stimulated by and savage behavior

Stimulation leads to obesity


Dorsomedial Regulates hunger
and savage behavior
POSTERIOR HYPOTHALAMUS

Posterior Thermoregulation (warming) Damage causes

Mammillary Memory Damage causes

r 39 1
En d of Sessio n Qu1 z
5. Which hypothalamic nudeus fits each of the following descriptions?
• . Savage behavior and obesity resu lt from stimulation
• Savage behavior and obesity result from destruction
:?: • The "master clock" for setting circadian rhythms
w
f---
Vl • Releases hormones affecting the anterior pituitary
>-
Vl
u
ro • Responsible for sweating and cutaneous vasodi lation in hot temperatures
:?:
_j
• Responsible for shiveringand decreased cutaneous blood flow in the cold
0 • Produces antidiuretic hormone (ADH) to regulate water balance
z:
<( • Regulates the release of gonadotropic hormones (i.e., LH and FSH )
lf)
::J
:?:
<(
_j
• Destruction results in inability to stay warm
<(
:c • Receives inputfrom the retina
f---
0 • Stimulation leads to eating and destruction leads to anorexia
o_
>- • Destruction results in diabetes insipidus
:c

6. Deficiency of what nutrient causes damage to the mammillary bodies?

7. A patient is evaluated for behavior and personality changes. She used to be


argumentative and aggressive but has become fairly quiet and docile. She has also
become socially disinhibited and has been making inappropriate sexual advances
toward friends, relatives, and strangers. DuringJhe interview, you note the
patient's tendency to chew on pens, her fingers, and her necklace. Damage to what
neurological structure might account for these behaviors?

f 40 l
CRANIAL NERVES PART 1
COA: Chapter 7
Identifying cranial nerves (FA15 p474-475) (FA16 p477-479) (SU p28)
Cranial nerve nuclei (FA15 p474) (FA16 p478)
Olfactory nerve (FA15 p475) (FA16 p479) (SU p41)
Optic nerve (FA15 p475) (FA16 p479) (SU p41) n
;;o
Visual field defects (FA15 p486) (FA16 p489) (SU p36) )>
Oculomotor nerve (FA15 p475) (FA16 p479) (SU p41) z
)>
Trochlear nerve (FA15 p475) (FA16 p479) (SU p41) ,--
Abducens nerve (FA15 p475) (FA16 p479) (SU p41) z
m
Extraocular muscles (FA15 p484) (FA16 p488) (SU p37) ;;o
Pupillary control (FA15 p483) (FA16 p487) (SU p35) <
m
(j)

-u
)>
;;o

Warm -Up Rev1ew ---i

2. What molecule targets proteins in the Golgi apparatus for lysosomes? (FA15 p73) (FA16 p59)

3. Which embryonic structure is derived from the hypoblast and serves as a secondary
energy source?

r4 l
4. Label the cranial nerves as they come off the brainstem:

---------A. ------------------------
-------8. ------------------------
,- - - -- - (, ------------------------
f- -------0. ------------------- - - - - -
er:
<(
o_
V)
E. ------------------------
w
>
cr:
w
F.
z
_j
<(
z
<(
cr:
u

~~---G. ________________________
H. ________________________
I.
J.

K. ------------------------
L.

Dorsal

------0. ------------------------
~--P.

- - - - - - Q. - - - - - - - - - - - - - - - - - - - - - - - -

R. ------------------------
5. ------------------------

T.
u. ------------------------
~------~ ------------------------
w. ________________________
X. ------------------------
¥.

z. ------------------------

Ventral

r 42 1
End of essJ on .Qulz
5. Which cranial nerve nuclei are locatedin each ofthe following locations?
• Medulla
• Pons
()
• Midbrain :Al
)>
z
)>
,-
z
m
6. Where is the pathology located in each of the following scenarios? :Al
<
• Right <:~nopsia m
U)

• Bilateral hemianopsia ""D


)>
:Al
• Right homonymous hemianopsia ---1

7. WhenUght isshone in a patient's right eye, his right pupil fails to constrictbuthis left
pupil constricts normally. When light is shone in the left eye, the left pupil constricts
butthedghtpupildoes not. Where is the likely pathology?

r '13 1
CRANIAL NERVES PART 2
COA: Chapter 7
Trigeminal nerve (FA15 p475) (FA16 p479) (SU p41)
Cavernous sinus (FA15 p477) (FA16 p481) (SU p42)
Facial nerve (FA15 p475) (FA16 p479) (SU p41)
N Bell's palsy (FA15 p478) (FA16 p480) (SU p34)
f- Vestibulocochlear nerve (FA15 p475) (FA16 p479) (SU p41)
~
<( Glossopharyngeal nerve (FA15 p475) (FA16 p479) (SU p41)
[L

Vl Vagus nerve (FA15 p475) (FA16 p479) (SU p41)


w
> Accessory nerve (FA15 p475) (FA16 p479) (SU p41)
~ Hypoglossal nerve (FA15 p475) (FA16 p479) (SU p41)
w
z
_.J
<(
z
<(
~
u
Warm -U p Rev 1ew
1. During which weeks of fetal development does organogenesis take place? (FA15 p55B)
(FA16p562)

2. Which group of genes is responsible for skeletal development? (FA15 p558) (FA16p562)

3. What is required for a molecule to enter into the nucleus through a nuclear pore?

r 441
4. What conditions can result in facial nerve palsy?
(Hint: 1ovely Bella !:!ad 6n STD)
• Lyme disease


• AIDS
• Sarcoidosis
• Tumors
z
• Diabetes rn
;:o
<
rn
(/)

-o
:t>
;:o
5. How do the symptoms of a facial nerve (or facial nerve nucleus) lesion differ from the symptoms of a -I
N
facial motor cortex lesion?
• Facial nerve/nucleus lesion - Paralysis of ipsilateral side of _ _ _ _ _ _ _ _ _ _ __ face
• Facial motor cortex lesion - Paralysis of contralateral side of face

6. Why does a motor cortex stroke not affect the upper face?
• The facial motor nucleus receives motor fibers for th e lower face only from the contralatera l motor
cortex, but it receives motor fibers for the upper face from both left and right motor cortices.
• In a left motor cortex stroke, only the right lower face will be paralyzed because the right upper face
still receives innervation from the right motor cortex.

7. If the right vagus nerve or nucleus is damaged, to which side will the uvula deviate?

8. A patient's uvula deviates to the right when she says, "Ah." What neurological structures might be
damaged?
Rightward deviation of the uvula means that th e muscles of the right palate are raising the palate, and
the muscles of the left p3l3te 3re not. This could be c3used by d3m3ge to:
• Left _ _ _ _ _ _ _ __ _ _ _ __ _ _ _ __
• Left - receives input from:
Right corticobulbar tract
Right motor cortex

r 45 1
9. If the right hypoglossal nerve or nucleus is damaged, to which side will the tongue deviate when
sticking out?

N
f---
0:::
<(
0...
Lf)
L1.J
>
0:::
En d. of Sess1on Qu1 z
L1.J
:z: 10. What two nerves are tested with the gag reflex?
_j
<(
:z:
<(
0:::
u

11. A 19-year-old man presents with a furuncle on his philtrum, and the cavernous sinus
becomes infected. What neurological deficits might you see in this patient?

12. How can a stroke of the facial motor cortex be disti11guished from aeU's palsy?

r 46 1
BRAINSTEM AID SPINAL CORD
COA: Chapter 7
H: Chapter 376
Spinal tracts (FA15 p470) (FA16 p473) (SU p31-32)
Brainstem anatomy
- Midbrain (SU p32) OJ
;;o
- Pons (SU p32) )>
- Medulla (SU p32) z
l/)
Vagal nuclei (FA15 p476) (FA16 p479) ---1
Horner syndrome (FA15 p483) (FA16 p487) (SU p35) m
Spinal cord anatomy (FA15 p469-470) (FA16 p472-473) (SU p31-33)
:s:
)>
Romberg test z
0
l/)
-u
z
Warm -Up Review · )>

'n
0
What molecules provide the structural frameWork fot DNA and the nuclear ;;o
0
envelope?

2. What substances are utilized by natural killer cells to induce apoptosis in other cells?
(FA15 p201) (FA16 p193)

3. What drugs interfere with microtubule functioning? (FA15 p74) (FA16 p60)

r 47l
4. Dorsal column/medial lemniscus

0
a:: "-" \
,,_,, '
~
~V e n t
nucl~us
r a l ·
posterolateral

0
u
·- -,~ , ,~
_.J

t ->
<(
z
a...
V)
Medial lemniscus rs-s; ·

•=~~ ~~:t:!~~:n~he
0
z
<(

~
ll.J
f-
V)
medulla
z C-8 ' . I
<( leg ~
a:: S-1 ,~
co

5. Spinothalamic tract

TS-30 ~ ......_,~~----J-"'""-. Thalamus

TS-13 l
\

Arm ~~- Decussates in the anterior


white commissure

6. Lateral corticospinal tract

' ~ '

TS-30 , ...,_ " J

~~
'~" ~-~
. rs- 8 "'. ••···• ·•· ·. / Corticospinal tract

TS-4~1 ~ Medulla ry pyramids

~ Decussates in the caudal medulla


1
Ts-

To muscle+-·-· C-8 '>~_.I

r 48 1
7. Rostral midbrain
~ Oculomotor (CN Ill) nucleus

OJ
;a
)>
z
Vl
-j
m
:s:
Substantia nigra )>
z
0
Vl
"""0
z
)>

'
n
0
;a
0

8. Pons

9. Medulla
Hypoglossal (CN XII} nucleus

Vestibular (CN VIII) nuclei


Inferior cerebellar peduncle
Spinal trigeminal tract •
and nucleus

Inferior olivary nucleus

Medullary pyramid

r 49 1
10. Label the cross-sectional view of the spinal cord:

0
0:::
0
u
_j
<(
z
o_
c.n
0
z
<(

~
L1.J
f-
lf)
z
<(
0:::
m

A. --------------------- -
B.

C. ------------- - - ---------------------------- -- --------

D. -----------------------------------------------------
E.

F.

G. ----- - - -------- -- -------- -- -- - - -- -- -------- ----------

H. --------- -

I.

J.
K.

r so 1
End Session Quiz
11. What artery supplies the medullary pyramids and the medial lemniscus in the medulla?

OJ
Al
)>
z
(J)
---j
rn
12. What arterysupplies the inferior cerebellar peduncle, nucleus ambiguus and lateral 5:
)>
spinothalamic tract in the medulla? z
0
(J)
-u
z
)>
r-
n
0
Al
0
13. Where does each of the following spinal tracts decussate?
• Lateral corticospinaltract
• Dorsal column-mediallemniscal pathway
• Spinothalamic tract

14. Which spinal tracts convey the following types of information?


• Light touch, proprioception, and vibration sensation
• Voluntary motor commands from the motor cortex to the body
• Pain and temperature sensation
• Important for postural adjustments and head movements
• Eroprioceptive information cerebellum

15. What is the triad ofHorner syndrome?

r s1 1
BRAINSTEM LESIONS
H: Chapter 370

The rule offours


Brainstem strokes (FA15 p464) (FA16 p467) (H p3288)
Internuclear ophthalmoplegia (FA15 p486) (FA16 p490) (SU p35) (H p239)
(./)
z Locked-in syndrome (FA15 p456) (FA16 p492) (H p2247)
0 Weber syndrome (H p3287)
(./)
UJ
_J

2:
UJ
f-
l/)
z Warm -Up Review
<(
~
ro 1. What effect might each of the following teratogens have on a developing fetus? (FA15 p560)
(FA16 p564) (SU p397)
• ACE inhibitor
• Aminoglycoside
• Diethylstilbestrol
• Folate antagonist
• Lithium
• Tetracycline
• Va lproic acid

I .'i? l
2. What is the hallmark sign of a brainstem lesion?
Alternating syndromes: long tract symptoms on one side (e.g., hemiparalysis) and cranial nerve
symptoms on the other

3. The Rule of Fours OJ


;;o
)>
1. There are 4 medial/ midline structures, beginning with M :z
lfl
• Motor pathway (corticospinal tract) -1
m
:;;::
Deficit:------------------------------ r-
m
• Medial lemniscus lfl
0
Deficit:------ - -- -- - - -- - -- -- - -- - -- - -- - - :z
lfl
• Medial longitudinal fasciculus
Deficit: _ _ _ _ _ _ _ _ _ __ _ __ _ _ __ __ _ __ _ _ __ _ __ _
• Motor crania l nerve nuclei
Deficit: _ _ _ __ _ __ _ __ _ __ _ _ __ _ _ _ _ _ _ _ _ _ __
2. There are 4 "side" (lateral) structures, beginning with S
• ~pinocerebe ll ar tract
Deficit:-------------------------------
• ~pinothalamic
tract
Deficit:---- - - - - - - - - - - - - - -- - - - - - - - - - - - -
• ~ensory
nucleus of CN V
Deficit:-------------------------------
• ~ympathetic
pathway
Deficit: - - - -- - - -- - - - - - - - - - - - - -- -- -- - --
3. 4 cranial nerves originate in the pons, 4 originate below the pons, and 4 originate above the pons
(2 in the midbrain)
• _ _ _ _ _ _ _ __ _ _ _ originate in the midbrain
• _ _ _ _ _ _ _ _ _ _ _ _ originate in the pons
• _ _ _ _ __ _ __ _ _ _ originate in the medulla
4. The 4 motor cranial nerve nuclei in the midline are those that divide equally into 12
(except for CN I and CN II)
• CN _ _ _ _ __
• CN _ _ _ _ __
• CN _ _ _ _ __
• CN _ _ __ _ _

[53]
4. What are the symptoms of lateral medullary syndrome (Wallenberg syndrome)?
• Loss of pain and temperature over contralateral body (spinothalamic tract damage)
• Loss of pain and temperature over ipsilateral face (spinal trigeminal nucleus damage)
• Hoarseness, difficulty swallowing, loss of gag reflex (nucleus ambiguus: CN IX and X damage)
• Ipsilateral Horner syndrome (descending sympathetic tract)
Vl
z • Vertigo, nystagmus, nausea/vomiting (vestibular nuclei damage)
0
Vl
• Ipsilateral cerebellar deficits (e.g., ataxia, past pointing) (inferior cerebellar peduncle damage)
w
_J

5. Which vessel is most likely occluded in a stroke of the lateral medulla?

6. What are the symptoms of medial medullary syndrome?


• Contralateral spastic hemiparesis (pyramid/corticospinal tract damage)
• Contralateral tactile and kinesthetic defects (med ial lemniscus damage)
• Tongue deviates toward the side of the lesion (hypoglossa l nucleus/nerve damage)
• Note that pain and temperature sensation are generally preserved

7. What are the symptoms of lateral inferior pontine syndrome?


• Ipsilateral facial nerve paralysis (facial nucleus and nerve fiber damage)
• Ipsilateral limb and gait ataxia (damage to middle cerebellar peduncle)
• Ipsilateral loss of pain and temperature sensation from the face (spinal trigeminal nucleus and nerve
fiber damage)
• Contralateral loss of pain and temperature sensation (damage to spinothalamic tract)
• Ipsilateral Horner syndrome (damage to descending sympathetic tract)
• No contralateral body paralysis or loss of light touch/vibratory/proprioceptive sensation

8. What are the symptoms of medial pontine syndrome?


• Contralateral spastic hemiparesis (corticospinal tract damage)
• Contralateral loss of light touch/vibratory/proprioceptive sensation (medial lemniscus damage)
• Ipsilateral internuclear ophthalmoplegia (damage to medial longitudinal fasciculus)
• Gaze away from side of lesion (damage to pontine gaze center: PPRF)
• Ipsilateral paralysis of lateral rectus muscle (damage to abducens nucleus)
• Note that pain and temperature sensation are preserved

r 541
9. Internuclear ophthalmoplegia
~
Patient can direct both eyes forward

A OJ
;;o
)>

Patient can direct both eyes toward the right z


Ul
-i
rn
:s:
r-
B rn
Ul
0
z
Ul
Patient cannot adduct right eye {m. rectus)

c ofCN

10. What causes Weber syndrome? What are the symptoms?


Anterior midbrain infarction resulting from occlusion of the paramedian branches of the posterior
cerebral artery:
• Cerebral peduncle lesion
Dysphagia, dysphonia, dysarthria (corticobulbar tract damage)
Contralateral spastic hemiparesis (corticobulbar tract damage)
• Oculomotor nerve (CN Ill) palsy ~ ipsilateral ptosis, pupillary dilation , and lateral strabismus (eye
looks down and out)

[55 ]
En d. of Session Qu iz
11. A patient comes to the ER with double vision and left-sided weakness. On physical
examination, she cannot abduct her right eye. In addition to motor weakness of the
left arm and leg, she also has loss of fme touch, proprioception and vibration sense in
l/)
z the left arm and leg. Where is the lesion, and which vessel is most likely involved?
0
l/)
LLJ
_J

~
UJ
f-
l/)
z
<(

g::; 12. A patient presents with vertigo, nystagmus and slurred speech. Physical examination
reveals right-sided ptosis and miosis, a decreased gag reflex on the right, deviation of
the uvula to the left, diminished pain/temperature sensation on the right side of his
face, and diminished pain/temperature sensation on the left side of his body. Where
is the lesion, and which vessel is most likely involved?

13. A patient presents with double vision, slurred speech, hoarse voice and acute right-
sided weakness. Physical examination shows weakness of the right arm and leg, and
the left eye is depressed and abducted. Where is the lesion, and which vessel is most
likely involved? ~,

r s6 1
CSFAIDICP
R: Chapter 28
CSF circulation (FA15 p468) (FA16 p471) (SU p27)
Dural venous sinuses (FA15 p467) (FA16 p470) (SU p28)
Hydrocephalus (FA15 p468) (FA16 p471) (SU p24, 27)
Idiopathic intracranial hypertension (FA15 p468) (FA16 p471) n
Vl
-n
)>
z
0

War -Up Rev1ew n


--o

What is the function of the lysosome? (FA15 p73) (FA16 p59)

2. What are the characteristicfeatures of a cell undergoing apoptosis? (FA15p222)


(FA16 p216)

3.. How.do Bax and Bcl-2 affect apoptosis?

4. Hydrocephalus
Cause ICP Signs/Symptoms

Noncommunicating
hydrocephalus

Communicating
hydrocephalus

Normal pressure
hydrocepha lus

Hydrocephalus ex
vacuo

f 571
5. What are the characteristic features of idiopathic intracranial hypertension (AKA pseudotumor
cerebri)?

• Headaches - daily, pulsatile, worse at night or early AM, possible retroocular pain worsened by eye
movement, possible N/V
CL •
u
0
• Most worrisome sequela is vision loss
z • CT scan: no ventricular dilation, no tumor, no mass
<t:
u..
Vl • Intracranial pressure (ICP) is _ _ _ _ _ _ _ _ _ _ (> 200 mm Hp in non-obese patients, >
u
250 mm Hp in obese patients)

6. How is idiopathic intracranial hypertension managed?


• Discontinue any inciting agents (e.g., excess vitamin A, isotretinoin, tetracyclines, danazol)
• _ _ _ _ _ _ _ _ _ _ __ _ _ in obese patients
• _ _ _ _ _ _ _ _ __ _ _ __ - first-line pharmacotherapy
• Invasive treatment options
Serial lumbar punctures
Optic nerve sheath decompression
Lumboperitoneal shunting (CSF shunt)

7. Lumbar puncture

L4

r .'18 1
End Sess ion Ou1z
8 . •. Where is CSF generated? Where is CSF reabsorbed?

(}
Vl
--n
)>
z
0

9. What is the difference between communicating and noncommunicating (}


-u
hydrocephalus?

10. What clinical features characterize normal pressure hydrocephalus?

r s9 1
HEADACHE
H: Chapter 14

Headache (FA15 p490) (FA16 p494) (SU p51)


- Tension headache (H p120)
LLJ
- Migraine headache (H p114)
:r:: - Cluster headache (H p122)
u Headache treatments (FA15 p490) (FA16 p494)
<{
0 - Sumatriptan (FA15 p502) (FA16 p505) (SU p455) (H p119)
<{
LLJ Other headaches (H p124-128)
:r::

Warm -Up Rev1ew


1. Which protein is involved in transporting an endocytosed vesicle from the plasma
membrane to the endosome? (FA15 p73) (FA16 p59)

2. Which developmental structure matches each of the following descriptions?


• .FetalplacentaLstructure that secretes hCG
• Maternal component of placenta

3. What is the main inhibitory neurotransmitter of the CNS? In which diseases are levels
altered? (FA15p453) (FA16 p455)

r 60 l
4. Patient Profile: Eddie's Exploding Eye
Eddie is a 27-year-old man who comes to the physician because of two weeks of recurrent headaches.
The headache begins around 10:00 AM and lasts about 90 minutes before resolving spontaneously.
The pain is always located behind the right eye and feels like a red hot poker is being jabbed into his
eye. The headache does not throb. During the headache, the right eye is red and watery, and the right
side of his nose runs profusely. He denies any nausea or vomiti ng. The headache is not worse ned by
:r:
bright lights, loud noises, or physica l activity. What therapy will treat this patient's headache most rn
l>
rapidly? 0
l>
n
:r:
m

5. Tension headache
• Constant, non-throbbing pain (lasts 4-6 hours, up to 7 days)
• Occurs in the _ _ _ _ _ __ _ _ __ _ _ _ _ regions (most often bilateral) or as a band
around the head
• No associated symptoms such as:

6. Migraine headache
• At least 5 attacks
• Headache lasting 4 -72 hours (2-48 hours in ch ildren)
• At least 2 of the fo llowing:
_ _ __ __ __ __ _ _ __ __ _ __ location
_ _ _ _ _ __ _ _ _ __ _ _ __ _ _ _ quality
Moderate to severe intensity (inhibits or prohibits daily activities)
- Aggravated by routi ne physical activity
• At least 1 of the fol lowing:
Nausea and/or vomiting

7. Cluster headache
• Strictly _ _ __ __ __ __ __ _ __ _ _ __
• Severe piercing/boring pain in the _ _ _ _ _ _ _ _ _ __ _ _ __ region
• Does not throb like a migraine. No aura.
• Duration of 15 minutes to 3 hours, occurs _ _ __ _ _ _ _ ___ (often at t he same time),
and continues for an interval of 4-8 weeks
• May be associated w ith:
Partial Horner syndrome(_ __ _ _ _ __ _ _ __ _ -'
Ipsilatera l , tearing, rh inorrhea, or nasal congestion

r 6:
8. What is the most likely cause of headache based on each of the following descriptions?

.
Made worse by foods containing tyramine

LLJ
J: Obese female with papilledema
u
<(
0
<(
LLJ
I Jaw muscle pain when chewing

Periorbital pain with ptosis and miosis

Photophobia and/or phonophobia

Bilateral frontal/occipital pressure

Patient using topical retinoic acid for acne

Lacrimation and/or rhinorrhea

Elevated ESR

"Worst headache of my life"

Headache + extraocular muscle palsies

Scintillating scotomata prior to headache

Headache occurring either before or after


orgasm

Responsive to 100% oxygen supplementation

Trauma to the head ~ headache begins days


after the event, persists for over a week and
does not go away

Severe headache + fever or neck stiffness

Constant headache for weeks + neuro deficits


and/or seizures
End of S s1on Quiz
9. What features allow you to distinguish a migraine headache from a cluster headache
or a tension headache?
:r:
rn
)>
CJ
)>
n
:r:
rn

10~ What is the mechanism of action of sumatriptan?

What are the contraindicaticms to sumatriptan use?

r 63 1
BRAIN TUMORS
R: Chapter 28
H: Chapter 379
Adult brain tumors (FA15 p492) (FA16 p496) (SU p50) (R p1306) (H p3382)
Pediatric brain tumors (FA15 p493) (FA16 p497) (SU p50)
Herniation syndromes (FA15 p493) (FA16 p498) (H p2248)

-UpRev1ew
How does having high cholesterol content in the plasma membrane affect the
function of the plasma membrane?

2. What are the stages that an embryo goes through between conceptic:m and the
development of an inner.cell mass?

What are the differences between oligodendroglia and Schwann .cells? (FA15 p451-452)
(FA16 p453-454) 7~

r 641
nd ..of Sess1on Quiz
What are the three most common primary brain tumors in adults? What are the three
most common in children?

CD
;;o
)>
z:
--i
c
s:
0
;;o
5. Which primary brain tumor fits each of the following descriptions? Ul

• Pseudopalisading necrosis
• · · Polycythemia
• Neurofibromatosis type 2
• Associated with von t-Jippei-Uildau syndrome
• Foamy cells, high vascularity
• Hyperprolai:tinemia ~ galactorrhea, amenorrhea, anovulation
• Psammoma.bodies
• Fried eggappearance
• Perivascular pseudorosettes
~ ~

. • Bitemporal hemianopia
• Worst prognosis of any primary brain tumor
• Child withhydrocephalus
• Homer-Wright pseudorosettes

r 6Sl
DELIRIUM AND DEMENTIA
R: Chapter 28
H: Chapter 371
Dementia (FA15 p487, 508) (FA16 p491, 513) (SU p48) (H p3300)
- Alzheimer disease (FA15 p487) (FA16 p491) (SU p48) (R p1287) (Phys p727) (H p3305)
<t - Alzheimer drugs (FA15 p502) (FA16 p505) (GG p619)
f-
z Vascular dementia
w
2: Dementia with Lewy bodies (FA15 p487) (FA16 p491) (SU p48) (R p1295) (H p3312)
w - Frontotemporal dementia (FA15 p487) (FA16 p491) (SU p48) (R p1292) (H p3310)
0
0
- Creutzfeldt-Jakob disease (FA15 p487) (FA16 p491) (H p3312, 3441)
z - Other causes of dementia
<t - Dementia work-up
2: Delirium (FA15 p508) (FA16 p512) (SU p58)
::::J
ex: Delirium vs. dementia (FA15 p508) (FA16 p512-513)
-'
w
0

Warm -Up Rev1 ew


1. Which cranial nerve relays the following information? (FA15 p475) (FA16 p479)
• Hypoxia measured by the carotid body
• Motor information for swallowing
• Blood pressure from the aortic arch
• Salivation from the sublingual glands
• Salivation from the parotid gland
• Blood pressure from the carotid body

2. Which type of collagen is abnormal in Alport syndrome? (FA15 p75) (FA16 p61)

3. Which type of collagen is abnormal in Ehlers-Dan los syndrome? (FA15 p75) (FA16 p61)

4. Identify the neuronal pigment/inclusion that matches each of the following statements:
• Intranuclear inclusions seen in herpes simplex encephalitis
• Cytoplasmic inclusions pathognomonic of rabies
• Neuronal inclusions characteristic of Parkinson disease
• Cytoplasmic inclusion bodies associated with aging
• Dark cytoplasmic pigment in neurons of the substantia nigra and locus coeruleus, not seen in
patients with Parkinson disease
• Eosinophilic, rod-like inclusions in hippocampus of Alzheimer patients
• Diagnostic of Alzheimer disease
• Filamentous inclusions that stain with silver, do not survive neuronal death
• Filamentous inclusions that stain with PAS and ubiquitin

r 66 1
5. Delirium vs. Dementia
Delirium Dementia

Onset

0
rn
Daily course ';;o
c
$
)>
Level of :z
0
consciousness 0
rn
$
rn
:z
--1
Thought production )>

Psychotic features

Prognosis

[ 671
En d of SessJon Qu1z
6. What are the usual components of a dementia work-up?

<t
f-
z
L1.J
2:
L1.J
0
0
z
<t 7. Why is Alzheimer disease more common in patients with Down syndrome?
2:
::::J
cr::
_j
L1.J
0

8. What is the mechanism of action of the drugs used in the treatment of Alzheimer
disease?

9. A 72-year-'old woman is brought into the clinic by her family because of strange
behaviors over the past week. She has been very agitated, takes many naps during
the day, occasionally urinates on herself, and has had a poor appetite. She will not
focus her attention on the questions you ask her. What is the most likely diagnosis?

RAP! O- Ft RE FACTS
Most common cause of dementia

Second most common cause of dementia

Extracellular amyloid deposits in the gray matter

Intracellular deposits of hyperphosphorylated tau


protein

Intracellular spherical aggregates of tau protein


seen on silver stain

r 681
SEIZURES
H: Chapter 369
Phys: Chapter 59
GG: Chapter 21
Seizures (FA15 p489) (FA16 p494) (SU p43-44) (Phys p725) (H p3251)
Tuberous sclerosis (FA15 p491) (FA16 p495) (R p1316) (J)
m
Sturge-Weber syndrome (FA15 p491) (FA16 p495) N
Anti-seizure medications (FA15 p496) (FA16 pSOO) (SU p44) (H p3261-3267) (GG p590-606) c
AJ
m
V>

Warm -Up Review


1. What are the three different G proteins and their downstream effects? Which
re(;eptors use these G proteins?(FA15 p24B) (FA16 p242! (5Up173)

2. Tuberous sclerosis
• Autosom;:JI domin;:Jnt; 1/ 5,000 - 1/10,000 live births
• Incomplete penetrance and variab le expressivity
• Most common mutations are in the TSC1 or TSC2 genes
TSCl gene ~ hamartin protein
TSC2 gene ~ tuberin protein
• Classic triad: seizures, intellectual disability, angiofibromas
• Additional findings: hypomelanotic macules (ash -l eaf spots), retinal hamartomas,
cortical tubers (glioneurona l hamartomas)
• Tumor associations: renal angiomyolipoma, cardiac rhabdomyoma, astrocytoma (subependymal giant
cell astrocytoma)

r 69 1
3. Trigeminal neuralgia (tic douloureux)
• "Lightning-like" pain, "electric shocks," along a division of the trigeminal (usually maxillary) triggered
by light touch (wind, bedsheets)
• Treatment: carbamazepine or other anticonvulsant (phenytoin, gabapentin, topiramate)

~
0:::
4. Drug of choice for absence seizures:
=>
N
w
(f)

5. Used to treat status epilepticus:

6. Used to treat eclampsia:

7. Additional SE of phenytoin:

8. Which antiepileptics are teratogens?

9. What drugs cause Stevens-Johnson syndrome?

10. What drugs are known to cause agranulocytosis?

11. Hepatotoxic antiepileptics:

r 70 1
12. What drugs induce the P450 system?

(J)
m
N
c
:;u
m
(J)

End of Session Qu1z


13. In regard to seizures, what the terms partial, simple, complex, and generalized
mean?

14. What are the lllostcommon causes of seizures in children?

15. A10-year-old child "spaces out" in class (stops talking midsentence <md tht:m
continues as if nc:>thing had happened). During the spells, there is slight quivering of
lips. What is the diagnosis?

16. What drugs are known for causing Stevens-Johnson syndrome?

17. What are the toxic side effects of phenytoin?

RAPID-FIRE FACTS
I Treatment for absence seizures

f71 l
SPINAL CORD LESIONS
COA: Chapter 4
R: Chapter 28
H: Chapters 374, 377, 380
Motor neuron signs (FA15 p470) (FA16 p473)
Vl Spinal cord lesions (FA15 p471-472) (FA16 p474-475) (SU p39)
z
0 Polio
Vl Multiple sclerosis
UJ
_J ALS (Lou Gehrig's disease)
0 Tabes dorsalis
0:::
0 Lesion of the anterior spinal artery
u - Syringomyelia
_J
<( - Brown-Sequard syndrome
z
0....
Vl

Warm- Up Rev1 ew
1. Which nucleus of the hypothalamus fits each of the following descriptions? (FA15 p454)
(FA16p456)
• Considered the "master clock" for most of our circadian rhythms
• Regulates the parasympathetic NS
• Regulates the sympathetic NS
• Produces antidiuretic hormone (ADH) to regu late water balance
• Mediates oxytocin production

2. A 43-year-old man presents with symptoms of dizziness and tinnitus. CT shows enlarged
internal acoustic meatus. What is the most likely diagnosis?

rn 1
EndofSess ion Qu 1z
3. What are the findings of Brown-Sequard syndrome?

(J)
--o
z
)>
,-
n
0
;;o
0
,-
m
Vl
0
z
4. What clinical presentation would lead you to suspect amyotrophic lateral sclerosis as Vl

a diagnosis?

5. ..What are some of the more classic presenting scenarios for multiple sclerosis?

6. What are the classic presenting symptoms of syringomyelia?

RAP/ O-F! RE FACTS


Conjugate lateral gaze palsy, with nystagmus and
diplopia during latera l gaze
Degeneration of the dorsal co lumns

,~ Demyelinating disease in a young woman

Mixed upper and lower motor neuron disease


·MOVEMENT
Phys: Chapters 54, 55, 56
Thalamus (FA15 p456) (FA16 p458) (SU p40)
Cerebellum (FA15 p457) (FA16 p459) (SU p34) (Phys p681)
Tremor (FA15 p459) (FA16 p461) (H p3327)
1-
z
UJ
2:
Warm-Up Review
UJ
>
0
2:
1. What is the initial treatmentfor ventricular fibrillation? (FA15 p284) (FA16 p27B)

2. Which organism is associated with each of the following dues?


• Rabbit hunter
• Pet prairie dog
• Lymphadenopathy + a new kitten
• Dog bite

3. Which bacterial structure has each of the following functions?


• Provides rigid support to the bacterialcell and protects against osmotic pressure differences
• Space between the inner and outer cellular membranes in gram-negative bacteria
• Mediates adherence of bacteria to the surface of a cell
• Protects against phagocytosis

r 741
4. What portions of the thalamus relay each of the following types of information?
• Sensory signals from body (via medial lemniscus and spinothalamic tract) --7 cortex
• Trigeminothalamic and taste signals --7 somatosensory cortex
• Retina (visual information) --7 occipital lobe
• Inferior colliculus (auditory information) --7 primary auditory cortex
• Mammillothalamic tract --7 cingulate gyrus (part of Papez circuit) :s
0
• Communications with prefrontal cortex; memory loss results if destroyed <
rn

• Cerebellum (dentate nucleus) and basal ganglia --7 motor cortex


:s
rn
z
-1
• Basal ganglia --7 prefrontal, premotor, and orbital cortices

5. Describe the general flow of information through the cerebellum.


Inputs (from cerebral cortex and spinal cord) --7 cerebellar cortex --7 deep nuclei of cerebellum --7
output targets (e.g., ventral lateral nucleus)

6. list the deep nuclei of the cerebellum from medial to lateral.

7. What structure provides the major output pathway from the cerebellum?
Superior cerebellar peduncle --7 contralateral ventral lateral nucleus (VL) of thalamus

8. On which side of the body (contralateral or ipsilateral) would motor control be affected by a lesion of
the cerebellum?
• Motor control ipsilateral to the side of the lesion would be affected
• Output from the cerebellum --7 contralateral thalamus --7 cortex --7 corticospinal tract --7
body contralateral to cortex

r 751
9. Organization of the cerebellum

f---
z
w
Lateral part of hemisphere - T---
::2:
w {Cerebrocerebellum)
>
0
::2:

Intermediate part lr---7-------


of hemisphere

Flocculonodular lobe
(Vestibulocerebellum)

10. Cerebellar function and lesions


Function Effect of Lesion
• Dysequilibrium

Vestibulocerebellum

• Truncal ataxia
• Nystagmus

• Postural control of trunk •


Spinocerebellum
• Coordination of legs (upper vermis) • Broad-based gait

Cerebrocerebellum Control of extremities

176 l
11. What are the differences between resting tremor, intention tremor, and essential tremor?
• Resting tremor
- Tremor occurs at rest but disappears with vo luntary movements
- Associated with Parkinson disease
• Intention tremor
- Tremor appears only with vo luntary movements :s:
0
- Associated with damage to cerebrocerebellum <
m
• Essential tremor :s:
rn
z
- Tremor occurs both with movement and at rest --1
- Associated with family history of tremor

12. What are the features of essential (familial) tremor?


• Rapid fine tremor of head, hands, arms, and/or voice
• Occurs both with movement and at rest
• 50% of patients have a fami ly history of tremor
• Treated with ~-blocker (propranolol), primidone (an anticonvulsant) or clonazepam. Some patients
self-medicate with alcohol

End ofSessron Ou1z


13. What kind of information is relayed by each of the following thalamic nuclei?
• Ventral lateral (VL) nucleus
• Ventral posterolateral (VPL) nucleus
• M.edial geniculate nucleus

14..A patient presents with involuntary flailing movements of one arm. Where is the lesion?

15. What are the differences between essential tremor, resting tremor, and intention tremor?

r 771
BASAL GAIGLIA
Phys: Chapter 56
R: Chapter 28
H: Chapter 372
Basal ganglia (FA15 p458) (FA16 p460) (Phys p690)
Parkinson disease (FA15 p459) (FA16 p461) (SU p45) (R p1294) (Phys p693) (H p3317)
Levodopa-carbidopa (FA15 pS00-501) (FA16 p504-505) (SU p46) (H p3321) (GG p614)
Other Parkinson disease drugs (FA15 pS00-501) (FA16 p504) (SU p46) (H p3321) (GG p614)
Movement disorders (FA15 p459) (FA16 p461)
Huntington disease (FA15 p459) (FA16 p462) (SU p47) (R p1297) (H p3330)
Huntington disease treatment (FA15 p502) (FA16 p505)

Warm -Up Rev1ew


1. What is the initial treatment for ventricular tachycardia when there is no pulse?

2. What is the function of catalase? (FA15 p122) (FA16 p112)

3. Which portions of the hypothalamus are inhibited by leptin? Which are stimulated?
(FA15 p454) (FA16 p456)

4. Basal ganglia

~ yreb·at '-"".. . J~ ]

el eL~·-------·------------~----~
e
(
Thalamus
VANLand
l
~-=N=~eo_s_t-ri-.at.: u::::
: m:,____j eI Substantia Nigra other nuclei
parsreticulata

J
. .

j indir,ect j direct !....,-..._,mol


ipathway ! pathway
\_

elte .,.
! )
1
'"'
Substantia Nigra
pars compacta
·-·-----·--- .~~--

J
···-·-...:...... ___ · ..

e~ _t
1_1 Globu• ''"'d"""· . ; c_.__ _ _ e_-_~_~_,~· P•IHdu• '"'· ~e
L-.~ -~ - . e;r Subthal~mk I.
........,_ _ _ _ _ _ _..,.mwm·_ (f)-..,.; L~~-
cl_
e~---·-j
)"" iPe<luowlo=lo~
~~deus __

r 781
End of Ses 1on Quiz
5. What are the cardinal features of Parkinson disease?

OJ
)>
(J)
)>

'GJ
)>
z
GJ
'
)>

6. A 28-year-old chemist presents with MPTP exposure. Which neurotransmitter is


depleted?

7. How does each ofthe following structures normally impact movement?


• .c;lobus pallid us. interna
· • • Subthalamic nucleus
• Substantia nigra pars ,...,-,rnn - .r-t--..

8. Amale patient presents with involuntary flailing of onearm. Where is the .lesion?

9. Which ne.~rotransmitters are altered in Huntington disease?

RAPID-FIR E FA CTS
Depigmentation of the substantia nigra

Chorea, dementia and atrophy of the caudate and


putamen
Eosinophilic inclusions in the cytoplasm of
neurons

f79l
SKELETAL MUSCLE
Phys: Chapters 6, 7
Types of muscle fibers (FA15 p423) (FA16 p424) (SU p247-248)
Muscle conduction to contraction (FA15 p423) (FA16 p424) (Phys p83)
Skeletal and cardiac muscle contraction (FA15 p424) (FA16 p425) (SU p246) (Phys p73)
Dantrolene (FA15 p499) (FA16 p503) (SU p417)

_J

~
LLJ
Warm-Up Review
_J
LLJ
:>.L 1. What infections are caused by group A streptococcus (Streptococcus pyogenes)?
l/)
(FA1S p129) (FA16 p120) (SU p323)

2. Which gram-positive organism is associated with each ofthe following statements?


(FA15 p127-134) (FA16 p118-125)
• Causes scalded skin syndrome
• Gray-white membrane in the posterior pharynx of an unvaccinated child
• Pharyngitis ~ glomerulonephritis
• Most common cause of meningitis
• Most common cause of osteomyelitis
• Serious newborn infections

r so 1
3. Label the sarcomere:

D
c
n

(j)
;;;><;
rn
r-
rn
:;;;!
r-
:s:
c
F Ul
n
r-
rn
E G

4. Muscle spindle control


Muscle spindle: monitors muscle length
• Extrafusal muscle fibers: functional unit of muscle
• lntrafusal muscle fibers: regulate length
Muscle stretch results in intrafusal stretch which stimulates 1a afferent--+ dorsal horn --+
a motor neuron, causing reflex muscle contraction
Hint: help you pick up a heavy suitcase
• Golgi tendons: monitor tension rather than length (perpendicular to intrafusal muscle fibers)
Provide inhibitory 1b afferent feedback
Hint: cue you to drop a heavy suitcase

Gamma loop: regulates sensitivity of reflex arc


• CNS --+they motor neuron --+ contracts intrafusal fiber (central part of spindle), increasing the
sensitivity of the reflex arc

muscle
fibers

muscle
fibers

r s1 1
End of Sess1on Qui z
5. Which two muscle receptors are responsible for opening the sarcoplasmic reticulum
in response to depolarization?
LLJ
_.J
u
Vl
::J
2:
_.J
<( 6. What drug prevents the release of calcium from the sarcoplasmic reticulum of
r-
L1.J skeletal muscle?
_.J
LLJ
l.L
Vl

7. What are the differences between type 1 and type 2 muscle fibers?

8. What are the differences between the la afferent motor pathway and the lb afferent
motor pathway?

RA PID - FIR E FACTS


Toe extension upon stimu lating the sole of the
foot with a blunt instrument

r s2 1
UPPER EXTREMITY NERVES PART 1
COA: Chapter 6
Neck anatomy
Brachial plexus (FA15 p419) (FA16 p420) (SU p238) (COA p721) (H p3471)
Brachial plexus lesions (FA15 p419) (FA16 p420) (SU p238)
Erb-Duchenne palsy (FA15 p419) (FA16 p420) (SU p238) c
Klumpke palsy (FA15 p419) (FA16 p420) (SU p238) -u
-u
m
:::0
m
X
___,
:::0
Warm -Up Rev1ew m
:s:
___,
1. V\{tiich gram-positive organism is associated with each ofthe following statements? -<
z
(FA.l5 p127-134) (FA16 p118-12S) m
:::0
• Infant with poor muscle tone <
m
(/)
• Diarrhea after using" ahtibiotics -u
)>
• Respiratory distress in a postaL worker :::0
___,
• Otitis media in children
•Cellulitis

2. What is the classic presenting symptom of a patient with Lyme disease? (FA1S p140)
(FA16 p130) (SU p333)

3. What is the function of macrophages in the spleen? (FA15 p199) (FA16 p191)

r s3 1
4. Label the following diagram of the brachial plexus:

Cords Terminal Branches

cs
f-
~
G
<{
0...
(/)
w
>
~
C6
w
z
>-
f- C7 J
~
w
~
f-
X
w
~
C8
w
0...
0...
::::::>
K
T1

A. -----------------------------------------------------
B.

C. - ---------------------- ---------- -- -------------------

D. ------------------------------------------------------
E.

F.

G. -----------------------------------------------------

H. -----------------------------------------------------
I.

J.
K.

.~

r 841
5. Upper extremity nerves
• Long thoracic --7 serratus anterior --7 holds the scapula to the thorax
• Suprascapular --7 infraspinatous --7 external rotation; supraspinatus --7 abducts the arm
• Lateral pectoral --7 pectoralis major --7 flexion , adduction and medial rotation
• Upper subscapular --7 subscapularis --7 internal rotation
• Thoracodorsal --7 latissimus dorsi --7 extension, adduction, transverse extension, flexion from an c
u
extended position and internal rotation u
rn
:;o
• Lower subscapular --7 teres major --7 medial rotation and adduction rn
X
• Musculocutaneous --7 biceps, coracobrachialis and brachialis --7 major flexors of the arm ~
:;o
rn
• Axillary --7 deltoid --7 abduction; teres minor --7 latera l rotation of the humerus :s
~
Radial --7 triceps and extensors --7 extend forearm -<
z
• Media l --7 pronators and thenar muscles --7 pronate the arm rn
:;o
<
• Ulnar --7 flexor carpi ulnaris, interosseus and hypothenar --7 flex and adduct the hand rn
Vl
u
)>
:;o
~

r 85 1
End of Sess1on Qu1z
6. Label the following diagram of the brachial plexus:

al Branches
r---
0:::
<(
o_
[/)
w
>
0::: G
w
z:
>-
f-
2:
w
0:::
f-
xw
0:::
J
w
o_
o_
:::J

A _____ _______~------------

B. ~---~-~----~---------~---

D. --------~--------~--------
E.

r
r.

G. _ _ _ _ _ _ _ _ _ _ _ _ _ _ _ _ _ _~-------

I.

J.
K.

(continued)

[ 86 J
End of Sess1on Quiz (cont inued)
7. What are the symptoms of a lesion to the CS and C6 nerve roots?

c
-o
-o
m
:::0
m
X
-i
:::0
m
:s
::::::j
-<
z
m
:::0
<
8. are the symptoms of a to the inferior trunk ofthebra<;hial plexus? m
Ul
-o
)>
:::0
-i
~

RAPID -FIRE FACTS


Newborn with arm paralysis following a difficult
labor

r 87l
UPPER EXTREMITY NERVES PART 2
COA: Chapter 6
Upper extremity nerves (FA15 p418) (FA16 p419)
- Radial nerve
Ulnar nerve
N Median nerve
1- Musculocutaneous nerve
~

~ - Axillary nerve
if) - Long thoracic nerve
L.LJ
>
~
L.LJ
z:
>-
1- Warm-Up Rev1ew
:2:
L.LJ
~
1. What patients are particularly susceptible to Listeria monocytogenes?
1-
><
L.LJ
~
L.LJ
0...
Cl...
::::J

2. What name is given to RBCs that have been partially digested by splenic
macrophages?

3. Which nerve is being tested with each of the fo11owing reflexes?


• Biceps reflex
• Triceps reflex
• Patellar reflex
• Achilles reflex

4. Radial nerve
• Motor
- Triceps brachii (extends forearm)
Supinator (supinates)
Brachioradialis (flexes forearm , supinates and pronates)
Extensor carpi radialis longus (extends wrist)
Extensor digitorum, extensor pollicis longus +other extensors (extends wrist and digits)
Mnemonic: BEST
• Sensation
Posterior arm
Dorsal side of the hand
• Pathology
"Saturday night palsy": compression of the radial nerve against the spiral groove of the humerus
~ wrist drop, difficulty straightening fingers, and numbness on the back of the hand and
posterior arm
Damage is associated with a fracture of the shaft of the humerus

i 88 l
5. Ulnar nerve
• Motor
Hypothenar muscles (control hand and 5th digit)
3rd and 4th lumbricals (flex at MCP and extend at interphalangeal joints)
Interosseous (abducts and adducts digits)
- Adductor pollicis (adducts thumb) c
"""0
• Sensation """0
rn
;;o
5th digit and half of 4th digit rn
X
Media l side of palm and dorsa l hand ---1
;;o
rn
• Pathology s:
Proximal injury 4 loss of flexion of the wrist and digits ---1
-<
"Ulnar claw" deformity: MCP joint hyperextension and IP joint flexion at 4th and 5th digits :z:
rn
;;o
- Weak abduction and adduction of digits <
rn
Hypothenar wasting Vl
"""0
)>
;;o
---1
N

6. Median nerve
• Motor
Pronator teres and pronator quadratus (pronate hand)
Flexor carpi radialis and palmaris longus (flex forearm)
Flexor digitorum superficia lis and lateral half of flexor digitorum profundus (flex wrist and digits)
Flexor pollicis longus (flexes thumb)
1st and 2nd lumbricals (flex MCP joints and extend IP joints)
• Sensation
Lateral palm
First 3Yz digits
• Pathology
"Ape hand" deformity: loss of thumb abduction
"Hand of benediction" deformity: loss of flexion of the 2nd and 3rd digits
Carpal tunnel syndrome: numbness of the first 3Yz digits

r 891
7. Musculocutaneous nerve
• Motor
Coracobrachialis, biceps and brachialis (flex and supinate forearm)
• Sensation
Lateral sid e of forea rm
N • Pathology
f---
0:::
<r:
Decreased flexion and supination of forearm
0...
lf)
Ll.J
>
0:::
Ll.J
z 8. Axillary nerve
>-
f- • Motor
2:
Ll.J
0:::
Deltoid (abducts shoulder)
f-
x Long head of triceps (extends forearm)
Ll.J
0::: Teres minor (externa lly rotates arm)
Ll.J
0...
0... • Sensation
:::::>
Lateral shoulder
• Pathology
Occurs with dislocation of shou lder or fracture of surgical neck of humerus
Loss of abduction; weak flexion, extension, and rotation of shoulder
Loss of sensation to latera l shoulder
Posterior circumflex artery runs with nerve

9. Long thoracic nerve


• Motor
Serratus anterior (holds the scapu la to the thorax)
• Sensation
None
• Pathology
Winged scapula
Due to a blow to the neck or shoulder or after radical mastectomy

t 90 l
End of Session Quiz
10. ¥Jilat nerve is damaged wheo.a patient presents with each of the following upper
extremity symptoms?
• Unable to extend the Dl Rand PIP joints of the 4th and 5th fingers ·•.
• Unableto flex the index and middle finger
• Wristdrop
• Scapular winging rn
__,
X
• Loss of sensation over fingers 1-4 ;;o
rn
• Cannot abduct or adduct fingers :s:
• Loss of shoulder abduction -1
-<
• Loss of elbow flexion and forearm supination z
rn
;;o
• Loss ofwrist extension <
rn
l.n
-u
)>
;;o
__,
11...What nerve is·mostat risk of injury with each of the following types of fracture/injury? N

• Fracture of the shaft of the humerus


• ·Fracture ofthe surgical neckofthe humerus
• Anterior shoulder dislocation
• Injury to the carpal tunnel

12. Apatient falls off a motorcycle and .lands on his right shoulder. On physical exam, you
notice his shoulder has an abnormal configuration. X-rays indicate an anterior dislocation
of his shoulder. What artery and nerve are most at risk of being damaged?

r 91 1
LOWER EXTREMITY
COA: Chapter 5
Lower extremity nerves and muscles (FA15 p421) (FA16 p422)
- Femoral nerve
- Obturator nerve
>- - Superior gluteal nerve
1-
Inferior gluteal nerve
2:
w - Sciatic nerve
0:::
I- - Tibial nerve
X
LLJ - Common fibular nerve
0::: - Deep peroneal nerve
w
s0 Sensation of the lower limbs
Arteries ofthe leg
__J

Warm - Up Rev1ew
1. Which type of hypersensitivity is responsible for each of the following clinical scenarios?
(FA15 p212) (FA16 p204-205)
• Poststreptococcal glomerulonephritis
• Rheumatic fever
• Polyarteritis nodosa
• Serum sickness
• Contact dermatitis

2. What are the differences between essential tremor, resting tremor and intention tremor?
(FA15 p459) (FA16 p461)
• Essential tremor
• Intention tremor
• Resting tremor

3. Femoral nerve
• Motor
Psoas, iliacus, pectineus and sartorius (hip flexion)
Vastus lateralis, vastus intermedius, vastus medialis and rectu s femoris (knee extension) .
• Sensation
Anterior thigh
Medial lower leg
• Pathology
Pelvic fracture

r 92 1
4. Obturator nerve
• Motor
- Adductor magnus, adductor longus and adductor brevis (hip adduction)
Gracilis (knee flexion)
• Sensation
Medial thigh r-
0
• Pathology ~
m
;;o
- Anterior hip dislocation m
X
-;
;;o
m
:s:
-;
5. Superior gluteal nerve -<
• Motor
- Tensor fascia latae, gluteus medius and gluteus minimus (hip abduction and medial rotation)
• Sensation
None
• Pathology
Posterior hip dislocation
Polio
- Trendelenburg gait

6. Inferior gluteal nerve


~ • Motor
Gluteus maximus (hip extension and lateral rotation)
• Sensation
None
• Pathology
Posterior hip dislocation
Hip replacement
Difficulty standing up or climbing stairs

7. Sciatic nerve
• Motor
Biceps femoris (tibia l branch), semitendinosus, semimembranosus and adductor magnus (hip
extension and knee flexion)
• Sensation
Posterior thigh and gluteal regions
Entire lower leg except medial aspect
• Pathology
Intervertebral disc herniation
Spinal stenosis
Spondylolisthesis

r 93 1
8. Tibial nerve
• Motor
Gastrocnemius, soleus and plantaris (plantar flexion)
Popliteu s (unlocks knee)
Flexor hallucis longus and flexor digitorum longus (toe flexion)
>- • Sensation
I-
~ Sole of the foot
UJ
o:::
1-
Back of the calf (sural nerve)
C;S • Pathology
0:::
UJ Kn ee injury
5
0_ j

9. Common fibular nerve (common peroneal nerve)


• Motor
Peroneus longus, peroneus brevis and short head of the biceps femoris
• Sensation
Lateral lower leg
Dorsal side of foot
• Pathology
Chronic compression
Habitual leg crossing
Fract ure of neck of fibula
Foot drop

10. Deep fibular nerve (deep peroneal nerve)


• Motor
Peroneus tertius (foot eversion)
- Tibialis anterior, extensor hallucis longus and extensor digitorum longus (foot dorsiflexion)
• Sensation
- Webbing between first and second digits
• Pathology
- Trauma to lateral knee
Foot drop

r 94 1
En Sess1on Qu
11. Which nerve would most likely be damaged with each of the following injuries?
• Pelvic fracture
• · Anteriqr hip dislocation
• Posterior hip dislocation
• Vertebral disc herniation
• Knee injury
• Habitual

12. Which nerve is responsible for each of the following functions?


• Hip flexion and knee extension
• Sensation to ithemedial thigh
• Helps to prevent Trendelenburg gait
• ..Sensation of the lower leg- except for the medial aspect
• Sensation of the sole of the foot

i g" l
SENSATION
Phys: Chapters 46-48
R: Chapter 28
Sensory corpuscles (FA15 p452) (FA16 p454)
Peripheral nerve layers (FA15 p452) (FA16 p454)
:z Schwann cells (FA15 p451) (FA16 p453) (SU p23)
0 Guillain-Barre syndrome (FA15 p488) (FA16 p493) (SU p47) (H p3473)
~ Other demyelinating diseases (FA15 p489) (FA16 p493) (SU p47)
(f)
z
w
(f)

Warm -Up Rev1 ew


1. Name eight different indirect cholinergic agonists, and state the use for each.
(FA15 p250) (FA16 p244)

2. Sensory corpuscles
Adaptation Sensation Location/other

Merkel

Meissner

Ruffini

Pacinian

Free nerve ending

r 961
En d of Session Qu 1z
3. Which sensory receptor communicates each of the following types of information?
• Pricking pain (fast, myelinated)
• Burning or duiLpain and itch (slow, unmyelinated)
(J)
• . Vibration and pressure m
z
(J)
• Dynamic/changing light. discriminatory touch
~
• Static/unchanging light touch 0
• Proprioception information, muscle length monitoring z
• Proprioception information, muscle tension monitoring

4. What CSF changes are present in Guillain~Barre syndrome?

5. What are the classic manifestations of Gulllain~Barre syndrome?

r 971
EYE
COA: Chapter 7
H: Chapter 28
Eye anatomy (FA15 p479) (FA16 p482) (COA p889)
Conjunctival pathology
~ Corneal pathology
w Uveitis (FA15 p480) (FA16 p484)
Aqueous humor (FA15 p479) (FA16 p483)
Glaucoma (FA15 p480) (FA16 p484) (SU p37) (COA p912) (H p234)
Glaucoma drugs (FA15 p494) (FA16 p498) (GG p1785)
Cataracts (FA15 p480) (FA16 p483) (SU p37)
Retina
- Age - related macular degeneration (FA15 p481) (FA16 p485) (SU p37) (H p235)
- Diabetic retinopathy (FA15 p481) (FA16 p485) (SU p37)
- Retinal detachment (FA15 p481) (FA16 p485) (COA p910)
- Retinoblastoma
Optic nerve
- Papilledema (FA15 p482) (FA16 p486) (SU p37)
- Central retinal artery occlusion (FA15 p482) (FA16 p486)
Brain lesions affecting the eyes

Warm -Up Revi ew


1. Which cytokine, more than any other, should be known as the macrophage-activating
cytokine? (FA15 p383) (FA16 p379) (SUp388)

2. Which complement is responsible for neutrophil chemotaxis?

3. Which pupillary sign suggests a diagnosis of syphilis?

4. Describe the flow of aqueous humor.


• Produced by the epithelium of the ciliary body
• Flows from the posterior chamber~ through the pupil ~ into th e anterior chamber
• Exits the anterior chamber via the trabecular meshwork (canal of Schlemm)

f 981
5. Acute angle-closure glaucoma
• Emergency
• Abrupt onset of pain, headache (temporal, eyebrow), nausea, colored halos, rainbows around light
• Red, teary eye with hazy cornea and fixed, mid-dilated pupil (not reactive to light) that is firm to palpation

m
-<
m
6. Open-angle glaucoma
• Common , insidious form; usually bilateral
• Risk factors: older than 40, African American, family history of glaucoma, myopia , and diabetes
• Early stage: asymptomatic, elevated intraocular pressure
• Late stage: gradual loss of peripheral vision, permanent blindness if untreated

7. What is the classic presentation of a patient who has a cataract?


• Usually bilateral
• Slowly progressive, painless decrease in vision (difficulty driving at night, reading road signs, or
reading fine print)
• Possible disabling glare from sunlight or oncoming headlights at night
• Near-sightedness is often an early manifestation ("second sight")

End of Sess1 on .Qu1z


8. What initial type of vision loss is commonly seen in patients with open-angle glaucoma?

9. What drug classes areused in the treatment of glaucoma?

10. What is the treatment for dry age-related macular degeneratic>n?


EAR
Phys: Chapter 55
H: Chapter 21
COA: Chapter 7
Auditory sensation (FA15 p477) (FA16 p481) (SU p34)
cr:: Hearing testing (FA15 p477) (FA16 p481)
:'5 Acute otitis externa (COA p 978)
Acute otitis media (COA p978)
Cholesteatoma (FA15 p478) (FA16 p482) (H p249) (R p740)
Vertigo (FA15 p490) (FA16 p495)

Warm - Up Review
1. Which spinal tracts conveythefollowinginformation? (FA15 p470) (FA16 p473)
• Pain and temperature sensation
• Important forpostural adjustments and head movements
• Proprioceptive information for the cerebellum

2. What molecules are expressed on the surface of antigen-presenting dendritic cells?

3. Auditory pathway
• Tympanic membrane
• Middle ear ossicles (malleus ~ incus ~ stapes)
• Auditory hair cells in the cochlea
Olivocochlear bundles send cholinergic signals to the cochlea, which cause contraction of the
outer hair cells, stiffening the basilar membrane and sensitizing the inner hair cells to a particular
frequency
- Aminoglycosides (e.g., streptomycin, gentamicin) cause hearing loss by dJmJging the outer hair
cells
• Spiral (cochlear) ganglion
• Cochlear nuclei
• Superior olivary nucleus (contralateral)
• Lateral lemniscus
• Inferior colliculus
• Medial geniculate nucleus (thalamus)
• Primary auditory cortex (temporal lobe)

r 100 1
4. Weber test
• Normal ~ midline
• Conductive hearing loss ~ lateralizes to the side of the affected ear
• Sensorineural hearing loss ~ lateralizes to the side opposite the affected ear

5. Rinne test
• Normal ~ air conduction > bone conduction (AC > BC)
• Conductive hearing loss ~ BC > AC

6. Given each of the following exam findings, what is the diagnosis?


Weber Rinne (left) Rinne (right) Diagnosis
Patient A Midline AC > BC AC > BC

Patient B Right AC > BC BC > AC

Patient C Left AC > BC AC > BC


Patient D Midline BC >AC BC > AC

7. Acute otitis externa (swimmer's ear)


• Inflammation/ infection of the ear canal
• Pain with manipulation of the ear (pinna) or instrumentation of the canal
• Most commonly caused by P. aeruginosa or S. aureus
• Treatment: irrigation and topica l antibiotics

8. Acute otitis media (AOM)


• Bacterial or viral infection of the middle ear space
• Diagnosis made by inspection of the tympanic membrane (TM)
Bulging
Middle ear effusion (opacity, air-fluid level, pus)
Erythema
TM immobility under positive pressure with a pneumatic otoscope
• Most common bacterial causes: S. pneumoniae, nontypable H. inf]uenzae, M. catarrhalis
• Treatment:
- Antibiotics (amoxicillin, amoxici llin + clavulanic acid, cephalosporins)
- Tympanostomy tubes
• Complications may include acute mastoiditis (typical AOM symptoms+ post-auricular swelling,
redness, and mastoid tenderness)

9. Cholesteatoma
• Overgrowth of desquamated keratin debris within the middle ear space that may eventually erode
the ossicular chain and the mastoid air cells
• Causes: negative middle ear pressure (chronic retraction pocket) from eustachian tube dysfunction
or direct growth of epithelium through a TM perforation
• Commonly associated with chronic middle ear infection
• Physical exam: grayish-white, "pearly" lesion behind or involving the TM, conductive hearing loss, vertigo
• Treatment: surgical removal (tympanomastoidectomy) and reconstruction of the ossicular chain

r 101 1
10. Vertigo
• Benign paroxysmal positional vertigo (BPPV)- presumed to be due to debris or misplaced otoliths
within the vestibular apparatus; the Dix-Hallpike maneuver and the Epley maneuver are used for
diagnosis and treatment
• Vestibular neuritis - inflammation of vestibular nerve
o::: • Meniere disease (endolymphatic hydrops) - imbalances of the fluid and electrolyte composition of
<t
w the endolymph cause a triad of intermittent vertigo, tinnitus, and hearing loss
• Central vertigo- brainstem and cerebellar lesions damaging the vestibular nuclei

End of Sess1on Qu1z


11. A patient presents with vertigo, tinnitus, and hearing loss. What is the diagnosis?

12. What bacterial organisms are most commonly responsible for acute otitis media?

13. What bacterial organism is most commonly responsible for otitis externa?

14. Chronic otitis media can sometimes result in a cystic lesion lined by keratinizing
squamo-us epithelium (which can be metaplastic) and filled with amorphous debris.
What is the diagnosis?

[ 102 J
SLEEP
Phys: Chapter 59
H: Chapter 27
Sleep stages (FA15 p455) (FA16 p457) (SU p42-43)
Sleep changes in the elderly (FA15 p60) (FA16 p44)
Sleep patterns of depressed patients (FA15 p511) (FA16 p515) Ul
Common sleep disorders (SU p43) 'rn
rn
- Insomnia -u
- Restless leg syndrome
Night terrors (FA15 p516) (FA16 p520)
Sleep apnea (FA15 p613) (FA16 p621)
- Narcolepsy (FA15 p517) (FA16 p521)
Nocturnal enuresis
Medications for insomnia

rm -Up Review
1. To which class of antiarrhythmics does each of the followingagents belong?
• Verapamil
• Pfocainamide
• Lidocaine
• Diltiazem

~. 2. What is the clinical use for tiotropium? (FA15 p251) (FA16 p245)

A patient h~s facial angiofibromas, ash-leaf spots of depigmentation, a history


of seizures, and intellectual disability. What is the most likely diagnosis? What
neoplasms is this patient at an increased risk of developing? (FA15 p235) (FA16 p495)

[ !03 J
4. What medications are common in the treatment of insomnia? What makes each one unique?
Melatonin Nonaddictive, OTC, good safety profile,
questionable efficacy
Antihistamines Commonly used OTC by patients first line,
(diphenhydramine) anticholinergic side effects (avoid in the elderly),
next-day fatigue
0..
w
w
_j
Antidepressants Anticholinergic effects, dry mouth
lf)
(amitriptyline, trazodone,
doxepin)
Benzodiazepines Increase GABA, addictive, short term only
(triazolam, temazepam,
lorazepam)
Zolpid em, zaleplon Act at the benzo receptor, short term only
(< 35 days), rebound insomnia when
discontinued
Eszopiclone Longer half-life than zolpidem, may be used long
term
Rame lteon Nonaddictive, works at melatonin receptors,
avoid if hepatic insufficiency; long-term studies
are lacking

Orexin receptor antagonists Block effects of orexin A and 8, which promote


(suvorexant) wakefulness

5. What is the treatment for narcolepsy?


• Avo idance of drugs that cause sleepiness
• Scheduled naps (once or twice a day for 10-20 min)
• Stimulants - modafinil, methylphenidate
• If cataplexy then use ven lafaxine, fluoxetine, or atomoxetine
• Sodium oxybate (GHB) is last line for cataplexy

f 104]
End of SeSSI On UIZ
6. Which EEG waveforms correspond to the different stages of sleep?

Vl

'rn
rn
-o

7. What drugs are used to shorten stage N3 sleep?

8. What is the sleep pattern in a patient with narcolepsy?

9. During which stage otsleep do nig~t terrors occur?

r 10s 1
ANESTHETICS
GG: Chapters 11, 17, 19, 20

Principles of anesthetics (FA15 p497) (FA16 p502)


Inhaled anesthetics (FA15 p498) (FA16 p502)
IV anesthetics (FA15 p498) (FA16 p502)
(f) Propofol (FA15 p498) (FA16 p502) (SU p448)
u
f-
Ketamine (FA15 p498) (FA16 p502) (SU p432)
w - Barbiturates (FA15 p497-498) (FA16 p501 -502)
:r::
f- - Benzodiazepines (FA15 p497-498) (FA16 p501-502)
(f)
w - Opioids (FA15 p498) (FA16 p502) (SU p244)
z Neuromuscular blocking agents (FA15 p499) (FA16 p503) (SU p248-249)
<[
Malignant hyperthermia (FA15 p498) (FA16 p502) (SU p75)
Dantrolene (FA15 p499) (FA16 p503) (SU p54, 417)
Local anesthetics (FA15 p499) (FA16 p503)

Warm-Up Revi ew
1. What is the classic presentation of amyotrophic lateral sclerosis? (FA15p471)(FA16p474)
(SU p47)

2. What are classic signs of an upper motor neuronlesion?JNhat are classic signs of a
lower motor neuron lesion? (FA15 p470) (FA16 p473)

;- 106 l
3. Inhaled anesthetics
• Nitrous oxide: low blood solubility, low lipid solubility
• Halothane: high blood solubility, hepatotoxicity
• Sevoflurane, desflurane: low blood solubility, high lipid solubility
• lsoflurane: moderate blood and lipid solubility
)>
z
m
(J)
-i
I
4. IV anesthetics m
-i
• Propofol: increases GABA, rapid onset, rapid recovery, high triglyceride content n
(J)

• Ketamine: dissociative anesthetic, hallucinations, increases cerebral blood flow


• Barbiturates: decrease cerebral blood flow, increase duration of chloride channel opening on GABA
receptors
• Benzodiazepines: increase frequency of chloride channel opening on GABA receptors , reverse with
flumazenil
• Opioids: analgesia; mu, delta , and kappa receptors
Butorphanol: partial opioid, causes less respiratory depression
Dextromethorphan: weak opioid used for cough suppression
- Tramadol: less-addictive opioid used for analgesia
Loperamide and diphenoxylate: treatment of diarrhea

5. Neuromuscular blocking agents


..~ • Non-depolarizing: competitive inhibitors of postsynaptic nicotinic acetylcholine receptors at
neuromuscular junction , can reverse with acetylcholinesterase inhibitor
• Depolarizing (succinylcholine):
Phase 1: rapid onset and recovery, cannot be reversed
Phase II : larger dose, longer recovery, can be reversed with acetylcholinesterase inhibitor
Succinylcholine + inhaled anesthetic in patient with RYR1 gene mutation --7 possible malignant
hyperthermia

r I07l
En d of Sess1on Qu 1z
6. Which anesthetic fits each of the following descriptions?
• IV, associated with hallucinations and bad dreams
• IV, most commonly used benzodiazepine for conscious sedation
tn
u • Inhaled, has side effect of hepatotoxicity
1-
w • IV, rapid anesthesia induction and short duration of action
I
1- • IV, decreases cerebral blood flow (important in brain surgery)
Vl
w
z • Opioid that does not induce histamine release
<{
• High triglyceride content increases risk of pancreatitis with long-term use

7. What is the mechanism of action of dantrolene?

8. What is the mechanism of action of the local anesthetics? Which nerve fibers are
blocked first with local anesthesia?

9. What drugs can be used to reverse neuromuscular blockade?

I 108 l
PHARM BASICS THERAPEUTICS ARE
DISCUSSED TH ROU
THE PART 2 VIDEOS
1 Parasympathetic Activation
ALONGSIDE THE DfSEASES
2 Parasympathetic Inhibition THEY TREAT THE P
BASICS SECTION
3 SympathetiC Activation
COVERS THE AUTONOMIC
4 Sympathetic Inhibi t ion NERVOUS SYSTEM AND
DRUGS THAT MODULATE
5 Neurotransmiss1on IT CELLULAR MESSENGER
I

6 G Protem Second Messengers SYSTEMS, DRUG


METABOLISM AND THE
7 Pharmacokinetics FOUR PHARMACOKINETI
8 Pharmacodynam ics NS THAT
ABSOLUTELY MUST BE
9 Drug Metabol ism UNDERSTOOD IN ORDER
10 Drug Elimination TO SURVIVE YOUR EXAM
THI SSECTION ALSO
11 Drug S1de Effects INCLUDES MATERfAL
ON TOXICOLOGY AND
12 Antidotes
IMPORTANT DRUG SIDE
EFFECTS
PARASYMPATHETIC ACTNATION
GG: Chapters 8-10
Phys: Chapter 60
Autonomic nervous system (FA15 p247) (FA16 p241)
Nicotinic vs. muscarinic receptors (FA15 p247) (FA16 p241)
Parasympathetic activation (SU p25)
Cholinergic agonists (FA15 p250) (FA16 p244)
Alzheimer disease
Myasthen ia gravis (FA15 p435) (FA16 p436)

Warm- Up Review -o
)>
;;o
1. Where are the different types of collagen found in the body? (FA15 p75) (FA16 p61) )>
(.f)
-<
:$:
-o
~
I
rn
-l
n
)>
2. A10-year-old man is broughtto the emergency department follovving the sudd~n n
-j

onset of a severe headache. He has a history of Ehlers-Danlos syndrome and · <


~
hypertension. Examination of the CSF shows numerous red blood cells. What is the 0
most likely cause of this man's headaches? (FA15 p77) (FA16 p63) :z:

3. How does the presentation of a dominant parietal lobe lesion differ from the
presentation of a nondominant parietal lobe lesion? (FA15 p461) (FA16p464)

[ iII J
4. Patient Profile: Old MacDonald Gets Leaky
A 38-year-old man presents to the emergency department with shortness of breath, diaphoresis and
agitation. He has excessive urination and diarrhea. He recently moved to the area and works at a local
farm . Physical exam shows he has constricted pupils, a heart rate of 55/min , mild inspiratory wheezes,
excessive salivation and muscle twitching throughout his body. What is the mechanism of action of the
medication most likely used to treat this patient's symptoms?

Parasympathetic activation Cholinergic excess


• Gl: increased digestion • D_________________________
• Bladder: wall contraction, relax sphincter • u-------------------------
z • Eye: miosis, ciliary muscle contraction • M
0
~
• Lung: smooth muscle contraction • B
> • Heart: decrease heart rate/contractility • B
1-
u
<( • E
u
1-- • Lacrimal glands: stimulate tears • L
w
:r:
~
• Salivary gland: watery secretion • s
0...
~
• s
>-
(/) • Uterus: contraction
<(
iY:
<(
• Penis/clitoris: erection/engorgement
0...

Summary
• Organophosphate poisoning ~ inactivation of _________________________
excess ~ DUMBBELSS (leaky)

• Treatment:
Goal: decrease stimulation of excess acetylcholine on muscarinic receptors
Atropine blocks muscarinic receptors
Pralidoxime regenerates acetylcholinesterase

5. Myasthenia gravis
• Antibodies to the acetylcholine receptor
• Most common board question presentation : _ _____________________ that worsens
throughout the day
• Tensilon test
• Thymu s pathology:
50% associated with thymic ______________________________________
20% associated with thymic ___ _ _ _ _ _ _ _ _ _ _ _ _ _ _ _ _ __
15% associated with thymic _ _____________________________________
• Myasthenic crisis: rapidly progressing weakness esp. in _ _ __ _ ___ muscl es
• Treatment:

[ I i2]
En d of S S!On UIZ
6. How does the parasympathetic nervous system affect the following body structures?
• Heart
• Eye
• Salivary glands
• Bronchiolar smooth muscle
• Bladder
• . Male GU system
• Gl tract

u
)>
7. What drug regenerates acetylcholinesterase after organophosphate poisoning? ;;o
)>
Ul
-<
:s:
u
2::;
I
rn
---i
n
)>
8. What is the antidote for organophosphate poisoning? n
---i
<
2::;
0
:z:

9. Which anticholinesterases are used in the treatment of Alzheimer disease?

RAP! O-F! RE FACTS


Amyloid deposits in gray matter of the brain

Drooling farmer

[ 113 J
PARASYMPATHETIC INHIBITION
GG: Chapters 9, 77
Parasympathetic activation review
Parasympathetic inhibition (SU p25)
Muscarinic antagonists (FA15 p251) (FA16 p245)
Other drugs with anticholinergic side effects

Wgrm -U p .Review
1. Anxiety is associated with altered levels of which neurotransmitters?
z
0
I--
co
-
:r::
:z
u
I--
w
:r:: 2. A patient's uvula deviates to the right when she says "ahh." Damage to what
I--
<(
()__
neurological areas would produce this abnormality?
:2:
>-
(/)
<(
a:
<(
()__

3. What prenatal testing is abnormal in a fetus with a neural tube defect? (FA15 p449)
(FA16 p451)

4. What are the symptoms of inhibiting parasympathetic activity?

r 114 1
5. Which anticholinergics are used in the treatment of urge urinary incontinence?
• Oxybutynin
• To lterodine
• Darifenacin
• Trospium
• Solifenacin

6. In which patient populations is atropine contraindicated?

-u
)>
;;;o
)>
(J)

7. What other medications have anticholinergic side effects? -<


5
-u
• First-generation H 1 blockers: diphenhydramine (Benadryl), doxylamine (Unisom), ch lorpheniramine ~
:r:
• Neuroleptics rn
-i
• Tricyclic antidepressants n
z
• Amantadine :r:
CD
-i
0
z

En d of Session Quiz •
Classify each of the following drugs as a direct cholinergic agonist, anticholinesterase,
antimuscarinic, or cholinesterase regenerator:
• Physostigmine • Tropicamide
· ·····pnocarpi ne • Benztropine
• Oxybutynin • Scopolamine
• Atropine • Edrophonium
• Donepezil • Tolterodine
• Pralidoxime • Trospium
• Bethanechol • Rivastigmine
• Neostigmine • Homatropine
• Darifenacin • Pyridostigmine
• lpratropium • Carbachol

9. Which of the muscarinic antagonists discussed could be used to improve FEV1 in a


patient with COPD?

r 115 1
SYMPATHETIC ACTNATION
Autonomic nervous system review
Adrenergic receptors (FA15 p247) (FA16 p241) (SU p25)
Sympathomimetics (FA15 p253) (FA16 p246)

Warm -Up Rev1ew


1. A patient cannot adduct her left eye on lateral gaze, but convergence is normal.
Which structure is damaged? (FA15 p4B6) (FA16 p490)

:z:
0
~
>
f-
u
<(
u
f-
2. A 2-year-old child who has had multiple viral and fungal infections is found to be
L1.J
I hypocalcemic. Which of the three germ layers (ectoderm, endoderm, or mesoderm)
~
Q_
gives rise to the structure missing in this child? (FA15 p214, 566) (FA16 p20B, 570)
2
>-
(f)

3. Describe the sensory innervation of the tongue. (FA15 p450) (FA16 p452)

r 116 1
nd of Sess io n u1z
Which receptors are stimulated by each of the following sympathomimetics?
• Clonidine
• Dopamine
• Phenylephrine
• Albuterol
• Norepinephrine
• Isoproterenol
• Epinephrine
• Dobutamine
Vl
• Terbutalirie -<
:5:
--o
~
:r:
rn
-i
5. Which sympathomimetic matches each of the following statements? ()

::t>
• Given as anebulizer for asthma ()
-i
• Drug of choice for anaptwlaxis <
• Most common first-line agent for patients in septic shock ~
0
• Given subQ for asthma z
• Used by ENT to vasoconstrict nasal vessels

r 117 1
SYMPATHETIC INHIBITION
GG: Chapter 12
a-blockers (FA15 p255) (FA16 p248) (SU p75-76)
{3-blockers (FA15 p256) (FA16 p249) (SU p77-78)
a2 -adrenergic agonists (FA15 p254) (FA16 p247)

War m-Up Rev1ew


1. Which structures are derived from the pharyngeal pouches? (FA15p566) (FA16 p570)

z
0
I:=
aJ
:r::
z
u
1-
UJ
:r::
~
o__

~ 2. Which structures run through the cavernous sinus? (FA15 p4 77) (FA16 p481)
(/)

3. What are the clinical features of osteogenesis imperfecta? (FA15 p76) (FA16 p63)

r 118 1
nd ess1 on Quiz
4. Which selective a-blocker has lesseffect on blood pressure?

en
-<
5. What are the common side eff~<:ts.of (3-blockers? Which patient pqpulations should :s:
--o
use caution when taking (3-blockers? ~
I
m
---1
n
z
I
CD
---1
0
z

6. Whatare the various clinical applications of (3-blockers?

RAP/ O-F! RE FACTS


Medical treatment for pheochromocytoma

Selective a 1 -antagonist used to treat BPH

r 119 1
NEUROTRAISMISSION
GG: Chapter 8
Phys: Chapt ers 45, 60

Cholinergic neurotransmission (FA15 p249) (FA16 p243) (SU p26) (GG p182) (Phys p731)
Catecholamine synthesis (FA15 p107) (FA16 p95) (SU p26)
Noradrenergic neurotransmission (FA15 p249) (FA16 p243) (SU p26)

Wa Rev1ew
z 1. Whatare the three different mechanisms that cells employ to breakdown proteins?
0
(f)
(FA15 p71) (FA16 p57)
(f)

::::;;:
(f)
z
<(
0:::
f-
0
0:::
::::J
LLJ
z What arethe components of the blood-brain barrier? (FA15p453) (FA16 p455)

3. A woman involved in an accidentcannotturn her headto the left alld has a right
shoulder droop. Which structure is damaged? (SUp41)

r 12.0 l
4. Fill in the diagram of acetylcholine synthesis and neurotransmission:

Cholinergic

:z
m
c
;:o
0
---1
;:o
)>
:z
Vl
s
Vl
Vl
0
Causes ____ paralysis :z

Causes _ _ __
4 Fates of ACh

5. Which enzymes and cofactors are required to convert phenylalanine to epinephrine?

[ i21 J
6. Fill in the diagram of norepinephrine synthesis and neurotransmission:

Noradrenergic

:z:
0
Ul
Ul
:2::
Ul
:z:
<(
00::
f--
0
00::
:::::J
w
:z:

E ession Ou1
Which three enzymes are required to convert phenylalanine to dopamine?

8. A patient develops paralysis after being bitten by a black widow spider. Is this a
spastic or flaccid paralysis? What other autonomic symptoms might be seen?

What substances inhibit the reuptake ofnorepinephrine?

10. What substances stimulate the release of norepinephrine from neurons?

r 122:
GPROTEIN SECOND MESSENGERS
GG: Chapter 8
Phys: Chapter 45
G protein second messengers (FA15 p248) (FA16 p242) (SU p173)

rm -Up Review
1. Damage to.which artery would produce each of the following presentations?
(FA15 p464-465) (FA16p467-468)
G)
• Eye looks down and out "U
:;;o
• Bilateral loss of lateral visual nelds 0
-I
• Broca or Wernicke aphasia m
:z:
Unilateral lower extremity sensOry and/or motor loss Vl
m
• Unilateral facial and upper extremity sensory and/or motor loss n
0
:z:
0
5
m
Vl
2. · Which diuretic is used in the treatment of idiopathic intracranial hypertension? Vl
m
:z:
G)
m
:;;o
Vl

3. When does implantation of the zygote take place? (FA15 p558) (FA16 p562)

r 1231
4. Signal transmission pathways

hormone binds growth factor binds


7-pass transmembrane receptor tyrosine kinase receptor

/
activated alpha
~
activated alpha activated
/ ~
adaptor protein
subunit of G-Protein subunit of G-Protein ---1.--.. phospholipase C
i
1
active adenylate cyclase inositol triphosphate (IP3)
A diacylglycerol (DAG)
Ras-activating protein

~
active Ras protein
~
(/")
~
LJ..J
!
increased levels
increased intracellular calcium ~
activation of protein kinase I
l9
z
LJ..J
of cyclic AMP ~ i
calcium bound calmodulin
(/")
(/")
LJ..J
~
0
z
!
activation of protein
kinase A
~
activation of
CaM-kinase
activation of protein
kinase C
activation of protein kinase II

~
activation of protein kinase Ill
0

~
u
LJ..J
(/")

z
LJ..J affected target proteins and/or gene regulatory proteins
f-
0
~
CL
l9

hormone binds growth factor binds


7-pass transmembrane receptor tyrosine kinase receptor

/ ~ / ~13
1 5 6
~
12 7 A 8
14
~
~
15
1 9 ~
16
3 ~ i
10
l
4
~
11 12
17
~
18

!
affected target proteins and/or gene regulatory proteins ~

r 12.4 1
End of Sess1on Qui z
5. Which G protein class does each of the following receptors stimulate?
• a1
• a2
• [31
• [32
• M1
• M2
• M3
• Dz
Cl
-o
A!
0
-1
m
6.. Outline the pathway by which stimulation of a G. receptor activates protein kinase A. z:
Ul
m
n
0
z
0
:s:
m
Ul
Ul
m
z
Cl
m
A!
Ul

r 12s 1
PHARMACOKINETICS
GG: Chapters 2-4
Enzyme kinetics (FA15 p242) (FA16 p236) (SU p9-10)
Enzyme inhibitors (FA15 p242) (FA16 p236) (SU p11)
Pharmacokinetics (FA15 p243) (FA16 p237) (SU p12, 395) (GG p28)
Bioavailability (FA15 p243) (FA16 p237)
Half-life (FA15 p243) (FA16 p237)

VJ
u 1. Label the following diagram of the circle of Willis:
f---
Ll.J
:z
~ A. ------------------------------
0
u
<(
B.
~
a::
<(
:r::
o_

D. ------~----~--~----------

H. ----------------------------~

L.

I 126 1
2. Label the following graphs of enzyme kinetics:

A-+ - - - - - - - - - - -- - - :..;
- -:,;-.;,..; - -~~:::====-
- -;..;.,-.-
~ E-- - - -

--o
I
)>
:::0
5
)>
n
0
;;;:s .
z
rn
---1
Substrate Concentration [5] n
c (J)

1
[S]

A
B.
c.
D. ,.
E.
F.
I~
G.

[ 127 J
3. Label the following plots as either competitive or noncompetitive inhibition:

(/)
u
f---
UJ
:z
~
0
u
<(
::?:
a::
<( Substrate Concentration [5]
:r::
Q_

1
[S]

A. -----------------------------
B.

C. --------------------------------

0. --------------------------------

r 12s l
4. What four pharmacokinetics equations are most important to know for Step 1?

--o
I
)>
AJ
s)>
n
0
;><;;
z
rn
---1
n
(J)

r 1291
End of Sess ion Qui z
A 60-kg man with status asthmaticus is being given an IV infusion of drug X at 60 mg/h.
The clearance of drug X is 2 L/h, and the volume of distribution is approximately 0.5 L/kg.
Forty-eight hours after administration was begun, the asthma attack is under control. At
this time, the concentration of drug X in his plasma is 20 mg/L.

5. What is the half-life of drug X in this patient?

l/)
u
f-
ll.J
z
::.::
0
u
<(
~
6. What loading dose could have been used to reach the target concentration
a::: of20 mg/L?
<(
:r:
0...

7. If the patient begins to show signs of toxicity, and the target dose is decreased to 10
mg/l, what would you do to get to this level?

8. Assuming 100% bioavailability, what oral ·dose of drug X would be necessary to


maintain the new target level?

9. If the patient has a kidney disease and the clearance is reduced to 1 L/h but Vd is
unchanged, what effect will this have on loading dose and maintenance dose?

r 13o 1
PHARMACODYNAMICS
GG: Chapter 3
Potency and efficacy (FA15 p245) (FA16 p239) (SU p12) (GG p45)
Antagonists (FA15 p246) (FA16 p240) (SU p13)
Therapeutic index (FA15 p246) (FA16 p240) (SU p12-13) (GG p73)

Wa~ . m-Op Rev1e.w


1. Which portion of the bra.in is supplied by the anterior cerebral.artery? Middle cerebral artery?
(FA15 p462) (FA16 p466)
"""0
:::r::
)>
:;o
:s
)>
n
0
2. What are the symptoms of nonnal pressure hydrocephalus? (FA15 p468) (FA16 p4/''1) 0
-<
z
)>
:s
n
Vl

3. A 76-year-old man who recently suffered a stroke is brought to the clinic by his wife.
She says that he now shaves only the right side of his face, and he eats off only the
right half of a plate. W.hen asked to draw a clock, he draws just the right half of the
clock face. A lesion in which area of the brain is most likely responsible for these
findings?

4. Potency and efficacy

Dose

r 131 1
5. Competitive antagonists

l/) OL_--~~~~~~~~~----~,------~,----
u 0 .1 1.0 10 100 1000
~
<(
z
>-
Agonist dose (log scale)
0
0
u
<(
:2:: 6. Noncompetitive antagonists
cr:
<(
:r: 100 ···················································-- --
0...

t j Effectof
antagonist

OL_--~~~~~------~----~,------.----
0.1 1.0 10 100 1000

Agonist dose (log scale)

7. Partial agonists

1
100 -~~
.....
u
£QJ
Decrea•ed efficacy

E
::l

·xraE 5 0 1 ························································································ A~·· · · · · · ·

r 132 1
nd ofS ess1on Gu1z
8. An investigator has developed two new preparations of insulin. Both preparations
cal'llowerthe blood glucose by a maximum of 100 mg/dl.lttakes 16 units of
insulin preparation Ato lower the blood glucose by 100 mg/dl. It takes 12 units of
preparation B to lower the blood glucose by 100mg/dLWhich insul.in preparation
has the higher efficacy, and which has the higher potency?

--o
:r:
)>
9. Your patient is taking experimental drug Z to lower his LDL cholesterol. When he is :;o
s)>
given the antagonist drug X, the.potency of drug Z isdecreased, butthe efficacy is n
unchanged. Doubling the dose of drug Z is shown to overcome the inhibition by drug 0
0
X. Is drug X a competitive or noncompetitive antagonist of drugZ? -<
z
)>
s
n
fJl

10. Drug A has a therapeutic index of 10:1, while Drug B has a therapeutic index of 2:1.
Which drug would be considered safer?

r 133 1
DRUG METABOLISM
GG: Chapter 6
Phase I vs. phase II metabolism (FA75 p244) (FA76 p238) (SU p74) (GG p724)
Cytochrome P450 (GG p727)
P450 interactions (FA75 p260) (FA76 p254) (SU p397) (GG p729)
Ethanol metabolism (FA75 p95) (FA76 p83) (SU p75) (R p477) (GG p630)

p Rev1ew
1. In which section of the brainstem is each of the cranial nerve nuclei located? (FA15
2':
(J) p474) (FA16 p478)
_j

0
m
<(
f--
LLJ
2':
l9
::::J
0::: 2. Which cranial nerve is responsible for each of the following actions? (FA15 p475) (FA16
0
p479) (SU p40-41)
• Eyelid opening
from anterior% of tongue
• Head turning
• M.usdes ()f mastication

chemo- and baroreceptors

Which regions ofthe brain compose the.lirnbic system? (FA15 p456)(FA16p4513) (SU

4. P450 inhibitors:
• c___________
• Ritonavir (protease inhibitors)
• Amiodarone
• Cimetidine
• Ketoconazole
• A___________
• Macrolides (erythromycin, clarithromycin)
• lsoni3zid
• G___________ __
• Omepr3zole
• s- - -- -- -------

f 1341
5. P450 inducers:
• Griseofulvin
• c__________________
• P__________________
• Barbiturates
• R___________________
• St. John's Wort
• c__________________

6. Ethanol metabolism 0
:;:o
c
Gl
Alcohol Acetaldehyde :s
dehydrogenase dehydrogenase rn
~
Ethanol ? ) • Acetaldehyde ---?"..,.-"'~~::,~--· Acetate OJ
0
NAO• NAOH NAO• NADH '
(J)
:s

End of Se ss1 on . .Q u1z


7. Which hepatic phase of metabolism is lost first by geriatric patients? Which phase is
mediated by cytochrome P~SO?

8. What medication inhibits acetaldehyde dehydrogenase?

9. .What medication inhibits alcohol dehydrogenase?

10. List 7 inducers and 11 inhibitors of cytochrome P450.

r 135 1
DRUG ELIMINATION
GG: Chapter 2
Drug elimination (FA15 p244) (FA16 p238) (SU p14)
Drug elimination and urinary pH (FA15 p244) (FA16 p238) (GG p18)
Drug suffixes (FA15 p261) (FA16 p255)

Warm.:.U Review
1.. · Complete the following diagram of the.cell cycle:
z
0
~
z
~

r 136 1
2. Drug elimination and urinary pH

HA
acidic drug anion

basic drug uncharged base


0
::0
c
• pKa (acid dissociation constant) is the pH at which the amount of the nonprotonated form (A- or B) C>
rn
=the amount of protonated form (HA or BH+) 's
• If pH is low (acidic environment) and less than pKa, there wi ll be more of the protonated form z
(HA or BH+) ~ basic drugs (BH+) get trapped ~
0
• If pH is high (basic environment) and higher than pKa, there will be more of the nonprotonated form z
(A- or B) ~acidic drugs (A) get trapped
• Treat acidic drug OD (i.e., sa licylates) with NaHCO, ~traps the acidic drug in the basic urine
• Treat basic drug 00 (i.e., amphetamines) with NH 4 CI ~traps the basic drug in the acidic urine

[ 137 J
3. Drug suffixes

Suffix Drug Class Examples


-atil Phosphodiesterase inhibitor Sildenafi l, tada lafil
-azepam Benzodiazepine Lorazepam , diazepam
-azine Phenothiazine Chlorpromazine, thioridazine, promethazine
-azolam Benzodiazepine Alprazolam , midazolam, triazolam
-barbital Barbiturate Phenobarbital, secobarbita l
-bendazole Benzimidazole Mebendazole, albendazole
-caine Local anesthetics Lidocaine, tetracaine
-chol Muscarinic agonist Bethanechol , carbachol
-cillin Penicillin Amoxicillin, methicillin
-conazole Imidazo le (antifungal) Fluconazole, ketoconazole
Non-depolarizing neuromuscular Atracurium, mivacurium
-curium
blocker
Non-depolarizing neuromuscular Vecuronium, rocuronium
-curonium
blocker
-cycline Protein synthesis inhibitor Tetracycline, doxycycline
Dihydropyridine calcium channel Nifedipine, am lodipine
-dipine
blocker
-dronate Bisphosphonate Alendronate, risedronate
-etine SSRI Fluoxetine, paroxetine, sertraline
-f1urane Inhaled anesthetics lsof1 urane, sevof1urane
-gliptin DPP-IV inhibitor Sitagliptin, saxagliptin
-glitazone Thiazolidinedione Pioglitazone, rosiglitazone
-ipramine TCA Clomipramine, imipramine
-mab Monoclonal antibody lnf1iximab, rituximab
-mustine Nitrosourea (nitrogen mustard) Lomustine
-navir Protease inhibitor Ritonavir, saquinavir
-olol ~-blocker Atenolol, metoprolol
-operidol Neuroleptic Haloperidol , droperidol
-phylline Methylxanthine Theophylline, aminophylline
-pril ACE inhibitor Enalapril, lisinopril
-prost Prostaglandi n analog Latanoprost, travoprost
-sartan Angiotensin receptor blocker Losartan , valsarta n
-statin HMG CoA reductase inhibitor Atorvastatin, simvastatin
-stigmine Anticholinesterase Neostigmine, physostigmine, rivastigmine
-terol s?-agonist Albuterol, salmeterol
-tidine H2 histamine antagonist Cimetidine, ranitidine
-triptan 5-HT, 8110 agonist Sumatriptan, rizatriptan
-triptylin e TCA Amitriptyline, nortriptyline
-tropin Pituitary hormone Somatotropin, thyrotropin (TSH)
-zosin G1-antagonist Prazosin , doxazosin

r 13s 1
nd of Sess1o n Ou 1z
4. Which meditation overdose can be treated with sodium bicarbonate?

Which medication overdose can be treated with ammonium chloride?

0
;;o
c
G)

6. To what class does each of the following drugs belong? m

• Methacholine
'
:s:
z
• Propranolol
~
• Pyridostigmine 0
• Formoterol z
• Terazosin

[ 139 J
DRUG SIDE EFFECTS
GG: Chapter 4

Drug side effects (FA15 p257-260) (FA16 p251 -254) (SU p396)

Warm - Up Rev1ew
1. What regulates the progression of the cell cycle from G2 phase to M phase? (FA15 p72)
(FA16 p58)

Vl
f-
u
LU
LL
LL
LU
LU
0
Vl
CJ
=:J
0::
0
2. The complexes of cyclin D + CDK4 and cyclin E + CDK2 assist in the progression of
the cell cycle into which phase?

3. The complexes of cyclin A + CDK2 and cyclin B + CDK1 assist in the progression of
the cell cycle into which phase?

r 1401
4. What drugs have the following potential side effects?
Flushing Hemolysis in G6PD deficiency
• • Isoniazid
Niacin •
• Adenosine • Primaquine
• Dihydropyridine calcium channel blockers • Aspirin (high-dose)
• Ibuprofen
Coronary vasospasm • Nitrofurantoin
• Cocaine • Dapsone
• Amphetamines • Fava beans
• • Naphthalene
0
• Ergotamine ;:o
c
Cl
Thrombosis (J)

Dilated cardiomyopathy • Oral contraceptives 0


rn
• Doxorubicin .,
rn
--n
• Daunorubicin Pulmonary fibrosis rn
n
• -l
(J)

Torsades de pointes • Bleomycin


• Potassium channel blockers (class Ill) • Busulfan
• Sodium channel blockers (class lA)
• Cough
• Haloperidol • ACE inhibitors
• Chloroquine
• Protease inhibitors Pseudomembranous colitis

Thrombocytopenia • Amoxici llin/ampicill in

• Cimetidine Hepatic necrosis
• Ha lothane

Agranulocytosis • Acetaminophen
• • Valproic acid
• Carbamazepine • Amanita phalloides
• Colchicine
• Propylthiouracil Hepatitis
• Isoniazid
• Methimazole
• Dapsone
Tertiary adrenal insufficiency
• Corticosteroid withdrawal
Aplastic anemia
• Chloramphenicol
Gynecomastia
• Benzene
• NSAIDs • Digoxin
• Propylthiouracil Estrogens
Methimazole • Cimetidine
• Alcoho l (chronic use)
Gray baby syndrome
Ketoconazole

• Marijuana (THC)

r 141 1
4. (continued) Tendinitis/cartilage damage
Hot flashes • Fluoroquinolones

• Clomiphene • Interstitial nephritis
• NSAIDs
Hypothyroidism • Furosemide
• •
• Lithium
• Sulfonamides Diabetes insipidus
• Lithium
Gout • Demeclocycline
U}
r- • Diuretics (loop and thiazide)
u
LLJ
LL • Niacin Fanconi syndrome
LL
LLJ
LLJ
• Cyclosporine • Tetracycline (expired)
0
Ul
• Pyrazinamide
lCJ Hemorrhagic cystitis
~
0:::
0 Gingival hyperplasia • Cyclophosphamide
• • Ifosfamide
• Verapamil
Parkinson-like syndrome
Osteoporosis •
• Corticosteroids • Antipsychotics
• Heparin • Reserpine

Photosensitivity Cinchonism
• • Quinidine
• Amiodarone • Quinine
• Tetracycline
Seizures
Stevens-Johnson syndrome •
• Carbamazepine • lmipenem/cilastatin
• Lamotrigine • Isoniazid
• • Tramadol
• Phenytoin • Metoclopramide
• Phenobarbital • Benzodiazepine/alcohol withdrawal
• Penicillin
• Ethosuxim ide Tardive dyskinesia
• Antipsychotics
• RASH
• Sulfa drugs Metoclopramide

Drug-induced lupus Disulfiram-like reaction


• Sulfonamides •
• • Certain cephalosporins
• Isoniazid • Procarbazine
• Procainamide
• First-generation sulfonylureas
• Phenytoin Disulfiram

r 14? 1
4. (continued)
Nephrotoxicity + Neurotoxicity Anticholinergic effects
• • Atropine
• Cisplatin • Tricyclic antidepressants
• Polymixins • H1 blockers
• Low-potency neuroleptics
Nephrotoxicity + Ototoxicity
• Aminoglycosides

• Loop diuretics
• Cisplatin
CJ
:::D
c
G)
{J)

CJ
m

End ess1 on Quiz ,,


m

m
()
5. What medication causes cardiotoxicity and bone marrow suppression? --1
{J)

6. All African-American man who goes to Africa develops anemia after taking
prpphylactic medicine for primary disease prevention. In what enzyme is this patient
deficient?

7. A 65-year-old male patienttaking multiple medications presents with gynecomastia.


Which medications could be responsible for this patient's gynecomastia?

8. A patient presents with tinnitus, dizziness, headaches, and Gl distress. What drug is
causingthese symptoms?

9. What medications are known for causing drug-induced lupus?

r 14l 1
AITIDOTES
GG: Chapter 4

Antidotes (FA15 p257) (FA16 p251) (SU p398)

Warm -U p Review
1. What information is communicated at each of the following nuclei? (FA15 p476) (FA16
p479)
• Nucleus solitarius
U) • Nucleus ambiguus
LLJ
f--- • Dorsal motor nucleus
0
0
f-
z
<(

r 144 1
End of Sess1on Qu1z
2. What are the antidotes to each of the folloyvingtoxins?

Acetarninophen '

Salicy'lates

Amphetamines
,,, ,,,,,,

.. )>
Anticholinesterases, organophosphates z
.. ---1
CJ
0
Anti muscarinic, anticholinergic agents ---1
.,,
' ',,,' '.',' '

'
"'
(J)

,,,,,, Digoxin
,''
'<
Iron
'''•
.,,

Lead
' ..
. .
Arsenic, mercury, gold

Copper, arsenic, gold ,,,


,,,,,

C::yanide

Methemoglobin
'

Carbon monoxide ,,
,,,. ,,, ..

Methanol, ethylene glycol (antifreeze)

Opioids

Benzodiazepines
'
',

Tricyclic antidepressants

.,.
Heparin
'
,.,,, ..

Warfarin

t-PA, streptokinase

Theophylline ,y

r 145 1
IT IS Dl FFICU LT TO

CARDIOVASCULAR OVERSTATE
IMPORTANCE OF
UNDERSTANDING THE
1 Embryology
CARDIOVASCULAR
2 Congenital Heart Defects SYSTEM FOR CENSING
EXAMS FROM COMMON"
3 Cardiac Output
IN/CAL PROBLEMS
4 Heart Failure (SUCH AS HYPERTENSION
AND ATHEROSCLEROSIS)
5 Treatment of CH F
TO PUZZLING
6 Ed ema and Shock ELECTROPHYSIOLOGICA
DISORDERS, AND
7 Cardiac Cycle
FROM CONGESTIVE
8 Diastolic Heart Murmurs HEARTFAJLU RE TO
Systolic Heart Murmurs MURMURSAND OTHER
9
AUSCULTATORY FINDINGS,
10 Electrophysiology THE CARDIOVASCULAR.
11 Ant1arrhyth miCS SYSTEM IS ONE OF THE
MOST HEAVILY-TESTED
12 Electroca rd 1ogra phy ORGAN SYSTEMS
13 Arrhythmias
14 Regulation of BP
15 Hypertension
16 Anti hypertensives
17 Atherosclerosis
18 Antianginal Therapy and
Lipid - Lowering Agents
19 Myocardial Infarction part 1
20 Myocardial Infarction part 2
21 Cardiomyopathies and Endocarditis
22 Other Card1ac Pathology

23 Vascu lit1s
24 Other Vascular Diseases
EMBRYOLOGY
R: Chapter 12
H: Chapter 224
Phys: Chapters 23, 83
Cardiac embryology (FA15 p268) (FA16 p262) (SU p61-62) (R p532, Fig. 12-3) (H p1809)
Atrial septum (FA15 p268) (FA16 p262) (SU p61-62)
Ventricular septum (FA15 p269) (FA16 p263) (SU p61-62)
Cardiac embryology- adult derivatives (FA15 p268) (FA16 p262) (SU p62)
Aortic arch derivatives (FA15 p564) (FA16 p568) (SU p62)
Fetal circulation (FA15 p271) (FA16 p264) (SU p63)

Warrn -LJ p Review


Which intracranial tumor is associated with each of the fc:)llowing statements? (FA15 p492-493)
(FA16 p496-497) (SUp50)
• Polycythemia
m
• Neurofibromatosis II s:
CP
• Hyperprolactinemia ~ galactorrhea, amenorrhea, anovulation :;;o
-<
• Psammoma bodies 0
• Perivascular pseudorosettes '
0
GJ
-<

Which cranial nerve is responsible for the motor innervation of the tongue?

3. A 28-year-old woman is brought to the emergency room following a motor vehicle


accident..Her passenger says thatJmmediately after impact;.she appeared fine;
however, five minutes later, she lost consciousness. A CT scan of the head shows an
intracranial hemorrhage that does not cross suture lines. Which bone and vessel were
most likely injured in theaccident?.(FA15 p466) (FA16 p469)(SU p29)

4. Outline the pathway by which the heart tube forms the atria of the four-chambered heart.
• Tube grows, elongates, and fo lds into an S-shape--+ atrial chamber lies posteriorly inS, and
ventricular chamber lies anteriorly in S
• Atrial chamber grows and incorporates superior vena cava and pulmonary vein
• Septum primum form s
• Septum secundum forms incompletely (leaving a hole called the foramen ovale)
• Ce ll death in septum primum forms a hol e called ostium secundum

!49 J
5. What divides the right and left atria?

6. How is blood shunted from the right atrium to the left atrium in the embryo?
Through the foramen ovale (of the septum secundum) and ostium secundum
(of the septum primum)

7. What are the three possible causes of an atrial septal defect?

>-
l.'J
0
_j

0
>-
~ 8. Which structure grows to close the opening/canal between the atrial chamber and ventricular
~ chamber into two smaller openings?

9. Which genetic abnormality is commonly associated with endocardial cushion defects?

10. Outline the pathway by which the ventricles and their outflow tracts are separated.
• Ventricular chamber lays anteriorly in the S-shaped heart tube ~ muscular ventricuiar septum
begins to divide the ventricles
• Truncoconical swellings (ridges) inside the truncus arteriosus meet and fuse in a 180° turn to form
the spiral septum (AKA aorticopulmonary septum)
• Inferior portion of spiral septum meets with muscular ventricular septum to divide the ventricles
and form the aorta and pulmonary arteries

11. Name six different truncoconical (spiral) septum defects.


• Tetralogy of Fallot
• Persistent truncus arteriosus
• Transposition of the great vessels (RV ~ aorta, LV ~ PA)
• Dextrocardia
• Ventricular septal defect (VSD)
• Fenestrae

[ i50 J
12. Describe how the ventricles are remodeled in order to form the atrioventricular valves.
• Myocardium erodes ~ventricles en large as a resu l t~ residual mesodermal tissue becomes fibrous
and forms the chordae tendineae
• Formation of papillary muscles and AV va lves

En ession Quiz
13. Which embryologic structure of the heartgives rise to each of the following adult
structures? -
• _A.sc~nding a9rta and pulmonary trunk
• Coronary
• svc
• Smooth parts of the and right ventricles
• Smooth partof the right atrium
• Trabeculated parts of left and right atria
• Trabeculated .partsof the left and right ventricles

14. Which structure divides the truncus arteriosus into the aortic and pulmonary trunks? What
is the cellular origin of this structure?

15. Which fetal vessel has the highest oxygenation?

16. What causes the ductus arteriosus to close?

17. What causes the foramen ovale to dose?

[ ! 51 l
CONGENITAL HEART DEFECTS
R: Chapter 12
H: Chapters 236, 238
Left-to-right shunts (FA15 p289) (FA16 p283) (SU p64)
Ventricular septal defect
- Atrial septal defect
- Patent ductus arteriosus
- Eisenmenger syndrome
Coarctation of the aorta (FA15 p289) (FA16 p283) (SU p64)
Right-to-left shunts (FA15 p288) (FA16 p282) (SU p64)
Persistent truncus arteriosus
Transposition of the great vessels
- Tricuspid atresia
- Ebstein anomaly
- Tetralogy of Fallot
Total anomalous pulmonary venous return
Conditions associated with cardiac malformations (FA15 p290) (FA16 p284)

Ul
f--
u
LLJ
LL
LLJ
0
f--
r:Y: Which cell types are richln smooth endoplasmic reticulum? (FA15 p72)(FA16 p58)
<t:
LLJ
:r::
__J

;::;
z
LLJ
(_:J
z
0
u 2. What is the tl"eatmentfor heparin overdose? (FA15 p257) (FA16 p251)

What is the treatmentforwarfarin overdose? (FA15 p257) (FA16 p251)

[ 152 J
4. Ventricular septal defect

n
0
5. Atrial septal defect z
Cl
rn
z
:r;!
,--
:r:
rn
J>
::u
~
0
rn
"Tl
rn
n
~
Vl

6. Patent ductus arteriosus

r 153 1
7. Coarctation of the aorta

Descending aorta

Coarctation

Vl
f-
~
LL
8. Truncus arteriosus
w
0
f---
0:::
<(
w
:r::
_j

~
z
w
l'J
z
0
u

9. Transposition of the great vessels

Aorta Pulmonary trunk

Left ventricle

r 154 1
10. Tricuspid atresia

n
0
:z
Cl
m
11. Ebstein anomaly :z
);!
• Tricuspid leaflets are displaced into right ventricle, hypoplastic right ventricle, tricuspid regurgitation r
I
or stenosis m
)>
• 80% have a patent foramen ova le with a right-to-l eft shunt ;;o
-l
0
• Dilated right atrium causing increased risk of SVT and WPW m
-n
• Physical exam: widely split 52 , tricuspid regurgitation m
n
-l
• Associated with maternal - - - -- -- -- - - -- -- -- - - -- -- -- - - - use l/)

I l.'i.S 1
12. Tetralogy of Fallot

U)
f-
u
LU
LL
LU
0 13. Total anomalous pulmonary venous return
f-
er:
<(
w
:::r::
_j

~
z
LU
l.')
z
0
u

r 156 l
End of Sess1 on Quiz
14. What arethe components of the tetralogy of Fallot?

15. Explain how the great vessels are attached in transposition ofthe great vessels.

n
0
z
Cl
m
16. A45-year-old·man presents with a blood pressure of 160/90 mm Hg on the right arm. and z
170/92 111m Hg on the left arm. There are ho palpable pulses in the feetor ankles. What );!
,--
problem does this patient most likely have? I
m
)>
:::0
--I
0
m
th -n
m
n
17. What heart defect is as~.ociated with each of the following disorders? --I
Vl
• Chromosome 22q11 deletions
• Down syndrome
• Congenital rubell3
• TUrner syndrome

RAP! o -F! RE FACTS


Continuous, machinery-like heart murmur

"Boot-shaped heart"

Rib notching

Most common congenita l cardiac anomaly

Most common congenital cause of early cyanosis

r 1sn
CARDIAC OUT~UT
Phys: Chapters 9, 20, 21
Cardiac output (FA15 p272) (FA16 p266) (SU p65, 68)
Preload and afterload (FA15 p273) (FA16 p267) (SU p63)
Cardiac output variables (FA15 p273) (FA16 p267) (SU p65, 68)
Starling curve (FA15 p274) (FA16 p268) (SU p146) (Phys p110)
Ejection fraction (FA15 p273) (FA16 p267) (Phys p106)
Resistance, pressure, flow (FA15 p274) (FA16 p268) (SU p65, 78) (Phys p158)
Cardiac and vascular function curves (FA15 p275) (FA16 p269) (SU p66) (Phys p168)

Warm-Up Rev1ew
1. A farmer suffering from organophosphate poisoning continues to have significant
symptoms despite administration of atropine. What additional drug should this
patient receive? (FA15 p250)(FA16 p244)
f--
:::J
o_
f--
6 2. Which enzymes are involved in the catabolism of norepinephrine?
u
<(
0
a:
<(
u

3. What are the various clinical applications of atropine? (FA15 p251) (FA16 p245)

4. Starling curve

~~~~~~~ ------------ - -­
\~e
~+.e._~

Walking

Rest
-- ------ -

Ventricular End-Diastolic Volume (Preload)

[ !58]
5. Cardiac and vascular function curves

-- +inotropic
- - - - - - - - • Increased
contractility

Cardiac output
Starling
relationship

,.,.--------- + afterload

Mean
systemic
pressure

Right atrial pressure (mm Hg)


(End-diastolic volume (L))
n)>
:;o
0
)>

En d ..of Sess ion Ou1z n


0
c
6. What is the pulse pressure in a patient with a systolic blood pressure of 150 mm Hg and a ---1
u
mean arterial pressure (MAP) of 90 mm Hg? c
---1

7. What is the basic equation for cardiac.output (CO)? What is the Fick principle?

8. How can the myocardial oxygen demand be decreased in circumstances where the heart is
ischemic?

9. What can make the stroke volume (SV) increase for a given preload?

10 What is the cardiac ejection fraction (EF)?

r 1'19 1
HEART FAILURE
Phys: Chapter 22
R: Chapter 14
H: Chapter 234
Cardiac pressures (FA15 p286) (FA16 p280)
CHF physiology (FA15 p297) (FA16 p292) (SU p83-85) (R p526)
Renin-angiotensin-aldosterone system (FA15 p534) (FA16 p540) (SU p65) (Phys p260)
CHF signs and symptoms (FA15 p297) (FA16 p292) (SU p84)
BNP (FA15 p285) (FA16 p279)

Warm-
1. What drugs are known to inhibit cytochrome P450? (FA15 p260)(FA16p2S4)

<(
LL

r-
et::
<(
w
:r::

2. What ls a disulfiram-like reaction? (FA15 p95, 260) (FA16 p83,

3. i.What drugs cause a disulfiram-like reaction? (FA15p95, 260)(FA16

I 160 -~
4. CHF compensation

:r:
rn
)>
;;o
-j
.,
d of s ·on Q z )>
,-
c
5. What are the signs of right-sided heart failure? ;;o
rn

6. What are the signs of left-sided heart failure?

7. How does poor cardiac output result in an increase in aldosterone?

( 161 J
TREATMENT OF CHF
GG: Chapters 27, 28
H: Chapter 234
Treatment of chronic CH F (FA15 p297) (FA16 p292) (SU p84)
Digoxin (FA15 p75, 307) (FA16 p61, 301) (SU p84) (Phys p259) (GG p838)
Treatment of acute CHF

Warm-Up Rev1ew
1. What are the symptoms of organophosphate poisoning? (FA15 p250) (FA16 p244)

LL
:r:
u
LL
0
f-
z
UJ
2 2. What are the symptoms of atropine overdose? (FA15 p251) (FA16 p245)
~
UJ
a:::
f-

3. How does hemicholinium inhibit the transport of choline into the nerve terminal?

4. Patient Profile: The Anaerobic Bodybuilder


A 76-year-old Caucasian man who is a competitive bodybuilder comes to the physician complain ing
that he has started "getting anaerobic" very quickly when exercising. Upon further questioning, you
determine that he is experiencing shortness of breath after only 2-3 minutes of exercise. Physical
examination shows mild pedal edema. His lungs are clear. An echocardiogram shows an estimated left
ventricular ejection fraction of 25% without valvular dysfunction. What are the mechanisms of action
of the drug classes proven to reduce mortality in this patient?

[ 162 J
5. Which classes of drugs are proven to improve survival and reduce hospitalizations in chronic CHF?
Which are used only for symptomatic relief?

Improved Survival Symptomatic Relief

6. Cardiac glycosides (digoxin)


lon Exchange ATPase
-I
;;o
m
~
sm
z
-I
0
--n
n
::r:
--n

;·@*13=
IW.dt.f ···*'·a ·*UP
*tttit·M --kw ·· tnt= .·
~

nd of Sess1on Ou1 z
7. What medications are used to treat chronic heart failure?

8. . . What medications are used to treat acute heart failure?

9. What is the mechanism of action of the cardiac glycosides (e.g., digoxin)?

r 163 1
EDEMA AND SHOCK
Phys: Chapters 16, 24, 25
R: Chapters 4, 11
Fluid exchange in the capillaries (FA15 p287) (FA16 p281) (Phys p184) (R p113-115)
Shock (FA15 p297) (FA16 p292) (R p131-134)
Locations of central lines (H 2218)
Femoral region anatomy (FA15 p350) (FA16 p347) (COA p204, Fig 2.15)

Wa rm- Up Rev1ew
1. What drugs have each of the following side effects? (FA15 p257-260) (FA16 p251-254)(SU p396)
• Agranulocytosis

• Osteoporosis

~ • Pulmonary fibrosis
u
0
I
if)

0
• Gynecomastia
z:
<(
<(
~
• Photosensitivity
l.J..J
0
LLJ

• Drug-induced lupus

2. Illustrate the Starling forces affecting capillaries.

r 164 1
3. How do systemic vascular resistance (SVR) and cardiac output (CO) change in each of the following
types of shock?
SVR co Rx

Hypovolemic

Cardiogenic

Sepsis/anaphylaxis

Neurogenic

/"Tl
0
rn
4. Locations of central lines: 3:
)>

• )>
z:
0
Pros: easiest site with the least risk (./)
:r:
Cons: cannot stay in place more than 5-7 days due to risk of infection 0
n
• ;:><;

Pros: good landmarks for easy placement. can remain in place 3 -4 weeks, less discomfort
Cons: highest risk of pneumothorax; not a great choice in patients with COPD (barrel-chested) or
lung tumors

Pros: good landmarks for easy placement, can remain in place 3-4 weeks
Cons: more discomfort, risk of puncturing the carotid or causing pneumothorax

[ 165]
En d of Sess 1on Qu1 z
5. How does each of the following circumstances impact the Starling forces of fluid movement
through capillaries?
• Heart failure
• Liver failure
· • Oliguric renal failure
• Infections and toxins
• Nephrotic syndrome
• Lymphatic blockage
• Burns
• Diuretic administration
• IV infusion of albumin or clotting factors
• Venous insufficiency

:,<::
u
0 6. How do the vascular resistance and stroke volume differ in hypovolemic shock compared to
:r:
(.f)

0
septic shock?
z:
<(
Vascular resistance Stroke volume
<( '

2:
w Hypovolemic shock
0
w

Septic shock

7. ,What are some of the causes of cardiogenic shock?

8. How is the skin of a patient different in cardiogenic shock compared to septic shock?

[ 166 J
CARDIAC CYCLE
Phys: Chapters 9, 20
Cardiac cycle (FA15 p276) (FA16 p270) (SU p66) (Phys p108, Fig. 9.8)
Pressure-volume loop variations (FA15 p276) (FA16 p270) (SU p66) (Phys p106-109)
Heart sounds (FA15 p276) (FA16 p270)
Splitting (FA15 p277) (FA16 p271)

Wa rm -Up Rev1 ew
1; Two drops of drug X are instilled into a patient's left eye. In the dark, bothpupi.ls
are dilated. In the light, the right pupil becomes miotic while the left .pupil remains
mydriatic. What isdrugX? (FA1Sp251) (FA16 p245)

n
)>
:;o
0
)>
n
n
-<
n
r-
m
2. What isthe antidote for poisoning with each of the following? (FA15 p257! (FA16 p251)
• Copper
• Mercury
• tPA, streptokinase
• Digoxin

f 1671
3. Pressure-volume loop (normal)

140

-en
J:
Aortic valve
closes

E 100

--
E
lsovolumetric
relaxation
lsovolumetric
contraction

20

w
_j
LJ Volume (ml)
>-
LJ
LJ
<t
0
cr:
<t
LJ

4. Pressure-volume loop (variations)

t Afterload (e.g. hypertension)


140
t Aortic pressure
-en
J:
+sv
tEsv
E 10 0

-E
<11
lo-
:I
VI
VI
<11 60
lo-
c..

20

Volume

r 1681
5. Cardiac cycle

Diastasis

120

'01
100 ..... "' ...... Aortic pressure
:I:
E 80
!
..
Q1

:I
60

.
Ill
Ill
Q1

0..
40
AV

20 Atrial pressure
(venous pulse)
r;;;,.;;;;.;;;.;;;;.;;;,;; Ventricular pressure
:r
!
Q1
Ventricular volume
n
E )>
:I ;;cJ

g 0
)>
n
n
-<
n
r
m

i69l
End of Session Qu 1z
6. When does isovolumetric contraction take place?

7. How does an increase in afterload affect the stroke volume of the heart assuming
contractility remains the same?

w
__]
u
>-
u
u
<(
8. What impact does an increase in contractility have on stroke volume assuming
0 preload and afterload .remain constant?
[)"
<(
u

9. Which heart sound is associated with dilated congestive heart failure? Which heart sound is
associated with chronic hypertension?

10. .What gives rise to the jug.ularvenous a, c, and v waves?

11. Where does the QRS complex fall in relation to valvular dynamics?

[ t70 J
DIASTOLIC HEART MURMURS
H: Chapter 227
Phys: Chapter 23
Cardiac auscultation (FA15 p278) (FA16 p272) (Phys p265)
Murmur physiology
Bedside maneuvers (FA15 p278) (FA16 p272)
Aortic regurgitation (FA15 p279) (FA16 p273) (SU p89)
Mitral stenosis (FA15 p279) (FA16 p273) (S U p89)
Patent ductus arteriosus (FA15 p279) (FA16 p273) (SU p64)

Wa -Up 1ew
1. What drug category has each of the following endings? (FA15 p261) (FA16 p255)
• -sartan
• ~stigmine
• -curium/-curonium
0
• -glitazone )>
• -dipine {J)
---1
0
• -azepam
• -azole
'
n
:r:
• -caine en
)>
;;o
• -cycline ---1
· • -navir 5
c
;;o
5
c
;;o
(/)

2. Which heart sounds are considered benign when there is no evidence of disease?
• Spl it \
• Split 52 on inspiration
• 53 heart sound in a patient < 40 yo
• Early, quiet systolic murmur

I 171 l
3. Diastolic heart murmurs

Aortic regurgitation

51 52 51

Mitral stenosis

Vl
~
:=:>
~
~
:=:>
::?:
f-
0:::
<t
LLJ
:r: 51 52 51
u
_j Opening snap (OS)
0
f-
Vl
<t
0

Patent ductus arteriosus

51 52 51 52

r 1121
n Sess1 on Quiz
4. Throughwhichheartvalves should blood be flowing during.systole? During diastole?

What are the diastolic murmurs?

6. Which murmurs are heard.bestir'l the left lateral decubitus position? 0


)>
Ul
-1
0
'
n
::c
rn
)>
;;o
-1
:s
c
:;o
:s
c
;;o
Ul

RAPID - FIRE FACTS


Bounding pulses, head bobbing, diastolic murmur

Continuous, machine-like murmur

[ 173 J
SYSTOLIC HEART MURMURS
H: Chapter 227
Phys: Chapter 23

Aortic stenosis (FA15 p279) (FA16 p273) (SU p89)


Mitral regurgitation (FA15 p279) (FA16 p273) (SU p89)
Mitral valve prolapse (FA15 p279) (FA16 p273) (SU p89)
Tricuspid regurgitation (FA15 p279) (FA16 p273)
Ventricular septal defect (FA15 p279) (FA16 p273) (SU p89)

ev1
1. Outline the pathway ofconverting phenylalanine to epinephrine.. (FA15 p107) (FA16 p95)

Vl
a::
:::J
~
a::
:::J
~
f---
a::
<t
LLJ
:r::
u
_j

0
f---
Vl
>-
Vl
r~

2. What are the most common causes of aortic valvular stenosis?

[ i74]
3. Systolic heart murmurs

Mitral Regurgitation

S1 S2

Mitral Valve Prolapse

IJ)
-<
IJ)
---1
0
'n
I
m
)>
51 S2 ;;o
Midsystolic ---1
dick(M() :s:
c
;;o
:s:
c
Aortic Stenosis ;;o
IJ)

51 S2

Ejection
dick{EC)

[ 175 J
End of Sess1on Ou1z
4. Which heart murmur is associated with weak pulses?

5. Which pathology is associated with each of the following murmurs?


Crescendo-decrescendo systolic murmur best heard
in the 2nd-3rd right interspace close to the sternum

Early diastolic decrescendo murmur heard best along


the left sternal border, with BP of 160/55 mm Hg

Late diastolic decrescendo murmur heard best along


the lower left sternal border
Vl
0:::
Holosystolic murmur best heard at the apex and often
:::::J radiates to the left axilla
2:
0:::
:::::J Late systolic murmur usually preceded by a mid-
~
f-
systolic click
0:::
<r:
LJ..J Crescendo-decrescendo systolic murmur best heard
:::r::
u in the 2nd-3rd left interspaces close to the sternum
__.J

0 Holosystolic murmur best heard along the left lower


1-
Vl
>- sternal border
Vl
Rumbling, late diastolic murmur with an opening snap,
loudest in the 5th interspace in the midclavicular line

Continuous, machine-like murmur (in systole and


diastole)

[ 176 J
ELECTROPHYSIOLOGY
Phys: Chapters 9, 10
H: Chapters 228, 258
GG: Chapter 29

Heart physiology (FA15 p280-281) (FA16 p274-275) (SU p66)


Ventricular action potential (FA15 p280) (FA16 p274) (SU p66)
Pacemaker action potential (FA15 p281) (FA16 p275) (SU p66)

Wa Up Re\/r ew
1. A gardener has shortness ofbreath, salivation, miosis, and diarrheC\. What is the most
likely diagnosis? What is the underlying mechanisrn?(FA15 p250) (FA16 p244)

rT1
.--
rn
n
--1
AJ
0
-u
::r:
-<
2. What is the mechanism of action of N-acetylcysteine when given asan antidote for Ul
0
acetaminophen overdose? (EA15 p444) (EA16 p446) .--
0
Gl
-<

3. What is the equation for half-life? (FA15 p243) (EA16 p237)

r !771
>-
l'J
0
_j

0
(./)
>-
:r:
o_
0
0:::
f-
u
LLJ
_j
LLJ

r 178 1
ANTIARRHYTHMIC$
GG: Chapter 29
Antiarrhythmics (FA15 p308-310) (FA16 p302-304)
Na' channel blockers (FA15 p308) (FA16 p302) (SU p71) (GG p840)
{3-blockers (FA15 p309) (FA16 p303) (SU p72) (GGp830)
K' channel blockers (FA15 p309) (FA16 p303) (SU p72) (GG p843)
Ca,. channel blockers (FA15 p310) (FA16 p304) (SU p72) (GG p830)
Adenosine (FA15 p310) (FA16 p304) (SU p71, 72) (GG p842)
Potassium (SU p72)
Magnesium (FA15 p310) (FA16 p304)

Warm -Up R.ev1ew


t. · What are the four important pharmacokinetic equations? (FA15p243) {FA16 p237)

J>
z
-i
J>
;;o
;;o
I
-<
-i
:r:
:s:
n
IJl

[ 179 J
2. Na+ channel blockers
• Decrease slope of phase 0 in myocytes
• Increase effective refractory period and QT interval
• "Double Quarter Pounder, Mayo, Lettuce, Tomato, Fries Please"
• 1A: disopyramide, quinidine, procainamid e
• 1B: mexiletine, lidocaine, tocainide
• 1C: flecainide, propafenone

3. P-blockers
• Decrease cAMP, decrease calcium currents
• Decrease the slope of phase 4 in pacemaker cells
• Propranolol, esmolol, metoprolol, atenolol , timolol
(f)
u
2:
:r:
1-
>-
:r:
a: 4. K• channel blockers
a:
<(
1- • Work at phase 3
z
<( Increase effective refractory period and QT interval
• Sotalol, ibutilide, bretylium, dofetilide, amiodarone

5. Ca2+channel blockers
• Decrease slope of phase 0 in pacemaker cells
• Increase effective refractory period
• Verapamil, diltiazem

6. Adenosine
Increases potassium efflux
• Prevents depolarization
• Used for supraventricular tachycardia

r 1so 1
d of Sessio n Qu1z
7. What is the mechanism of action of each class of antiarrhythmic?

)>
z
---1
)>
;;u
;;u
Which antiarrhythmic has the side effect of cinchonism? :r:
-<
---1
:r:
3:::
n
Vl

9. What are the potential side effects of amiodarone use?

10. What is the mechanism of action of adenosine as an antiarrhythmic?

11. To which class of antiarrhythmics does each of the following agents belong?
• Sotalol
• Propranolol
• Bretylium
• Quinidine
• Verapamil
• Procainamide
• Lidocaine
• Diltiazem

[ 181 J
ELECTROCARDIOGRAPHY
~\
Phys: Chapters 10, 11
H: Chapter 232
Electrocardiogram (FA15 p282) (FA16 p276)
Axis

p Review
1. What is the mechanism ofaction offomepizole? (FA15 p95) (FA16p83)

2. Whateffects will a noncompetitive antagonist have on Vmax a.nd Km? (FA15p242)


(FA16 p236)
>-
:r::
Q_
<(
0::
l:J
0
0
0::
protein class does each of the following receptors stimulate? (FA15p248)
<(
u
0
0::
f--
u
w
_j
w

4. ECGaxis

The net e lectrical signal (cardiac axis) will fall


within the shaded region in normal cardiac physiology.

[ 182
5. Common conditions that result in axis deviation:
• Left axis deviation
Inferior wall myocardial infarction
Left anterior fascicu lar block
Left ventricular hypertrophy (sometimes)
Left bund le branch block (LBBB)
• Right axis deviation
Right ventricu lar hypertrophy
- Acute right heart strain (e.g., massive pulmonary embolism)
Left posterior fascicular block
Right bundle branch block (RBBB)

End of Sess1on Ou1z m


r-
m
6. How does the cause of a narrow QRS complex differ from the cause .of a wide QRS n
-i
complex? ;;o
0
n
)>
;;o
0
0
Cl
;;o
)>
-o
I
-<

7. What is theECG axis given the QRS deflections in each of the following scenarios?
• Positive in lead I, positive in lead II
• Positive in lead I, negative in lead Ill
• Negative in lead I, positive in lead Ill
• Positive in)ead I, negative inaVR

8. How does hyperkalemia affect the shape ofT waves?

9. What are the most common causes of left axis and right axis deviation?

r 1s3 1
ARRHYTHMIAS
Phys: Chapters 10, 77, 23
H: Chapters 23, 232

Atrial fibrillation (FA15 p284) (FA16 p278) (SU p69)


Atrial flutter (FA15 p284) (FA16 p278) (SU p69)
AV block (FA15 p284-285) (FA16 p278) (SU p70)
Wolff-Parkinson-White (WPW) syndrome (FA15 p283) (FA16 p277) (SU p69)
Ventricular arrhythmias (H p1890)
Torsades de pointes (FA15 p283) (FA16 p277) (SU p68-69) (H p1891, Fig 233-9)
Ventricular fibrillation (FA15 p284) (FA16 p278) (SU p69) (Phys p149-151)

Warm-Up Review
1. Which adult structures are derived from the 3rd, 4th, and 6th aortic arches?
(FA15 p564) (FA16 p568) (SU p62)
<./)
<(

2:
::r:
I-
>-
::r:
0:::
0:::
<(

2. What agents stimulate the release of norepinephrine from the presynaptic bouton?
(FA15 p249) (FA16p243)

3. What agents potentiate the action of norepinephrine by inhibiting its reuptake? (FA15 p249)
(FA16p243)

4. Atrial fibrillation

r 184 1
5. Atrial flutter

6. First degree AV block

)>
AJ
AJ
I
-<
---1
I
:s:
)>
V1

8. Second degree (Mobitz II) AV block

r 1ss 1
9. Third degree AV block

10. Wolff-Parkinson-White (WPW)

11. Paroxysmal supraventricular tachycardia (PSVT)

12. Premature ventricular contractions (PVCs)


""""" " "~~»~"~"~~"~, »~-~

[ i 86 J
13. Ventricular escape rhythm

14. Junctional escape rhythm

)>
;:o
;:o
:r:
-<
-j
:r:
s
}>
Vl

15. Monomorphic ventricular tachycardia


~ ~
~ ~
I i! I
i ~
I I, .1,
I
1--J-

l,J tft-l.J: _,_"~

~ v~ -
I !.i w' J J
I
"t [i
1
1 1\
i
v
• r-..
v
'
w
I
1
;
r...l
v
.. -
jr'
1\/i ;
\.
rv ~
'
,~

'
\l
v v
l \ r·
v
;
\i
~

16. Torsades de pointes

[ 187]
17. Drugs that prolong the QT interval
• Anti-infectives: macrolides, chloroquine
• Antipsychotics: haloperidol, risperidone
• Methadone
• Anti-HIV protease inhibitors (-navirs)
+
• Antiarrhythmics: class lA (quinidine) and class Ill (K channel blockers such as sotalol)

nd e 1on z
19.·· ·What is the initial treatment forventricular fibrillation?

20. What are the major characteristics. of atrial fibrillation?

21. What is the hallmark of a third degree AV. block?

22. •What drugs are known prolong the QT interval, increasing the likelihood of torsades de
pointes in those atrisk?

23. What are. the two different types of second degree AV block? How do they differ?

24. Why is warfarin anticoagulation important in patients with chronic atrial fibrillation?

[ 188 J
REGULATION OF BP
Phys: Chapters 18, 19, 20
Regulation of BP (FA15 p534) (FA16 p540)
Baroreceptors (FA15 p286) (FA16 p279)
Chemoreceptors (FA15 p286) (FA16 p279)
Physiology of vasoconstriction (FA15 p424) (FA16 p425)
Atrial natriuretic peptide (FA15 p285) (FA16 p279)
Autoregulation (FA15 p287) (FA16 p280)

Warm -Up Review


vyhat gastrointestinal problem is most likely associated with each of.the following
scenarios?
• 50-year-old woman, pruritus without jaundic;e, positive AMA
• Gl bleeding, buccal pigmentation
• 60-year-old woman, rheumatoid arthritis, no alcohol history, fatigue, and right abdominal ;;o
rn
pain; elevated ANA and ASMA, eleyated serum lgG levels, no viral serologic markers GJ
c
• 23-year~oldwoman, no alcohol history; elevated levels ofLKM-1 antibodies, noviral r-
serologic markers; liver biopsy withinfiltration of the portal and periportal area with ~
lymphocytes 0
z
• Fatal disease of unconjugated bilirubin resulting from a complete lack of U DPGTactivity
.,
0
OJ
-u

2. Physiology of vasoconstriction
Sildenafil
• Bradykinin Smooth muscle cell
• ACh
• a 2 agonists
• Histamine J ..
cGMephosphodiesterase

.~rOMP ®• ;:;:~~a-ta_s_e.....---m·T"
- - - • relaxation

myosin-P04 actin • contraction


t
myosin light- ___...
chain kinase
. · ·l
/'"(±/
..... L.Y. o'\
·. ·.:h
.· ~ ·"'. : myp·s·in
· o~ m ··~

· calmodulin-Ca cAMP

Nitrates Epinephrine ((3)


LPS
Prostaglandin E2

[ 189 J
Of eSSIOn QUIZ
3. Outline the mechanism bywbich the kidneys regulate blood pressure.

4. What substances act on smooth muscle myosin light-chain kinase? How does this affect
blood pressure?

CL
co
u_
0
z
0
~
~ 5. Describe the chain ofevents in which hypotension causes reflex tachycardia.
~
w
a:

!90 J
HYPERTENSION
H: Chapter 247
Phys: Chapter 19
Hypertension risk factors (FA15 p290) (FA16 p284) (SU p75)
Left ventricular hypertrophy (R p552)
Aortic dissection (FA15 p293) (FA16 p287) (SU pBO)
Severe hypertension (FA15 p290) (FA16 p284) (SU p75)

Warm -Up 1ew


1. . Which immunosuppressant matches each ofthe following statements?
• Precursor of 6-mercaptopurine
·• Antibody.that binds to CD3 on T cells
• Antibody that bindsJL-2 receptor On activated T~ells
• Inhibits inosine monophosphate dehydrogenase
• Inhibits calcineurin ~ loss of IL-2 production ~ blockage ofT cell differentiation and :r:
activation -<
-o
m
• Metabolized by xanthine oxidase, therefore allopurinol increases its toxicity ;o
--i
m
z
Ul
2. What are the 2s of a Meckel diverticulum? (FA15 p364) (FA16 p361) (SU p118) 0
z

3. What is the treatmentfor Zollinger-Ellison syndrome? (FA15 p336-337) (FA16 p332)

4. What causes secondary hypertension?


• (most common)
• Chronic kidney disease (CKD) or end stage renal disease (ESRD)
• Medications(_ _ _ _ _ __ , NSAIDs, antidepressants, glucocorticoids)
• Illicit drugs (e.g., cocaine, amphetamines)
• Adrenal diseases (hypercortisolism, hyperaldosteronism, pheochromocytoma)
Hypercortisolism (Cushing syndrome)
Hyperaldosteronism (Conn syndrome)

• Hypothyroidism or hyperthyroidism
• Hyperparathyroidism
• Coarctation of the aorta

[ 191 l
eSSIOn QUIZ
5. What is the blood pressure cutoff for the diagnosis of hypertension?

6. What would you most suspect the cause of hypertension to be in a patient with each of the
foUOVIfing clinical clues?
~ • Paroxysms of increasedsympathetictone: anxiety,. palpitations, diaphoresis
•••• • Age .of onset between 20 and ·so
• Abdominal bruit
• Blood pressure in arms >
• Family history of HTN
•• Tachycardia, heat intolerance, diarrhea

z
0 • Hyperkalemia
lf)
z • Hypokalemia
UJ
f--- • Central obesity, moon-shaped face, hirsutism
0:::
UJ
o_ • <Young individual with acute onset tachycardia
>-
:r:: • Proteinuria

7. . What chestx·-ray finding is a possible sign for aortic dissection?

8.. Which category of blood pressure medications is preferred in the treatment of aortic
dissection?

r 192 1
ANTIHYPERTENSIVES
GG: Chapters 25, 26, 27
Phys: Chapter 19
ACE inhibitors (FA15 p555) (FA16 p559) (SU p76)
Angiotensin receptor blockers (FA15 p555) (FA16 p559) (SU p76)
Aliskiren (FA15 p555) (FA16 p559)
Calcium channel blockers (FA15 p304) (FA16 p298) (SU p77-78)
Hydralazine (FA15 p304) (FA16 p298) (SU p77, 79)
Minoxidil (FA15 p598) (FA16 p603) (SU p77, 79) (GG p781)
Nitrates (FA15 p305) (FA16 p299) (SU p431)
Choosing an antihypertensive regimen (FA15 p304) (FA16 p298) (GG p772-784)

Warm-Up··. Review
\'Vhich nerve is affected in each of the following upper extremity symptoms?
(FA15 p417-41B) (FA16 p420-421)
)>
• Scapular winging z
-1
• loss of forearm pronation :r:
-<
• Inability to abduct or adductfingers -o
m
;:o
• Inability to abduct arm beyond 10 degrees -1
m
• Paresthesias and pain over digits 1-3 z
Vl
<
m
Vl

2. Guillain-Barre syndrome is classiccllly preceded by what infections? p4BB)


(FA16 p493)

3. Calcium channel blockers (CCBs)


• Dihydropyridines: amlodipine, felodipine, nicardipine, nifedipine, nisoldipine
Mechanism: act on vascu lar smooth muscle to cause vasodilation
- Th erapeutic use: hypertension, angina, vasospasm (Prinzmetal's angina, Raynaud phenomenon),
esophageal spasm, migraine prophylaxis
- Adverse effects: peripheral edema, flushing, dizziness, constipation, reflex tachycardia
• Nondihydropyridines: diltiazem, verapamil
Mechanism: block calcium channels at pacemaker cells
- Therapeutic use: hypertension, angina, arrhythmias
- Adverse effects: cardiac depression, AV block, flushing, dizziness, constipation

4. Minoxidil
• Mechanism: opens K+ channe ls and hyperpolarizes smooth muscle, resulting in rela xatio n of vascular
smooth muscle
• Therapeutic use: severe hypertension , topical application for hair loss
• Adverse effects: hypertrichosis, hypotension , reflex tachycardia, fluid retention /edema

r 1931
5. What are the JNC-8 recommendations for initial antihypertensive pharmacotherapy in each of the
following patient populations?
Population Initial pharmacotherapy

CKD patients

Black patients (without CKD)

Non-black patients (without CKD)

6. For each comorbid condition, which antihypertensives might be considered first, and which
CJ antihypertensives should be avoided (either used with caution or absolutely contraindicated)?
>
(f)
z
w
Hypertension plus ... Initial Therapy Options Avoid
f---
0::: • ACE inhibitor/ARB • CCB
w
Q_
CHF • ~-blocker • ~-blocker (in acute decompensated
>-
::r:: • Aldosterone antagonist CHF or cardiogenic shock)
f---
z
<(
ACE inhibitor/ARB ~-blocker
DM

• Thiazide
• ~-blocker
• ACE inhibitor/ARB
Post-MI/CAD
• CCB (as needed for angina)
• Nitrates (as needed for angina)

• ~-blocker
Atrial fibrillation
• Diltiazem/verapamil
• ~-blocker
Bradycardia
• Diltiazem/verapamil
a-blocker
BPH

• Hydralazine ACE inhibitor/ARB


• Methyldopa
Pregnancy
• Labetalol
• Dihydropyridine CCB
• CCB
Migraines • ~-blocker

Propranolol
Essential tremor

r 194
End of ess.ion Quiz
7. Which antihypertensive class or drug fits each of the following side effects?
Hypertrichosis -, - ----·

Cyanide toxicity

Reflex tachycardia

Cough
'''·
_,_, .

Possible development of drug"induced lupus


-

Possible angioedema
_.··

)>
z
8. Which antihypertensives are particularly beneficial to heart failure patients? -1
:r:
-<
-o
rn
;;u
-1
rn
z
(f)

<
rn
(f)

9. Which antihypertensives are safe to use in pregnancy?

[ 195 J
ATHEROSCLEROSIS
R: Chapters 11, 72
H: Chapters 243, 244, 245

Arteriosclerosis (FA15 p291) (FA16 p285) (R p491)


Atherosclerosis (FA15 p292) (FA16 p286) (SU p73)
Pathogenesis of atherosclerosis (SU p73) (R p494)
Abdominal aortic aneurysm
Coronary arteries (FA15 p272) (FA16 p265) (SU p65) (R p540, Fig . 12-11)
Ischemic heart disease (FA15 p293) (FA16 p287) (SU p81) (R p538)
Causes of chest pain (R p539)

Warm-Up Rev iew


1. Where do neurons of the lateral corticospinal tract synapse prior to exiting the spinal
cord to affect movement? (FA15 p470) (FA16 p473)
VJ
VJ
0
0:::
L.U
_J
u
VJ
0
0:::
L.U
I
f-
<I: 2. What drugs and endogenous hormones regulate the secretion of gastric acid?
(FA15 p354) (FA16 p351)

r 196 1
3. Pathogenesis of atherosclerosis
• Endothelial dysfunction: leads to increased vascular permeability, leukocyte adhesion and
thrombosis
• Accumulation of lipoproteins: occurs in the vessel wall and is mostly LDL
• Monocyte adhesion to the endothelium: migration of the monocytes into the intima and then
transformation of these cells into macrophages and foam cells
• Factor release : activated platelets, macrophages and inflammatory mediators
• Smooth muscle cell proliferation: migration of smooth muscle cells into intima
• Lipid accumulation: occurs extracellularly and within macrophages and smooth muscle cells

c
Ill
E
:s
...1

::;
I
rn
:;u
0
Vl
n
,-
rn
:;u
Ill 0
....E
E Vl
Vl

4. Abdominal aortic aneurysm


• Caused by atherosclerotic plaque compressing the underlying media
• Nutrient and waste diffusion is compromised
• Media degenerates and necroses, leading to arterial wall weakness
• More common in men over 50 and in smokers
• Presents as a pulsating mass in the abdomen
• Consequences include: rupture leading to fatal hemorrhage, embolism from atheroma, obstruction
of a branch vessel and impingement on an adjacent structure (ureter)

5. Deadly causes of acute chest pain


• Aortic dissection (or dissecting aortic aneurysm)
• Unstable angina
• Ml
• Tension pneumothorax
• PE

[ 197 J
6. What is the most likely cause of chest pain in each of the following scenarios?
• ST segment elevation only during brief episodes of chest pain
• Patient is able to point to localize the chest pain using one finger
• Chest wall tenderness on palpation
• Rapid onset sharp chest pain that radiates to the scapula
• Rapid onset sharp pain in a 20-year-old and associated with dyspnea
• Occurs after heavy meals and improved by antacids
• Sharp pain lasting hours-days and is somewhat relieved by sitting forward
• Pain made worse by deep breathing and/or motion
• Chest pain in a dermatomal distribution
• Most common cause of non-cardiac chest pain

Vl
Vl
0
0:::
UJ
_j
End ofSession /Quiz
u
Vl
0
7. A 68-year~old man has an abdominal ultrasound perfOrmed to check Ol'l~lje status of
0:::
w his abdomin.aJ aortic.aneur'{sm(AAA).His l.ast ultrasound from 6months ago showed a
:r::
maximum dilation of 4.5 em. This most recent ultrasound reveals that thisAAA has enlarged
~
em. He is asymptomatic. What is thebest ne}(tstep in hismanagement?

8. A patientwithpoorlymanagedhypertension has acute, sharp substernal pain that radiates


to theback and progresses over a few hours. Death occurs in a few hours. What is the most
likely diagnosis?

9. What is the most likely cause of chest pain in each of the following scenarios?
• Acute onset dyspnea, tachycardia, and confusion in the hospitalized patient
• Rapid onset of sharp pain in the 20-year-old and associated with dyspnea
• Pain began the day following an intensive new exercise program
• ST segment elevation only during brief episodes of chest pain
• Sharp pain lasting hours to days and is somewhat relieved by sitting forward
tenderness on palpation

r 198 J
ANTIANGINAL THERAPY AND LIPID-LOWERING AGENTS
GG: Chapters 27, 31
Antianginal therapy (FA15 p305) (FA16 p299) (SU p81) (GG p761)
Lipid-lowering agents (FA15 p306) (FA16 p300) (SU p74) (GG p884)

War -Up. Review


1. label the following diagram of lung volumes. (FA15 p602) (FA16 p610)

H
A
"'
Cll
E
:l
g
0'1
!:
:I
Ja J>
z
--' -I

lc J>
z
Cl
z
Io Time
J>
,--
-I
:r:
m
;;o
J>
A \J
-<
J>
-~ B. z
CJ

'=
c. \J
CJ
I
,--
D. 0
~
m
;;o
E. z
Cl
J>
F. Cl
m
z
-I
G. V>

H.

2. What substance is important for relaxing the lower esophageal sphincter?

3. What is the most common type of tracheoesophageal (TE) fistula? (FA15 p342)
(FA16 p338)

[ 199 J
4. What effects would nitrates and ~-blockers have on each of the following parameters?
Time frame Nitrates ~-blockers

End-diastolic volume or preload

Blood pressure

Contractility

Heart rate

(/)
f-
Ejection time
z
LLJ
l'J
<(
l'J
z Myocardial oxygen demand or consumption
a::
LLJ
s0
__j

0
'
Q_
__j

0
0
z
<(
>-
Q_ • SE: facial flushing
<(
a::
LLJ
SE: elevated LFTs, myositis
:r: • Gl discomfort, bad taste
f-
__j
<(
• effect on HDL
z • effect on triglycecidesLVLDL
l'J
z
<(
• Best effect on LDL/cholesterol
f- Binds C diffici/e toxin
z
<(

6. A 50-year-old man starts on lipid-lowering medication; Upon his first dose, he develops a
rash~ pruritus, and d~rrhea. What drug is he taking?

How can the flushing reaction ofniadn be prevented?

[ 200 J
MYOCARDIAL INFARCTION PART 1
R: Chapter 12
H: Chapters 243-245
GG: Chapters 25-27
Evolution of an Ml (FA15 p294) (FA16 p288)
Diagnosis of an Mf (FA15 p295) (FA16 p289) (SU p81-83)

Wa rm-Up Review
1...·. What is.the classic presentation of congenital pyloric stenosis? (FA15 p342) (FA16 p339)

2. For what type of information is each of the following thalamic nuclei a relay station?
(FA15 p456) (FA16 p458)
• Ventral posterior lateral
• Lateral geniculate
• Ventral posterior medial
• .Ventral anterior

3. How does fetal hemoglobin differ from ~dult hemoglobin?(FA15 p603)(FA16p611)

r 201 1
4. Evolution of an Ml
Time Frame Gross Features Microscopic Features Risks and Other Findings

0-4 hrs

4-12 hrs

12-24 hrs

1-3 days

f---
0:::
<t
o_
3-14 days
z
0
f---
u
0:::
<t
LL
z > 2 w eeks
_j

<t
0
0:::
<t
u
0
>-
2:
5. ECG evolution of an Ml
Weeks later

6. ECG changes with an Ml (always obtain a previous ECG for comparison!)


• ST segment elevation of at least 1 mm in t w o contiguous leads
• T wave inversion
• New LBBB
• New Q w aves (at least one block wide or % height of the tot al QRS complex)

r 202 1
End.of Se ss ion Qui z
Which serum lab markers are commonly used to diagnose an Ml?

8. Which corollary artery is moSt commonly occluded in anMI?

...

9. What is the most common lethal complication after a myocardial infarction? :s


-<
0
n
)>
AJ
0
)>
.--
z
...,..,
)>
AJ
n
__,
0
z
-o
)>
__,
AJ

RAPID-FIR E FA CTS
Chest pain , pericardia! friction rub, and persistent
fever occurring several weeks after an l'v1 1

[ 203]
MYOCARDIAL INFAROION PART 2
R: Chapter 12
H: Chapters 243-245
GG: Chapters 25-27
Type of infarcts (FA15 p295)
Coronary artery anatomy and Mls
Ml complications (FA15 p295) (FA16 p290) (SU p82)
Ml treatment

-Up
Which hereditary hyperbilirubinemia matches each of the foll()wingstatements?.CFA15 r372)
(FA16p370)
• Mildly decreased UDPGT
• Completely absent UDPGT
N
• Grossly black liver
f-
a: • Responds to phenobarbital
<(
o__ Treatment includes plasmapheresis and phototherapy
z • •Asymptomatic unless under physical stress (alcohol, infection)
0
f-
u
0:::
<(
LL
z
_j
2. What infectious complication may arise from pulmonarysilicosis? (FA15 p612)
<(
0
0:::
<(
u
0
>-
2:
3. What are the pathological characteristics of the arteries in pulmoncu·y hypertension?
(FA15 p605, 614) (FA16 p622)

4. Arteries - Walls - ECG Leads


Artery Wall perfused ECG leads
Left anterior descending Anterior wall V1-V4, VS
Left circumflex Lateral wall aVL, VS, V6
Right coronary Inferior wall * II , Ill, aVF
Right coronary Posterior wall R precordial ECG: V4
* Always obtain a right-sided ECG (V1-V6 on right chest) in inferior wall Ml ~if ST segment elevation
in V4, then posterior right ventricle also affected. This indicates a right-sided Ml ~ fluids and avoid
nitroglycerin .

r 2o4 1
5. Myocardial infarction treatment
• ABCs
• rv!ONA:
IV morphine
- Supplemental 0 2
Nitroglycerin
- Aspirin
• ~-blocker: oral metoprolol if no signs of heart failure or severe asthma
• Statin: atorvastatin, preferably before percutaneous intervention (PCI)
• Anti platelet therapy to all patients: clopidogrel or ticagrelor
• Anticoagulant therapy to all patients:
Unfractionated heparin to all patients undergoing PCI
- Enoxaparin for patients not managed with PCI
• Potassium and magnesium
• If ST elevation myocardial infarction (STErv-11):
- PCI :s:
-<
0
If PCI is unavailable: fibrinolysis within 90-120 minutes n
)>
• If non-ST elevation myocardial infarction (NSTErv-11): ;;o
CJ
- Avoid fibrinolysis )>
r
PCI :z:
• If significant coronary artery disease on PCI: stenting or coronary artery bypass graft ")>
;;o
n
~
0
:z:
"""0
)>
;;o
End of Sessi on Qu1 z ~
N

6. What would cause each of the following findings after an Ml?


• Cardiac tamponade
• Severe mitral regurgitation
• New VSD
• Stroke

7. Which ECG leads will show evidence of ischemia in an anterior waiJ Ml?

8. How would you manage a patient presenting with an acute myocardial infarction?

r 2os 1
CARDIOMYOPATHIES AND ENDOCARDITIS
R: Chapters 6, 72
H: Chapter 238

Cardiomyopathies
- Hypertrophic cardiomyopathy (FA15 p296) (FA16 p291) (SU p89)
- Restrictive cardiomyopathy (FA15 p296) (FA16 p291) (SU p89)
- Dilated cardiomyopathy (FA15 p296) (FA16 p291) (SU p88)
Myocarditis (SU p86) (R p570) (H p1953-1961)
Infective endocarditis (FA15 p298) (FA16 p293) (SU p87) (R p559)
Noninfective endocarditis (FA15 p298) (FA16 p293) (SU p87) (R p224, Fig. 6-28)

Warm-Up Rev1ew
1. At what age should patients begin receiving screening colonoscopies?

V)

i=
0
0:::
3 2. What is the triad of Kartagener syndrome? What is the underlying defect? (FA15 p74)
2; (FA16 p60) (SU p102)
z
w
0
z
<(
V)
w
:r: 3. What are the classic symptoms of carcinoid syndrome? (FA15 p336) (FA16 p332)
~
0... (SU p114)
0
>-
:2:
0
0
0:::
<(
u

4. Myocarditis
• Generalized inflammation of the myocardium (not resulting from ischemia)
• Most common cause (in the U.S.): _ _ __ _ __ _ __ __ __
• Histo: diffuse interstitial infiltrate of _ _ _ _ _ _ _ _, with _ _ __ _ _ _ _ __

5. What are the typical signs and symptoms of endocarditis?


• Fever, chills, weakness, anorexia
• New regurgitation heart murmur or heart failure
Mitral valve is most common
- Tricuspid is most common in IV drug users ~septic pulmonary infarcts
• Splinter hemorrhages in fingernails
• Osier nodes (painful red nodules on finger and toe pads)
• Janeway lesions (erythematous macules on palms and soles)
• Roth spots (retinal hemorrhages with clear central areas)
• Signs of embolism: brain infarct ---7 focal neuro defects, renal infarct ---7 hematuria, splenic infarct~

abdominal or shoulder pain


• Systemic immune reaction: glomerulonephritis, arthritis

r 2061
1on Qu iz
What are the differences between acute and subacute infective endocarditis?

7. An IV drug user presents with chest pain, dyspnea, tachycardia, and tachypnea. What is
mOst likelythe cause?

n
)>
A young girlwith congenital valve disease is give11 penicillin prophylaq:ically.. In the ER, ;u
0
infective endocarditis is diagnosed. What is the next step in her management? 0
s:
-<
0
--o
~
:::r:
rn
Ul
)>
z
0
rn
z
0
0
n
)>
;u
RAPID-FIRE FACTS 0
=i
Ul

Splinter hemorrhages under the fingernails

Retinal hemorrhages with pale centers

Heart va lve most commonly involved in infective


endocarditis

Heart valve most commonly involved in an IV


drug user with infective endocarditis

r 2071
OTHER CARDIAC PATHOLOGY
R: Chapter 12
H: Chapters 237-239
Rheumatic heart disease (FA15 p299) (FA16 p294) (SU p87-88) (R p557)
Pericarditis (FA15 p299) (FA16 p294) (SU p91) (R p573) (H p1971)
Cardiac tamponade (FA15 p299) (FA16 p294) (SU p91)
Syphilitic heart disease (FA15 p299) (FA16 p294)
Cardiac tumors (FA15 p300) (FA16 p300) (SU p91) (R p575) (H p1981)

Wa rm- Up Review
1. Identify the hepatitis B status of each of the following patients based on their hepatitis B
serologic markers. (FA15 p167) (FA16 p157) (SU p134)
HepBsAg HepBsAb HepBcAb Status

>- Negative Positive Positive


(._')
0_.J
0 Negative Negative Positive
:r:
~
0... Positive Negative Positive lgM
u
<{
0 Positive Negative Positive lgG
~
<{
u
~
w
Negative , Positive Negative
:r:
l-
0

2. Which tumors arise centrally in the lung and are linked to smoking? Which tumors tend to
arise peripherally in the lung?

3. A 20-year-old man contracts influenza then presents with an idiopathic hyperbilirubinemia.


What is the most likely cause?

r 2os 1
4. What are the diagnostic criteria (Jones criteria) for rheumatic fever?
With evidence of a group A Strep infection, there is a high probability of acute rheumatic fever if a
patient has two major criteria (or one major and two minor criteria).
• Major criteria :
- Joints (migratory polyarthritis)
Heart (pancarditis)
Nodules (subcutaneous)
Erythema marginatum (serpiginous skin rash)
Sydenham chorea (chorea of the face, tongue, upper limb)
• Minor criteria:
Prolonged PR interval
Elevated temperature (fever)
- Arthralgia
Elevated CRP or ESR

0___,
5. Kussmaul's sign vs. Pulsus paradoxus: :::r::
m
:::0

Kussmaul's sign Pulsus paradoxus n


)>
:::0
Decreased SBP by more than 10 mm Hg during 0
Event JVD during inspiration )>
inspiration n
--o
~
Mechanism Decreased capacity of RV Decreased capacity of LV :::r::
0
r
0
Cl
Disease Constrictive pericarditis » tamponade Cardiac tamponade » pericarditis -<

r 2091
6. Underwhat circumstances might you see pulsus paradoxus?

What are the major criteria for the diagnosis of .acute rheumatic fever?

>-
CJ
What heart pathology fits each of the following statements?
0__j
0
Focal myocardial inflammationwith
I multinucleate giant cells
~
CL Chest pain and coarse rubbing heart sounds in
u patient with Cr of 5.0
<r:
0
0::: Tree-barking of the aorta
<r:
u
0:::
UJ Ch.ild with fever, joint.p<;Jin, cutaneous nodules
I
1-- four weeks after a throat infection
0
elevation in all ECG leads

ECG shows electrical alternans

RAPID -FIRE FACTS


Granulomatous nodules in the heart

Most common primary cardiac tumor in adults

Most common primary cardiac tumor in children

Most common cause of constrictive pericarditis

i" 210 1
VASCULITIS
R: Chapter 11
H: Chapter 326
Large-vessel vasculitis (FA15 p302) (FA16 p296) (SU p90)
- Takayasu arteritis
- Giant cell arteritis (temporal arteritis)
Medium-vessel vasculitis (FA15 p302) (FA16 p296) (SU p90)
- Kawasaki disease
- Thromboangiitis obliterans (Buerger disease)
- Polyarteritis nodosa
Small-vessel vasculitis (FA15 p302-303) (FA16 p296-297) (SU p90)
Henoch-Schonlein purpura
Eosinophilic granulomatosis with polyangiitis (Churg-Strauss)
- Granulomatosis with polyangiitis (Wegener's)
- Microscopic polyangiitis

Warm-Up Rev1ew <


)>
V1
n
1. What is the difference between primary biliary cirrhosis and primary sclerosing c
cholangitis? (FA15 p374) (FA16 p371) (SU p135) '--;
V1

2. A patient develops ARDS from an occupational inhalation of nitrogen dioxide. What


~istologic change would bei~een in a patient recovering from ARDS?

3. What is Reynold's pentad for cholangitis? (SU p135)

[ 211 ]
End ofSess ion Quiz
4. Which type of vasculitis fits each of the following high-yield characteristics?

Weak pulses in upper extremities

Granulomas of lung, glomerulonephritis,


c-ANCA positive
Necrotizing immune complex inflammation
of visceral/renal vessels

2-year-old East Asian female

20 -year-old East Asian female


'

male smokers

Young asthmatics

Infants and young ch ildren; involves the


coronary Clrteries

Most common vasculitis

AssoCiated with hepatitisB infection


.

Occlusion of ophthalmic artery can lead to


blindness

Perforation of nasa l septum

Unilateral headache, jaw cla udication

RA P/ D- FI RE FACTS
c-ANCA

p-ANCA

Treatment for thromboangiitis obliterans

Treatment for giant cel l arteritis

r 212 1
OTHER VASCULAR DISEASES
R: Chapter 11
Raynaud phenomenon (FA15 p301) (FA16 p437)
Sturge-Weber disease (FA15 p491) (FA16 p495)
Vascular tumors (FA15 p301) (FA16 p441)

Warm -Up Review


1. Which autoimmune disorder is associated with each of the following types of
antjbodies? {FA15 p213) (FA16 p207J
Anti-TSH receptor antibodies
• Antimitochondrial antibodies
• Anti centromere antibodies
,. • Antihistone antibodies
• Anti-smooth ml.lsde
0
-l
:::r::
rn
:;;o
<
2. What is the cause of chronic granulomatous disease? What are the consequences of )>
(J)

chronic granulomatous disease? (FA15 p21SJ (FA16 p209) (SU p383) n


c
':;;o
)>

0
(J)
rn
)>
(J)
rn
3. What are the mechanism and time frame of acute transplant rejection? (FA15 p217) (J)

(FA16p211)

4. Patient Profile: The Shivering Scoutmaster


A volunteer leader in a local youth organization comes to the physician complaining that when he is
camping in the winter months, his right middle finger becomes quite cold and turns first white then
dusky blue. This discoloration often spreads to other fingers on both hands. After about 15 minutes
of warming his hands, the fingers become quite red before returning to their normal color. What is the
most likely diagnosis, and how can this disorder be t reated?

f2.13l
QUIZ
5. Which disorders are commonly discovered in patients with Raynaud phenomenon?

6. Which vascular tumor fits each of the following descriptions?


size of a

Raised, red area .present at birth, increases in


size initially then regresses over .months.to years

(J)
Lesion caused by lymphoangiogenic growth
LLJ
(J)
factors in anHIV Pi3tient
<(
LLJ
(J) Polypoid red lesionfound in pregnancy or
0 trauma
0::::
<(
_j
::::J painful, red-blue tumor under fingernails
u
(J)
<(
> Cavernous lymphangioma associated with
0::::
LLJ
Turner syndrome
::r:
f--- Skin papule in AIDS patient caused by Bartonella
0
spp.

r /!4 1
LOGY IS,

RHEUM AND OERM ESSENCE, A HODGEPODGE


OF INFLAMMATORY
DISEASES, AUTOIMMUNE
1 Bone Formation and Bone Tumors
DISORDERS ON -
INFLAMMA /NT
2 Bone Disorders
DISEASES, OFTEN
WITH PERPLEXING,
3 Musculoskeletal Injuries
OVERLAPPING
PRESENTATIONS AND
4 Osteoarthritis and Rheumat oid Arth r1 t 1s
WIDE -:- RANGING SY
MANI FESTATIONS
5 Other Types of Arthr1t1s
DISEASES THAT OFTEN
6 System 1c Diso rd ers DON'T SEEM TO FIT
ANYWHERE ELSE.
7 Dermatology part 1 LiKEWISE, CUTANEOUS
MANIFESTATIONS OF
8 Dermatology part 2 SYSTEMIC DISEASE CAN
OFTEN BE DIFFICULT TO
DIAGNOSE CORRECTLY .
BOTH DISCIPLINES ARE
COMMONLY TESTED,
HOWEVER OFTEN
ASKING THE EXAMINEE
TO INTERPRET CLINICAL
PHOTOGRAPHS AND
RADIOGRAPHS.
BONE FORMATION AND BONE TUMORS
R: Chapter 26
Formation of bone (FA15 p424) (FA16 p425) (SU p226)
Achondroplasia (FA15 p425) (FA16 p426) (SU p228)
Primary bone tumors (FA15 p428) (FA16 p429) (SU p231)
Clinical manifestations of bone mets (FA15 p240) (FA16 p233) (SU p231)

Warm~Up Review
1. Which drug categories have the following endings? (FA15 p261) (FA16 p255)
• -olol
• -oxin
• -pril
• cfiaine
• -tropin

2. Which a-adrenergic antagonists are used in the treatment of pheochromocytoma?


(FA15 p255) (FA16 p248) CD
0
z
rn

"
0
;o
5
~
3. Why do the kidneys retain fluid in patients with heart failure? 0
z
)>
z
0
CD
0
z
rn
---1
c
5
0
;o
lf)

r 2171
End of Sess1on Quiz
4. Where does new bone formation take place in growing long bones?

5. Which primary bone tumor fits each of the following descriptions?


• Most common malignant primary bone tumor in children
• Most common benign bone tumor
• . 11;22 translocation
• Rb mutation
• "Sunburst" pattern on x-ray
• "Soap bubble" appearance on x-ray
• "Onion skin" appearance of bone
• May actual ly be a hamartoma
• Cadman's triangle on x-ray

6.. What mutation leads to achondroplasia?


VI
a:::
0
2:
:=J
f-
LLJ
z
0
CD
0
z
<(
z
0
~
2:
a:::
0
LL
LLJ
z
0
CD

r 7.18 1
BONE DISORDERS
R: Chapter 26
Bone biology (FA15 p424) (FA16 p425) (SU p227-229) (R p1180-1181)
Osteoporosis (FA15 p425) (FA16 p426) (SU p230)
Osteoporosis treatment (FA15 p425, 445) (FA16 p426, 447)
- Bisphosphonates (FA15 p445) (FA16 p447) (SU p230)
Osteopetrosis (FA15 p425) (FA16 p426) (SU p230)
Paget disease of bone (FA15 p426) (FA16 p427) (SU p230)
Rickets and osteomalacia (FA15 p425) (FA16 p427) (SU p230)
Osteitis fibrosa cystica (FA15 p332, 427) (FA16 p326, 428)
Polyostotic fibrous dysplasia
Bone disorder labs (FA15 p427) (FA16 p428)

Warm-Up Revrew
·1. Name five drugsithat inhibit acetylcholinesterase. What the dil'lical application for
each? (FA15 p250) (FA16 p244)

2. When placing a femoral venous catheter, while palpating the pulsatile femoral artery,
where is the femoral catheter placed in relation to the artery? (FA15 p350) (FA16 p347)

CD
0
z
rn

3. What are the different mechanisms by which heart contractility can be increased? CJ
l/)

(FA15p273) (FA16 p267) 0


;;o
CJ
rn
;;o
l/)

r? 19 1
4. Osteoporosis treatment
• Stop smoking, stop steroids (if able), increase weight bearing
• Calcium and vitamin D supplement ation
• Bisphosphonates (inhibit osteoclastic resorption, reduce fracture risk by 40-50%)
• Conjugated estrogen (WHI trial -4 hip fracture reduction of 33%): USPSTF recommends against
using estrogen for the sole purpose of fracture reduction due to the risk of other side effects (e.g.,
Ml and CVA)
• SERM (raloxifene) decreases vertebral fracture by 40% in women with osteoporosis, no effect on
risk of non-vertebral fractures, reduces risk of breast cancer.
• Teriparatide (recombinant PTH analog that stimulates osteoblasts) decreases hip fracture rate
by 53%. Duration of therapy not to exceed two years. Must use bisphosphonate after stopping
teriparatide to maintain bone mineral density.
• Denosumab: RANK-L inhibitor, inhibits osteoclast activity

5. Osteitis fibrosa cystica (AKA von Recklinghausen disease of bone)


• A bony manifestation of an endocrine disorder
• Can be caused by:
Hyperparathyroidism
• High PTH -4 high serum calcium, low serum phosphate, high alkaline phosphatase
- Type 1A pseudohypoparathyroidism (AKA Albright hereditary osteodystrophy)
• PTH resistance at the renal tubules -4 low serum calcium and high phosphate
• Low calcium ~ high PTH
• High PTH ~ excess osteoclastic activity -4 "brown tumors" in bone (cystic spaces lined by
osteoclasts and filled with fibrous stroma and blood)
• DEXA scan reveals low bone mineral density, but the mechanism of bone loss is different from
osteoporosis
(/)
(Y
LLJ
0
(Y
0
(/)

0
LLJ
6. Bone disorder labs
z 2
0
m
Serum Ca + Serum Phos Alk Phos PTH
Osteoporosis
Osteopetrosis
Paget disease
Rickets/osteomalacia
Renal insufficiency
Vitamin D intoxication
Primary hyperparathyroidism
Osteitis fibrosa cystica

r 2201
End ofSess1on Quiz

8. Which bony disease fits each ()f the following descriptions?


• Reversib le when vitam in D isreplaced
• Excess osteoclastic activity results.in disorganized bony architecture
• Bone is replaced by fibroblasts, collagen, and irregular bc:iny trabeculae
due to defective mineralization of osteoid
of bone resorption · ~. thickened and dense bones
of II

9. What measures can be taken to prevent osteoporosis?

OJ
10. FiiUn the following table: 0
z:
•• 2• rn
Serum Ca Sert.Jm Phos Alk Phos PTH CJ
Vl
Paget disease 0
:::0
Osteomalacia/rickets CJ
rn
:::0
Osteitis fibrosa cystica ···•·
Vl

Osteoporosis

RAPID -FIRE FACTS


Bone enlargement, bone pain, arthritis

Vertebral compression fractures

r 221 1
MUSCULOSKELETAL INJURIES
COA: Chapters 3, 5, 6
R: Chapter 26
Bones of the wrist (FA15 p417) (FA16 p418)
Epicondylitis (FA15 p417) (FA16 p417) (SU p242)
Biceps tendon rupture
Rotator cuff (FA15 p417) (FA16 p417)
Shoulder dislocation and separation (SU p242)
Hip dislocation
Unhappy triad (FA15 p416) (FA16 p417) (SU p242)
Ankle sprain (SU p242)

Warm-Up Rev iew


1. Which drug category has each of the following endings? (FA15 p261) (FA16 p255)
• -dronate
• -chol
• -mustine
• -statin
• -bendazole

2. What are the side effects of amiodarone? (FA15 p309) (FA16 p303) (SU p405)

(/)
w
0:::
:::::>
z
--'
~
w
--'
w
~
3. What _conditions are associated with diastolic murmurs? (FA1 5 p279) (FA16 p273) (SU p89)
(/)
0
--'
:::::>
u
(/)
:::J
::2:

r 222
4. Epicondylitis
• Lateral epicondylitis ("tennis elbow")
• Medial epicondylitis ("golfer's elbow")

5. What is the difference between a dislocated shoulder and a separated shoulder?


• Dislocated - head of the humerus rotates out of the glenoid cavity
• Separated - clavicle separates from the acromion and coracoid process of the scapula

6. Which structures can be damaged in an anterior shoulder dislocation?


• Axillary nerve and posterior circumflex artery
• Supraspinatus tendon
• Anterior glenohumeral ligaments and glenoid labrum separation from the articular surface of the
anterior glenoid neck (AKA Bankart lesion)
• Posterolateral humeral head defect due to forceful impact against the anterior rim of the glenoid
(AKA Hill-Sachs lesion)

7. What is the most common type of hip dislocation? What structures can be damaged with this type of
dislocation?
• Most common type: _ __ __ _ _ _ _ __ _
• Structures injured:
s::
c
Medial and lateral circumflex femora l arteries V1
vein n
- - - - -- ---- c
r-
- - -- -- - - - - nerve 0
V1
?<:
rn
r-
rn
);!
r-
z
8. How should an ankle sprain be treated? c
;;o
• R- - - - - - - - - - - - rn
V1
• I___________
• C._ _________________
• E.__________________
• NSAIDs
• Range of motion exercises

r 2231
End of.Session Quiz
9. Which muscles of the rotator cuff are responsible for each of the following actions?
• Initial 15 degrees of arm abduction
• External rotation of arm
• Internal rotation of

10... Which antibiotic class is known to increase the risk of tendon rupture in adults?

11. What ligaments are typically injured during an ankle sprain?

RAP! o-F! RE FACTS


_j
Positive anterior drawer sign
f"S
LLJ
_j Tenderness in the anatomical snuffbox
LLJ
:>L
(f)
0
_j

:=J
u
(f)
::::>
2

[ 224 J
OSTEOARTHRITIS AND RHEUMATOID ARTHRITIS
H: Chapters 321, 332
R: Chapter 26
GG: Chapter 34
Osteoarthritis (FA15 p429) (FA16 p430) (SU p232)
Treatments
- Acetaminophen (FA15 p444) (FA16 p446) (SU p243)
- NSA/Ds (FA15 p445) (FA16 p447) (SU p243-244) (H p2835, Table 332-1)
- COX-2 inhibitors (FA15 p445) (FA16 p447) (SU p244-245)
- Aspirin (FA15 p445) (FA16 p447) (SU p244)
Rheumatoid arthritis (FA15 p429) (FA16 p430) (SU p232)

Warm-Up Rev1ew
1. How does adding a competitive antagonist affect the efficacy and potency of a
receptor agonist? (FA15 p246) (FA16 p240) (SU p10-11)

0
(Jl
---1
rn
0
)>
:::0
---1
2 . .What is the competitive antagonist used to treatbenzodiazepine overdose? (FA15p25?) :c
:::0
:::j
(Jl

)>
z
0
:::0
:::r:
rn
c
s:
3. An 80-year-old man is found to have a systolic crescendo-def;rescendo murmur. ~
0
What is the most likely cause? (FA15 p279) (FA16 p273) (SU p89) 0
)>
:::0
---1
:c
:::0
--1
(Jl

r ns l
E of Sess1on Ou1 z
4. What is the classic feature of osteoarthritis?

5. What two antibodies are most useful in diagnosing rheumatoid arthritis?

6. By what mechanism do NSAIDs cause renal disease?

(f)

1-
0:::
:r::
1-
0:::
<(
0
0
~
~
::::J
LU
:r::
RAP! D-F! RE FACTS
0:::
o Swollen, hard, painful finger joints
z
<(
(f)

1- Swollen, boggy, painful finger joints


0:::
:r::
1-
~ Cartilage erosion with polish ed bone ben eath
0
LU
I-
V)
0

f 226 l
OTHER TYPES OF ARTHRITIS
H: Chapters 325, 333, 334, 336
R: Chapter 26
GG: Chapter 34
Gout (FA15 p430) (FA16 p431) (SU p233)
Gout drugs (FA15 p446) (FA16 p448) (SU p233-234) (GG p994)
Pseudogout (FA15 p431) (FA16 p431) (SU p233) (H p2839)
Seronegative spondyloarthropathies (FA15 p432) (FA16 p433) (SU p232)
TN Fa inhibitors (FA15 p446) (FA16 p448)
Juvenile idiopathic arthritis
Septic arthritis (FA15 p432) (FA16 p432) (SU p235)

Warm -Up Review


Place the following agents in the appropriate categories: (FA15 p250-256) (FA16 p244-249)
norepinephrine, metopro/o/, timolol, scopolamine, phenoxybenzamine, bethanechol, isoproterenol, donepezil,
prazosin, atropine: propranolol, labetalol, atenolol, terbutaline, dopamine, pilocarpine, carbachol, edrophonium,
phenylephrine, glycopyrrolate, phentolamine, neostigmine, benztropine, terazosin, pralidoxime, echothiophate,
ipratropium, epinephrine, esmolol, oxybutynin, doxazosin

Direct chollnergic agonists:


Cholinesterase regenerator

a-adrenergic antagonists (a-blockers)

Cholinergic antagonists·
0
-i
:r:
m
;::o
-i
-<
u
m
U1
0
.,
~-adrenergic antagonists (~-blockers) )>
;::o
-i
:r:
;::o
Nonselective
-i
Indirect cholinergic agonists (132 ~ 13) U1
(anti-acetylcholinesterases)

.... · t31 selective

Sympathomimetics:
a, a 2 j31 {3 2 agonist
t\ a 2 {3 1 agon ist

D1 ~ D2 > t3 1 > a 1 agonist


(3 1 = (32 agonist

J\ > 13, agonist


a 1 > a 2 ;Jgon1st

r 2211
2. Juvenile idiopathic arthritis (JIA)
• Affects chi ldren (onset before age 10)
• Persistent joint swell ing (synovial thickening, accumulation of synovia l fiu id)
• Subtypes
_ _ _ __ _ __ _ _ _ _ : severe symmetrical arthritis, dactylitis
_ _ _ _ __ __ _ __ _ :typicall y involves large joints; 20-25% have uveitis
_ _ _ __ _ _ _ _ _ __ (Still's disease): begins with systemic symptoms (fever, rash ,
elevated WBC, anemia, hepatosplenomegaly, lymphadenopathy); arthritis later

End of Sess1on Quiz


3. What medications are used in the treatment of an acute gout exacerbation?

4. What drugs can be used in the treatment of chronic gout?

l/)

f---
0::: 5. A sexually active 19-year-oid man presents with pink eye, arthritis of the right knee,
I
f---
0:::
and dysuria. What is the most likely diagnosis?
<(
u__
0
l/)
w
o_
>-
r-
0:::
w
I
f---
0

RAP! o-F! RE FACTS


Swol len, red , acutely painful great toe joint

Positively birefringent rhomboid-shaped crystals

Negatively birefringent need le-shaped crystals

Bamboo spine on x-ray

HLA-B27

Pencil-in-cup deformity on x-ray of the finger

r na1
SYSTEMIC DISORDERS
H: Chapters 323-324, 326, 335, 388
R: Chapter 26
GG: Chapter 34
Systemic lupus erythematosus (FA15 p433) (FA16 p434) (SU p235-236)
Scleroderma (FA15 p436) (FA16 p437) (SU p236)
Sjogren syndrome (FA15 p430) (FA16 p432) (SU p236)
Muscular dystrophies (FA15 pBS) (FA16 p73) (SU p249) (R p1242)
Polymyalgia rheumatica (FA15 p434) (FA16 p435)
Polymyositis/dermatomyositis (FA15 p435) (FA16 p436) (SU p236) (R p1238-1239)
Fibromyalgia (FA15 p434) (FA16 p435) (H p2849)

Warm -Up Review


1. What are the clinical uses for each of the following antimuscarinic drugs? (FA15 p251)
(FA16p245)
• lpratropium
• Atropine, homatropine, tropicamide
• Benztropine
• Scopolamine

2. Which receptors use a Gs protein? (FA15 p24B)

(J)
-<
(J)
---1
rn
:s:
n
0
(J)
0
:::0
0
rn
:::0
(J)

[ 229]
3. Systemic lupus erythematosus = 4/11
• Skin disorders
Malar rash
Discoid rash
Photosensitivity
Painless oral ulcers
• Inflammatory disorders
Arthritis (nonerosive, two joints)
Serositis (pleuritis, pericarditis)
(+)ANA
• Organ system disorders
Renal (proteinuria, cellular casts)
Neuro (seizures, psychosis)
Heme (hemolytic anemia, leukopen ia, lymphopenia , thrombocytopenia)
Immune (antiphospholipid antibodies, anti-dsDNA Ab, anti-Smith Ab, fa lse -positi ve VDRL)

4. Fibromyalgia
• Excess muscular tenderness in 11 of 18 particular sites
• Chronic generalized pain, fatigue, sleep disturbances, HA, cognitive difficulty, mood disturbances
• 30% also have depression and/or anxiety
• Pharmacological treatment
FDA-approved: pregabalin, milnacipran
Traditiona l (not FDA-approved): amitriptyline, low-dose analgesic, fluoxetine
• Non-pharmacological treatment
(/) Reassurance t hat it is a real disease and that it is benign, exercise and stretching, sleep,
0::
w relaxation techniques, stress reduction
0
0::
0
(/)

0
u
~
w
f-
(/)
>-
(/)

[ 230 J
End of Sess1o n Quiz
5. A patient presents with photosensitivitY, arthritis, hemolytic anemia and recurrent oral
ulcers. What is the most likely diagnosis?

6. What drugs are known for causing drug-induced lupus?

7. What are some of the characteristics of polymyositis that distinguish itfrom


polymyalgia rheumatica?

(J)
-<
(J)
--1
rn
s
n
RAP! O-Ff RE FACTS 0
(J)
0
Anti-Smith and anti-dsDNA antibodies ;;o
0
rn
;;o
(J)
Anti-histone antibodies

Anti-centromere antibodies

Anti -topoisomerase antibodies

Anti-Jo-1 antibodies

Anti-Ro antibodies

Arthritis, dry mouth, and dry eyes

[ 231 ]
DERMATOLOGY PART 1
H: Chapters 52, 53, 54
Epidermis layers (FA15 p436) (FA16 p437) (SU p251)
Epithelial cell junctions (FA15 p437) (FA16 p438)
Melanocytes (R p1143)
Atopic dermatitis (FA15 p439) (FA16 p440) (SU p252)
Verrucae (FA15 p439) (FA16 p440) (SU p252)
Melanocytic nevus (FA15 p439) (FA16 p440)
Allergic contact dermatitis (FA15 p439) (FA16 p440)
Acne (FA15 p439) (FA16 p440)
Psoriasis (FA15 p439) (FA16 p440) (SU p252)

Warm - Up Review
1. How does a drug's dose-response curve change with the addition of a competitive
antagonist as compared to the addition of a noncompetitive antagonist? (FA15 p246)
(FA16 p24D)

2. What are the clinical uses for each of the following sympathomimetics? (FA15 p253) ·
(FA16 p246)
• Dopamine
l-
ex: • Clonidine
<{
o_ • Amphetamine
>- • Terbutaline
\..?
0 • Epinephrine
--'
0
~
z
0:::
LLJ
0

3. Melanocytes
• Melanin-producing cells
• Located in the stratum basale (bottom layer) of the epidermis
• Responsible for skin color

r n21
4. Atopic dermatitis (eczema) treatment
• Mild cases: switch to a moisturizing soap and add an emollient
• Calcineurin inhibitors (tacrolimus or pimecrolimus)
• Topical steroids
• Antibiotics for open lesions (cover Staphylococcus aureus and Streptococcus spp.}
• Antihistamines
• Leukotriene inhibitors
• UV light therapy
• Severe cases: consider systemic steroids
• Very severe cases: consider methotrexate, cyclosporin , azathioprine

5. Acne
Pathophysiology Treatment
Hyperkeratosis

Sebum overproduction

Propionibacterium acnes proliferation

Inflammation
0
rn
:::0
5:
)>
---1
0
,--
0
Gl
-<
-o
)>
:::0
---1

[ 233 J
End of Sess1on Quiz
6. Ust the layers of the epidermis.

7. Which structure connects epithelial cells to the basement membrane?

8. What are the four primary causes of acne?

f-
a:
<(
9. Which skin disorder matches each of the following statements?
CL
• Pruritus associated with asthma
>-
l'J • Allergy to nickel
0__J
0 • Parakeratotic sca ling
~ • Caused by HPV types 2 and 4
::?:
0:::
w
0

[ 234 J
DERMATOLOGY PART 2
H: Chapters 52, 53, 54, 87
Seborrheic keratosis (FA15 p439) (FA16 p440) (SU p252)
Pigmented skin disorders (FA15 p438) (FA16 p439) (SU p252)
- Albinism
- Vitiligo
- Melasma
Infectious disorders (FA15 p440) (FA16 p442)
Impetigo
Cellulitis
- Necrotizing fasciitis
- Staphylococcal scalded skin syndrome (SSSS)
- Hairy leukoplakia
Blistering skin disorders (FA15 p441) (FA16 p443)
Pemphigus vulgaris
- Bullous pemphigoid
- Dermatitis herpetiformis
- Erythema multiforme
- Stevens-Johnson syndrome
Lichen planus (FA15 p442) (FA16 p444)
Actinic keratosis (FA15 p442) (FA16 p444) (SU p252)
Acanthosis nigricans (FA15 p442) (FA16 p444) (SU p252)
Erythema nodosum (FA15 p442) (FA16 p444)
Pityriasis rosea (FA15 p442) (FA16 p444)
Skin cancer (FA15 p443) (FA16 p445) (SU p253)
- Squamous cell carcinoma
- Basal cell carcinoma
- Melanoma

Review
1. What are normal blood pressures in the right and left ventricles? (FA15 p286) (FA16 p2BO)
0
rn
:::0
:s
~
0
'0
Cl
-<
2. Which primarybone tumor..flts.each of the following descriptions? (FA15 p428) -o
:r>
:::0
___,
• Most common malignantprimary bone tumor in children
N
• Most common benign bone tumor
• 11;22 translocation
• "Soap bubble" appearance on x-ray
• "Onion skin" appearance ofbone
• Codman'striangle on x-ray

r 235 1
End" of Sess ion Quiz
3. What are some of the hallmark features of necrotizing fasciitis? What organism causes this
infection?

4. What pathogen causes painless white patches on the tongue that cannot be scraped off?

5. Which skin disorder matches each of the following statements?


• Pruritic, purple, polygonal papules
• Pruriticvesicles associated with celiac disease
• Antibodies against hemidesmosomes
• Antibodies against desmosomes

N
• Keratin-filled cysts
f----
0:::
• Sandpaper; predisposition to squamous cell cancer
<(
o_ • Honey-crusting lesions common about the nose and lips
>-
\..C) • Histology shows palisading nuclei
0
_j

0
~
~
0:::
w
0

RAP! O- F! RE FACTS
Keratin pearls on skin biopsy

IV1ost common malignant skin tumor

r 2.36 1
THERE IS MUCH MORE

REPRODUCTION SYSTEM THAN JUST


RODUCTIVE

OBSTETRICS. THERE ARE


1 Reproductive Anatomy
IMPORTANT ENDOCRINE
CONCERNS, AS WELL
2 Gen1tal Embryology
AS SOME OF THE
MORE COMMON AND
3 Androgens
OCCASIONALLY MORE
DEADLY CANCERS,
4 Testicular Pathology
SUCH AS
- PROSTATE-,
- -- -~ - - -

BREAST AND OVARIAN


5 Pen1s and Prostate Pathology
CANCER ALSO
6 Female Reproductive Cycle COVERED ARE VARIOUS
GENETIC DISORDERS
7 Menstruation and Menopause AND CHROMOSOMAL
DISORDERS, ·WHICH MAY
8 Vulva, Vag1na and Cerv1x BE DETECTED IN UTERO
OR SHORTLY AFTER BIRTH
9 Utenne Pathology AND OFTEN REQUIRE
GENETIC COUNSELING.
10 Ovar1an Pathology

11 Ovarian Neoplasms

12 Pregnancy part 1

13 Pregnancy part 2

14 Chromosomal Disorders

15 Genetic Disorders AD and


Trmuc leot1de Repeats

16 Genet1c Disorders AR and X-Lmked

17 Breast
REPRODUCTIVE ANATOMY
R: Chapters 21, 22
H: Chapters 346, 347, 349
COA: Chapter 3

Male reproductive anatomy (FA15 p571) (FA16 p575) (SU p198)


Female reproductive anatomy (FA15 p570) (FA16 p574) (SU p197)
Innervation (COA p411)
Male sexual response (FA15 p571) (FA16 p575)
Gonadal drainage (FA15 p569) (FA16 p573)
Groin hernias (FA15 p352) (FA16 p349) (SU p250-251)
Urinary incontinence (FA16 p551)

Warm pReview
1. •Where are nicotinic

2. Which drug category has each of the following endings? (FA15 p261) (FA16 p255)
• -operidol
• -phylline
• -terol
• -triptyline
-zosin

3. Whatare the systolic heart murmurs? (FA15 p279) (FA16 p273)

:::0
m
v
:::0
0
0
c
n
-1
<
m
4. Types of urinary incontinence )>
z
Cause Symptoms Treatment ~
0

Weakening of
. ~
-<
Stress • Pessary
pelvic floor
• Surgery

• Lifestyle modifications
Urge •
(e.g., oxybutynin,
darifenacin, solifenacin)

Inability to void
• Treat underlying cause
normally~
Overflow • Pessary
overdistention of
• Surgery
bladder

r 239 1
End of Sess1on 0u1z
5. A professional cyclist develops erectile dysfunction and perineal pain and numbness.
Which nerve was most likely injured?

6. A 67-year-old man with prostate cancer develops erectile dysfunction after


undergoing a radical prostatectomy. Which nerves were most likely injured?

7. Describe the autonomic control of erection, emission, and ejaculation.

8. To where does testicular cancer first metastasize?

Which structures make up Hesselbach's triangle?

>-
~
0
~
z
<(

> 10. Which type of urinary incontinence matches each of the following descriptions?
tJ
:::>
• Treated with antimuscarinics (e.g., oxybutynin)
o • Occurs with coughing and sneezing
~
CL
• Presents as a continuous leakage of urine
w
0::

RAP! O- F! RE FACTS
Main artery of the pelvis

Main nerve of the pelvis

"Bag of worms" scrotum

[ 240 J
GENITAL EMBRYOLOGY
H: Chapter 346
Phys: Chapter so

Genital embryology (FA15 p567) (FA16 p571) (SU p196-198)


Vaginal atresia
Bicornuate uterus (FA15 p568) (FA16 p572)
SRY gene (FA15 p567) (FA16 p571) (SU p196)
Congenital penile abnormalities (FA15 p569) (FA16 p573) (SU p201)
Genital homologues (FA15 p568) (FA16 p572)
Gonadal descent (FA15 p569) (FA16 p573) (COA p205)

Warm -Up Rev iew


1. What are the common side effects of 13-blockers? (FA15 p256) (FA16 p249)

2. Describe how the murmur of mitral regurgitation is different from the murmur of
aortic regurgitation. (FA15 p279) (FA16 p273)

3. In which order of elimination (zero or first) would you see a linear decrease in the .
plasma concentration of a substance when plotted against time? (FA15 p244) (FA16 p238)

Cl
rn
z
):;!
,-
rn
sCD
:;;o
-<
0
,-
0
Gl
-<

4. Exstrophy of the bladder - congenital gap in the anterior bladder wall and abdominal wall in front of it
~ exposure of the bladder interior to the outside world

[ 24 1 ]
End of Session Quiz
5. Which structures develop from the mesonephric duct system?

6. What is the male homologue to each of the following female structures?


• Vestibular bulbs
• Labia minora
• Bartholin glands
• Urethral and paraurethral glands

7. What gene product comes from the SRY gene that underlies male genital
development?

8. Which structure anchors the testes to the scrotum?

>-
\C)
0
_J

0
>-
0:::
co
2:
w
_J

~
z
w
\C)

[ 242 J
ANDROGENS
Seminiferous tubules (FA15 p572) (FA16 p576)
Spermatogenesis (FA15 p577) (FA16 p583) (SU p199)
Regulation of spermatogenesis (FA15 p572, 577) (FA16 p576, 583)
Control of reproductive hormones (male) (FA15 p595) (FA16 p600) (SU p171)
Androgens (FA15 p577) (FA16 p582)
Testosterone, methyltestosterone (FA15 p597) (FA16 p603) (SU p223-224)
Antiandrogens (FA15 p597) (FA16 p603) (SU p223-224) (GG p1204)
Androgen insensitivity syndrome (FA15 p579) (FA16 p586) (SU p201) (H p3051)
Sa-reductase deficiency (FA15 p579) (FA16 p586) (SU p201)
Pseudohermaphroditism (FA15 p579) (FA16 p586) (SU p201)
True hermaphroditism (FA15 p578) (FA16 p585) (SU p201)

War p .Review
1. .What mediCations have anticholinergic side effects?

2. Which drug categories have the following endings? (FA15 p261) (FA16 p255)
• -ane
• -azine
• -barbital
• -cillin
• -ipramine

3. A 21-year-old man presents with five days of fever, chills, and an enlarged, painful
knee. Which organism is most likely responsiblefor his symptoms? (SU p235)

)>
z
0
:;:I:J
0
Cl
m
z
Vl

r 2'131
End of Sess1on Quiz
4. How does the dosing schedule of a GnRH agonist affect gonadotropin release?

5. What is the difference between androgen insensitivity and Sa-reductase deficiency?

6. What are the roles of Sertoli cells and Leydig cells in spermatogenesis?

U)
z
ll.J
<..'J
0
0:::
0
z
<t:

r 7441
TESTICULAR PATHOLOGY
H: Chapters 96, 346
Epididymitis (SU p218-219) (R p973)
Torsion (SU p218-219) (R p974) (H p1097)
Cryptorchidism (FA15 p592) (FA16 p597) (SU p218-219) (R p972)
Testicular germ cell tumors (FA15 p593) (FA16 p598) (SU p218-219) (R p975)
Testicular non-germ cell tumors (FA15 p593) (FA16 p598) (SU p219) (R p979)
Tunica vagina/is lesions (R p980)

Revi ew
1. Which elbow injury is common in tennis players? Which elbow injury is common in ·
golfers? (SU p242)

2. a is the general by-product of phase I metabolism? What is the general by-


product ofphase II metabolism? What reactions take place in phase I metabolism?
What reactions take place in phase II metabolism? (FA15 p244) (FA16 p238)
Phase I Phase II

-1
rrl
(J)
-1
()
c
3. Which three enzymes are required to convert phenylalanine to dopamine? (FA15 p107) ';;o
)>

(FA16 p95) --o


~
:r:
0
,-
0
Gl
-<

245 I
4. Epididymitis
• Inflammation of the epididymis
• Diagnosis: support of the testes ~ some relief
• Treatment < 35 yo = GC/Chlamydia
Ceftriaxone IM then doxycycline x 10 days
• Treatment > 35 yo or history of anal interco urse = Enterobacteriaceae
Fluoroquinolone x 10-14 days

5. Testicular torsion
• Twisting of the spermatic cord ~ ischemia
• Diagnosis: support of testis ~ no re lief, US
• Treatment: surgical detorsion with bilateral orchiopexy with in six hours

6. Cryptorchidism
• Failure of testis to descend into scrotum
• Usually unilateral
• Descent usually complete in first year of life
• 35x increased risk of malignant tumor in the undescended testicle (usua lly a germ cell tumor)

End of Sess ion Quiz


7. A 23-year-old patient presents with one testicle. For what is this patient at risk?

>-
(..?
0
-'
0
I
~
[L 8.. Which testicular tumor is described by each of the following statements?
0:::
<I: • Composed of cytotrophoblasts and syncytiotrophoblasts
-'
:::J
u
• May present initially with gynecomastia
f-
(f)
• Elevated AFP
w • Elevated S-hCG
f-
• Most common testicular tumor
• Most common testicular tumor in infants and children up to 3 years of age
• Most common testicular tumor in men over age 60
• Histologic appearance simi lar to koilocytes (cytoplasmic clearing)
• Histologically may have alveolar or tubular appearance sometimes with papillary
convolutions
• Composed of multiple tissue types
• Histologic endodermal sinus structures (Schiller- Duval bodies)
• 25% have cytoplasmic rod-shaped crystalloids of Reinke
• Androgen-producing and associated with precocious puberty

r 2461
PENIS AND PROSTATE PATHOLOGY
COA: Chapter 3
Phys: Chapter 80
H: Chapters 48, 95
R: Chapters 21, 22

Sexual dysfunction (FA15 p516) (FA16 p520) (SU p221) (H p374)


Erectile dysfunction (COA p427) (H p375)
Phosphodiesterase inhibitors (FA15 p598) (FA16 p603) (SU p224-225) (H p377)
Prostatitis (FA15 p594) (FA16 p599) (R p981) (H p2391)
Benign prostatic hyperplasia (FA15 p594) (FA16 p599) (SU p217) (R p982)
Prostatic adenocarcinoma (FA15 p594) (FA16 p599) (SU p217) (R p983) (H p799)

Warcn -'-U p Review


1. WhCi1:.isthe function of the protein that is absent in Duchenne muscular dystrophy? (FA15p85)
(FAf6p73)

2. What is the most likely presentation of methanol toxicity?

3. skin disorder matches each of the following statements? (FA15 p439-443)


(FA16 p440"445) (SU p252)
• Pruritic, purple polygonal papules
• Pruritic vesicles associated witfl celiac disease -o
m
• Thickened scar especially around the face and chest z
Vl
• .Parakeratotic scaling )>
• Keratin-filled cysts z
0
• Skin rash and proximal muscle \MP-:ll/r)/0 u
:::0
• Honey-crusting lesions common around the nose and 0
Vl
• Hyperkeratosis and koilocytosis g
m
u
~
:r:
0
4. Erectile dysfunction (ED) '
0
Gl
-<
• Failure to initiate (psychogenic, endocrinologic or neurogenic)
• Failure to fill (atherosclerosis)
• Failure to store adequate blood volume within the lacunar network
• Diabetic, atheroscl erotic and drug-related causes account for > 80% of cases of ED in older men

r :>47 1
5. What are the different types of Bowen disease?
Bowen disease
• Gray, solitary, crusty plaque on the penile shaft or the scrotum
• Can occasionally progress to invasive squamous cell carcinoma
Erythroplasia of Queyrat
• Red, velvety plaque usually involving the glans
Bowenoid papulosis
• Multiple papular lesions which do not become invasive
• Typically affects younger individuals

6. Condyloma acuminatum
• Benign genital warts
• Caused by HPV types 6 and 11

7. Benign prostatic hyperplasia (BPH)


• Present in 80% of men over age 80
• Diagnosis based on the following symptoms:
Sensation of incomplete voiding
Increased urinary frequency (less than every two hours)
Straining to void
Intermittent or weak urine stream
Urgency
Nocturia (at least 2-3 times a night)
• Palpable prostate size may not correlate with degree of obstruction or symptom severity
>-
(_:J
0__J
• Medical intervention
0 Nonselective a -blockers: doxazosin, prazosin, terazosin
1
::r:
~ • Decrease prostate smooth muscle tone ~ immediate improvement in urine flow
0..
w • Side effects: dizziness, postural hypotension, fatigue, asthenia
~
(/)
Selective a 1A.D -blocker: tamsulosin
0 • Fewer side effects than nonselectives, has no antihypertensive effects
er::
0..
Sa-reductase inhibitors: finasteride, dutasteride
0
z • Slowly reduce dihydrotestosterone levels ~ 20% decrease in prostate volume
<(
(/) over 3 -6 months
z
w • Side effects: decreCJsed libido, ejCJcuiCJtory disorder, impotence
0..

• Surgical intervention

[ 248 J
End of Session Quiz
8. Whatist

9. What are the side effects of sildenafil?

10. Which organism is commonly implicated in balanitis?

e~ mechanism of action and clinical use of flutamide differ from finasteride?

12. A 55-year-old man undergoing treatment for BPH has increased testosterone and
decreased dihydrotestosterone (DHT) as well as gynecoma · and edema. With what
medication is he being treated?
--o
m
z:
(J)

)>
z:
0
--o
;;o
0
13. What are the diagnostic symptoms of BPH? (J)

g
m
--o
~
::r
0
,---
0
Cl
-<

RAP/O-F! RE FACTS
Most common cancer in men

~, Most common cause of urinary obstruction in men

Most common treatment of erectile dysfunction

r 749 1
FEMALE REPRODUCTNE CYCLE
Phys: Chapter 81

Oogenesis (FA15 p575) (FA16 p578) (SU p199)


Hypothalamic-pituitary-ovarian axis (FA15 p595) (FA16 p600) (SU p771)
Estrogen (FA15 p573) (FA16 p577) (SU p204)
Progesterone (FA15 p573) (FA16 p577) (SU p205)
Female reproductive cycle (FA15 p574) (FA16 p579) (SU p204)
Pregnancy and hCG (FA15 p576) (FA16 p580) (SU p209)
Lactation (FA15 p576) (FA16 p581) (SU p209)

Warm-Up
1.. What are some examples of substances eliminated at a constant rate (zero-order
elimination)? (FA15 p244) (FA16 p23B) (SU p14)

2 . . ·Which enzymes are used to metabolize alcohol? (FA15 p95) (FA16 pB3) (SUp304)

3. A30-year-old woman presents with a low-grade fever, a·rash across her nose that gets
worse when she is out in the sun, and widespread.edema. Which blood test should be
ordered to confirm the diagnosis? (FA15p433) (FA16 p434)
w
_J
u
>-
u
w
>
f-
u
:::J
0
0
0:::
[}__
w
0:::
LLJ
_J
<(
:2:
w
u_

r 2501
4. Two-cell theory of estradiol production
lflllllll/ll ------ Pulsatile GnRH------------
.. Cholesterol
\ Theca cell Granulosa cell

Desmolase
Estrone

IAromatase
Androstenedione ----l~'l-----<.... Androstenedione
' ' '11'--- - Basement membrane

5. Female reproductive cycle

Follicle Maturing Follicle Ovulation Corpus luteum Degenerative


~ I
C. Luteum

"

"T1
rn
:s
2
MENSTRUATION
4 6 8 '
10 12 14 t 16 18 20 22 24 26 28
)>
r
rn
Spike in body ::0
rn
temperature. u
::0
0
0
c
()
---1
6. Outline the steps of the female reproductive cycle. <
rn
• 1' FSH ()
-<
()
• Follicle maturation r
rn
• 1' estrogen
• Negative feedback -7 positive feedback
• LH surge triggers ovulation
• Formation of corpus luteum
• 1' progesterone (and estrogen)
• J_ FSH and LH
• Degeneration of corpus luteum
• l progesterone and estrogen
• Menstruation, t inhibition of FSH
• I FSH

r 2s1 1
End.of Sessio n Qu iZ
7. During which stage of meiosis is the primary oocyte arrested? The secondary oocyte?

8. Which hormone (estrogen or progesterone) is responsible .for each of the following


actions?
• Production of thick cervical mucus that inhibits entry of sperm into the uterus
• Induces LH surge
• Relaxes uterine smooth muscle
• Stimulates endometrial proliferation
• Inhibits endometrial proliferation

9. Outline the steps that occur during the female reproductive cycle.

w
__J
u
>-
u
w
>
10. Human chorionic gonadotropin has an alpha subunit identical to which other
1-
u
hormones?
:::::J
0
0
0::
()_
w
0::
w
--'
<(
::2:
w
LL

11. What hormonal changes facilitate lactation?

r 2521
MENSTRUATION AND MENOPAUSE
R: Chapter 22
H: Chapter 97
Menstruation (FA15 p574) (FA16 p579) (SU p203-205)
Estrogens (FA15 p596) (FA16 p601) (SU p209)
Progestins (FA15 p596) (FA16 p602) (SU p209)
Contraception (FA15 p597) (FA16 p602) (SU p209)
Menopause (FA15 p576) (FA16 p582) (SU p208-209)
Hormone replacement therapy (FA15 p596) (FA16 p601) (SU p225)

Warm-Up Rev1ew
1. A patient falls off a motorcyc;le and lands on his right shoulder. On physical exam,
yg~ notice his shoulder hasanabnormal configur~tion. X·rays indicate ~nanteri()r
disle>cati.on of his shoulder. Which artery and nerve are most at risk of being
darn aged?

2. A 75-year-old man presents with acute knee pain and swelling. An x-ray reveals
absence of erosion of the joint space, but there are .calcium deposits in the menisci.
What is the diagnosis? Whatwould you find on aspiration of the joint? (FA1S p431)
(FA16 p431) (SU p233)

s:
rn
:z
(J)
3. What are some of the inducers of cytochrome P450? (FA15 p260) (FA16 p254) ___,
:::0
c
~
0
:z
)>
z
0
s:
rn
:z
0
-o
)>
c
Ul
rn

i ?.S.i l
4. Methods of contraception
Class Examples Notes
• Condom
• Diaphragm
Barrier
• Sponge

• Pil l ("OCPs") •
• Patch • HTN
Combined
• Vaginal ring • Ml and stroke
estrogen-progestin

• Pill ("M inipill") • Irregular bleeding


• DMPA injection (Depo-Provera) • DMPA: weight gain, mood
Progestin-only • Etonogestrel implant (Nexplanon) changes,
• Levonorgestrel IUD (Mirena)


• Increased risk of PID w it hin 3
Other Copper IUD
weeks of insertion

5. Menopausal hot flashes


• Occur in 75% of menopausa l women
• Presentati on: starts in face/chest t hen generalizes, associated w it h diaphoresis and pa lpitati ons,
fol lowed by chi lls and sh ive ri ng; lasts 2-4 minutes
• May cause sleep disturbances
• Treatment: > SSRI, SNRI, gabapentin
• Herbal treatment: soy isoflavonest, red clovert, black cohosht, vitam in E
w
(f) tpotentia l estrogenic effect on the breast, much like estrogen replacement
:::>
<I:
0...
0
z
w
z: 6. Hormone replacement therapy for menopause
0
z • Use estrogen + progesterone if uterus present
<I:
z Unopposed estrogen increases risk o f - -- -- - - -- - - -- - - - -- - -
0
:;: • For relief of menopausal symptoms, not to prevent chronic disease
:::>
0:: • Use for only shortest amou nt of ti me and at lowest dose needed
1-
tn
z • Avoid if:
w
z: Coronary heart disease
- Active liver disease
History of _ _ _ __ __ _ _ _ _ _ _ __ _ __

r 2541
En d ofSession Quiz
7. What is a common cause of irregular bleeding in the first few years after menarche?
What is the treatment?

8. A 23-year-old woman who uses OCPs for contraception has a positive screellillg
test for TB. She is started oil the appropriate therapyfor latent TB. Despite good
cOmpliance with all her medications, she becomes.pregllant. What is the most likely

9. VVhat hormonal changes are seen during menopause?

10. Why is progesterone used in combination with estrogen during hormone


replacement therapy? :s
m
z:
(J)
___,
:::0
c
~
0
z:
)>
z:
0
:s
m
z:
0
\J
)>
c
(JJ
m
RAPID -FIRE FACTS
I Hepatic adenoma

r 255 1
WLVA, VAGINA AND CERVIX
H: Chapter 97
R: Chapter 22
Epithelial histology (FA15 p570) (FA16 p574)
Lichen sclerosus
Vulvar cancer (SU p215)
Vaginal tumors (FA15 p584) (FA16 p591) (SU p215)
Cervical dysplasia and cancer (FA15 p584) (FA16 p591) (SU p215)

Warm- Up Rev1ew
1. What are the by-products of MAO and COMT enzymatic activity on dopamine,
norepinephrine, and epinephrine?

2. What locations are known for tophi formation in patients with gout? (FA15 p430)
(FA16 p431)

3. What is the mechanism of action of the bisphosphonates? (FA15 p445) (FA16 p447)

4. Lichen sderosus
• Inflammatory condition of vu lva characterized by thinn ing of the epidermis
• Produces smooth white plaques
• Can lead to shrinking of introitus and fusion of labia majora and minora
• Symptoms: pain, _ __ _ _ _ ____ , dyspareunia
• Diagnosis: biopsy
• Treatment: topical corticosteroids
• Associated with increased risk of _ _ _ _ _ _ _ _ _ _ _ __

1256 l
En Session Qui
5. Which type of epithelium covers each of the following structures?
• Ectocervix
• Endocervix
• Fallopian tube

6. What conditions are associated with diethylstilbestrol use during pregnancy?

7. What are the risk factors for cervical cancer?

8. What is the mechanism by which HPV facilitates oncogenesis?

<
c

~
<
)>
Gl
z
RAP! O-Ft RE FACTS )>
)>
z
0
"Grape-like" vaginal tumor n
m
:::D
<
Koilocytes X

12S7l
UTERINE PATHOLOGY
H: Chapter 97
R: Chapter 22
Uterine basics
Endometrial hyperplasia and carcinoma (FA15 p588) (FA16 p594) (SU p215)
Endometritis (FA15 p588) (FA16 p594)
Endometriosis (FA15 p588) (FA16 p594) (SU p206-207)
Danazol (FA15 p597) (FA16 p602) (SU p417)
Adenomyosis (FA15 p588) (FA16 p594)
Leiomyomas (FA15 p588) (FA16 p594) (SU p215)
Leiomyosarcoma (SU p215)
Leuprolide (FA15 p596) (FA16 p601) (SU p225)

Warm - Up Rev1ew
1. What are some medications that have a low therapeutic index? (FA15 p246) (FA16 p240)

2. Which bone disorder re~ults from excess PTH?

3. What are the layers of the epidermis? (FA15 p436) (FA16 p437) (SU p251)

>-
l'J
0
--'
0
I
~
o_
LLJ
z
0:::
LLJ
1---
:::J

4. What are the layers of the endometrium? Which layers are shed during menstruation?

r 2ss 1
5. leiomyosarcoma
• Rare malignant tumor that arises de novo from the myometrium
• Suspect in woman with a rapidly enlarging uterus

6. Uses of leuprolide
• Leiomyomas (short term to shrink tumors prior to surgery)

• Central precocious puberty
• Advanced prostate cancer
• Infertility
- Given continuously to suppress intrinsic hormone production during IVF
- Given once to induce ovulation

End of Sess· n Quiz


7. What causes endometrial hyperplasia?

8. What uterine pathology matches each of the following descriptions?


• Excess unopposed estrogen is the main risk factor
• Characterized by plasma cells in the endometrium
• Causes cyclic pe!vicpain +/- bowel and bladder symptoms
• Menorrhagia + nontender, enlarged uterus with irregular contours
• Menorrhagia +tender, enlarged, globular uterus
• Malignant tumor that arises de novo from the smooth muscle of the uterus
c
---1
m
/0
:z
m
9. What is the mechanism of action of leuprolide? "D
~
:r:
0
,--
0
Gl
-<

RAPID - FIRE FACTS


Most common benign tumor in women

~ "Chocolate cyst" of the ovary

"Powder burn" lesion

[ 259]
OVARIAl PATHOLOGY
H: Chapter 97
R: Chapter 22
Ovarian basics (Phys p989-991)
Ovarian cysts (FA15 p585) (FA16 p592)
Ovarian torsion
Polycystic ovarian syndrome (FA15 p585) (FA16 p591) (SU p206)
Clomiphene (FA15 p596) (FA16 p601) (SU p224)
Infertility

Warm - Up Revrew
1. What are the risk factors for osteoporosis? (FA15 p425) (FA16 p426)

2. A man presents with exquisite pain and swelling in the metatarsophalangeal joint of
his right great toe. Joint aspiration reveals needle-like crystals. What is the diagnosis?
(FA15 p430) (FA16 p431)

3. Which structure connects epithelial cells to the basement membrane? Which


structure connects adjacent epithelial cells to each other? (FA15 p437) (FA16 p43B)

>-
\.:J
0
--'
0
I
~
0... 4. Ovarian torsion
z:
-<t: • Twisting of supportive ligam e nts~ impaired vascular supply~ ischemia and necrosis
a:::
<t • Risk increases with _ __ _ __ _ _ _ _ _ _ _ _ _ _ __ _ __
>
0
• Presentation: acute onset of severe pelvic pain (sharp, stabbing) with possible radiation, +/- nausea
and vomiting
• Diagnosis: clinical , pelvic ultrasound
• Treatment: _ _ _ _ __ _ __

[ 260 J
5. What hormonal abnormalities are seen in polycystic ovarian syndrome?

6. Evidence of ovulatory cycles:


• Cyclic menses
• : midcycle pelvic pain associated with ovulation
• LH surge: tested for by OTC ovulation prediction kits
• Biphasic basal body temperature

nd of Sessio·n.··.·Qu iz
7. Which type of ovarian cyst matches each of the following descriptions?
• Forms from an unruptured Graafian follicle
• Forms if the corpus luteum fails to involute
• Associated with high hCG levels
• C::.ontains tissue from multiple germ cell layers
• "Chocolate cyst" of the ovary

8. What are the clinical manifestations of polycystic ovarian syndrome?

0
<
)>
9. A patient with polycystic ovarian syndrome is at an increased risk for developing :::0
)>
which type of cancer? :z:
u
~
::r:
0
,---
0
Gl
-<
10. What is the mechanism of action of clomiphene?

RAP ID - FIRE FACTS


Most common cause of infertility in women

Midcycle pelvic pain associated with ovulation

r 261 1
OVARIAN NEOPLASMS
R: Chapter 22
Ovarian cancer (FA15 p586) (FA16 p592)
Epithelial tumors (FA15 p586-587) (FA16 p592-593) (SU p214)
Germ cell tumors (FA15 p586-587) (FA16 p592-593) (SU p214)
Sex cord-stromal tumors (FA15 p586-587) (FA16 p592-593) (SU p214)
Metastatic tumors (FA15 p587) (FA16 p593) (SU p214)

Warm- Up Review
1. Patients with which congenital heart defects would benefit from an increase in
afterload? (FA15 p2BB-289) (FA16 p282-283)

2. What drugs can be used to reverse neuromuscular blockade?

3. Which psychiatric disorder is associated with increased levels of norepinephrine?


(FA15 p453) (FA16 p455) (SU p55)

l/l
~
l/l
<(
___J
0..
0
LU
z
z 4. What are the four main categories of ovarian tumors?
<(
0:::
<(
>
0

r 262 l
5. Ovarian neoplasms

Epithelial Germ Cell


Sex Cord-Stromal Tumors
Tumors Tumors
• Serous • Teratoma • Granulosa cell
• Mucinous • Dysgerminoma • Sertoli - Leydig
Subtypes • Endometrioid • Yolk sac • Fibroma
• Clear ce ll • Choriocarcinoma • Thecoma
• Brenner

Age

Unilateral/
Bilateral

• Serous cystadenoma • Fibroma


• Mucinous cystadenoma • Thecoma
Benign •

• Serous • •
cystadenocarcinoma • Dysgerminoma •
• Mucinous • Yolk sac
Malignant cystadenocarcinoma • Choriocarcinoma

Prognosis

0
<
)>
:;;o
)>
z
z
rn
0
-o
,-
)>
(J)

~
(J)

f 263 l
End of Sess1on Qu1z
6. What are the risk factors for ovarian cancer?

7. Which ovarian neoplasm is associated with each of the following statements?


• Estrogen-producing ~ precocious puberty or postmenopausal bleeding
• Androgen-producing ~ virilization
• Contains fallopian tube-like epithelium
• Contains urinary tract-like epithelium
• Historically associated with pseudomyxoma peritonei

8. Which ovarian neoplasm is associated with each of the following histological


findings?
• Schiller-Duval body
• Cali-Exner body
• Psammoma body
• "Fried egg" cells

9. Which ovarian neoplasm is associated with each of the following tumor markers?
• AFP
• hCG
• LDH
U)
2:
U)
<(
_j
0...
0
L!.J
z
~ RA P! o - Ff RE FACTS
0:::

~ Ovarian tumor+ ascites + hydrothorax

Teratoma~ hyperthyroidism

[ 264]
PREGNANCY PART 1
H: Chapter 7
Twinning (FA15 p561) (FA16 p565)
Ectopic pregnancy (FA15 p583) (FA16 p589) (SU p211)
Gestational trophoblastic disease (FA15 p580) (FA16 p587) (SU p211-212)
Umbilical cord (FA15 p563) (FA16 p567)
Amniotic fluid abnormalities (FA15 p583) (FA16 p589)
Rh alloimmunization
Prenatal testing (FA15 p86) (FA16 p74) (SU p210-211)
Physiologic changes in pregnancy

Warrn-.Up Rev1ew
Which cardiac abnormality is associated with each the following murmurs?
(FA15 1]278:279) (FA16p272-273)
• .Crescendo -'decrescendo systolicmurmur best heard in the2nd .and 3rd right intercostal
spaces close to the sternum
• Rumbling late diastolic murmur with an opening snap
• Pansystolic (or holosystolic) murmur best heard in the LI-th, 5th, and 6th left intercostal
spaces, adjacent to the sternum
• Continuous, machine-like murmur

2. Outline the pathway by which stimulation of a Gq .r~ceptor activates protein kinase C.


(FA1Sp248) (FA16 p242) (SUp173)

u
;;;o
m
Gl
3. Gestational trophoblastic disease z
)>
z
• Hydatidiform mole n
-<
• Invas ive mole u
)>
;;;o
• Choriocarcinoma -i

Partial Mole Complete Mole


Karyotype
Fetal parts
Chorionic villi
Uterine size
hCG
Risk of invasive mole
Risk of choriocarcinoma

[ 265 J
4. Rh alloimmunization
• Rh(D)-negative woman develops lgG antibodies against Rh(D)-positive fetus
• In subsequent pregnancies, anti-D antibodies cross placenta and attack fetal RBCs ~ hemolytic
disease of the fetus and newborn (erythroblastosis fetalis)
• Prevent by administering anti-D immune globulin (e.g., Rhogam) when risk of fetomaternal
hemorrhage

5. Physiologic changes in pregnancy


• Basal metabolic rate increases 10-20%
• Plasma volume increases 30 -50%, RBC volume increases 20 -30%
• Cardiac output increases 30-50%
• Blood pressure decreases in early pregnancy ~ nadir at 24-26 weeks, return to pre-pregnancy
levels by term
• Increased GFR ~decreased BUN and Cr
• Increased procoagulation factors~ hypercoagulable state

End of Sess1.on Quiz


6. What are the risk factors for ectopic pregnancy?

7. Which type of ovarian cyst is associated with molar pregnancies?

8. What is the most common location for metastasis in choriocarcinoma?

1--
0:::
9. Failed involution of which embryological ·structure is associated with each of the
<(
o_ following statements?
>-
u • Drainage of urine from the umbilicus
:z:
<( • Drainage of meconium from the umbilicus
:z:
l.9
w
0:::
o_
10. What condition is suggested by each of the following abnormalities on second
trimester maternal serum screening?
• i AFP
• .! AFP, ! estriol, i hCG
• ! AFP, ! estriol, ! hCG

RA PID-FIR E FACTS
"Snowstorm" appearance on ultrasound

[ 266]
PREGNANCY PART 2
H: Chapter 7
Abnormal placentation (FA15 p582) (FA16 p588) (SU p211)
Placental abruption (FA15 p582) (FA16 p588) (SU p211)
Postpartum hemorrhage
Hypertension in pregnancy (FA15 p581) (FA16 p590) (SU p211)
Diabetes in pregnancy (SU p212)
Terbutaline (FA15 p597) (FA16 p602) (SU p225)
Prostaglandin drugs (SU p225)
Mifepristone (FA15 p597) (FA16 p602) (SU p223)

Warm-Up Review
1. What effect win stimulatingthe following receptors have on heart rate? (FA15 p24B)
(F,A.16p242)
• . f)l
• ()2
• Mz

2. What are the primary mechanisms of ac.tion of the different classes of


antiarrhythmics? (FA15 p308-310) (FA16 p302 304) 2

• Class I
• Class II
• Class Ill
• Class IV

3. A .patient COIJl~S to the clinic ~omplaining of anterior shoulder paln .that radiates
down the forearm. On examination, there is swelling of the biceps muscle. What is
the most likely diagnosis?

--o
;;o
rn
Cl
z
)>
z
()

4. What are the causes of postpartum hemorrhage? -<


u
)>
• (most common cause) ;;o
---1
Enlarged, soft, boggy uterus N

Risk factors: overdistended uterus (large fetus, multifetal gestation), induced or augmented labor,
prolonged labor
• Retained pl acental tissue
• Genital lacerations
• Abnorma l placentation (placenta accreta / increta/ percreta)
• Uterine rupture
• Coagulation defects

[ 267]
5. Diabetes in pregnancy
• Gestational diabetes
Diabetes that develops during pregnancy but resolves postpartum
Screen with ora l glucose tolerance test between 24-28 weeks gestation
Management: diet +/- insulin
Complications: _ _ _ __ _ _ __ , hypoglycemia
• Pregestational diabetes
Patient has diabetes prior to pregnancy
Management: insulin
Complications: macrosomia, hypoglycemia, congenital anomalies
(e.g., - -- -- -- - -- - - -- - - - - -- - - -- - - ' ' sti llbirth

End of Sess1on Ou1 z


6. Which placental abnormality matches each of the following descriptions?
• Placenta covers internal cervical os
• Premature separation of placenta
• Abnormal adherence of placenta to myometrium
• Invasion of placenta into myometrium
• Invasion of placenta through myometrium and serosa
• Presents as painless vaginal bleeding late in pregnancy
• Presents as painful vaginal bleeding late in pregnancy

7. What is the most common cause of postpartum hemorrhage?

N
f-
8. V\fhat condition is best described by each of the following statements?
0:::
<( • Patient with PMH of HTN becomes pregnant
o_

>- • New-onset HTN during pregnancy with no proteinuria


u
z • New-onset HTN during pregnancy + proteinuria or end-organ dysfunction
<(
z • Patient with preeclampsia has a seizure
(_:J
w
0:::
• Patient with preeclampsia has low hemoglobin, low platelet count, and high AST
Q_

9. Whatis the mechanism of action of mifepristone? What is the mechanism of action of


terbutaline?

[ 268 J
CHROMOSOMAL DISORDERS
R: Chapter 5
H: Chapters 61, 62
Autosomal trisomies (FA15 p86) (FA16 p74) (SU p202)
Down syndrome (FA15 p86) (FA16 p74) (SU p202)
- Robertsonian translocation (FA15 p87) (FA16 p75) (SU p202)
- Edwards syndrome (FA15 p86) (FA16 p74) (SU p202)
- Patau syndrome (FA15 p86) (FA16 p74) (SU p202)
Sex chromosome disorders (FA15 p578) (FA16 p585) (SU p202-203) (R p164-167)
Cri-du-chat syndrome (FA15 p87) (FA16 p75) (SU p202)
Williams syndrome (FA15 p87) (FA16 p75)
22q11 deletion syndromes (FA15 pBB) (FA16 p76) (SU p382) (R p163)

Wa -Up Rey1ew
1. What are the signs ar'ld symptoms of excessive cholinergic stimulation? (FA15 p250)
(FA16 p244) .

2. Which psychiatric disorder is associated with decreased norepinephrine? (FA15 p453)


(FA16 p455)

n
Patients who have migraines with aura are at an increased risk for what other :r:
:::0
neurologic condition? 0
s:
0
Ul
0
s)>
,--
0
Ul
0
:::0
0
m
:::0
Ul

r 2691
End .of Sess1on Quiz
4. What prenatal ultrasound finding is a clue that the fetus may have Down syndrome?

5. What is the most common event that causes Down syndrome: meiotic
nondisjunction, mosaicism, or Robertsonian translocation?

6. What are the clinical features of Williams syndrome?

7. What are the possible defects in cases of chromosome 22q11 deletion?

8. What are the distinguishing characteristics of Klinefelter syndrome compared to Turner


if) syndrome?
0:::
LL)
0
er::
0
if)

0
_J
<{
2:
0
if)
0
~
0
0:::
I
u
RAP/ D-FI RE FACTS
Most common genetic cause of intellectual
disability

Second most common genetic cause of


intellectual disabi lity

Horseshoe kidney, congenita l heart defects,


streak ovaries, and cystic hygroma

Rocker-bottom feet, clenched hands, microcephaly


with prominent occiput and sma ll jaw

r 27o 1
GENmc DISORDERS: AD AND TRINUCLEGnDE REPEATS
R: Chapter 5
Autosomal dominant diseases (FA15 p83) (FA16 p71) (SU p395)
Trinucleotide repeat diseases (FA15 p83, 85) (FA16 p71, 73) (SU p395)
Fragile X syndrome (FA15 pBS) (FA16 p73) (SU p203)
Friedreich ataxia (FA15 p472) (FA16 p475) (SU p34)

Warm -Up Review


1. Which serum lab marker is helpful in assessing the appropriate oxygenation of tissues?

2. What are the five branches of the facial nerve? (SU p41)

Cl
m
:z
_,
m

n
0
3. What should be suspected as a cause of headache in a patient using topical retinoic (})
0
· acid forthe treatment of acne? (FA15 p46B) (FA16 p471) (SU p312) ;:<J
0
m
;:<J
(})

)>
0
)>
:z
0
_,
;:<J
:z
c
n
r
m
_,
0
0
m
4. Autosomal dominant polycystic kidney disease ;:<J
rn
--o
• Bilateral enlargement of kidneys due to multiple cysts m

• Mutation of APKD1 on chromosome 16 ~


(})

• Associated with berry aneurysms

5. Achondroplasia
• Defect of fibroblast growth factor receptor
• Associated with advanced paternal age

[ 27 1 l
I

6. Familial adenomatous polyposis


• High risk of co lon cancer
• Mutation in APC gene

7. Gardner syndrome
• Osteomas, lipomas, sebaceous cysts, co lon polyps, colon cancer

8. Familial hypercholesterolemia
• Type IIA hyperlipidemia
• Markedly elevated LDL
• Xanthomas, xanthelasmas, early Ml

9. Hereditary hemorrhagic telangiectasia (Osler-Weber-Rendu syndrome)


• Telangiectasias, recurrent epistaxis, skin discoloration, AVMs , Gl tract bleeding

(f)

~ 10. Hereditary spherocytosis


UJ
a...
UJ • Spheroid erythrocytes
0:::
UJ
0
• Defect of spectrin or ankyrin
1-
0
• Hemolysis leading to anemia and jaundice, increased MCHC
UJ
___J
• Treatment: splenectomy
u
::::J
z
0:::
1-
0
z
<[
11. Huntington disease
0
<[
• Neurodegenerative disorder
(f) • Decreased GABA and acetylcholine in the brain
0:::
UJ
0 • Cogn itive decline, caudate atrophy, chorea, CAG repeats , chromosome 4, average age 40
0:::
0
(f)

0
u
i=
UJ
12. Marfan syndrome
z
w • Fibrillin gene mutation
l'J
• Pectus excavatum, hyperextensible joints, long fingers and toes
• Cystic medial necrosis of aorta, aortic va lve regurgitation, aortic aneurysm, aortic dissection, mitral
valve prolapse
• Subluxation of lenses

[ 2.72 J
13. Multiple endocrine neoplasias
• MEN1 associated with MEN1 gene
• MEN2A and 28 associated with RET gene mutation
• MEN1: (PPP) parathyroid ad enomas, pituitary adenomas, pancreatic tumors
• MEN2A: (PPM) parathyroid adenomas, pheochromocytoma and medullary thyroid cancer
• MEN28: (PMM) pheochromocytoma , medullary thyro id cancer, mucosal neuromas

14. Neurofibromatosis type I (von Recklinghausen)


• Mutation of a gene on chromosom e 17
• Cafe -au -lait spots, neural tumors, Lisch nodules, scoliosis

15. Neurofibromatosis type II


• Bilateral acoustic neuroma (crania l nerve V III)
• Hearing loss, tinnitus, balance problem
• Mutation on chromosome 22

Gl
16. Tuberous sclerosis m
z
m
• Mutation of hamartin or tuberin gene -I
n
• Non-malignant tumors in the brain and other organs. facial lesions (adenoma sebaceum), 0
(J)
hypopigmented ash-leaf spots, cortica l and retina l hamartomas, seizures, intellectual disability, 0
;;o
cardiac rhabdomyomas and astrocytomas 0
m
;;o
(J)

)>
0
17. von Hippel-lindau disease )>
z
0
• Hemangioblastomas of the retina, cerebel lu m and medu lla -I
;;o
• 50% develop bilateral renal cell carcinoma z
c
• Pheochromocytomas n
r-
m
0
-I
0
m
;;o
18. Fragile X syndrome m
u
m
• X- linked disorder, trinuc leotide repeat disorder )>
-i
• Mutation of FMR1 gene which codes for FMRP (J)

• Intellectual disability, macro-orchid ism, long face, large jaw, everted ears. mitral valve prolapse

19. Friedreich ataxia


• Autosomal recessive, trinucleotide repeat disorder
• Mutation of gene that codes for frataxin
• Degeneration of sen sory neurons, hypertroph ic cardiomyopathy, diabetes mellitus
• Staggering gait, frequent falling, nystagmus, dysarthria , pes cavus and hammertoes
• Presents in childhood w ith kyphoscoliosis

r 2n 1
End of Sess1.o n Ou1z
20. What genetic defect is responsible for achondroplasia?

21. A genetic defect in which proteins results in hereditary spherocytosis?

22. What autosomal dominant disease fits each of the following statements?
• Associated with floppy mitral valve, dissecting aortic aneurysm
• Associated with mitral valve prolapse, liver disease, berry aneurysms
• Neural tumors and pigmented iris hamartomas
• Very strong association with colon cancer
• Ml before age 20
• Hemangioblastomas of retina/cerebellum/medulla
(/)
• Increased MCHC, hemolyticanemia
~
w • Bilateral acoustic neuromas
CL
w • Facial lesions, seizure disorder, cancer risk
0:::
w
0
• Caudate atrophy, dementia
1-- • Cystic medial necrosis of the aorta
0
w
_j
• Defect of fibroblast growth factor (FGF) receptor 3
u
~
z
0:::
1--
0
z 23. What are some of the disorders resulting from trinucleotide repeats?
<(
0
<(
(/)
0:::
w
D
0:::
0
(/)
D
u
1--
w
z
w
(C)

r 2741
GEimC DISORDERS: ARAND X·UNKED
R: Chapter 5
Autosomal recessive diseases (FA15 p84, 103) (FA16 p72)
Lysosomal storage diseases (FA15 p111) (FA16 p100)
X-linked recessive disorders (FA15 p84) (FA16 p72)

Warm-Up Review
1. What are the defects intetralogy offallot? (FA15 p2BB) (FA16 p282) (5Up64)

2. What are the.defects in Ebstein anomaly? (FA15 p269) (FA16p282)

3. Which antibiotic class is known to increase the risk of tendon rupture in adults? Cl
(FA15 p187) (FA16 p178) (SU p341) m
z
m
---;
n
0
(J)
0
;;cJ
0
m
;;cJ
(J)

)>
;;cJ
4. Cystic fibrosis )>
:z:
0
Lumen ~
r-
:z:
A
m
0

r 275 1
5. Cystic fibrosis
• Defect in CFTR gene on chromosome 7
• Recurrent pulmonary infection (Pseudomonas), bronch iectasis, pancreatic insufficiency, meconium
ileus, infertility in males (absence of vas deferens), fat soluble vitamin deficiencies, salty taste
• Diagnose with sweat choloride test (>60 mEq per liter)
• Treatment: N-acetylcysteine, antibiotics (f1uoroquinolones), pancreatic enzymes, fat soluble vitamins

6. Fabry disease
• X-linked recessive
• Deficient enzyme: alpha-galactosidase leading to accumulation of ceramide trihexoside
• Pain (damage to peripheral nerves), renal failure, hypertension, cardiomyopathy, angiokeratomas

7. Gaucher disease
• Autosomal recessive
• Most common lysosoma l storage disease
• Deficient enzyme: glucocerebrosidase leading to accumulation of glucocerebroside
• Hepatosplenomegaly, painful bony lesions, anemia, fatigue and thrombocytopenia
• Prominent blue cytop lasmic fibrils (crumpled tissue paper)

8. Niemann-Pick disease
0 • Autosomal recessive
L1.J
~
z
• Deficient enzyme: sphingomyelinase leading to accumulation of sphingomyelin
_.J
, • Hepatosplenomegaly, thrombocytopenia, ataxia, dysarthria, dysphagia , gradual worsening of
X
0 intellectual function
z
<( • Cherry red spot on the macula and foam cells
0:::
<(
Vi
0:::
L1.J
0
0::: 9. Tay-Sachs disease
0
(./)
• Autosomal recessive
0
u • Deficient enzyme: hexosaminidase A leading to accumu lation of GM2 ganglioside
r-
w
z • Worsening mental and physical abilities, death by age 4 years, cherry red spot on macula
L.U
\..9 • No hepatosplenomegaly

10. Krabbe disease


• Autosomal recessive
• Deficient enzyme: ga lactocerebrosidase leading to accumulation of galactocerebroside
• Affects myelin sheath (peripheral neuropathy), seizures, optic atrophy, weakness, developmental
delay
• Symptoms begin at 3-6 months, death by 2 years

r 276 1
11. Metachromatic leukodystrophy
• Autosomal recessive
• Deficient enzyme: Arylsulfatase leading to acculumulation of cerebroside sulfate
• Affects myelin sheath, muscle wasting, weakness. progressive vision loss, dementia

12. Hurler syndrome


• Autosomal recessive
• Deficient enzyme: alpha-L-iduronidase leading to accumulation of heparan sulfate and dermatan
sulfate
• Progressive deterioration, coarse facial features, hepatosplenomegaly, intellectual disability, poor
growth (resembles dwarfism), corneal clouding
• Hurler-Scheie and Scheie are lesser forms that have later onset

13. Hunter syndrome


• X-linked recessive
• Deficient enzyme: iduronate sulfatase leading to accumulation of heparan sulfate and dermatan
sulfate
• Milder form of Hurler, later onset of 1-2 years, no corneal clouding, aggressive behavior

14. X-linked recessive disorders


" Oblivious Female Will Give Her Boys Her (x)Linked Disorders"
C)
• Ocular albinism m
z
m
• Fabry disease ---1
n
• Wiskott-Aidrich 0
(./)
• G6PD deficiency 0
:::0
• Hunter syndrome 0
m
:::0
• Bruton agammaglobulinemia (./)

p
• Hemophilia A/B ;;o
p
• Lesch-Nyhan Syndrome :z:
0
• Duchenne muscular dystrophy >;<
,-
:z:
7'
m
0

r 2771
End of Sess1on Qui z
15. What gene is defective in cystic fibrosis?

16. What is the traditional test used to diagnose cystic fibrosis?

17. What two diseases can present with a cherry red spot on the macula?

18. Which lysosomal storage disease has prominent cytoplasmic fibrils that resemble
crumpled tissue paper?

o
UJ
19. Which two lysosomal storage diseases have myelin sheath pathology leading to
~ peripheral neuropathy?
_J
I
X
0
z
<(
0::
<(
Vi
0::
UJ
0
0::
0
VJ
0
u
1-
UJ
z
UJ
l'J

r 21s 1
BREAST
R: Chapter 23
Breast basics
Acute mastitis (FA15 p590) (FA16 p595) (SU p216) (R p1046)
Gynecomastia (FA15 p590) (FA16 p595) (SU p216) (R p1049) (H p3019)
Benign epithelia/lesions (FA15 p590) (FA16 p595) (SU p216)
Benign breast tumors (FA15 p589) (FA16 p595) (SU p216)
Malignant breast tumors (FA15 p590-591) (FA16 p596) (SU p216)
SERMs (FA15 p596) (FA16 p601) (SU p224) (GG p1761)
Aromatase inhibitors (FA15 p596) (FA16 p601) (SU p223) (GG p1761)

Warm-Up Rev1ew
1. Describe theichanges seen in serum calcium, serum phosphate, alkaline phosphatase,
and PTH in each of the following diseases: (FA15 p427) (FA16 p428)
· . ...

· .... ········
.. Disease Serum Ca2+ Serurn Phos AlkPhos PTH
...... .···

Primary
.
hyperparathyroidism ..
.
.· · ·

Paget disease of bone . ....


. ...
Vitamin D toxicity ..,
..

2. What is the role of Sertoli cells and Leydig cells in male spermatogenesis? (FA15 p572)
(FA16 p576)

3. What is the mechanism of action of sildenafll? (FA15 p598) (FA16 p603) (SU p453)

f 279 j
4. Benign epithelial lesions
Non-proliferative breast (flbrocystic) changes
• Fibrosis- _ _ _ _ _ _ _ _ _ _ _ _ _ _ _ _ _ _ _ _ _ _ _ _ _ __
• Cysts- _ _ _ __ _ _ _ _ _ _ _ _ _ _ _ _ _ _ _ _ _ _ _ __

Proliferative breast disease without atypia


• Sclerosing adenosis - _ __ _ _ _ _ _ _ _ _ _ _ _ _ _ _ _ _ _ _ __
• Epithelial hyperplasia - _ __ _ _ _ _ _ __ _ _ _ _ _ _ _ _ __
• Complex sclerosing lesion (radial scar) - _ _ _ __ __ _ _ _ __ _ _ _ _
• Fibroadenoma

5. Breast carcinoma in situ (15-30%)


Ductal carcinoma in situ (DCIS, intraductal carcinoma)
• Comedocarcinoma - _ _ _ _ _ _ _ _ _ _ _ _ _ _ _ _ _ _ _ _ _ __
• Solid
• Cribriform
• Papillary
• Micropapillary
Lobular carcinoma in situ (LCIS)

6. Invasive breast cancer (70-85%)


• Invasive ductal carcinoma (75-80%) - _ _ _ _ _ _ _ _ _ _ __ _ __ __
• Invasive lobular carcinoma - _ _ _ _ _ _ _ _ _ _ __ _ _ __ _ _ __
• Medullary carcinoma - _ _ _ _ _ _ _ _ _ _ _ __ _ _ __ _ _ _ _ _
• Tubular/cribriform carcinoma
• Mucinous carcinoma
• Papillary carcinoma
• Metaplastic carcinoma
• Inflammatory carcinoma - signs of inflammatory breast cancer may include:

f--
(/)
<1:
w
0:::
co

r 2so1
End ofSess1on 8u1z
7. What are some of the causes of gynecomastia?

8. What organism is most commonly responsible for acute mastitis?

9. What is the classic presenting complaint in a patient with intraductal papilloma?

·10. What breast pathology fits each ofthe following descriptions?


• Most common breast tumor in women under 25
• Most common breast mass in postmenopausal women
• Most common breast mass in premenopausal womeh
• Most common form of breast cancer
• Small, mobile, firm mass with sharp edges in 24-year-old woman
• Histological "leaf-like projections"
• Signet ring cells
• Loss of e-cadherin cell adhesion gene on chromosome 16
• Always ER(+) and PR(+)
• Commonly presents with nipple discharge
• Eczematous patches on nipple
• Multiple bilateral fluid-filled lesions with diffuse breast pain
• Firm, fibrous mass in a 55-year-old woman

11. A 58-year=old postmenopausal woman is on tamoxifen. What is she at increased risk of


acquiring?

co
;;o
rn
)>
(J)

RAP! O- F! RE FACTS --1

Most common cancer in women in the U.S.

Most common benign breast tumor

Most common malignant breast t umor

Blue dome cyst in the breast

Treatment for ER(+) breast cancer

Red, itchy, swollen rash on the areola and nipple

r 2.81 1
r~-
THERE IS NO PART OF

MICRO THE HUMAN BODY THAT


IS NOT SUSCEPTIBLE TO
INFECTION AND INVASION
1 Bacterial Basics
BY MICROORGANISMS
2 Bacterial Toxms WHILE MOST SPECIFIC
3 Staphylococcus INFECTIONS ARE
4 Strept ococcus DISCUSSED WITHIN .THE
5 Other Gram-Positive Bactena APPROPRIATE ORGAN
SYSTEMS, BOARD EXAMS ·
6 Gram - Negative Bacteria part 1
WILL LSO TEST YOUR
7 Gram - Negative Bacteria part 2 KNOWLEDGE OF THE
8 Spirochetes and Zoonot1cs INFECTIOUS PATHOGENS
9 Mycobacteria THEMSELVES AND TH

10 Nonstainmg Bacter ia MECHANISMS OF TH


VARIOUS ANTIMfCROB/ALS
11 Pen1cillms
USED TO ERADICATE
12 Cephalospor1ns THEM
~
13 Other Cell Walllnh1bitors
14 Protem Synthesis lnh1b1tors
15 Other Anti b1ot1cs
16 M1cro by Systems part 1
17 M1cro by Systems part 2
18 M1cro by Systems part 3
19 V1ral Bas1cs
20 DNA Viruses part 1
21 DNA V1rus es part 2
22 RNA Vtruses part l
23 RNA V1ruses part 2
24 HIV
25 HIV Drugs
26 Mycology part 1
27 Mycology part 2
28 Ant1fungals
29 Protozoa
30 Helminths and Ectoparasites
BACTERIAL BASICS
R: Chapter 8
Bacterial cell structures (FA15 p118) (FA16 p108) (SU p317-318)
Encapsulated bacteria (FA15 p122) (FA16 p112) (SU p318) (R p345)
Ribosomes
Genetic transfer (FA15 p126) (FA16 p114)
Bugs with unusual cell membranes/walls (FA15 p119) (FA16 p109)
Spores (FA15 p130) (FA16 p114) (SU p318)
Bacterial staining (FA15 p120) (FA16 p110)
Bacterial genetics (FA15 p126) (FA16 p114)

.warm -Up Review


.Which medication overdose is treatedwith sodium bicarbonate? (FA15 p257) (FA16 p251J

2. A patient presents with tinnitus, dizziness, headaches, and Gl distress. What drug is
potentially causing these symptoms? (FA15 p259) (FA16p253)

3. Which testicular tumor is associated with each of the following statements? (FA15 p593)
(FA16 p598) (SU p219)
• Composed of cytotrophoblasts and syncytiotrophoblasts
• May present initially with gynecomastia
• Elevated AFP
• Elevated ~-hCG
• Histologic appearance similar to koilocytes (cytoplasmic clearing)

OJ
)>
(J)

n
V1

r 2ss 1
End of Sess ion Ou1z
4. What bacterial structures have the following functions?
• Mediates adherence of bacteria to the surface of a cell
• Protects against phagocytosis
• Provides rigid support to bacterial cell and protects against osmotic pressure differences
• Space between the inner and outer cellular membranes in gram-negative bacteria
• Motility
• Bacterial form which provides resistance to dehydration, heat, and chemicals
• Forms attachment between two bacteria during transfer of DNA material (AKA conjugation)
• Independent pieces of genetic material within bacteria that may contain genes for antibiotic
resistance

5. What stain is required to see the following organism (or substance)?


• Cryptococcus neoformans
• Pneumocystis jirovecii (PCP)
• Chlamydia

6. Which organisms are not well visualized with Gram stain?

7. Which bacteria are encapsulated?

(/)
u
:2 8.. By what method are plasmids exchanged between bacteria?
CD
_j
<t
0:::
l.J..J
f-
u
<t
CD

f 286 l
BACTERIAL TOXINS
R: Chapter 8
Endotoxin (FA15 p125) (FA16 p117) (SU p322)
Endotoxin vs. exotoxin (FA15 p123) (FA16 p115) (SU p322)
Bacteria with exotoxins (FA15 p124-125) (FA16 p116-117) (SU p323-324) (R p350)
- Staphylococcus au reus (FA15 p125) (FA16 p117) (SU p323)
- Streptococcus pyogenes (FA15 p125) (FA16 p117) (SU p323)
- Corynebacterium diphtheriae (FA15 p124) (FA16 p116) (SU p324)
- Pseudomonas aeruginosa (FA15 p124) (FA16 p116) (SU p324)
- Shigella (FA15 p124) (FA16 p116) (SU p323)
- Escherichia coli (FA15 p124) (FA16 p116) (SU p323)
- Yersinia enterocolitica (FA15 p139) (FA16 p129) (SU p323)
- Bacillus anthracis (FA15 p124) (FA16 p116) (SU p323)
- Vibrio cholerae (FA15 p124) (FA16 p116) (SU p323)
Bordetella pertussis (FA15 p124) (FA16 p116) (SU p324)
- Clostridium tetani (FA15 p124) (FA16 p116) (SU p324)
Clostridium botulinum (FA15 p124) (FA16 p116) (SU p324)
Clostridium perfringens (FA15 p125) (FA16 p117) (SU p324)
Bacterial growth curve (SU p320)

Warm Up Rev1ew
1. What is the antidote.for each of the following substances? (FA15 p257) (FA16 p251) (SU p219)
• Acetaminophen
• Salicylates
• Amphetamines
• Anticholinesterases and organophosphates
• Antimuscarinic, anticholinergic agents

2. Which testicular tumor is associated with each of the following statements? (FA15 p593)
(FA16 p598) (SU p219)
• Histologically may have an alveolar or tubular appearance, sometimes with papillary
convolutions
• Composed of multiple tissue types
• Histologic endodermal sinus structures (Schiller-Duval bodies)
• 25% have cytoplasmic rod-shaped crystalloids of Reinke
• Androgen-producing, associated with precocious puberty

3. What organism is commonly implicated in balanitis? OJ


)>
n
-l
rn
;:o
)>
,.
-l
0
X
z
(j)

[ 287 J
4. What known toxins are secreted by Staphylococcus aureus? What is the action of each?
• a-hemolysin/a-toxin ~ hemolysis
• ~-hemolysin/sphingomyelinase C ~ hemolysis
• y-hemolysin (leukocidin ~ toxic to leukocytes, hemolysin ~ hemolysis)
• 6-hemolysin/6 -toxin ~toxic to erythrocytes and other ce lls
• Panton-Valentine l e ukocidin~ tissue destruction with MRSA, affects both neutrophi ls and
macrophages
• Enterotoxins ~food poisoning~ vomiting and diarrhea
• Toxic shock syndrome toxin-1 (TSST-1) ~release of cytokines ~toxic shock syndrome (high fever.
hypotension , diffuse rash)

5. What known toxins are secreted by Streptococcus pyogenes (group A strep)? What is the action of
each?
• Streptolysin 0 ~ hemolysis (Oxygen labile)
• Streptolysin S ~ hemolysis (oxygen ~table)

• Streptococcal pyrogenic exotoxins type A, B, (/erythrogenic toxins ~ red rash (erythro -) and fever
(pyro-) of scarlet fever

En of Session Quiz
6. What exotpxin matches each of the following characteristics?
• · Inhibits ACh release ~flaccid paralysis
• Phospholipase that causes gas gangrene
• Inhibits the inhibitor of adenylate cyclase ~ whooping cough
• Stimu lates adenylate cyclase ~ chloride and water into gut~ diarrhea
• Destroys leukocytes
• Composed of edema factor, lethal factor, and protective antigen
• Enterotoxin causing rice-water diarrhea
• Causes scarlet fever
• Ci')uses toxic shock syndrome
•Inactivates EF-2 ~ pseudomembranous pharyngitis
• Blocks the release of the inhibitory neurotransmitter glycine

Vl
z 7. What seven bacteria secrete enterotoxins (exotoxin that causes water and electrolyte
>< imbalances of intestinal epithelium resulting in diarrhea)?
0
f-
_J
<(
c,:
LW
f-
u
<(
a:l

[ 288 J
STAPHYLOCOCCUS
H: Chapters 134-142
Gram-positive lab algorithm (FA15 p127) (FA16 p118)
Coagulase (SU p321)
Staphylococcus aureus (FA15 p128) (FA16 p119) (SU p324)
Pigment-producing bacteria (FA15 p122) (FA16 p113)
Staphylococcus epidermidis (FA15 p128) (FA16 p119) (SU p324)
Staphylococcus saprophyticus (FA15 p128) (FA16 p120) (SU p324)

Warm -lJ p Review


1. .. What is the clinical use of clomiphene? How does this drug work? (FA15 p590) (FA16 p601}

2. What drugs inhibit prolactin secretion? (FA15 p316) (FA16p310)

3. What are the signs and symptoms of anticholinergic activity? (FA15 p251) (FA16 p245)

4. Gram-positive algorithm

U1
~

<
\J
I
-<
'n
0
0
n
n
c

<
U1

r 2891
End
bel the following gram-positive algorithm:

Gram(+) .-

/~
~

if)
::::J
u
u
0
u
0
_j
>-
::r:::
o_
;':I:
if)

[ 290 J
STREPTOCOCCUS
H: Chapter 136
Gram-positive algorithm (FA15 p127) (FA16 p118)
Streptococcus pneumoniae (FA15 p129) (FA16 p120) (SU p324)
Bacterial virulence factors (FA15 p123) (FA16 p113)
Viridans group streptococcus (FA15 p129) (FA16 p120) (SU p325)
Group A streptococcus (Streptococcus pyogenes) (FA15 p129) (FA16 p120) (SU p325)
Group B streptococcus (Streptococcus agalactiae) (FA15 p129) (FA16 p121) (SU p325)
Enterococci (FA15 p130) (FA16 p121) (SU p325)
Streptococcus bovis (FA15 p130) (FA16 p121) (SU p325)

Warm-Up Review
1. ..· Which medication overdose may be tfeatect with ammonium chloride? (FA15 p257) (FA16 p251)

2. Luteinizing hormone (LH) acts on which target cells?

Which ovarian tumor matches each ()f the following statements? {FA15 p5B6-5B7! (FA16 p592-593)
Estrogen secreting leading to precocious puberty
• Produces AFP
• Psammoma bodies
• Testosterone secreting leading to virilization

4. What are the different viridans group streptococci?


• S. mutans
Most prominent organism in dental plaque
• S. sa livarius
• S. anginosus
• 5. mitis Vl
--1
;;o
• S. sanguinis rn
-o
May enter circulation during dental procedures and cause subacute endocarditis in those with --1
0
turbulent flow heart problems (pre-existing endothelial damage) n
0
n
If a patient has a prosthetic heart valve, a history of endocarditis, or congenital heart disease, n
then use prophylaxis with amoxicillin prior to dental procedures c
Vl

i" 29i l
5. What human diseases can be caused by group A streptococcus (Streptococcus pyogenes)?
• Superficial/ deep infections
- Acute pharyngitis, cellulitis, invasive infections (bacteremia, pneumonia, necrotizing fasciitis)
• Immunologic illnesses
- Acute post-streptococcal glomerulonephritis, rheumatic fever
• Toxin -mediated illnesses
- Streptococcal toxic-shock syndrome, necrotizing fasciitis

6. What skin infections can be caused by both Streptococcus pyogenes and Staphylococcus aureus?
• Cellulitis
Deep skin infection
• Impetigo
- Superficial skin infection characterized by redness and small vesicles or blisters
- Vesicles can rupture and leak yellowish fluid (honey-crusted lesions)
Usually affects fa ce, nose, mouth , and cheeks

7. Why is group B streptococcus (Streptococcus agalactiae} so common in neonates? What type of


infections does this organism cause in neonates?

End. .of Sess 1on Qui z


8. Which organisms are most commonly implicated in subacute endocarditis?

Vl
:=J
u
u 9. What types of disease processes are caused by Streptococcus pyogenes?
0
u
0
f---
0...
w
0:::
f-
Vl

f 292 l
OTHER GRAM-POSITNE BAOERIA
H: Chapters 137-142
Corynebacterium diphtheriae (FA15 p130) (FA16 p123) (SU p325)
Spores (FA15 p130) (FA16 p114) (SU p318)
Clostridia (FA15 p131) (FA16 p122) (SU p326)
Clostridium difficile (FA15 p131) (FA16 p122) (SU p326)
Obligate anaerobes (FA15 p121) (FA16 p111)
Anthrax (FA15 p132) (FA16 p121) (SU p327)
Listeria monocytogenes (FA15 p132) (FA16 p123) (SU p327)
Actinomyces vs. Nocardia (FA15 p133) (FA16 p123) (SU p327)

Warm -up Revtew


Which embr}tologic structure of the heart gives rise to each of the following adult
structures? (FA15 p26B) (FA16p262)
• A?CE;nding aorta ¢nd pulmoni'lry trunk
• Coronary sinus
• Superior vena cava

2. What is the karyotype in Klinefelter syndrome? What is the karyotype in Turner syndrome?
(FA15 p578) (FA16 p585)

3. Which amino acids are found in large concentrations in collagen? (FA1S p76) (FA16 p62) (SU p294)

0
-i
I
• m
;;o
C)
;;o
)>
3:
I
'lJ
0
(J)

-i
<
m
CXJ
)>
n
-i
rn
;;o
)>

[ 2931
End of Sess1on Qui z
4. Which gram~positive organism matches each of the following statements?
• Causes scalded skin syndrome
• Gray-wh.ite membrane in the posterior pharynx of an unvaccinated child
• Pharyngitis resulting in glomerulonephritis
• Most common cause of meningitis
• Most common cause of osteomyelitis
• Serious newborn infections
• Infant with poor muscle tone
• Diarrhea after using antibiotics
• Respiratory distress in a postal worker
• Otitis media in children
• Cellulitis

5. What patients are susceptible to Listeria monocytogenes?

6 . .· Which bacteria are spore formers?

7. What medications are commonly used to treat Clostridium difftcile colitis?

<(
0:::
UJ
f-
u
<(
m
UJ
>
f-
U)
0
()__

'
::20
<(
RAP! D- F! RE FACTS
0:::
(.9
Branching, filamentous , gram-positive rod s with
0:::
UJ su lfur granules
I
f-
0

i 2941
GRAM-NEGATIVE BACTERIA PART 1
H: Chapters 143-161
Gram-negative algorithm (FA15 p135) (FA16 p125)
Neisseria (FA15 p136) (FA16 p126) (SU p330)
Haemophilus influenzae (FA15 p136) (FA16 p126) (SU p330) (H p1233)
Legionella pneumophila (FA15 p137) (FA16 p127) (SU p331) (H p1236)
Pseudomonas aeruginosa (FA15 p137) (FA16 p127) (SU p331) (H p1266)
Helicobacter pylori (FA15 p140) (FA16 p130) (SU p332) (H p1261)

Warm-Up Rev1ew
1. What is the time frame for each of the following steps of wound healing? (FA15 p229) (FA16 p223)
• Infiltration of macrophages and epithelial cell migration
• Granulation tissue
• ...Infiltration of polymorphic neutrophils

2. .What are the signs and symptoms of.left-sided heart failure? (FA15 p297) (FA16p292) (SU p84)

3. Which genetic abnormality is associated with endocardial cushion defects? (FA15 p290)
(FA16p284)

Cl
:;D
)>
sI
z
rn
Gl
~
<
m
co
)>
n
-j
m
:;D
)>
--o
)>
:;D
-j

[ 2951
4. Gram-negative algorithm

Gram(-) ----+

E Session Quiz
5. Aculture reveals gram-negative,. oxidase-positive diplococci. What is the likely organism?

6. A 22•year-old medical student has a burning feeling in his gut after meals. An EGO is
performed and biopsy of the gastric mucosa shows spiral-shaped gram-negative bacteria.
.What is the organism?

r---
0:::
<(
0..
<(
0:::
7. A 50-year-old male smoker presents with a new cough and flu-like symptoms. Gram stain of
UJ
r- the sputum shows no organisms but silver stain shows rods. What is the diagnosis?
u
<(
ro
UJ
>
~
lCl
UJ
z
I
8. What gram-negative organism matches each of the following descriptions?
~ • Sepsis, DIC, adrenal hemorrhage
<(
0::: • 5-year-old with pharyngitis, drooling, and x-ray reveals thumb sign
\.:J
• Burn wound infection
• Life-threatening meningitis + purpura
• Septic arthritis in young, sexually active patients

[ 296]
GRAM·NEGATNE BACTERIA PART 2
H: Chapters 149, 153-156
R: Chapters 8, 17
Escherichia coli (FA15 p138) (FA16 p128) (SU p328)
Shigella (FA15 p139) (FA16 p129) (SU p327)
Salmonella (FA15 p139) (FA16 p129) (SU p328)
Campylobacter jejuni (FA15 p138) (FA16 p128) (SU p329)
Yersinia enterocolitica (FA15 p139) (FA16 p129) (SU p329)
Vibrio cholerae (FA15 p139) (FA16 p129) (SU p329)
Klebsiella pneumoniae (FA15 p138) (FA16 p128) (SU p329)
Proteus

wa -Up iFeyiew
1. What substances do cytotoxic T ce.lls and Nl( cells use to induce apoptosis in virus~infected
cells? (FA15 p201-202) (FA16 p193-194) (SU p374~375)

2. What highly damaging events can cause irreversible cell injury?

3. Which ovarian tumor matches each of the following statements? (FA15 p586-587) (FA16 p592-593)
• Llned with fallopiantube-like epithelium
• Ovarian tumor+ ascites + hydrothorax
• Cali-Exner bodies
• Mu ltiple different tissue types
• Elevated ~-hCG
Cl
• Resembles bladder epithelium ::::0
)>
:sI
z
rn
Cl
~
<
rn
ClJ
)>
n
-i
rn
::::0
)>
-o
)>
::::0
-i
"-'

r 297 1
4. Escherichia coli
• Enterotoxigenic E. coli (ETEC)
Watery diarrhea due to heat-labile and heat-stable enterotoxins
Does not infect the intestinal wall
Most common type of travelers' diarrhea
• Enterohemorrhagic E. coli (EH EC)
Bloody diarrhea and severe illness from contaminated meat
Most common is 0157:H7
Produces shiga toxin
Can lead to hemolytic uremic syndrome (HUS): anemia, thrombocytopenia, acute renal failure
- Treatment: supportive
• Enteropathogenic E. coli (EPEC)
- W atery diarrhea, but no toxin produced
Common in children
• Enteroinvasive E. coli (EIEC)
Bloody diarrhea and fever due to intestinal wall invasion
Closely related to Shigella
- Treatment: fluoroquinolones, TM P-SMX, azithromycin

End of Sess1on Quiz


5. Which form of Escherichia coli causes hemolytic uremic syndrome (HUS)?

6. What infectious agent most likely corresponds to the following statement?


• Diarrhea caused by gram-negative, nonmotile organism that does not ferment lactose
N • Rice-water stools
1-
0::
<(
• Diarrhea caused by S-shaped organisms
0..
<(
• Diarrhea transmitted from household pet
0::: • Diarrhea caused by gram-negative, motile organism that does not ferment lactose
UJ
1-
u • Most common cause of travelers' diarrhea
<r • Bloody diarrhea after eating undercooked hamburger meat
m
UJ
> • Diarrhea and right lower quadrant pain mimicking appendicitis
1-
<( • Diarrhea after consuming eggs or handling raw chicken
(.9
UJ • Osteomyelitis in a sickle cell patient
z
I

~
<(
0:::
(.9

r 298 1
SPIROCHETES AID ZOONOTICS
H: Chapters 171-173
Spirochetes (FA15 p140) (FA16 p130) (SU p333)
Leptospira interrogans (FA15 p140) (FA16 p130) (SU p333)
Lyme disease (FA15 p140) (FA16 p130) (SU p333) (H p1403)
Syphilis (FA15 p141) (FA16 p131) (SU p334)
Argyll-Robertson pupil (FA15 p141) (FA16 p131)
VORL false positives (FA15 p141) (FA16 p131) (SU p335)
Zoonotic bacteria (FA15 p142) (FA16 p132)

Warm -LJp Rev1ew


1. Activation of fibroblast growth factor (FGF) receptor 3 is assOCiated with what disorder?
(FA15 p425) (FA16 p426)

2. A football player who was kicked .in the leg suffers from a•damaged lateral meniscus. What
else is likely to have been damaged? (FA15 p416) (FA16 p417) (SU p242)

3. What is the basic equation for cardiac output? What is the Fick principle? (FA15 p272) (FA16 p266)
(SU p68)

(/)
-u
;:u
0
n
:r::
rn
-1
m
lfl
)>
z
CJ
N
0
0
z
0
--j

n
(/)

f 29 9 l
End of Sessi u1z
4. Whatis the Classic presenting symptom in a patient with Lyme disease?

What pupillary sign might point you to a diagnosis of syphilis?

6. Whatorganism is associated with each of the following clues?


• Lymphadenopathy+ a new kitten
• Dog bite
• . Ixodes tick
• Rabbit

RAP! o-Ff RE FACTS


Standard treatment forT. pal/idum

Cellulitis from a dog or cat bite

A non -painful , indurated, ulcerated genital lesion

Moist, smooth, flat, white genital lesion


(J)
u
5 Large bull's-eye rash
:z:
0
0
N
0
z
<(
U)
LLJ
f-
LLJ
:r:
u
0
er::
o_
(/)

r 3oo l
MYCOBACTERIA
H: Chapter 165
GG: Chapter 56
Primary and reactivation tuberculosis (FA15 p133) (FA16 p124) (SU p332)
Antimycobacterial drugs (FA15 p188-189) (FA16 p179-180) (SU p340)
Mycobacteria (FA15 p134) (FA16 p124) (SU p332)
Leprosy (Hansen disease) (FA15 p134) (FA16 p125) (SU p333)

Warm -Up Review


1. Why should ~-blockers never be used in cases of suspected cocaine intoxication?
(FA15 p253) (FA16 p246)

2. Whatls the embryologic origin of each of the following adult structures? (FA15 p559)
·(FA16 p563)
• Olfactory epith elium
• Mammary glands
• Salivary glands
• Sweat glands

3. · What defects are seen in a patientwith transposition of the great vessels? (FA15 p2BB)
(FA16 p282) (SU p64)

4. What is the treatment for latent tuberculosis?

:s:
-<
n
0
Cil
)>
n
-I
5. What is the treatment for active tuberculosis? rn
:::0
5>

f 30! l
6. What antibiotic is used for prevention of Mycobacterium avium-intracellulare (MAl) in AIDS patients?
When should this prophylaxis begin?

End of Session Quiz


7. What is the distinction between a Ghon complex and a Ghon focus? Are these seen in
primary or reactivation tuberculosis?

8.. .·\tVhat does a positive PPD skin test indicate?

9..·•. •What does a negative PPD skin test indicate?

10. Apatientis attempting to recall for youwhat medications he is taking. He says there is one
medication. that he is taking to "keep his lungs healthy" that he says can "hurt his nerves"
unless he takes his B vitamin. To what medication is he referring?

<(
00::
LLJ
f-
RAPID - FIRE FACTS
u
<(
a:J
Associated w ith a Ghon focus or Ghon compl ex +
0 hilar lymphadenopat hy
u
>-
~
Associated w ith lung infiltrate in the apical-
posterior segments of upper lobes

Standard initial t reatment regim en for TB

r 302 1
NONSTAINING BACTERIA
Gardnerella vagina/is (FA15 p142) (FA16 p132) (SU p335)
Rickettsial diseases (FA15 p143) (FA16 p133) (SU p335)
Rocky Mountain spotted fever
- Typhus
- Ehrlichiosis and anaplasmosis
- Q fever
Chlamydiae (FA15 p144) (FA16 p134) (SU p336)
Chlamydia trachomatis serotypes (FA15 p144) (FA16 p134) (SU p336)
Mycoplasma pneumoniae (FA15 p144) (FA16 p134) (SU p337)

War m-Up Rev1ew


- .

1. A defect in the expression of the FMR1 gene is associated with what disorder?
(Ff.\15 p85) (FA16p73)

2. What is the antidote for each of the following substances? (FA1 5 p257) (FA16 p251)
• Iron
• Lead
• Cyanide
• Methemoglobin
• Carbon monoxide
• Methanol or ethylene glycol

z
0
z
U1
:;;
z
z
G)
co
)>
n
---4
rn
A)
)>

r 303 1
End of Sess ion Qu iz
3. What is the Rickettsial triad of symptoms?

4. A 25-year-old with Mycoplasma atypical pneumonia exhibits anemia due to cold


agglutinins. What type of immunoglobulins is responsible for the anemia?

5. Which antibiotic should be used to treat each of the following infections?


• Mycoplasma pneumoniae
• Rocky Mountain spotted fever
• Bacterial vaginosis

<t:
0::
LU
f-
u
<(
CD
(.9
z
z
~
(/)
z
0
z

[ 304 J
PENICILLINS
GG: Chapter 53
Penicillin (FA15 p180) (FA16 p170) (SU p337) (GG p1477)
Penicillinase-resistant penicillins (FA15 p181) (FA16 p171) (SU p337) (GG p1487)
Aminopenicillins (FA15 p181) (FA16 p171) (SU p337) (GG p1487)
Antipseudomonal penicillins (FA15 p181) (FA16 p171) (SU p337) (GG p1490)
~-lactamase inhibitors (FA15 p181) (FA16 p171) (SU p337) (GG p1501)

War -LJp Rev1ew


1. Wbat molecule does the Golgi apparatus add to proteins in order to direct the proteins to
thEflysosomes? (FA15 p73)(FA16 pS9)

2. What are the signs of right-sided heart failure? (FA15 p297) (FA16 p292) (SU p84)

\J
m
z
n
r-
r-
z
lfl

r 3os 1
3. What are the clinical uses for ampicillin and amoxicillin? Which has a greater oral bioavailability?

4. Which penicillin drug would you use given the following infection?
• Syphilis
• UTI
• Pseudomonas
• Neonatal infection

End of Sess ion Quiz


5. What organisms can be treated with ampicillin or amoxicillin?

6. Which three penicillins discussed can be used to treat Pseudomonas infections?

7. What are the two different mechanisms of resistance bacteria use against penicillin agents?

lfl
z
--'
--'
u 8. What are the P-lactamase inhibitors?
z
LJ..J
0...

r 306l
CEPHALOSPORINS
GG: Chapter 53
Cephalosporins (FA15 p182) (FA16 p172) (SU p338) (GG p1493)

Wa rm-Up "Rev iew


1. What is the difference betWeena case-control study, a cohort study, and aclinical
trial? Which studies use odds ratios, and which use relative risks?
(FA15 p48) (FA16 p32) (SU p17)

2. Which type of vasculitis fits each of the following descriptions? (FA15 p302-303) (FA16 p296-297)
(SUp90)
• Necrotizing granulomas of lung and necrotizing glomerulonephritis
• Necrotizing immune complex inflammation of visceral/renal vessels
• Young Asian women
• Young asthmatics
• l.hfants and yoUng children; involved coronaryarteries
• Most common vasculitis
• Associated with hepatitis B infection

()
m
IJ
:r:
)>
r
0
Vl
IJ
0
:::0
:z:
Vl

[ 307 J
End of Sess1on Qu1z
3. Which generation of cephalosporin would you choose to use for each of the following
infections?
• UTI prevention
• Serratia UTI
• N. meningitidis
• Pseudomonas
• Resistant otitis media

4. Classify the following cephalosporins as either 1st, 2nd, 3rd, 4th or 5th generation.
• Cefepime
• Cephalexin
• Ceftriaxone
• Cefuroxime
• Ceftazidime
• Ceftaroline

5. What is the mechanism of action of the cephalosporins?

(/)
:z
0::::
0
n...
(/)
0
---'
<(
:r
0...
w
u

[ 308 J
OTHER CELL WALL INHIBITORS
GG: Chapter 53

Aztreonam (FA15 p183) (FA16 p173) (SU p33B)


lmipenem-cilastatin, meropenem (FA15 p183) (FA16 p173) (SU p338) (GG p1500)
Vancomycin (FA15 p183) (FA16 p173) (SU p338)

Warm-Up Rev1ew
1. Assuming a normal Gaussian distribution for the results of a particular test, an, average value
of .35, and SO of 4, what percentage of people will be in the interval between .31 and 4.3?
(FA15 p53) (FA16 p37)

2. A small study of USMLE test takers for a particular school reveals scores of 225, 225, 225,
229, 2.30, 240, and 250, with the average score being 232. What is the mean, median, and
mode for these values? Would this create a positively skewed curve or a negatively skewed
curve? (FA15 p53)(FA16 p37)

0
-1
:r:
rn
;;;o
n
rn
r-
r-
::2:
)>
r-
r-
:z:
:r:
3. Which organisms cause infective endocarditis? (FA15 p274) (FA16 p293) o:J
=i
0
;;;o
Vl

r 309 1
End of Sess ion Quiz
4. What cell wall inhibitor matches each of the following statements?
• Can cause "red man" syndrome
• Aminoglycoside pretender
• Inpatient treatment for MRSA
• Hospitalized patient with new gram-positive cocci clusters in blood
• Treatment for C. difflci/e colitis
• Broad -spectrum coverage for appendicitis
• Cell wall inhibitors effective against Pseudomonas

5. How does an organism develop resistance to vancomycin?

6. A patient is receMng an IV infusion of an antibiotic and the nurse calls you when his face
becomes flushed 15 minutes after the infusion started. What is the treatment for this
patient?

(.f)
a:::
0
f-
ro
:r:
z

_J
_J
LL.J
u
a:::
LLJ
:r:
f--
0

r 31 o 1
PROTEIN SYNTHESIS INHIBITORS
GG: Chapters 50, 55
Protein synthesis inhibitors (FA15 p184) (FA16 p174) (SU p339)
Aminoglycosides (FA15 p184) (FA16 p174) (SU p339)
Tetracyclines (FA15 p185) (FA16 p175) (SU p339)
Macro/ides (FA15 p186) (FA16 p176) (SU p339)
Chloramphenicol (FA15 p185) (FA16 p175) (SU p339)
Clindamycin (FA15 p185) (FA16 p175) (SU p339-340)
Streptogramins (SU p340)

Warrn""Up··· Revlew
1. Which antihypertensive drug fits each of the following side effects?
·Bradycardia, asthma exacerbation
Reflex tachycardia
• Elevated antichistoneantibodies
• Hypercalcemia

2. In a study of 100 people, the study results show an average of 500 with a standard
deviation of 50. Calculate the 95% confidence interval. (FA15 p54) (FA16 p3B)

--o
::0
0
--;
rn
:z
(J)
--<
z--;
:::r:
rn
(J)
(J)

:z
:::r:
CJ:J
--;
0
::0
(J)

f ~II 1
3. Linezolid
• Mechanism of action: binds 23S RNA and interacts with the bacterial initiation complex
• Uses: M RSA, VRE

4. What are the clinical uses for tetracyclines?

5. What are the clinical uses for macrolides?

6. Streptogramins
• Quinupristin/Dalfopristin
• Synthesized by the bacteria Streptomyces virginiae
• Mechanism of action: bind to the 23S portion of the 50S ribosome
• Uses: MRSA, VRE, Staphylococcus and Streptococcus skin infections
• Side effects: hepatotoxicity, pseudomembranous colitis
• Inhibit cytochrome P450

(J)
0:::
0
f--
0]

::r::
z
Vl
(J)
w
::r::
f-
z
>-
(J)

z
w
f--
0
0:::
CL

r 3121
End of Sess1on Qu1z
7. What are the side effects of the aminoglycosides?

8. What drugs have photosensitivity reactions?

9. What causes gray baby syndrome? Gray marlSyndrome? Red lllan syndrome?

10. What are the clinical uses for macrolides?

"""0
;;o
0___,
rn
:z:
l/)
-<
z___,
:::r:
rn
l/)
l/)

z:
:::r:
co
___,
0
;;o
l/)

[ 313 J
OTHER ANTIBIOTICS
GG: Chapters 50, 55

Sulfonamides (FA15 p186) (FA16 p177) (SU p341)


Sulfa drug allergies (FA15 p260) (FA16 p254)
Trimethoprim (FA15 p186) (FA16 p177) (SU p341)
Nitrofurantoin (SU p341)
Fluoroquinolones (FA15 p187) (FA16 p178) (SU p341)
Polymyxins
Nonsurgical antimicrobial prophylaxis (FA15 p189) (FA16 p181)
Antibiotics to avoid in pregnancy (FA15 p195) (FA16 p187) (SU p341)

1. Which physiologic changes are sensed by peripheral chemoreceptors? And which


()lies by central chemoreceptors? (FA15 p286) (FA16 p279) (SU plOD)

2. An investigative test uses a lab markerSCSQ to identify early small and squamous
cell lung cancers. In a population .of...100smokers, 20test positive. Of the 20 who r-,
test positive, five actually have either of the two lung cancers..Of those who do not
test positive, it is determined that Jive have either of the two cancers. Whatar~ the
sensitivity, spedficity,PPV, and NPV of this stlldy? (FA15 p49) (FA16 p33)(SU p16)

(f)
u
f--
0
OJ
f-
z
<r
cr::
w
I
f--
0

31 4 ]
3. What other drugs should be avoided in patients with an allergy to sulfa?

4. What drugs can cause Stevens-Johnson syndrome (SJS)?

5. Nitrofurantoin
• Mechanism: Bactericidal; reduced by bacterial proteins to a reactive intermediate that inactivates
bacterial ribosomes
• Indication: UTI cystitis (not pyelonephritis) by Escherichia coli or Staphylococcus saprophyticus
• Side effects: rarely nausea, headache, diarrhea
• Safe in pregnancy

6. Polymyxins
• Cationic detergents
~ • IV: last resort for resistant gram-negative infections
• Top ical: used in many OTC antibiotic ointments
• Toxicity: IV is neurotoxic and nephrotoxic

0
-l
I
rn
;;o
)>
z
-l
co
0
-l
n
Vl

r 3 i5 1
En of Sess1o n Qu1 z
7. What agent is used for prophylaxis for each of the following infections?
• Gonorrhea
• Syphilis
• Recurrent UTis
• Pneumocystis jirovecii pneumonia
• Exposure to H. influenzae type B meningitis
• Endocarditis prevention inpati.ent with mechanical heart valve

8. What are the major side effects of the IV polymyxins?

9. When should trimethoprim/sulfamethoxazole be started for prophylaxis against


Pneumocystis jirovecii in patients with HIV?

10. What is the mechanism of action of each of the following antibiotics?

• Sulfamethoxazole

• Trimethoprim

• Levofloxacin

• Nitrofurantoin
VJ
u
f- • Metronidazole
0
m
f- • Polymyxin B
:z:
<(
0:::
w
:r:
f-
0

iilf,l
MICRO BY SYSTEMS PART 1
H: Chapters 221, 257, 381
R: Chapters 15, 28
Normal dominant flora (FA15 p171) (FA16 p161)
Common causes of pneumonia (FA15 p172) (FA16 p162) (SU p109)
Meningitis (FA15 p173) (FA16 p163) (SU p49)
Bioterrorism (H p1768)

rm -U Review
- -
1. In a randomized, placebo-controlled study of a new blood pressure medication,
members of the st~dy group receive their supply of pills (it a monthly dinicvisit.
During this visit, the blood pressure is checked, and the clinic nurses are available to
answer questions and instruct the patients on the importance of controllfng blood
pr~ssure through diet, exercise, and weight loss. Members ofthe control group·
receive their supply of placebo pills in the mail each month. After 6_months, the
study group's.mean systolic blood pressure has decreased 11 points from baseline,
comp~red to a 2-point decrease in the control group. Whic:h type of bias lllay have
contributed to the difference in outcomes between the two groups?

2. What is plllsus paradoxus, and what are the causes? (FA15 p299) (FA16 p294)

:s:
n
:::0
0
OJ
-<
Ul
-<
__,
Ul
m
s
Ul
-o
)>
__,
:::0

r 3171
3. What organism should be considered as the cause of pneumonia in each of the following scenarios?
• Alcoholi c patient
• Patient w ith decreased level of consciousness
• Patient with cystic fibro sis
• Patient with HIV and CD4 count < 200
• Post-influenza
• Acquired from patio-cooling water mister
• Travel to southwestern U.S.
• Exposure to bird / bat droppings in Ohio/ Mississippi River valleys
• Patient with pet parrot

4. What organism should be considered as the cause of meningitis in each of the following scenarios?
• Unvaccinated child
• Outbreak in a dormitory
• Meningitis with petechial rash
• Patient with HIV
• RBCs in CSF +temporal lobe involvement on MRI

5. CSF findings in meningitis


' Bacterial TB/fungal Viral
Opening
pressure
WBC count

Differential

Protein

Glucose

Gram stain

1-
cr:::
<(
0...
(j)
:2::
w
1-
U)
>-
U)

>-
co
0
0:::
u
:a

r 318 1
6. Anthrax
• Caused by Bacillus anthracis, a spore-forming, gram-positive rod
• Clinical syndromes:
- Cutaneous anthrax: necrotic ulcer~ eschar w ith surrounding erythema and edema
Pulmonary anthrax: mediastinal l ymphadenopathy~ hemorrhagic mediastinitis~ bacteremia~
circulatory collapse and death
- Anthrax meningitis
• Management of exposure
Remove clothing and wash patient with soap and water

Raxibacumab - monoclonal antibody that neutralizes B. anthracis toxins

7. Smallpox
• Caused by Variola virus
• Transmitted by respiratory droplets or contact with contaminated objects (fomites)
• Rash on mouth and face --+ spreads to trunk and extremities --+ becomes vesicular/pustular
• Management of exposure
Remove clothing and wash patient with soap and water
- Vaccine available through the CDC and state health departments

End of Sess1on Qu1z


8 . .·.·What two organisms are important causes of pneumonia and meningitisJn neonates?

9. A 32-year""old man comes to the hospital after the abrupt onsetof fever, chills, and a
productive cough. An x-ray of the chest is consistent with pneumonia. Gram stain of
sputum shows gram-positive diplococci. What is the most likely causal organism? :s:
n
;;o
0
OJ
-<
(f)
-<
(f)
-I
rn
:s:
10.What CSFfmdings help to distinguish bacterial meningitis from viral meningitis? (.j)

u
)>
;;o
-I

[ 319 ]
MICRO BY SYSTEMS PART 2
H: Chapters 130
R: Chapter 22
Vaginal infections (FA15 p174) (FA16 p164) (SU p335, 362)
Sexually transmitted infections (FA15 p177) (FA16 p167) (SU p213)
Pelvic inflammatory disease (FA15 p178) (FA16 p167) (SU p213)
Urinary tract infections (FA15 p174) (FA16 p164) (SU p155-157) (H p2388)

Warm-Up Review
1. What are the common side effects of 13-blockers? Which patient populations should
use caution when taking 13-blockers? (FA15 p256) (FA16 p249) (SU p77-78)

2. What immunopathology is associated with each of the following statements?


(FA15 p213~215)(FA16 p207-209)
• Antimitochondrial antibodies
• Antiplatelet antibodies
• Newborn with chronic diarrhea , failure to thrive, and chronic candidiasis
• Child with eczema, coarse facial features, and cold abscesses
• Child with partial albinism, peripheral neuropathy, and recurrent infections

3. Vaginal infections
N Vaginal
Organism Discharge Wet mount Treatment
t:;::: pH
<(
0...
if)
:2:
w
Gardnerella
1-
(/)
vagina/is
>-
(/)

>-
co
0
0:::
u Candida spp.
:2:

Trichomonas
vagina/is

[ 320 J
End of Sessi on Quiz
4. What infection matches each of the following descriptions?
• Green; frothy vaginal discharge+ flagellated cells on wet prep
• Koilocytes on biopsy of lesion
• Multiple tender vesicles? shallowulcers
~ . painless genital ulcer followed by. rash that involves palms .a ndsoles
• Mucopurulent cervical discharge +cervical motion tenderness

What are the symptoms of cystitis? What are the symptoms of pyelonephritis?

6> What urjnalysis findings would.help to confirm the diagnosis of cystitis?

RAP! o -Ft RE FA CTS


"Strawberry" cervix

Clue cells

Bacteria with "school of fish" appearance

:s
n
:;o
0
OJ
-<
(J)
-<
(J)
--1
rn
:s
(J)

-u
)>
:;o
--1
N

r 32: 1
MICRO BY SYSTEMS PART 3
H: Chapters 126, 737
R: Chapters 26
TORCHeS infections (FA15 p175) (FA16 p165) (SU p212)
Food poisoning (FA15 p771) (FA16 p161)
Osteomyelitis (FA15 p173) (FA16 p163) (SU p230)
Nosocomial infections (FA75 p178) (FA16 p168)

pRev iew
What are the clinical. uses for the cephalosporins? (FA15 p182) (FA16 p1 72) csu p337,338)

2. What lab finding helps with the diagnosis ofgiant cell arteritis? (FA15 p302)

(Y)
f-
er::
<(
Q_

(_(}
::2:
w
f-
(f)
>-
tn
>-
m
0
~
u
~

[ 322 J
3. What are the features of congenital syphilis?
Early manifestations (onset during first 2 years of life)
• Hepatomegaly, elevated LFTs
• Rash followed by desquamation of hands and feet
• Snuffles (blood-tinged nasal secretions)
• Skeletal abnormalities
Late manifestations (onset after first 2 years of life)
• Frontal bossing
• Interstitial keratitis
• Saddle nose deformity
• Hutchinson teeth (notching of upper incisors)
• Perforation of hard palate
• Saber shins (anterior bowing of tibia)

4. What is the treatment for syphilis in a pregnant patient who is allergic to penicillins?

5. What is the most likely cause of food poisoning given each of the following statements?
• Vomiting 1-6 hrs after eating potato salad at picnic
• Vomiting 1-6 hrs after eating reheated rice
• Vomiting and watery diarrhea on cruise
• Watery diarrhea 12 hrs after eating meat or poultry from cafeteria
• Inflammatory diarrhea 1-3 days after eating undercooked meat/poultry, eggs, or produce
• Inflammatory diarrhea after eating undercooked hamburger meat, followed by acute kidney injury
• Inflammatory diarrhea after eating raw seafood
• Inflammatory diarrhea followed by ascending paralysis
• Descending paralysis after ingestion of homemade canned goods

:s
n
6. What organism is the most likely cause of osteomyelitis given each of the following situations? :;:o
0
• Patient with sickle cell disease OJ
-<
(J)
• Patient with prosthetic device -<
(J)

• Involvement of the vertebrae -l


rn
• Following cat bite
:s
(J)

-o
• IV drug user )>
:;:o
-l
• Diabetic with foot ulcer w
• Puncture wound to foot through sneaker

[ 323 J
JOn QUIZ
Which TORCHeS infection is the most likely cause of each of the following findings?
• Chorioretinitis + hydrocephalus + intracranial calcifications
• Hydrops fetalis
• PDA + cataracts
• Saddle

8. An infant develops flaccid paralysis after p~ing f~d cereal sweetened..with honey.
What is the most likely diagnosis?

9. lhthe U.S., what is the most common cause of osteomyelitis in patients with sickle
cell disease?

gro.up of picnic goers develops vomiting 2 hours after eating. What is the most
likely cause?

(Y")

f-
er::
<(
0...
(/)

2:
ll.J
r-
(/)
>-
(f)

>-
co
0
er::
u
:2:

[ 324]
VIRAL BASICS
H: Chapter 177
Viral structure (FA15 p156) (FA16 p146) (SU p342)
Viral envelopes (FA15 p157) (FA16 p147) (SU p342)
Viral replication (FA15 p157) (FA16 p147) (SU p342, 344)
Viral genetics (FA15 p156) (FA16 p146) (SU p342)
Virus ploidy
DNA viral characteristics (FA15 p157) (FA16 p147)
RNA viral characteristics (FA15 p157) (FA16 p147)
Naked viral genome infectivity (FA15 p157) (FA16 p147) (SU p344)
Viral vaccines (FA15 p156) (FA16 p146)

Warm -Up .Revi ew


1. In osteomalacia and rickets, how do the serum levels of calcium, phosphate, alkaline
phosphatase, and PTH, and the urine levels ofcaldum and phosphate compare to the
normal values? JFA15 p4Z5, 427) (FA16 p427-42B)

2. What is the medication of choice for the treatment of atrial fibrillation due to
hyperthyroidism?

3. What effect does stress have on adipocytes?

<
;;o
)>
I
(lJ
)>
(J)

n
(./)

[ 325 J
End of Session Qu1z
4. What type of vaccine uses a live virus that has been modified? What are some
examples of Uve virus vaccines?

5. What is the DNA structure of most DNA viruses? What are the exceptions?

6. When should live virus vaccines be considered for HIV-positive patients?

7. Which viral families do not have an envelope?

(J)
u
(J)
<t
co
_J
<(
a::
>

[ 326 J
DNA VIRUSES PART 1
H: Chapters 177, 179-182
GG: Chapter 58
Herpes simplex virus 1 (FA15 p158-159) (FA16 p148-149) (SU p344)
Herpes simplex virus 2 (FA15 p158-159) (FA16 p148-149) (SU p344-345)
Varicella-zoster virus (FA15 p158-159) (FA16 p148-149) (SU p345)
Tzanc/? smear (FA15 p160) (FA16 p150)
Epstein-Barr virus (FA15 p159) (FA16 p149) (SU p345)
Cytomegalovirus (FA15 p159) (FA16 p149) (SU p345)
Human herpesvirus-6 (FA15 p158-159) (FA16 p148-149) (SU p345)
Human herpesvirus-7 (SU p346)
Human herpesvirus-a (FA15 p158-159) (FA16 p148-149) (SU p346)
Acyclovir (FA15 p193) (FA16 p184) (SU p402)
Valacyclovir (FA15 p193) (FA16 p184) (SU p459)
Famciclovir (FA15 p193) (FA16 p184)
Ganciclovir (FA15 p193) (FA16 p185) (SU p426)
Foscarnet (FA15 p193) (FA16 p185) (SU p426)

Warm -Up Review


What pathology fits each of the following high-yield statements?
• Signetring cells
• Nutmeg liver
• Maternal elevations of AFP
• RBC casts in urine
• Currant jelly sputum
• Dog or cat bite

2. HHV-8 or Kaposi sarcoma-associated herpesvirus (KSHV)


• Only about 5% of the total U.S. population is infected with the virus
• In men who have sex with men, the prevalence of the virus is much higher
• Infects spindle cells coming from vascular and lymphatic endothelial tissue (inner lining of blood
vessels and lymphatic vessels)
• Causes highly vascular tumors

3. Kaposi sarcoma tumors


0
• Cutaneous tumors are most common, causing erythematous or violaceous skin lesions
z
)>

• Can also affect other organ systems (especially Gl tract and lung) <
;;o
c
• Treatm ent IJl
rn
IJl
HAART
\J
)>
Topical agents: alitretinoin :::0
--i
lntralesional: vinblastine
Liposomal anthracyclines: daunorubicin, doxorubicin
• Different types
Classic: slow-growing tumor without much potential for metastasi s
Epidemic: found in HIV/AIDS patients
lmmunocompromised: found in transplant patients

r ~77 1
En d of Sess1on Quiz
4. What is the treatment for the different herpes viruses?

5. What group of patients is prone to esophagitis caused by CMV?

6...• Where is latent herpes infection found in the body?

7. What are Downey cells?

f---
0:::
<(
CL
U)
w
U)
=:J
0:::
RA P! O-F! RE FACTS
> Dark purple lesions on the skin in an HIV patient
<(
z
0

Temporal lobe encephalitis

"Owl's eye" inclusions in organ tissue

Intranuclear eosinophilic droplets

r ~7R 1
DNA VIRUSES PART 2
R: Chapter 8
H: Chapters 184-185
Parvovirus (FA15 p158) (FA16 p148) (SU p347)
Human papillomavirus (FA15 p158) (FA16 p148) (SU p346)
Adenovirus (FA15 p158) (FA16 p148) (SU p346)
Poxvirus (FA15 p158) (FA16 p148) (SU p346)
Polyomavirus (FA15 p158) (FA16 p148) (SU p346)
Hepadnavirus (FA15 p158) (FA16 p148) (SU p347)

Wa rrn -Up Rev1ew


· 1. What is the name of the genetic syndrome that fits each of the following descriptions?
• Abnormal type I collagen synthesis
• Heinzbodies
• Musty/mousy odor, albinism, intellectual disability, eczema
• Hyperextensible skin, loose joints, .bleeding tendency
• Hypoglycemia, jaundice, cirrhosi~
Bloating, cramps, osmotic diarrhea
• Dark brown urine, organs, and connective tissue; benign disease
• Multiple fractures and blue sclerae

2. What is the mechanism of action of lactulose? (FA15 p106, 379) (FA16 p93, 375) (SUp29B)

3. What are the characteristic features of alkaptonuria? (FA15 p108) (FA16 p96) (SU p292)

0
z
)>
<
;:o
c:
(J)
m
(J)

-o
)>
;:o
-l
N

r 3291
End of Sess 1on Qu 1z
4. Which DNA virus is associated with each of the following statements?
• Erythema infectiosum (AKAfifth disease)
• Heterophile-positive mononucleosis
• Can cause conjunctivitisor diarrhea
• Enlarged cell with "owl's eye" inclusions
• Identified with a Pap smear
• Milkmaid's blisters

• Burkitt lymphoma and nasopharyngea l carcinoma


• Gingivostomatitis
• Hides in sensory ganglia of 52 and 53
• Hides in trigeminal ganglia
• Hides in dorsal root ganglia

• Viral family of JC virus


• Downey cells
• H:Uman progressive multifocal leukoencephalopathy
• Oral hairy leukoplakia

• Multinucleate giant cells on Tzanck test


• Only DNA virus that is not double stranded
• ·· Roseola
• Heterophile~negative mononucleosis

5. A 45-year-old man with squamous cell carcinoma of the penis had exposure to what virus?

6. A 20-year-old college student presents with lymphadenopathy, fever, and


hepatosplenomegaly. His serum agglutinates sheep red blood cells. What cell type is
infected?

0J
f-
0:::
<{
0...
(./)
w

~ RAP! o~Ff RE FACTS


>
<!: Aplastic anemia in a sickle ce ll patient
z
0

Child w ith a fever and "slapped cheek" rash on the


face that spreads to his body

r 33o 1
RNA VIRUSES PART 1
H: Chapters 190 -197
Reoviruses
- Rotavirus (FA15 p161-162) (FA16 p151-152) (SU p347)
- Coltivirus (FA15 p161-162) (FA16 p151-152) (SU p347)
Pi coma viruses
Poliovirus (FA15 p161 -162) (FA16 p151-152) (SU p348)
Echovirus (FA15 p161-162) (FA16 p151-152) (SU p348)
- Coxsackie virus (FA15 p161-162) (FA16 p151-152) (SU p348)
- Hepatitis A virus (FA15 p161-162) (FA16 p151-152) (SU p347)
- Rhinovirus (FA15 p161-162) (FA16 p151-152) (SU p348)
Caliciviruses
- Norovirus (FA15 p161) (FA16 p151) (SU p348)
Flaviviruses (FA15 p161) (FA16 p151)
Yellow fever (FA15 p162) (FA16 p152) (SU p348)
- Dengue fever (FA15 p161) (FA16 p151) (SU p348-349)
- West Nile virus (SU p349)
- St. Louis encephalitis (SU p349)
Togaviruses
- Rubella (FA15 p163) (FA16 p153) (SU p349)
Coronaviruses (FA15 p161) (FA16 p151) (SU p349)
Retroviruses (FA15 p161) (FA16 p151) (SU p351)

Warm -Up Rev1ew


1. What pathology fits each of the following high-yield ~tatements?
• Opacities seen on x -ray on both sides of the carina
• Dermatitis, diarrhea, dementia, possibly death
• Greenish rings around the periphery of the iris
• Elastic skin, joint hypermobility
• Enlarged, hard left supraclavicular lymph node

2. Which hormones share a common alpha subunit? (FA15 p313) (FA16 p307)

:;o
z
)>

<
:;o
c
Vl
m
Vl
-u
)>
:::0
___,

[ 331 ]
3. Dengue fever
• Most prevalent mosquito-borne viral disease worldwide
• > 50 million annual, worldwide infections
• Disease severity ranges from mild to life-threatening
• Classic dengue fever ("break bone fever"): muscle/joint pain, headache, retro-orbital pain
• Hemorrhagic fever: developed by< 20% of dengue patients
• Tourniquet test:
- WHO field test for hemorrhagic fever
BP inflated to a point between SBP and DBP
If excess petechiae = increased cap illary wa ll fragi lity and thrombocytopenia

4. West Nile virus (WNV)


• Birds are the reservoi r and mosquitoes are the vectors. Humans, horses, and dogs are incidental
hosts.
• Symptoms: usually only headache, malaise, back pain, myalgia, and anorexia for 3 -6 days
• Severe symptoms in 1/ 150: meningitis± encephalitis including muscle weakness and flaccid
paralysis (via anterior horn involvement), alterations in consciousness, possibly death
• Diagnosis: serology for lgM anti-WNV antibodies
• Treatment: supportive

End of Sess1on Quiz


5. What is the number one cause of fatal infantile gastroenteritis?

6. What virus is commonly associated with outbreaks of gastroenteritis on cruise ships?

f-
0:::
<(
0...
(/)
w
(/) 7. Which RNA virus matches each of the following statements?
=:J
0::: • Hand, foot, and mouth disease
> "Break bone fever"
<(
z • Common cold
0:::
• Fever, jaundice, black vomit
• Meningitis in summer months
• Tourniquet test helps diagnose hemorrhagic disease
• Infects motor neurons of the anterior horn

[ 332 J
RNA VIRUSES PART 2
H: Chapters 186-187, 192-194, 197
Influenza (FA15 p163) (FA16 p153) (SU p349)
- H5N1 (avian influenza) (SU p349)
- H1N1 (swine influenza) (SU p349)
- Anti-influenza antivirals (FA15 p193) (FA16 p184)
Paramyxoviruses
Parainfluenza virus (FA15 p163) (FA16 p153) (SU p350)
Respiratory syncytial virus (FA15 p163) (FA16 p153) (SU p350)
Ribavirin (FA15 p195) (FA16 p187) (SU p451)
Rubeola virus (FA15 p164) (FA16 p154) (SU p350-351)
Mumps virus (FA15 p165) (FA16 p155) (SU p351)
- Rabies virus (FA15 p165) (FA16 p155) (SU p351)
Hantavirus (FA15 p161) (FA16 p151) (SU p351)
Ebola (FA15 p161) (FA16 p151, 156)

Warm -Up Revi ew


1. What pathology fits each ofthe following phrases?
• Antiplatel~t antibodies
• Bamboo spine on x-ray
• Webbed neck, short stature
• Painful, raised lesions on finger pads +fever
• Dry eyes, dry mouth, arthritis
• Posterior cervical adenopathy
• Low serum ceruloplasmin

2. Avian influenza
• HSN1
• It is spread only from bird to human (but there are fears of mutation that would allow human-to-human
spread)
• Symptoms: URI, Gl symptoms (diarrhea), fever, pancytopenia, elevated aminotransferases (ALT, AST)
• Diagnosis: reverse transcriptase PCR or viral culture
• Treatment: oseltamivir

3. Swine flu
• H1N1
• Derived from two swine flu strains, one human strain, and one avian strain
• Symptoms: typical tlu + Gl symptoms
• Treatment: oseltamivir or zanamivir to high-risk or severely ill patients (but local surveillance data
shou ld guide drug choice)

r 333 1
4. Parainfluenza (croup)
• Laryngotracheobronchitis
• Primarily ages 6 months-3 years; the leading cause of hospitalization in children younger than 4
years
• Symptoms: characteristic "barking seal" cough, respiratory distress (may mimic asthma), inspiratory
stridor
• Diagnosis: clinical , x-ray may show steeple sign
• Treatment: cool mist humidifier (no proven benefit), racemic epinephrine, one dose of
dexamethasone, supportive, oxygen as needed

5. Respiratory syncytial virus (RSV) bronchiolitis


• Bronchiolitis ± pneumonia
• Season: winter (same as f1u season)
• Symptoms: characteristic brassy cough, wheezing, respiratory distress (may mimic asthma)
• Passive immunization with palivizumab (monoclonal RSV immunoglobulin) monthly during the winter
months in premature infants or infants with chronic lung disease
• Treatment: supportive

f-
er:
<t
0...
U)
u..J
U)
:=J
cr:
>
<t
z
cr:
En d of Session Quiz
6. Which antiviral matches each of .the following statements?
• Prophylaxis for influenza A
• L,Jsed in treatment for chronichepatitis C
• First-line for herpes simplex virus or varicella-zoster virus
• Inhibits viral DNA polymerase when phosphorylated by viral thymidine kinase

Which RNAvirus matches each of the following statements?


• · Characteristic "barkingseal" cough
• .Childhood diarrhea in winter.months
• Asthma-sounding infection in infants (esp. preemies)

8. A patient presents with fever, vomiting, anddiarrhea.Shewas in Sierra Leonetwo


weeks ago visiting family. What viral infection do you suspect? What type of isolation
should you recommendwhen the patient enters the hospital?

9. What virus {RNA or DNA) fits the following statement?


• Cancause conjunctivitis or diarrhea
• Fever +jaundice + black vomit
• Enlarged cell with uowl's eye" inclusions
• Identified with a Pap smear
• "Barking seal" cough
• Bronchiolitiswith cough and wheezing in babies
• Negri bodies
• Hides in trigeminal ganglia
• Diarrhea in children during winter months
• Two most common causes of common cold
• Downey cells
• Aseptic meningitis

:;o
z
)>

<
RAP /O-Ff RE FACTS ;;o
c
(/)
Fever, runny nose, cough, conjunctivitis, and m
(/)

diffuse rash -o
)>
:::0
--1
Small , irregular, blue-gray spots on the buccal N
mucosa, surrounded by a base of red

r 335 1
HIV
H: Chapter 189
R: Chapter 6
HIV viral structure (FA15 p168) (FA16 p158) (SU p351-352)
Diagnosis of HIV (FA15 p168) (FA16 p158) (SU p353)
Diagnosis of AIDS (FA15 p168) (FA16 p158) (SU p353)
Course of HIV infection (FA15 p169) (FA16 p159) (SU p352-353)
Common diseases of HIV patients (FA15 p170) (FA16 p160) (SU p353-354)
Prophylaxis (FA15 p190) (FA16 p181) (SU p354-356)

W arm~
1. What are the clinical manifestations of Addison disease? What is the cause of
Addison disease? (FA15 p 318) (FA16p31B! (su p186)

2. What is the clas~icclinicafpresentation of a branchial cleft cyst? (FA15 p312)(FA16 p306)


(StJ p167-168)

>
::r:

I 336 l
End ofSess1on Quiz
3. When should a patient with HIV begin prophylactic medication for opportunistic infections?

.· 4. ·What organisms are known to cause infections inthe mouths of patientswith AIDS?

5. What are the three structural genes that code forHIV proteins?

RAPID - FIRE FACTS


Dark purple nodules on the skin in an HIV patient

Large cells w ith owl's eye inclusions

Treatment for CMV

Most common opportunistic infection in HIV patients

Drug used to prevent Pneumocystis pneumonia :r:


<

r 3371
HIVDRUGS
H: Chapter 189
GG: Chapter 59

Highly active antiretroviral therapy (FA15 p194) (FA16 p186) (SU p354 -355)
Protease inhibitors (FA15 p194) (FA16 p186) (SU p355)
Nucleoside reverse transcriptase inhibitors (NRT/s) (FA15 p194) (FA16 p186) (SU p355)
Non-nucleoside reverse transcriptase inhibitors (NNRT!s) (FA15 p194) (FA16 p186) (SU p355)
En[uvirtide (FA15 p194) (FA16 p186) (SU p355)
Integ rase inhibitors (FA15 p194) (FA16 p186)
Maraviroc (FA15 p194) (FA16 p186) (SU p356)

Warm -U p Rev 1ew


1. What pathology is associated with each of the following key phrases?
• Cherry-red spot on th~ macula
• ... Slapped cheek" rash on child
• Organism associated with dog or cat bite
• Facial muscle spasm upon tapping the cheek

2. What regulates prolactin secretion from the pituitary? (FA15 p316) (FA16 p310) (SU p170)

3. What is the classic presentation of a thyroglossal duct cyst? (FA15 p312) (FA16 p306) (SU p167)

4. Protease inhibitors: saquinavir, ritonavir, indinavir, nelfinavir, amprenavir, fosamprenavir, lopinavir,


tipranavir, atazanavir, darunavir

Side Effect HIV Drug or Class


Gl intolerance
Vl
CJ Inhibit cytochrome P450
=>
cr:
0 Hyperlipidemia/hypertriglyceridemia
>
:r:: Lipodystrophy
Pancreatitis
Nephrolithiasis
Increased bilirubin

[ 338 J
5. NRTI: zidovudine, didanosine, lamivudine, abacavir, emtricitabine, tenofovir
Side Effect HIV Drug or Class
Bone marrow suppression
Pancreatitis and peripheral neuropathy
Hepatic steatosis
Hypersensitivity reaction

6. NNRTI: nevi rapine, efavirenz, etravirine, rilpivirine


Side Effect H IV Drug or Class
Rash
Neuropsychiatric symptoms:
nightmares, vivid dreams, dizziness, worsening depression
False-positive drug test to cannabinoids
Teratogenic

7. List the drugs that make up each of the following HIV drug classes.
Protease inhibitors:

NRTI:

NNRTI:

I
<
0
:::0
c
Q
Ul

r 339 1
End of Ses sion Qu iz
8. Categorize the following agents as a protease inhibitor, NRTI, NNRTI, or integrase inhibitor:
• Ritonavir
• Didanosine
• Rilpivirine
• Zidovudine
• Abacavir
• Lamivudine
• Darunavir
• Efavirenz
• Raltegravir

9. What are the two HIV envelope proteins and the drugs that interfere with them?

10. Which antiretroviral or antiretroviral class matches each of the following statements?
• Side effect: lactic acidosis
• Side effect: Gl intolerance
• Side effect: pancreatitis
• Side effect: peripheral neuropathy
• Side effect: megaloblastic anemia
• Side effect: rash ·
• Side effect: hyperglycemia, diabetes mellitus, and lipid abnormalities
• Side effect: bone marrow suppression
• Given to pregnant women with HIV

l/)
<-CJ
::)
0::
0
>
:r

r 34o 1
MYCOLOGY PART 1
H: Chapters 163, 764, 769
R: Chapter 8
Yeast
Candida albicans (FA15 p147) (FA16 p137) (SU p356)
- Cryptococcus neo[ormans (FA15 p147) (FA16 p137) (SU p356-357)
- Pneumocystis jirovecii (FA15 p148) (FA16 p138) (SU p357)
Molds
- Aspergillus fumigatus (FA15 p147) (FA16 p137) (SU p357-358)
- Mucor and Rhizopus (FA15 p147) (FA16 p137) (SU p358)
- Dermatophytes (FA15 p146) (FA16 p136) (SU p358)
- Tinea versicolor (FA15 p146) (FA16 p136) (SU p357-358)

Warm- pRev1ew
1. What pathology matches each of the following statements?
• Lens-shaped lesion on head CT
• No milk production in the postpartum period
Pigmented hamartomas in the iris
• Howell-Jolly bodies
• Cancer associated with asbestos
• Owl's eye inclusions
• Owl's eye nucleus
• Owl's eye protozoan
• 50 -year-old man withnew, unexplai ned skin yellowing and no other symptoms

2. Which enzyme is deficientin PKU? What are the symptoms?(FA15 p107) (FA16p95)

3. What is the precursor molecule ofACTH synthesis? (FA16 p307)

3::
-<
n
0
,---
4. What are four molds that are considered dermatophytes (fungal species commonly found invading 0
Gl
superficial layers of the skin)? -<
-u
)>
;:o
-I

r 341 1
5. What infections are commonly caused by dermatophytes?

End of Sess ion Qu1


6. Which fungus matches each of the following statements?
• Causes diaper rash
• Opportunistic mold with septate hyphae that branch at a 45° angle
• Opportunistic mold with irregular nonseptate hyphae that branch at wide
angles (> 90°)
• Causes thrush in immunocompromised patients and vulvovaginitis in women
• Known for causing pneumonia in AIDS patients
• i Yeast known for causing meningitis in AIDS patients

7. What is the appearance ofa KOH skin scraping in a patient with tinea versicolor?

8. When should pneumocystis pneumonia prophylaxis begin in HIV patients?

f-
0:: RAP/ O- F! RE FACTS
<t
0...
>-- Treatment for oral candidiasis
<..9
0
_J

0 Most common opportunistic infection in HIV patients


u
>--
2:
Prophylaxis for pneumocystis pneumonia in AIDS patients

Treatment for systemic candidiasis

r 3421
MYCOLOGY PART 2
H: Chapters 163, 164, 169
R: Chapter 8
Systemic mycoses (FA15 p145) (FA16 p135)
Histoplasma capsulatum (FA15 p145) (FA16 p135) (SU p358-359)
Blastomyces dermatitidis (FA15 p145) (FA16 p135) (SU p359)
- Coccidioides immitis (FA15 p145) (FA16 p135) (SU p359-360)
Paracoccidioides brasiliensis (FA15 p145) (FA16 p135) (SU p360)
Sporothrix schenckii (FA15 p148) (FA16 p135) (SU p360-361)

Warm-Up Review
1. Which organism is associateclwith each of the following statements? (FA15 p142) (FA16 p132)
• Cat scratch
• Cat bite
• Cat feces
• Puppy feces .
• Animal urine

.
2. Which hormones arise from the anterior pituitary? (FA15 p313) (FA16 p307) (SU p171)

3. What effect does adipocyte-generated leptin have on the hypothalamus? (FA15 p454)
(FA16p456)

5:
-<
n
0
,.
0
Q
-<
"U
)>
;;lJ
--1
N

[ 3431
End of Session Qu1z
4. Which fungus matches each of the following statements?
• Found in southwestern U.S. including West Texas and California
• Found in Mississippi and Ohio River basins
• Causes San Joaquin Valley fever
• Found in rura l Latin America
• Associated with plant thorns and cutaneous injury

• Found in states east of the Mississippi River


• Found in bird and bat droppings
• Mold form contains barrel-shaped arthroconidia
• Associated with dust storms
• Broad-based budding of yeast
• Multiple budding of yeast form

• 5. What infections are associated with birds?

RAP! o - F! RE FACTS
I Treatment for Sporothrix schenckii

N
1--
c:r::
<t
CL
>--
(.!)
0_J
0
u
>-
~

[ 341 J
ANTIFUNGAL$
GG: Chapter 57
Amphotericin B (FA15 p190) (FA16 p182) (SU p361)
Nystatin (FA15 p191) (FA16 p182) (SU p361)
-azoles (FA15 p191) (FA16 p182) (SU p361)
Flucytosine (FA15 p191) (FA16 p182) (SU p362)
Caspofungin (FA15 p191) (FA16 p183) (SU p362)
Terbinajine (FA15 p191) (FA16 p182) (SU p362)
Griseofulvin (FA15 p191) (FA16 p183) (SU p362)

rm -Up Rev1ew
1. ·· What pathology matches each of the following statements?
Anti-desmoglein antibodies
• Cough, conjunctivitis, coryza, fever
• Councilman bodies
• Green/yellow pigment just within the co rneoscleral margin
• Dementia+eosinophilic inclusions inneurons
• Anti-dsDNA antibodies

2. What are the side effects of orlistat? (FA15 p380) (FA16 p376) (SU p443)

3. Which enzyme converts glucose to sorbitol? (FA15 p104) (FA16 p92)

)>
z
---1
-n
c
z
Q
)>
r-
Ul

[ 345 l
4. Label the following antifungal diagram:

Blocked by: B Blocked by: C


A +-@-- Lanosterol ~ Squalene
Blocked by: 0
Nucleic Acids <lllcilt---- Pyrimidines ..--®-- Precursors

Microtubules
/
Disrupted by: E

Membrane disruption by:


F
G

A. -------------------------------------------
B.

C. --------------------------------------------

D. --------------------------------------------
E.

F.

G. ------------- ---------------------- -- -- -- - - -

U)
_j
<(
CJ
z
::::J
LL
1--
z
<(

r 346 1
En d of Sess1on Quiz
5; Which antifungal is associated with each of the following mechanisms of .action?
• Interferes with microtubule function
• Binds ergosterol, forms mernbrane .pores
• Converted to fluorouracil, inhibits DNA synthesis
• .Inhibit? ergosterol :;ynthesis

6; Which antifungal is associated with each of the following side effects?


• ArrhytQmias and nephrotoxicity
• Ieratogenic ·... .
• Inhibits hormone synthesis and cytochrome p450

7. Which antifungal is used for each of the following indications?


• ..Cell waH synthesis inhibitor used in invasive aspergillosis
• ''Swish and swallow" for oral candidiasis or thrush
• Most common treatments for onychomycosis
• Deposits in keratin-containing tissues to treat tinea capitis and onychomycosis
• Used for cryptococcal meningitis in AIDS
• Drug of choice for sporotrichosis

8. Which antifungal can be used intrathecally for fungal meningitis?

J>
z
-l
,
c
z
Cl
)>
r-
Ul

[ 347 J
PROTOZOA
R: Chapter 8
Cryptosporidium (FA15 p149) (FA16 p139) (SU p362)
Giardia Iamblia (FA15 p149) (FA16 p139) (SU p362)
Entamoeba histolytica (FA15 p149) (FA16 p139) (SU p362)
Trichomonas vaginalis (FA15 p152) (FA16 p142) (SU p362)
Toxoplasma gondii (FA15 p150) (FA16 p140) (SU p363)
Naegleria fowleri (FA15 p150) (FA16 p140) (SU p363)
Trypanosoma brucei (FA15 p150) (FA16 p140) (SU p363)
Trypanosoma cruzi (FA15 p152) (FA16 p142) (SU p363)
Leishmania donovani (FA15 p152) (FA16 p142) (SU p363)
Malaria (FA15 p151) (FA16 p141) (SU p363)
Babesia microti (FA15 p151) (FA16 p141) (SU p363)

Warm -Up .Review


1. What high-yield pathology fits each of the folloWing statements?
• Large "bull's eye" rash
• Strawberrytongue
• Restingtremor,rigidity

2. < Whatisthe name given to athyroid hormone-secretingteratoma?

3. What medications are used to shrink prolactinomas? (FA15 p316) (FA16 p310) (SU p46)

<(
0
N
0
1--
0
a::
CL

[ 348 J
4. Malaria life cycle

Mosquito
drinks
gametocytes
Plasmodium fa!cipmum
has a banana-shaped
gametocyte

Late
Earlytrophozoite scl1lzont
resembles
diamond ring
Trophozoite

"""lJ
;o
0
--1
0
N
0
l>

[ 3491
End ofSess ion u1z
5. What protozoan matches each of the following statements?
• Cause of malaria
• Most commonprotozoal infection in the U.S.
• Cause of Chagas
• Amoebic dysentery
• African sleeping sickness _,
•·. Diarrhea in campers and hikers
• Transmitted in raw meat or infected cat feces
• Transmitted by sand

6. Apatient who visited Mexico presents with.bloody diarrhea. What infectious form is found
in the stool?

7. A32-year-old manwentcamping in.northern California 3 weeks ago, had a stint of diarrhea


a couple of weeks after the trip, and now presents with symptoms of abdominal pain in
the ri&h1: upper quadrant, fever:, <311~ abnormal liver function tests. What is the infecp11g
organism? ·

What are the names of the following stages in the .malaria Ufe cycle?
• ..Looks like.a diamond
• Ruptures the cell host
• Replicating intracellularly
" Fbrm injected from the Anopheles mosquito
• Banana shaped

9. A. patientrEiliJming from .a 2-week vacation in West Africa presents with typical malaria
symptoms and recurrent fever. What is the mechanism responsible .for this patient's
recurrent fever?
<(
0
N
0
§ RAP! O- Ff RE FACTS
Ring-enhancing brain lesion in an HIV patient

Treatment for Trichomonas vagina/is

Most common protozoal diarrhea

[ 350 J
HELMINTHS AID ECTOPARASITES
H: Chapters 218, 219, 220, 357
R: Chapter 8
Enterobius vermicularis (FA15 p153) (FA16 p143) (SU p365)
Ascaris lumbricoides (FA15 p153) (FA16 p143) (SU p365)
Trichinella spiralis (SU p365)
Strongyloides stercora lis (FA15 p153) (FA16 p143) (SU p365)
Ancylostoma and Necator (FA15 p153) (FA16 p143) (SU p365)
Taenia solium (FA15 p154) (FA16 p144) (SU p366)
Diphyllobothrium latum (FA15 p154) (FA16 p144) (SU p366)
Echinococcus granulosus (FA15 p154) (FA16 p144) (SU p366)
Schistosoma (FA15 p155) (FA16 p144) (SU p366)
Paragonimus westermani (SU p366)
Clonorchis sinensis (FA15 p155) (FA16 p144) (SU p366)
Wuchereria bancrofti (FA15 p153) (FA16 p143) (SU p365)
Lice (FA16 p145)
Scabies (FA16 p145)
Bedbugs

r p Review
1. Classify each of the foJiowing antibiotics as an aminoglycoside, a macrolide, or a
tetracycline. (FA1S p184-186) (FM6 p174 -176) (SU p339)
• Minocycline
• Gentamicin
• Erythromycin
• Tobramycin
• Azithromycin
• Doxycycline
• Amikacin
• Neomycin
• Clarithromycin

2. Acanth~is of RBCs and excess lipids·in enterocytes are seen in which genetic disorder?
(FA15 p388) (FA16 p386)
:r:
rn
r-
:s:
:z:
-1
:r:
l/)

)>
z
0
rn
n
-1
0
"""0
)>
;;o
)>
l/)
-1
rn
l/)

,~

[ 351 l
3. Which three roundworms are transmitted through a fecal-oral route by ingesting the eggs? Which
three roundworms are transmitted through the soil?

End of Sess 1on Ou1 z


4. What helminth matches each of the following statements?
• Snail host, "swimmer's itch"
• Fluke associated with hematuria and bladder cancer
• Contracted by eating undercooked fish and causes inflammation of the biliary tract
• Soil ~ enters through skin ~ venous blood supply ~ lungs ~ coughed into pharynx ~
intestines
• Responsiblefor lymphatic filariasis
• Adult patientfrom Mexico with new onset seizures and brain calcifications

5. What medication is used to treat each ofthe following parasitic infections?


• Giardia, Entamoeba, Trichomonas
• Plasmodium vivax or ovale
• Chloroquine-resistant malarias
• Most flukes, like Schistosoma and Clonorchis
• Hookworm, pinworm, roundworm
• Chagas disease
(/)
w • Leishmaniasis
r-
(/)
<(
a:
<(
0...
0
r-
u
w
0
z
<(
(/)
::r:
1--
z
~
_j
w
::r:

r 352 1
MANY M /CAL

IMMUNOLOGY STUDENTS SEEM TO VIEW


THE IMMONE SYSTEM
AS A COMPLETELY
1 Immune System Ba s1 cs UNFATHOMABLE A
UNKNOWAB YSTERY,
2 Ant1gen Presentation
A DEEP OCEAN OF
INFORMATION ·wHOSE
3 T Cells
DEPTHS ARE NEVERTO
BEF2LUMBED. BU TFEAR
4 B Cells and Antibodies
NOT 1 OUR PHYSICIAN
EDUCATORS BREAK lT
5 Immun ization and Autoantibodies
DOWN SYSTEMATICALLY,
6 Hypersens1t1v1ty and Complement ADDING LAYERS 0
KNOWLEDGE FOCUS
7 Macrophages, Granulocytes and ON TH E PRECISE FACTS
Cytokmes YOU NEED TO MASTER
FO R YOUR EXAM
8 Immu nosu ppressants

9 Immunodeflc 1enc1es
IMMUNE SYSTEM BASICS
R: Chapter 6
H: Chapter 314
Lymphoid organs (SU p367)
Lymph nodes (FA15 p198) (FA16 p190) (SU p367) (R p193)
MALT (SU p369) (H p2675)
Peyer's patches (FA15 p356) (FA16 p352) (SU p369)
Lymphatic drainage (FA15 p198) (FA16 p190) (SU p369) (COA p44)
Innate and adaptive immunity (FA15 p200) (FA16 p192) (SU p369)

,war -Up Review


1. Which muscles and nerves are derived Jrom the first branchial arch? (FA15 p565)
(FA16 p569) (SU p226)

2; Which amino acids are found in large concentrations in elastin? (FA1Sp77) (FA16 p64)
(SU p296)

3: What isthe mechanism of action of adenosine as an antiarrhythmic? (FA1Sp310)


(FA16 p304) (SU p402)

:s:
:s:
c
z
m
(J)
-<
(J)
-1
m
:s:
CD
):>.
Ul
()
Ul

r 355 1
En d of Sess1on Qu1z
4. To which lymph nodes do the sigmoid colon and the rectum drain?

5. In what parts of the lymph node would 8 cells and T cells be located?

6. Where would you find Peyer's patches?

Vl
u
Vl
<(
m
::2:
w
f-
Vl
>-
(/")
w
z
=:J
::2:
::2:

r 356 1
ANTIGEN PRESENTATION
R: Chapter 6
Antigen presentation
T cell surface markers (FA15 p209) (FA16 p202) (SU p372)
Major histocompatibility complex (FA15 p200) (FA16 p192) (SU p371) (R p194)
HLA subtypes (FA15 p201) (FA16 p193) (SU p371) (R p194)
Dendritic cells (FA15 p384) (FA16 p380) (SU p372) (R p191)
Langerhans cell histiocytosis (FA15 p403) (FA16 p403) (SU p372) (R p621)

Warm .- Up Review
1. Which muscles and nerves are derived from the second branchial arch? (FA15 p565) (FA16 p569)
(SUp226)

2. What is the antidote for each of the following substances? (FA15 p257) (FA16 p251)
(SU p398)
• Opioids
• Benzodiazepines
• Jricyclic antidepressants
• ·Heparin
• Warfarin
• Theophylline

3. Which ECG finding is associated with hyperkalemia? (FA15 p537) (FA16 p542)

4. Dendritic cells
• Covered with long membranous extensions that resemble the dendrites of nerve cells
• All display both class I and II MHC and a B7 protein (either CD80 or CD86)
• All have CD40 which can interact with T cells to further activate the antigen-
presenting cell
• Purpose: capture antigen at one location and present antigen at another location; this is
accomplished by migration to lymph nodes for presentation toT cells
• When acting as peripheral sentinels (immature or antigen-capturing state), antigens are captured by
three different mechanisms:
Phagocytosis
Receptor-mediated endocytosis
Pinocytosis )>
z___,
• Immature sentinels make a transition to an antigen-presenting state and travel into circulation, and
Cl
into a state with maximal capacity to present to Th cells (increased expression of class II MHC and rn
z
co -stimulatory molecules) >:J
Al
rn
• Different types: (/)
rn
Langerhans cells: found in epidermal layers of skin (cutaneous-associated lymphoid tissue, CALT) z
Interstitial dendritic cells: found in the interstitial .spaces of virtually all organs (except brain) E
0
z

[ 357 J
End of QUIZ
5. What molecules are expressed on the surface of antigen-presenting dendritic cells?

6. What are three cell types that are known for presenting antigens to CD4+ Tcells?

7. What characteristiC: findi11gwould you see on electron micr()scopy of a dendritic cell


in a patient with langerhans cell histiocytosis?

Which MHC class is recognized by helper Tcells? Which MHCdass is recognized by


cytotoxic TceUs?

:z
0

~
:z
w
l/)
LW
a:
Q_

:z:
w
lCJ
f-
z
<(

r sse1
TCELLS
R: Chapter 6
H: Chapter 314
Thymus (FA15 p199) (FA16 p191) (SU p367) (R p625)
T cell selection (FA15 p202) (FA16 p194) (SU p372)
Helper T cell differentiation (FA15 p202) (FA16 p194) (SU p373) (R p197)
T cell activation (FA15 p203) (FA16 p195) (SU p374)
Cytotoxic T cells (FA15 p202) (FA16 p194) (SU p374) (R p211)
NK cells (FA15 p201) (FA16 p193) (SU p375) (R p192)
Memory T cells
Regulatory T cells (FA15 p202) (FA16 p194) (SU p374) (H p1010)

Warm - Rev1ew
1. Whichmusclesand nerves are derived from the third branchial art:h? (FA15 p565)
(FA16 p569) (SU p226)

2. Which drug category is used in the treatment of myasthenia gravis?


(FA15 p250) (FA16p244)

3. To which class of antiarrhythmics does each of the following agents belong?


(FA15 p30Bc309) (FA16 p302-303) (SU p70)
• Sotalol
• Propranolol
• Bretylium
• Quinidine

--1
n
m
r
r
Ul

r 3591
End of Session Qu 1z
4. Which cytokines are produced by Th1 cells? By Th2 cells?

5. Which cytokines inhibit Th1 cells? Which inhibit Th 2 cells?

6. How do cytotoxic T cells kill virus-infected cells and neoplastic cells?

lf)
_J
_J
w
u
f-

r 360 1
BCELLS AND ANTIBODIES
R: Chapter 6
H: Chapter 112
Phys: Chapter 34
B cells (FA15 p384) (FA16 p380) (SU p375) (R p191)
B cell surface markers (FA15 p209) (FA16 p202) (H p2690)
Antibodies (FA15 p204) (FA16 p196) (SU p377)
Antibody diversity (FA15 p204) (FA16 p196) (SU p378)
Antibody isotypes (FA15 p205) (FA16 p197) (SU p379)

Wa _cUp Review
1. Wijich cardiac defect is suggested by e.ac:h of the following types of murmur?
(fA1?p279) (FA16 p2(3) (SU
• Pansystolic or hOiosystolic heard best at the apex, often radiates to the
• Crescendo/decrescendo systolic murmur heard best in the 2nd and 3rd right intercostal
spaces close to the sternum ·
• Late systolic murmur usually preceded by amid-systolic click
• Early diastolic decrescendo murmur heard best along the+eft side ofthe sternum
• ~ate diastolic decrescendo murmur heard best along the left side ofthe sternum

2. Which heart defect is associated with each of the following disorders?


• Chromosome 22q11 deletion
• Trisomy 21 (Qown syndrome)
· Congenital rubella
• Turner syndronie
• Marfan syndrome ·

3. V(D)J recombination
• Rearrangements of the DNA segments named variable (V), diversity (D), and joining (J) compose the
coding regions for each specific antigen receptor on B and T cells
• The rearrangement process begins with breaks in the dsDNA at Recombination Signal Sequences
(RSSs) that flank the V, D, and J coding regions OJ

• V(D)J recombination is initiated by the recombination activating gene complex (RAG-1 and RAG-2); n
IT1
,-
RAG-1 and RAG-2 proteins recognize the RSSs ,-
Vl
• Mutation in either of the RAG genes in mice causes an inability to initiate V(D)J rearrangements and )>
z
an arrest of B and T cell development 0
)>
z
-i
OJ
0
0
m
Vl

r 361 1
En d of Session Qu 1z
4. Which surface markers are found on B cells?

5. Which immunoglobulin isotype fits each of the following statements?


• Associated with allergies because it binds to mast cells and basophils and causes them to
degranulate and release histamine
• Comprises 70-75% of the total immunoglobulin pool
• Present in large quantities on the surfaces of many B cells
• Crosses the placenta and, additionally, confers immunity to neonates in the first few months
of life
• Can occur as a dimer
• Largely confined to the intravascular pool and is the predominant early antibody frequently
seen in the immune response to infectious organisms with complex antigens
• Distributed evenly between the intravascular and extravascular pools
• The predominant immunoglobulin in mucoserous secretions .such as saliva, colostrum, milk,
tracheobronchial secretions, and genitourinary secretions
• Can be a pentamer

6. What initiates recombination of V(D)J sequences?

(f)
w
0
0
m
1--
:z:
-"(
0
:z:
<(
(j)
__J
__J
LJ_J
u
m

[ 362 J
IMMUNIZATION AND AUTOANTIBODIES
R: Chapter 6
H: Chapter 112
Phys: Chapter 34
Passive and active immunity (FA15 p210) (FA16 p203) (SU p379) (Phys p233, 442)
Vaccination (FA15 p156, 210) (FA16 p146, 203) (SU p379)
Thymus-dependent antigens (FA15 p205) (FA16 p197) (Phys p434, 438)
Autoantibodies (FA15 p213) (FA16 p207) (SU p380) (R p211)
Amyloidosis (FA15 p231) (FA16 p225)

Warm-U Rev1ew
Which organism produces an exotoxin that works via each of the following
mechanisms? (FA15 p124) (:FA16p116) (SU p323)
• Inactivates EF-2
• Activates G,
• Disables G;
• Bacterial adenylate cyclase protein involved)
• E3locks GABA and glycine

2. What drug is used to help make the diagnosis of myasthenia gravis? (FA15 p250)
(FA16 p244)

3. Which heart sound is associated with dilated congestive heart failure? Which heart
sound is associated with chronic hypertension and a stiffened left ventricle?

:;;:
3:
c
z
N

~
0
z
)>
z
0
)>
c
--1
0
)>
z
---1
o:J
0
0
rn
Ul

r 363 1
4. Autoantibodies
Autoantibody Disease
Anti-lgG antibodies

Anti-citrullinated protein antibodies (ACPA)

Antinuclear antibodies (ANA)

Anti-Smith

Anti-dsDNA

Anti-histone

Anti-centromere

Anti-Scl-70

Anti-Jo-1

Anti-SSA (anti-Ro)

Anti-SSB (anti-La)

Anti-U1-RNP

Anti-desmoglein

Anti-acetylcholine receptor

Anti-endomysia! (anti-tissue transglutaminase)

Anti-gliadin

Anti-mitochondrial

Anti-smooth muscle

Anti-glutamate decarboxylase

Anti -thyrotropin receptor

Anti-thyroid peroxidase (anti-TPO)

Anti-thyroglobulin
Vl
w
0 Anti-basement membrane
0
co
f--
z
c-ANCA
<t
0
f-- p-ANCA
:::::>
<t
0
z
<t
z
0
!;:(
N
z
::J
:2:
2:

r 3641
-~ End .o f Session Quiz
5. Which viral vaccines are potentially dangerous to patients with an egg allergy?

6. Whkh autoimmune disease associated with anti-dsDNA antibodies?

7. Which stain is used to identify amyloid microscopically, and how is its appearance
described?

:s:
:s:
c
z
N
:J:j
0
z:
)>
z:
0
)>
c
---1
0
)>
:z:
---1
ClJ
0
0
m
(/)

r 365'
HYPERSENSITMTY AID COMPLEMENT
R: Chapter 6
Phys: Chapter 34
H: Chapter 314
Hypersensitivity reactions (FA15 p211-212) (FA16 p204 -205) (SU p381) (R p201)
Complement (FA15 p206) (FA16 p199) (SU p370) (R p88) (Phys p438) (H p2665)
Complement dysfunction (FA15 p206) (FA16 p199) (SU p371)
Paroxysmal nocturnal hemoglobinuria (FA15 p394) (FA16 p392) (SU p265) (R p642)

Warm-Up Review
1. What gives rise to the jugular venous a, c, and v waves? (FA15 p276) (FA16 p270) (SU p66)

2. Which heart murmur is associated with weak pulses? (FA15 p279) (FA16 p273) (SU p89)

3. What findings are seen in Ehlers-Danlos syndrome? (FA15 p77) (FA16 p63) (SU p236, 294)

1--
z
LLJ
:?:
LLJ
_j

0...
:?:
0
u
0
z
<(
>-
1--
>
1-
Vl
z
u.J
(f)
0:::
LLJ
0...
>-
:::r:

r 366 1
4. Paroxysmal nocturnal hemoglobinuria (PNH)
• RBCs are deficient in two surface molecules that help protect against the MAC:
- Glycosylphosphotidylinositol (GPI), wh ich ancho rs _ _ _ _ __ _ _ __ _ ___ (CD 55)
to the plasma membrane
MAC-inhibitory protein ' - - - -
• Clinical featu res:
- Chronic intravascu lar hemolysis

- Thrombosis
• Diagnosis: Ham's test, flow cytometry
• Treatment:

nd Sess1on UI Z
5. Which type of hypersensitivity is responsible for each of the following clinical
problems?
• Poststreptococcal glomerulonephritis
• Asthma
• Rheumatic fever
• Tuberculosis skin test

• Allergies, anaphylaxis, and hay fever


• Polyarteritis nodosa
• Serum sickness
• ABO blood type incompatibility

• Poison ivy
• Eczema
• Contact dermatitis
• Goodpasture syndrome

6. A patient suffers from recurrent Neisseria infections. Which complement proteins are
deficient?
:r:
-<
-o
rn
:;u
Ul
rn
z:
Ul
=i
7. Which complement is responsible for neutrophil chemotaxis? <
-1
-<
)>
z:
0
n
0
:s:
-o
r-
m
:s:
rn
z:
-1

r 1(..7 1
MACROPHAGE$, GRAIULOCYTES &CYTOKINES
GG: Chapters 35, 37, 62
Phys: Chapter 33
R: Chapter 6
Spleen (FA15 p199) (FA16 p191) (H p367) (R p623)
Monocytes (FA15 p383) (FA16 p379) (H p376) (Phys p426) (H p2654)
Macrophages (FA15 p383) (FA16 p379) (H p376) (R p192)
Granulocytes
Eosinophils (FA15 p383) (FA16 p379) (H p376) (R p96) (Phys p430)
Mast cells (FA15 p383) (FA16 p379) (H p376)
Cyto!?ines (FA15 p207) (FA16 p200) (H p376) (R p196, 203)
Interferons (FA15 p195, 207-208) (FA16 p187, 200-201) (H p376) (R p221)
Recombinant cytokines (FA15 p219) (FA16 p213) (H p377)

Warm-Up Rev1ew
1. Which heart sounds are c::onsidered benign if there is no evidence of disease?

2. Which organisms do not take.Gram stain? (FA15 p120) (FA16 p110) rsu p319)

3. What is the cause of Duchenne muscular dystrophy? (FA15 p85) (FA16p73) (SU p24'lJ

(/)
w
z
~
0
1-
>-
u
o<5
(/)
w
1-
G 4. Monocytes
0
~ Leave bone marrow for blood ~ circulate 8-12 hours to mature ~ migrate into tissues
~ • Skin , connective tissue - dendritic cells (histiocytes)
f3 • Alveoli, intestines, spleen - macrophages
(/)
w • Liver - Kupffer cells
CJ
<r: • Brain - microglia
I
o_
0 • Bone - osteoclasts
0:::
u • Joints - type A svnoviocytes
<r:
::;;;:

I .lAB l
5. What is the differential diagnosis for eosinophilia?
CANADA-P
• Collagen vascular disease (PAN, dermatomyositis)
• Atopic diseases (allergies, asthma, Churg-Strauss, allergic bronchopulmonary aspergillosis)
• Neoplasm
• Adrenal insufficiency (Addison disease)
• Drugs (NSAIDs, penicillins, cephalosporins)
• Acute interstitial nephritis
• Parasites (Strongyloides, Ascaris~ Loeffler eosinophilic pneumonitis)
(Other causes: HIV, hyper-lgE syndrome, coccidioidomycosis , etc.)

Se on UIZ
6. What cytokines match each of the following statements?
• Produced by Th1 cells
• Produced by Th2 cells
• Promotes growth and differentiation of B cells
• Promotes growth and activation of eosinophils
• Pyrogens secreted by macrophages

• Enhances synthesis of lgE and lgG


• Enhances synthesis of lgA
• Mediate inflammation
• Secreted byTh1 cells and activates macrophages
• Secreted byTh2 cells and inhibits macrophage activation

• Inhibits production of Th1 cells


• Ihhibits production ofTh:z cells
• Released by virus-infected cells
• Supports growth and differentiation of bone marrow stem cells
• Supports T cell proliferation, differentiation, and activation

7. What is the function of macrophages .in the spleen? :s:


)>
n
;;o
0
-o
I
)>
Cl
rn
_lf)

Cl
;;o
)>
z
c
,.--
0
n
-<
-1
rn
lJ)

QO
n
-<
-I
0
:::":
z:
rn
lJ)

r 369 1
IMMUNOSUPPRESSAITS
R: Chapter 6
Phys: Chapter 33
GG: Chapters 35, 37, 62
R: Chapter 6
Transplant rejection (FA15 p217) (FA16 p211) (SU p384) (R p231)
Immunosuppressants (FA15 p218) (FA16 p212) (SU p385-386) (GG p1006)
Cyclosporine (FA15 p218) (FA16 p212) (SU p385)
- Tacrolimus and pimecrolimus (FA15 p218) (FA16 p212) (SU p385)
Sirolimus (FA15 p218) (FA16 p212) (SU p385)
Azathioprine (FA15 p218) (FA16 p212) (SU p385)
Mycophenolate (FA16 p212) (SU p385)
Thalidomide (SU p386)
Therapeutic antibodies (FA15 p220) (FA16 p214) (SU p386) (GG p1023)

Warm - Up Rev1ew
1. A pregnant patient comesto the clinic complaining of new dark pigmentation on her
face. What is the most likely diagnosis? (FA15 p43B) (FA16 p439) (SU p252)

2. A woman who iS breastfeeding develops redness and swelling of her right breast.
Examination shows a localized area of erythema and edema that is tender to palpation. What
is the most likely diagnosis? (FA15 p590) (FA16 p595) (SU p216)

3. A 55-year-old man who is a smoker and heavy drinker presents with a new cough and flu-like
symptoms. Gram stain shows no organisms, but silver stain of sputum shows numerous short
rods. What is the most likely diagnosis? (FA15 p137) (FA16 p127) (SU p320)

<J)
f-
z
<{
<J)
<J)
LW
0::
0...
0...
~
<J)
0
z
::::)

~
~

r 37o 1
4. Sirolimus
• Mechanism: binds _ _ _ _ _ _ _ _ _ _ _ _ _ _ _ _ __ _ _ _ _ ~ inhibits mTOR
(mammalian target of rapamycin) ~ inhibits T cell proliferation
• Not nephrotoxic

5. Mycophenolate (mycophenolate mofetil)


• Mechanism: inhibits inosine monophosphate (IMP) dehydrogenase ~ inhibits _ _ _ _ _ __
synthesis ~ prevents lymphocyte proliferation
• Uses: transplants, lupus nephritis (off-label)
• Side effects: hyperglycemia, hypercholesterolemia , hypertension, infection, lymphoma

6. Thalidomide
• Mechanism: suppresses TN F-a production, increases NK cells and IL-2
• Uses: erythema nodosum leprosum (Hansen disease), multiple myeloma
• Side effects: phocomelia (prior use as a sedative during pregnancy)

End Session 8u1z


7. Which immunosuppressant matches each of the following statements?
• Precursor of 6-mercaptopurine
• Inhibits IMP dehydrogenase
• Binds to cyclophilins and inhibits calcineurin resulting in the loss of IL-2 production

• Binds FK-binding protein (FKBP) leading to loss of IL-2 prodwction


• Binds FKBP-12 leading to inhibition of mTOR and inhibition ofT cell proliferation
• Used for lupus nephritis
• Metabolized by xanthine oxidase, therefore allopurinol increases its toxicity

3::
3::
c
z
0
Vl
c
-u
\J
:;o
rn
Vl
Vl
J>
:z
-1
Vl

r 371 1
IMMUNODEFICIENCIES
R: Chapter 6
T cell deficiencies
- Thymic aplasia (DiGeorge syndrome) (FA15 p214) (FA16 p208) (SU p382)
- Chronic mucocutaneous candidiasis (FA15 p214) (FA16 p208) (SU p382)
- IL-12 receptor deficiency (FA15 p214) (FA16 p208)
8 cell deficiencies
- Bruton agammaglobulinemia (FA15 p214) (FA16 p208) (SU p381)
- Selective immunoglobulin deficiencies (FA15 p214) (FA16 p208) (SU p382)
Combined B and T cell deficiencies
- Severe combined immunodeficiency (SCID) (FA15 p215) (FA16 p209) (SU p382)
- Ataxia-telangiectasia (FA15 p215) (FA16 p209) (SU p383)
- Wislwtt-Aldrich syndrome (FA15 p215) (FA16 p209) (SU p383)
- Hyper-lgM syndrome (FA15 p215) (FA16 p209) (SU p382)
Phagocyte disorders
Chronic granulomatous disease (FA15 p215) (FA16 p209) (SU p383)
- Chediak-Higashi disease (FA15 p215) (FA16 p209) (SU p384)
Hyper-tgE (Job syndrome) (FA15 p214) (FA16 p208) (SU p384)
- Leukocyte adhesion deficiency syndrome (FA15 p215) (FA16 p209)

Warm-Up Rev1ew
1. What is the ECG axis given the QRS deflections in each of the following scenarios?
• Positive in lead I, positive in lead II
• Negative in lead I, positive in lead Ill
• Positive in lead I, negative in aVR ·

2. Which antiarrhythmic has the side effect of cinchonism? (FA15 p259) (FA16 p253) (SU p450)

3. Which stain is required to see each of the following organisms? (FA15 p120) (FA16 p110)
(SU p319-320)
• Cryptococcus neoformans
• Pneumocystis jirovecii (PCP)
• Chlamydia trachomatis

U)
w
u
z
w
u
LL
w
0
0
z
::J
2:
::2:

[ 372 J
T Cell Deficiencies
4. Thymic aplasia (DiGeorge)
• 3rd and 4th pharyngeal pouches fail to develop
No thymus --+ no mature _ __ _ __ _
No parathyroids --+ --+ hypocalcemia and tetany
• Recurrent viral, fungal , and protozoal infections
• Congenital defects in heart/great vessels
• 90% have a chromosome 22q11 deletion (detect with FISH)

5. Chronic mucocutaneous candidiasis


• T cell dysfunction vs. C. albicans
• Treatment: ketoconazole

6. IL-12 receptor deficiency


• tv1ycobacterial and fungal infections

B Cell Deficiencies
7. Bruton agammaglobulinemia
• X-linked (boys)
• B cell deficiency --+ defective tyrosine kinase gene --+ low levels of all immunoglobulins
• Recurrent bacterial infections after 3-6 months

8. Selective immunoglobulin deficiencies


• lgA deficiency is most common
tv1ost appear healthy
Sinus and lung infections
1/600 European descent
- Associated with atopy, asthma
Possible anaphylaxis to blood transfusions and blood products

:s
:s
c
z
0
0
rn
:1J
n
rn
z
n
rn
(J)

r 3731
Combined B and T Cell Deficiencies
9. Severe combined immunodeficiency (SCI D)
• Defect in early stem cell differentiation
• Can be caused by at least seven different gene defects:
_ _ _ _ _ _ _ _ _ _ _ _ _ _ _ _ _ _ _ deficiency
• Last defense is NK cells
• Presentation triad:
Severe recurrent infections
I) Chronic mucocutaneous candidiasis
2) Fatal or recurrent RSV, VZV, HSV, measles, flu , parainfluenza
3) Pneumocystis jirovecii pneumonia (PCP)
Chronic diarrhea
Failure to thrive
• No thymic shadow on newborn chest x-ray

10. Ataxia-telangiectasia
• lgA deficiency and T cell deficiency ~ sinus and lung infections
• Cerebellar ataxia and poor smooth pursuit of moving target with eyes
• Telangiectasias of face (after 5 years of age)
• Radiation sensitivity (try to avoid x-rays)
• Increased risk: lymphoma and acute leukemias
• Elevated AFP (after 8 months of age)
• Average age of death: 25 years

11. Wiskott-Aidrich syndrome


• WAITER
- Wiskott
- Aldrich
Immunodeficiency
- Thrombocytopenia and purpura
Eczema
Recurrent pyogenic infections
• X-linked
• No lgM vs. bacterial capsular polysaccharides
• Low lgM, high lgA

12. Hyper-lgM syndrome


• Increased lgM; other antibody isotypes decreased
U)
LLJ
• The two most important variants include:
u
zLLJ - AR ~ no CD40 on B cells
u - X-linked ~ no CD40 ligand on helper T cells (most common)
LL
LLJ
0
0
z
:::J
~
~

[ 374 J
Phagocyte Disorders
13. Chronic granulomatous disease (CGD)
• X-linked inheritance (65-70%)
• Lack of NADPH oxidase ~ phagocytes cannot destroy catalase-positive microbes
• Especially susceptible to infections
• Diagnosis: negative nitroblue tetrazolium (NBT) test
No yellow to blue-black oxidation
• Treatment:
Prophylactic TMP-SMX and itraconazole
IFN-y also helpful

14. Chediak-Higashi syndrome


• Defective LYST gene (lysosomal transport)
• Defective phagocyte lysosomes ~giant cytoplasmic granules in PMNs are diagnostic
• Presentation triad:
Partial albinism
Recurrent respiratory tract and skin infections
Neurologic disorders

15. Hyper-lgE syndrome (Job syndrome)


• Mutation in the gene for STAT3 signaling protein leading to:
Impaired differentiation of Th 17 cells
Impaired recruitment of neutrophils
• High levels of lgE and eosinophils
• Presentation triad:
Eczema
Recurrent cold 5. aureus abscesses (think of biblical Job with boils)
- Coarse facial features: broad nose, prominent foreh ead ("frontal bossing"), deep-set eyes, and
"doughy" skin
• Also common to have retained primary teeth resulting in two rows of teeth

16. Leukocyte adhesion deficiency syndrome


• Abnormal integrins ~ inability of phagocytes to exit circulation
• Delayed separation of umbilical cord

:s
:s
c
z
0
CJ
,rn
n
m
:z:
n
rn
Vl

f 375 l
End of Sess1o n Qui z
17. What are the X-linked immunodeficiencies?

18. What are the clinical features of Wiskott-Aidrich syndrome?

19. What is the cause of chronic granulomatous disease? To what infections are these
individuals susceptible?

20. A young child presents with tetany (from hypocalcemia) and candidiasis (resulting
from immunosuppression). What cell type is deficient in this patient?

(/")
LLJ
u
z
LLJ
u RAPID - FIRE FACTS
LL
w
0 Ecze ma, recurrent co ld abscesses, high serum lgE
0
z
=>
~ Large phagosoma l vesicles in neutrophi ls

f 376 l
)0/NUS ON AJOU

G/ THROUGH THE
GASTROINTESTINAL
TR ACT, FROM THE MOU TH
1 Oropharynx
TO THE ANUS AND ALL
POINTS IN BETWEEN .·
2 Es ophagus
INCLUDES EXCURSIONS
TO SCENIC DESTINATIONS
3 Stom ach
SUCH AS THE LIVER, THE
BILIARY SYSTEM AND TH
4 Upper Gl Pathology
NCREAS:

5 Duodenum

6 Enterocytes and Absorption

7 Disea ses of the Sm all Intestine

8 Large Inte stme part 1


.~'._

9 Large Intestine part 2

10 Large Inte stine part 3

11 Pancreas

12 L1ver Bas1cs

13 Alcoholic L1ver D1sea se and Cirrhos is

14 L1ver Pathology

15 Hepat itis

16 B1l1 ary Tract


0
OROPHARYNX ::0
0
"""0
:r:
R: Chapter 16 )>
::0
Phys: Chapter 64 -<
H: Chapter 317 z
><
Tongue development (FA15 p450) (FA16 p452) (SU p3B)
Tongue pathology
Salivary glands
Salivary gland pathology
Tumors of the salivary glands (FA15 p357) (FA16 p353)
Cleft lip and cleft palate (FA15 p566) (FA16 p570)
Nose
Paranasal sinuses
Tonsils and adenoids

p Rev1ew
1. What are the functions of interleukins 1-5? (FA15 p207) (FA16 p200)

2. After taking a course of amoxkillin, an adult patient develops toxic megacolon and
diarrhea. What is the mechanism of this diarrhea? (SUp326)

[ 379]
X
z 3. Tongue development
>-
0::
<t:
I Sensation
Q_
0 • Glossopharyngeal nerve (CN IX)
0::
0 3rd and 4th branchial arches
Taste
• Mostly from CN IX
• Very posterior area from
vagus nerve (CN X)

Sensation
1st branchial arch • Mandibular division branch of
trigeminal nerve (CN V3 )

Taste
• Facial nerve (CN VII)

4. Tongue pathology
• Glossitis: inflammation of the tongue caused by vitamin B and iron deficiencies
• Leukoplakia: areas of keratosis , more common in smokers
• Oral thrush: caused by Candida albicans, seen in infancy and immunocompromise
• Oral herpes: caused by HSV-1 primarily (rarely HSV-2)
• Aphthous ulcers: painful ulcer associated with trauma , citrus fruits, stress, food allergies and vitamin
812 deficiency

! 380 j
5. Salivary gland pathology 0
;;cJ
0
• Xerostomia: severe drying of the mouth , associated with Sjogren syndrome v
:r:
)>
• Sialolithiasis: caused by ston es blocking th e salivary gland duct; treat with sour candy ;;cJ
-<
• Sialadenitis: most common bacteria are Staphylococcus aureus and viridans group streptococci z
X

6. Tumors of the salivary gland


• Pleomorphic adenoma: most common, comprised of epithelial and mesenchymal cells, benign, can
lead to facia l nerve injury
• Warthin tumor: second most common benign tumor
• Mucoepidermoid carcinoma: most common malignant tumor and second most common overall
• Location of tumor: if located in parotid then < 30% malignant, if located in sublingual then > 70%
malignant, other glands between these two

7. Nose
• Infectious rhinitis: common cold caused by coronavirus, rhinovirus, adenovirus, and echovirus
• Allergic rhinitis : triggered by airborne allergens; treat with intranasal steroids
• Nasal polyps: overgrowth of mucosa; treat with intranasal steroids
• Cocaine: potent vasoconstrictor that can cause perforation of nasal mucosa
• Eustachian tube dysfunction: blocked by mucus or inflammation; treat with intranasal steroids

r 381 1
X
z
>-
0:::
<(
I
End of Sess1on Quiz
0...
0
cr:
8. What is the most common Joc::ation of salivary gland tumors?
0

9. What is the most common salivary gland tumor? What is the histological appearance
of this tumor?

10. What is the second most common benign salivary gland tumor?

11. What is the most common malignant salivary gland tumor (the second most common
tumor overall of the salivary gland)?

12. What medication is often used in the treatment of allergic rhinitis, nasal polyps, and
eustachian.tube dysfunction?

r 382 1
rn

ESOPHAGUS (/)
0
""0
:r:
R: Chapter 17 )>
Cl
H: Chapters 91, 292 c
Vl

Innervation and blood supply of the G/ tract (FA15 p346) (FA16 p342) (SU p116)
Layers of the Gl tract (FA15 p345) (FA16 p341) (SU p122)
Gl developmental pathology (FA15 p342) (FA16 p338) (SU p116)
Achalasia (FA15 p357) (FA16 p354) (SU p121)
Esophageal diverticula
Esophageal variceal bleeding (FA15 p358) (FA16 p354) (SU p119)
Boerhaave syndrome (FA15 p358) (FA16 p354) (SU p119)
Mallory-Weiss tear (FA15 p358) (FA16 p354) (SU p119)
Gastroesophageal reflux disease (GERD) (FA15 p358) (FA16 p354)
Hiatal hernia (FA15 p352) (FA16 p349) (SU p125)
Esophagitis (FA15 p358) (FA16 p354)
Esophageal strictures (FA15 p358) (FA16 p354)
Plummer-Vinson syndrome (FA15 p358) (FA16 p354)
Esophageal cancer (FA15 p359) (FA16 p355) (SU p120)

Warm -Up Rev1ew


1. On which type of immune cell is each of the following cell surface proteins found?
(FA1Sp209) (FA16 p202)
• CD4
• CD14
• CD16
• CD19
• CD3
• CDS

2. A child with an immune disorder has recurrent staphylococcal abscesses. It is found


thatthe neutrophi.ls fail to respond because the chemotactic stimuli are deficient.
What is the most likely diagnosis? (FA15 214) (FA16 p208)

r 383 1
(/)
=::> 3. Abdominal wall defects
CJ
<{
:r:: • Omphalocele
Q_
0 Extruding viscera covered by sac (sac composed of peritoneum and amnion)
(/)
UJ Liver often found protruding
Other anomalies (GI, GU, CV, CNS , MS) common (50%)
• Gastroschisis
Extruding viscera not covered by sac
Liver never found protruding
Other anomalies less common (10-15%)
Defect lateral to umbilicus (R > L)

4. Esophageal diverticula
• Zenker diverticulum: immediately above the upper esophagea l sphincter
• Traction diverticulum: near the midpoint of the esophagus
• Epiphrenic diverticulum: immediately above the lower esophageal sphincter

5. What upper Gl problem is associated with the following findings/symptoms?

Specialized co lumnar epithelium seen in a biopsy from distal


esophagus

Biopsy of a patient with esophagitis reveals large, pink,


intranuclear inclusions and host cell chromatin that is pushed
to the edge of the nucleus

Biopsy of a patient with esophagitis reveals enlarged


cells, intranuclear and cytoplasmic inclusions, and a clear
perinuclear halo

An esophageal biopsy reveals a lack of ganglion cells between


the inner and outer muscular layers

Protrusion of the mucosa in the upper esophagus

Outpouching of the esophagus found just above the LES

Goblet cells seen in the distal esophagus

A PAS stain on a biopsy obtained from a patient with


esophagitis reveals hyphate organisms

Esophageal pouch found in the upper esophagus


m
IJ)
0
-o
End of Sess1on Quiz I
)>
Cl
6. ·What is the histologic change found in Barrett esophagus? c
IJ)

7. What are the names of the diverticula found in the esophagus based on their anatomical
locations?

8. •A 60-year-old man with chronic reflux presents with esophageal cancer. What is the most .
likely histologic subtype?

RAPID -FIRE FACTS


Dysphagia, glossitis, and iron deficiency anemia

Hematemesis with retching

r -:sr1s 1
:r:
u
<(
2:
STOMACH
0
f- H: Chapter 293
(/)
R: Chapter 17
Phys: Chapter 64
GG: Chapter 45

GJ blood supply (FA15 p346) (FA16 p342) (SU p116) (COA p250)
Celiac trunk (FA15 p347) (FA16 p343)
Gastric secretory products (FA15 p354) (FA16 p351) (SU p119)
Gastric acid physiology (FA15 p354-355) (FA16 p351)
Zollinger- Ellison syndrome (FA15 p336) (FA16 p332) (SU p139)

Warm-Up Review
1. Which.cytokine(s) matches each of the following statements? (FA15 p207) (FA16 p2DO! (SU p376-377}
• Promotes 8 cell growth and differentiation
• Produced by Th1 cells
• Produced by Th2 cells
• Secreted by helper T cells and activates macro phages
• Pyrogens secreted by monocytes and macrophages
• Enhances synthesis of lgE and lgG
• Enhances synthesis of lgA
• ·Released by virus-infected cells

[ 386 J
Ul
--1
0
En Sess1 on Quiz :s
)>
n
:r:
2. What is the artery of the foregut? Of the.midgut? Of the rectum and distal third of the colon?

3. What disorder is characterized by an excess of gastrin? What is the treatment?

4. What receptors found on gastric parietal cells regulate acid secretion?

5. Whathappens to serum pH at the time of gastric acid secretion?

r 1 R7 1
>-
UPPER Gl PATHOLOGY
(..9
0_ j
0
:r:
H: Chapters 39, 91, 293
~
o_ Phys: Chapter 64
(..9
GG: Chapters 45, 46
0:::
w Acute gastritis (FA15 p359) (FA16 p356) (SU p120)
o_
o_
Chronic gastritis (FA15 p359) (FA16 p356) (SU p120)
::::>
Gastric ulcer (FA15 p360) (FA16 p357) (SU p120)
Duodenal ulcer (FA15 p360) (FA16 p357) (SU p120)
Peptic ulcer complications (FA15 p360) (FA16 p357)
H. pylori treatment (FA15 p140) (FA16 p130) (SU p120)
Pharmacotherapy for disease related to gastric acid
- Antacids (FA15 p379) (FA16 p375) (SU p140)
- H2 blockers (FA15 p378) (FA16 p374) (SU p139)
- Proton pump inhibitors (FA15 p378) (FA16 p374) (SU p140)
- Bismuth and sucralfate (FA15 p378) (FA16 p375) (SU p140)
Misoprostol (FA15 p379) (FA16 p375) (SU p140)
Menetrier disease (FA15 p359) (FA16 p356)
Gastric cancer (FA15 p360) (FA16 p356) (SU p120)
Pyloric stenosis (FA15 p342) (FA16 p339) (SU p118)
Ondansetron (FA15 p380) (FA16 p376) (SU p140)

Warm-Up Rev1 ew
1. Label the following Gram(+) algorithm. (FA15 p127) (FA16 p118) (SU p324-327)

Gram(+) _ __,._.
~
/~
~ <
<
2. What upper Gl problem is associated with each of the following findings?
Stomach biopsy reveals neutrophils above the basem ent
membrane, loss of surface epithelium , and fibrin-containing
purulent exudate
Stomach biopsy reveals lymphoid aggregates in the lamina
propria, columnar absorptive cel ls, and atrophy of glandular
structures
Diffuse thickening of gastric folds, elevated serum gastrin levels,
biopsy revea ls glandular hyperplasia without foveolar hyperplasia

[ 388]
d of Session Quiz
What are the two most common underlying causes of a hospital admission for a --o
bleeding peptic ulcer? ~
::r::
0
,--
0
Gl
-<

4. Whkh antacid matches each of the following statements?

A patient taking NSAIDs for the management of her gout develops anemia, has pain
with eating, and is positive on occult blood test. What drug would most directly
address t anism behind this patient's current problem?

6. Where is Virchow's node located?

RAP! O- Ff RE FACTS
Mucin-filled cell with a peripheral nucleus

Most common type of stomach cancer

Ovarian metastasis from gastric cancer

Gastric ulcerations and high gastrin levels

Acute gastric ulcer associated with elevated ICP


or head trauma

Acute gastric ulcer associated with severe burns

r 3891
:2:
=:J
z
L1.J
DUODENUM
0
0 R: Chapter 17
=:J
0 Phys: Chapter 64

Duodenal atresia (FA15 p365) (FA16 p338) (SU p118)


Sphincter of Oddi and ampulla of Vater (FA15 p350) (FA16 p347) (SU p136)
Gl hormones (FA15 p353-354) (FA16 p350-351) (SU p117)
Brunner glands (FA15 p345, 360) (FA16 p341, 357)
Prokinetic agents
Glligaments (FA15 p344) (FA16 p340)
Retroperitoneal structures (FA15 p343) (FA16 p339)
Abdom i nal CT

Warm-Up Rev1ew
1. Distinguish the followingtypes of organ transplant rejection. (FA15 p217) (FA16 p211)
(SU p384-385)

Type of Rejection Mechanism Time Frame

Hyperacute

Acute
I
...

Chronic

..,

2. Which enzymes do obligate anaerobes lack? (FA15 p121) (FA16 p111) (SU p318)

3. Which Mycobacterium species fits each of the following descriptions? (FA15 p133 -134)
(FA16 p124"125)
• Causes leprosy
• Causes pulmonary TB-Iike symptoms in COPD patients
• Causes cervical lymphadenitis in children
• Causes a disseminated disease in AIDS patients
• Associated with hand infections in aquarium handlers

r 390 1
0
c
4. Prokinetic agents 0
0
t ACh , t 5-HT (carcinoid syndrome), ! 0 2 rn
z:
c
• Cholinergic agonists (bethanechol) :s
• Acetylcholinesterase inhibitors (neostigmine)
• Metoclopramide (Reglan): stimu lates 5-HT4 , inhibits 0 2
• Macrolides: stimulate smooth muscle motilin receptors

End of Sess1on Quiz


5; What important secretory products are secreted from each ofthe following cells of
the Gl tract?
• G cells
•I cells
• S cells
• D cells
• Gastric parietal cells

6: •· What Glligarnent matches each of the following descriptions?


• Contains the portal triad and may be compressed to control bleeding
• Attaches the spleen to the posterior abdominal wall
• Attaches the spleen to the stomach

7. What is the mechanism of action of metoclopramide?

[ 391 ]
z
0
t-
o_
ENTEROCYTES AND ABSORPTION
a:::
0 R:Chapter17
V)
co Phys: Chapter 65
<(
H: Chapter 294
0
z
<( Small bowel histology
(f)
L1.J Digestion and absorption offood (SU p122)
1- Location of absorption of vitamins, minerals and nutrients (FA15 p356) (FA16 p352) (SU p124)
>-
u Iron
0
a::: - Folate
L1.J
1- - Vitamin 8 12
z Schilling test
L1.J
Malabsorption syndromes of the small intestine (FA15 p361) (FA16 p358) (SU p129)
Abetalipoproteinemia
- Celiac disease
- Disaccharidase deficiency
- Tropical sprue
- Whipple disease
Bacterial overgrowth
- Pancreatic insufficiency

Warm -Up Review


1. Which type of hypersensitivity is responsible for each of the following clinical
problems? (FA15 p212) (FA16 p204-205)
• Asthma
• Tuberculosis skin test
• Allergies, anaphylaxis, and hay fever
• ABO-blood type incompatibility
• Poison ivy
• Eczema
• Goodpasture syndrome

2. What are the four obligate aerobic bacteria? (FA1 5 p121) (FA16 p111)

l'· ..3<)')
L ]
rn
3. What is the Schilling test? z
---1
• Stage 1: Radiolabeled vitamin 8 12 + IM unlabeled vitamin 8 12 rn
:::0
0
Measurement of urinary excretion of radiolabeled vitamin 8 12 over a 24-hour period n
-<
Normal = 10% of the oral dose recovered in the urine ---1
rn
V>
• Stage 2: Radiolabeled vitamin 8 12 + intrinsic factor )>
z
Normal indicates pernicious anemia 0
)>
• Stage 3: Radiolabeled vitamin 8 12 + antibiotics CD
V>
Normal indicates bacterial overgrowth 0
:::0
"lJ
• Stage 4: Radiolabeled vitamin 8 12 + pancreatic enzymes ---1

Normal indicates malabsorption due to pancreatitis 0


z

En d of Sess 1on Ou1 z


4. What serum antibodies are associated with celiac ctlsea~;e!

5. What is the treatment for celiac sprue?

6. What organism is associated with Whipple disease?

7. What pathology of the small intestine is most closely associated with each of the
following statements?
• Srnall intestinal mucosa laden with distended macrophages in the lamina propria
• Defect in chylomicron exportation
• Cramping associated with milk products

RAP! O-F! RE FA CTS


Weight loss, diarrhea, arthritis, fever, adenopathy
and hyperpigmentation
Anti-transglut.::Jmin.::Jse/anti-gliadin antibodies

Vitamin given to pregnant women to prevent


neural tube defects

r 393 1
L1.J
z
f-
l/)
DISEASES OF THE SMALL INTESTINE
L1.J
f- H: Chapters 295, 296, 298, 299
z
_j
R: Chapter 17
_j
<(
:E Irritable bowel syndrome (FA15 p363) (FA16 p360)
(./)
Meckel diverticulum (FA15 p364) (FA16 p361) (SU p118)
L1.J
:r: Intussusception (FA15 p365) (FA16 p362)
f- Small bowel obstruction
LL
0 Intestinal ileus (FA15 p365) (FA16 p362)
(./) Meconium ileus (FA15 p365) (FA16 p362) (SU p113)
L1.J
(./) Necrotizing enterocolitis (FA15 p365) (FA16 p362)
<(
L1.J Intestinal ischemia (FA15 p365) (FA16 p362) (SU p125)
V)
Angiodysplasia (FA15 p365) (FA16 p362)
0
Carcinoid tumor (SU p125)

Warm -Up. Revi ew


1. The mother of a patient recently hospitalized for Haemophilus meningitis is started
on a prophylactic antibiotic. Two days later, the mother calls the clinic complaining
of blood in her urine. What is the most likely cause of her symptoms? (FA15 plBB)
(FA16 p1 79) (SU p340)

2. V\fhich immunosuppressant fits each of the following descriptions? (FA1Sp218)


(FA16 p212) (SU p385-386)
" Prod rug of 6-mercaptopurine
• Causes phocomelia
• Nephrotoxic in 75% of patients
• SE: acne, osteoporosis, HTN, hyperglycemia, immunosuppression ~ infection
• Inhibits secretion of IL-2 and other cytokines

3. What hematologic changes would be seen in a patient without a functional spleen?


(FA15 p199) (FA16 p191) (SUp257)

[ 394 J
4. Irritable bowel syndrome (IBS) 0
Vl
rn
• Clinical manifestations: chronic abdominal pain and altered bowel habits )>
Vl
rn
• Abdominal pain described as crampy with variable intensity that improves with defecation Vl

• Altered bowel habits include diarrhea, constipation, and alternating diarrhea and constipation ,0
___,
• Other Gl symptoms: GERD, dysphagia, early satiety, nausea and chest pain I
rn
• Non-GI symptoms: urinary frequency and urgency, dysmenorrhea, dyspareunia and fibromyalgia Vl
s::
)>
• Not associated with IBS: rectal bleeding, nocturnal abdominal pain, weight loss, anemia, elevated r-
r-
inflammatory markers, electrolyte abnormalities
z
-l
• Treatment: dietary modification (avoid lactose or gluten), fiber supplement, antispasmodics rn
Ul
___,
(dicyclomine and hyoscyamine), antidepressants (TCA, SSRI), guanylate cyclase agonists (IBS with
constipation)
zrn

5. What are the common causes of small bowel obstruction (SBO)?


• ABCs
- Adhesions: surgical adhesions from previous surgeries (about 75% of cases)
- Bulge: hernia (second most common cause)
- Cancer: tumors (most commonly metastatic colorectal cancer)
• Other less common causes: volvulus, intussusception, Crohn's disease, gallstone ileus, bezoar, bowel
wall hematoma from trauma, inflammatory stricture, congenital malformation, radiation enteritis

6. What intestinal disorder is common in the NICU to premature babies that receive oral
feeds too soon?

7. .What are the common causes of small bowel obstruction?

8. What are the classic symptoms of carcinoid syndrome?


r---
0:::
<(
0...
LARGE INTESTINE PART I
w R: Chapters 7, 17
z
r--- H: Chapters 91, 300, 350
<.1)
LLJ COA: Chapter 2
r-
z
Hirschsprung disease (FA15 p365) (FA16 p361) (SU p118)
w
l.9 Volvulus (FA15 p364) (FA16 p362)
0:::
<( Appendicitis (FA15 p363) (FA16 p360) (SU p125)
__J
Pectinate line (FA15 p349) (FA16 p345)

Warm-Up Review
1. Which heart defect is associated with each of the following disorders? (FA1S p290)
(FA16 p284)
• Chromosome 22q11 deletion
• Down syndrome
• Congenital rubella
• Turner syndrome
• Marfan syndrome

2. What four-drug regimen is used to treat TB? (FA15 p1BB) (FA16 p179)

3. Where are B cells and TceUs found in the spleen? In the lymph nodes? (FA15 p19B-199)
(FA16 p190-191)

r 3961
I
)>
:::0

End o f Sess ion u1 z Cl


m
:z
-1
4. What is the classic presentation of appendicitis? m
(J)
--l
z
rn
-o
)>
:::0
-1

5. What is the fundamental problem in Hirschsprung disease?

6. Where in the distal Gl tract does endodermal tissue make anabrupttransition to


tissue derived from surface ectoderm?

RAP/ O-F! RE FACTS


Most common surgical emergency

Severe RLQ pain with rebound tenderness

r 197 1
N
1-
0:::
<(
0...
LARGE INTESTINE PART 2
w
z H: Chapters 295, 297
1- R: Chapter 17
(/)
w
1-
z Colon polyps (FA15 p366) (FA16 p363) (SU p130)
w Peutz-Jeghers syndrome (FA15 p366) (FA16 p363) (SU p130)
l:J Colon cancer (FA15 p367) (FA16 p364) (SU p125)
0:::
<( Genetics of colon cancer (FA15 p367) (FA16 p364)
__j
Polyposis syndromes (FA15 p366) (FA16 p363) (SU p131)

Warm -Up Review


1. Which cytokines are secreted by the two different types of helper T cells? (FA15p2D2)
(FA16 p194)

2. What are five classes of medications used to treat glaucoma? (FA15 p494) (FA16 p498)

3. Damage to which nerve causes each of the following upper extremity symptoms?
(FA15 p418-419) (FA16 p420-421)
• Wrist drop
• Scapular winging
• Inability to wipe bottom
• Loss of forearm pronation
• Weak external rotation of arm
• Loss of elbow flexion and forearm supination
• Loss of wrist extension

r 398 1
E.nd of Sess1on Quiz
4, Which pathologicallesions.of the colon are considered precursors to malignancy?
Which colon pathology has the most malignant potential?

5. What gene mutations are commonly seen in the development of colorectal cancer?

6, · What are the risk factors for colon cancer?

RAP! O- F! RE FACTS
Gl hamartomas, hyperpigmentation of the mouth and hands

Multiple colon polyps, osteomas, soft tissue tumors

"Apple core" lesion on barium enema

[ 399 J
(Y")

r--
0::
<(
CL
LARGE INTESTINE PART 3
Ll.J
:z R: Chapters 7, 17
r- H: Chapters 91. 300, 350
tf)
Ll.J
COA: Chapter 2
r-
z Diverticulosis (FA15 p363) (FA16 p360) (SU p125)
Ll.J Diverticulitis (FA15 p363) (FA16 p360) (SU p125)
\C)
0:: Inflammatory bowel disease (FA15 p362) (FA16 p360) (SU p128-129)
<( - Crohn 's disease (FA15 p362) (FA16 p360) (SU p128-129)
---'
- Ulcerative colitis (FA15 p362) (FA16 p360) (SU p128-129)

Warm-Up Review
1. A patient with a urinary tract infection caused by Proteus vulgaris is at risk for what
type of renal stones? (FA15 p544) (FA16 p549) (SU p162)

2~ Which reflexes are evaluated during a newborn examination in order to assess healthy
neurological function? (FA15 p473) (FA16 p476)

3. How can dementia with Lewy bodies be distinguished from other forms of dementia?
(FA15 p487) (FA16 p491)

4. What treatments are available for managing Crohn's disease?


• 5-ASA agents (e.g., mesalamine, sulfasalazine) - usual initial therapy for mild disease
• Azathioprine or mercaptopurine > methotrexate
• Anti-TNF agents (e.g., infliximab, adalimumab)
• Steroids +/- antibiotics for acute exacerbations

[ 400 J
,-
)>
:::0

En d of Ses.si on.i Qu.1z Cl


m
:z
___,
5. What is the classic presentation for diverticulosis? m
Vl
___,
:z
m
--o
)>
:::0
___,
w

6. What is the treatm~nt for diverticulitis?

7. Determine which of the following characteristics are associated with Crohn's disease,
ulcerative colitis, or both.
• Skip lesions
• Loss of haustra
• Transmural inflammation
• Fistulas
• Increased risk ofC:olorectal cancer
• Noncaseating granulomas
• Always involves rectum
• Associated with primary sclerosing cholangitis
• Associated with ankylosing spondylitis

RAP ID- FIRE FACTS


Most common site of colonic diverticula

"String sign" on contrast x-ray

"Lead pipe" appearance of colon on contrast x-ray

[ 40 I ]
(J)

PANCREAS
<(
w
0:::
u
z
<( Phys: Chapter 64
o_
R: Chapter 19
H: Chapters 93, 312, 313
Pancreas and spleen embryology (FA15 p343) (FA16 p339) (SU p167, 118)
Pancreatic enzymes (FA15 p355) (FA16 p352)
Carbohydrate digestion and absorption (FA15 p355) (FA16 p352)
Cystic fibrosis (FA15 p84) (FA16 p72) (SU p113)
Acute pancreatitis (FA15 p376) (FA16 p373) (SU p138)
Chronic pancreatitis (FA15 p376) (FA16 p373) (SU p138)
Pancreatic adenocarcinoma (FA15 p377) (FA16 p373) (SU p138)

Warm -Up Rev1ew


1. What infections are caused by Chlamydia and Chlamydophila species? (FA15 p144)
(FA16 p134) (SU p336)

2. What is the treatment for most Chlamydia and Chlamydophila species infections?
(FA15 p14 4) (FA16p134)

3. Annular pancreas
• Two-thirds of patients remain asymptomatic throughout the ir lives
• Symptom onset can occur at any age of life and depends on the severity of duodenal constriction
• Children present with gastric outlet obstruction (bilious vomiting, feeding intolerance, abdomina l
distention)
• In infants, it is associated with polyhydramnios, Down syndrome, esophageal and duodenal atresia,
imperforate anus, and Meckel diverticulum
• Adults usually present at 20-50 years of age with abdominal pain, postprandial fullness and nausea,
peptic ulceration, pancreatitis, and rarely biliary obstruction

[ 402 J
LJ
)>
z
End 'Of Sessio Quiz n
AJ
rn
)>
4. Which hormones stimulate pancreatic secretion? (J)

5. What enzyme catalyzes the rate"'Jimiting step in carbohydrate digestion?

6. What are the possible etiologies bf acute pancreatitis?

7. What is the typical presentation of a patient with pancreatic insufficiency? What is


the treatment for pancreatic insufficiency?

RAP! o - F! RE FACTS
Painless jaundice

Most common cause of acute pancreatitis

Most common cause of chronic pancreatitis

[ 403 J
(f)
u
(f)
<(
LIVER BASICS
co
0::: Phys: Chapter 70
UJ
R: Chapter 18
>
_J H: Chapter 303

Liver embryology
Liver anatomy (FA15 p349) (FA16 p346) (SU p131)
Hepatocyte functions
Hereditary hyperbilirubinemia (FA15 p372) (FA16 p370) (SU p132)

Warm -Up Rev1ew


1. A young chUd has recurrent lung infections and granulomatous lesions. This is most
likely due to what underlying defect in neutrophils? (FA15 p215) (FA16 p209) (SU p383)

2. Which bacteria form spores? (FA15 p130) (FA16 p114) (5Up318, 321)

3. A 6-month-old child develops flaccid paralysis after ingesting honey. What is


the most likely causal organism? What is the underlying mechanism of disease?
(FA15 p124, 130-131) (FA16 p116, 122) (SUp324, 326)

4. Patient Profile: The Yellow Kid


A 4-day-old ma le neonate is brought to the physician because of jaundice. His mother had adequate
prenatal care. Labor and delivery were uncomplicated. His Apgar scores were 8 and 9 at 1 and 5
minutes, respectively. The neonate and his mother were discharged from the hospital 24 hours after
delivery. Physical examination shows a somewhat lethargic neonate. There is icterus of the skin and
sclerae. Muscle tone is mildly decreased. The liver is not enlarged or tender. The serum total bilirubin
is 22.6 mg/dl (normal 0.1-1.0 mg/dL) and direct bilirubin is 0.2 mg/dl (normal 0.0-0.3 mg/dL). The
hematocrit is 44%. A 14-day trial of phenobarbital does not significantly alter the bilirubin levels. What
is the most likely underlying cause of this disorder?

r 404 1
r-
<
n1

End of Session.Quiz :::0


O:J
)>

From which .embryonicgerm layer is the liver derived? (./)

n
(./)

6. What tree structures lllake ~P a ported triad? Where is the portal triC)d located
relative to the flow of arterial blood and the flow of bile?

What enzyme is responsible for the conjugation of bilirubin?

8. Which hereditary hyperbilirubinemia matches each of the following ~tatements?


• Mildly decreased UDP-GT
• Completelyabsent UDP-GT
• Grossly black liver
• Responds to phenobarbital
• Treatment includes plasmapheresis and photothera py
• Asymptomatic unless under physical stress (e.g., alcohol, infection)

RAPID-FIRE FACTS
Severe hyperbilirubinemia in a neonate

Mild, benign hyperbilirubinemia

[ 40S ]
U)
U)
0
:r:
ALCOHOLIC LNER DISEASE AID CIRRHOSIS
cr:
cr: R: Chapter 18
u
0
H: Chapters 307, 308
:z:
<(
Alcoholic liver disease (FA15 p369) (FA16 p367)
LLJ
U) Cirrhosis and portal hypertension (FA15 p368) (FA16 p365) (SU p136)
<(
LLJ
U)

0
cr:
LLJ
>
Warm -U p Rev1ew
_J

u 1. After undergoing marrow transplantation, a patient develops dermatitis, enteritis,


_J
0 and hepatitis. What is the most likely cause? (FA15 p217) (FA16 p211) (SU p3B5)
:r:
0
u
_J
<(

2. What are eight examples of encapsulated bacteria? What test can be used to detect
encapsulated bacteria? (FA15 p122) (FA16 p112) (SU p318)

3. Serum-ascites albumin gradient (SAAG)


SAAG = [albumin] serum - [albumin]ascites
• SAAG ;::: 1.1 in portal hype rtension
• SAAG < 1.1 in cancer, nephrotic syndrome, tuberculosis, pancreatitis, biliary disease, connective
tissue disease

! 4011 l
)>
r-
n
End of Session Qu1z 0
:r:
0
r-
4. What are the signs of portal hypertension? n

<
rn
;o
CJ
Ul
rn
)>
Ul
rn
)>
z
0
n
;o
;o
:r:
0
5. An alcoholic presents to the ER with an upper Gl bleed. His transaminC)ses are l/)

elevated. Which transaminase (ALT or AST) do you expect to be higher, and by how Ul

much? · ·

6. What cocktail of medications is commonly taken by patients suffering from severe


cirrhosis?

RAP! o ~F! RE FACTS


Triglyceride accumulation in hepatocytes

Eosinophilic inclusions in the cytoplasm of hepatocytes

Cancer closely linked to cirrhosis

r 4071
>-
l.'J
0_ j LIVER PATHOLOGY
0
:r: R: Chapter 18
~
CL Phys: Chapter 70
cr: H: Chapters 92, 301, 302, 309
LLJ
>
_j
Liver studies (FA15 p368) (FA16 p366)
Nutmeg liver (SU p135)
Budd-Chiari syndrome (FA15 p370) (FA16 p368)
Reye syndrome (FA15 p369) (FA16 p366)
Wilson disease (FA15 p373) (FA16 p371) (SU p134)
Hemochromatosis (FA15 p373) (FA16 p371) (SU p134)
a,-antitrypsin deficiency (FA15 p371) (FA16 p369)
Hepatic adenoma (FA15 p370) (FA16 p368)
Hepatic angiosarcoma (FA15 p370) (FA16 p368)
Hepatocellular carcinoma (FA15 p370) (FA16 p368) (SU p135)

Warm-U p Review
1. To which lymph nodes does each of the following regions drain? (FA15 p19B) (FA16 p190)
• Upper rectum
• Lower rectum
• External anus
• Testes

2. What is the function of MacConkey's agar? (FA15 p121) (FA16 p111)

3. Which portion of the eye is opacified in a cataract? (FA15 p4BO) (FA16 p483)

4. Hepatic adenoma
• Most often in femal es 20-44 (OCP years)
• Risk factors: OCP use, anabolic steroids, glycogen storage disease types I and Ill
• Symptoms: right upper quadrant pain, but usually there are no symptoms because it is often an
incidental finding on imaging
• Malignant transformation in 10% of patients
• Treatment: discontinue the OCP, serial imaging and AFP, +/- resection (esp. if > Scm)

5. Hepatic angiosarcoma
• Malignant endothelial neoplasm in liver
• Risk factors: vinyl chloride, arsenic

r 40R 1
.--
<
m
End of Sess1on Qu iz :::0
-o
~
6. What is seen in Budd-Chiarisyndrome? What conditions are associated with Budd~ ::r:::
0
Chiari syndrome? .--
0
Cl
-<

7. What is the mechanism by which aspirin can cause Reye syndrome?

8. A young man·presents with ataxia and trE!mors. He has brown pigmentation in a ring
around the periphery .of his cornea. What treatment should he receive?

What is the underlying problem in Wilson disease? What are the characteristics of
Wilson disease? What is the treatment for Wilson disease?

,r----.. 10. What is the classic triad of symptoms in hemochromatosis? What lab tests are used
to diagnose hemochromatosis? What is the treatment for hemochromatosis?

11. What are.the risk factors for the development of hepatocellular carcinoma?

RAP/ O-F! RE FACTS


Hepatomegaly, abdomina l pain, ascites

Green/yellow cornea l deposits

~ Low serum ceruloplasmin

Cirrhosis, diabetes and hyperpigmentation

r 409 1
HEPATITIS
R: Chapter 18
H: Chapters 304-306

Viral hepatitis (FA15 p166-167) (FA16 p156-157) (SU p133-134)


Autoimmune hepatitis

Warm -Up Review


1. Which three bacteria are obligate intracellular bacteria? (FA15 p122) (FA16 p112)
(SUp335-336)

2. A 20-year-old dancer has decreased plantar flexion plus decreased sensation over her
posterior proximal lower leg, the lateral margin of her foot, and the plantar surface of
her foot. Damage to which nerve is the most likely cause of these findings?

3. · What substances serve as chemotactic agents for leukocytes? (FA15 p227) (FA16 p221)

4. Hepatitis A
• Transmission: - - - - -- -
route, poor sanitation
• Risk factor: international travel
• Labs:
Hep A lgM antibodies during illness
Hep A lgG after resolution or vaccination
• Treatment: supportive
• Vaccine available

5. Hepatitis E
• Transmission: fecal/ora l route, contaminated water
• More likely to cause fulminant hepatic failure in - -- - - - - -- - - -- - - -
• Labs: PCR, Hep E lgM antibodies
• Treatm ent: supportive

f 410 l
6. Hepatitis D
• Delta virus, defective pathogen
• Infects only in presence of - - - - - --
• Transmission: blood, sexual contact
• Highest mortality
• Treatment: pegylated IFN-a

7. Hepatitis C
• - - - -- wi ll remain chronically infected
• Increased risk of hepatocellular carcinoma (US every - - - - -- for surveillance)
• Extrahepatic manifestations: membranoproliferative glomerulonephriti s, essential mixed
cryoglobulinemia, lymphoma , thyroiditis, porphyria cutanea tarda, lichen planus, diabetes mellitus
• Transmission: blood, sexual contact (rare)
• Diagnosis: Hep C antibody, RNA
• Treatment:
Led ipas vir -sofosbuvir
Ombitasvir-paritaprevir-ritonavir + dasabuvir

8. Hepatitis B characteristics
• Transmission: perinatall y, sexual contact (most common), blood
• Chronic infection
< 5% of patients who acquire virus as adult
90% of patients who acquire virus _ _ _ __ _ _ _ _
• Extrahepatic manifestations: polyarteritis nodosa, nephropathy, aplastic anemia
• Increased risk of hepatocellular carcinoma (US every 6 months, +/ - a-fetoprotein)

9. Hepatitis 8 serology
Marker Abbrev. Significance
Hep B surface antigen HBsAg
Hep B surface antibody HBsAb Recovery from active infection or immunization
Hep B core antibody HBcAb History of infection (lgM early, lgG late)
Hep 8 envelope antigen HBeAg Active vira l replication, high transmissibility
Hep B envelope antibody HBeAb Low transmissibility
Hep B DNA HBV DNA Active viral replication; treatment indicated when high

r 4 1: 1
<f) 10. Hepatitis B clinical scenarios
f-

~
0... HBsAg HBsAb HBcAb HBeAg HBeAb
HBV
L1.J DNA
:r
Acute infection

Acute infection
(window period)
Past infection
(recovered)

Vaccination

Chronic infection
(immune tolerance)
Chronic infection
+ (lgG)
(immune clearance)
Chronic infection
+ (lgG) Low
(inactive carrier)

11. What is the treatment for hepatitis B?


• Tenofovir, entecavir, telbivudine, lamivudine, adefovir
• Pregnant women:
If Hep B DNA is elevated, then consid er antiviral (lam ivudine)
• Give baby and Hep B immune globulin within 12 hours of birth

12. Autoimmune hepatitis


• Type 1: ANA, anti -smooth muscle antibody
• Type 2: anti-liver-kidney microsoma l antibody, anti-liver cytosol antibody

r 417 ·1
En d .of Sess1on Qui z
13. ldentifythe hepatitis B status of each ofthe following patients based on their
hepatitis B serologic markers.
HepBsAg HepBsAb .·. . HepBcAb Status
Negative Positive Positive
Negative Negative Positive
Positive Negative Positive lgM
Positive Negative Positive lgG
Negative Positive Negative

14. Which hepatitisviruses aretransmittedthrough the fecal/oral route?

15. Which antibodies can.be used to help make the diagnosis ofautoimmune hepatitis?

r 4u 1
1--

~ BILIARY TRACT
1--
~ R: Chapter 18
<t H: Chapters 303, 377

Biliary anatomy (FA15 p350) (FA16 p347) (SU p136)


Bile (FA15 p356) (FA16 p353) (SU p132)
Enterohepatic circulation (FA15 p357) (FA16 p353) (SU p132)
Hyperbilirubinemia
Primary sclerosing cholangitis (FA15 p374) (FA16 p371) (SU p157)
Primary biliary cirrhosis (FA15 p374) (FA16 p371) (SU p157)
Secondary biliary cirrhosis (FA15 p374) (FA16 p371)
Gallbladder pathology terminology (SU p136)
Cholelithiasis (FA15 p375) (FA16 p372) (SU p135)
Cholangitis (FA15 p375) (FA16 p372) (SU p135)
Cholecystitis (FA15 p375) (FA16 p372)

Warm -Up Rev1ew


1. Which immune deficiency is associated with each of the following findings?
(FA15 p206-207) (FA16 p208 -209)
• Anaphylaxis on exposure to blood products with lgA
• .Coarse facial featu res, abscesses, eczema
• Thrombocytopenia, purpura, infections, eczema
• Delayed separation of the umbilicus
• Neurologic defects, partial albinism, recurrent infections

2. Bilateral Bell's palsy should raise suspicion for what condition? (FA15 p140) (FA16p130)

[ 4141
3. What is the overall differential diagnosis for hyperbilirubinemia?
)>
Unconjugated hyperbilirubinemias Conjugated hyperbilirubinemias ;;o
-<
• Increased bilirubin production • Impaired transport ---1
;;o
)>
n
---1

• Biliary epithelial damage


• Impaired bilirubin uptake and storage

• Decreased UDP-GT activity • Intrahepatic biliary obstruction

• Extrahepatic biliary obstruction

4. Gallbladder pathology terminology


• Cholelithiasis: gallstones (hint: lith = stone)
• Cholecystitis: inf1ammation/infection of the gallbladder (-itis = inf1ammation)
• Cholangitis: inf1ammation/infection of the biliary tree
• Choledocholithiasis: gallstones in the bile ducts

r 415 l
1-
u
<(
a::
1-
>-
En d of Sess1on Qui
a::
<r:
_j
5. What is the fate of bilirubin after it is conjugated and secreted into the Gl tract?
rn

6. What are some of the intrahepatic and extrahepatic causes of biliary obstruction?

7. What is the difference between primary biliary cirrhosis and primary sclerosing
cholangitis?

8. What are some of the risk factors for the development of cholesterol gallstones?

r 4161
DESP/TE.THE MOVE

PULMONARY TOWARD MOR E CLINICAL


QUES TIONS ON BOARD
EXAMS, PULMONARY
1 Anatomy and Physiology
PHYSIOLOGY AN D
OXYG ENATION REMAIN
2 Lung Volumes and Pulmonary
VERY TESTABL E COMMON
Circulation
CLINICAL PROBL EMS LI KE
COPO, ASTHMA AND
3 Oxygen and Hemoglob in
LUN G CAN CER AREALSO
POPULAR SUBJECTS FOR
4 Oxygenation
TEST ITEMS BE SURETO
5 Ventilation an d Pe rfus1 on PAY SPECIAL ATTENTIO
TO THE /EXTREME
6 Extreme Env iron men ts ENVI RON MENTS "
SECTION ON HIGH
7 PE, DVT and Pneumothorax ALTITUDE, DEEP-SEA AND
AEROSPACE MEDICINE,
8 COPD and Asthma THES E ARE FR EQUENTLY~
-------
TESTED TOPICS TH AT
9 Pulmonary Medications OFTEN DO NOT RECEIVE
TH E ATTENTION THEY
10 Restr1ct1ve Lung D1 sease DESERVE

11 Lung Cancer

12 Lung In fe ct ion s
ANATOMY AND PHYSIOLOGY
COA: Chapter 1
Phys: Chapters 37, 39
Development (SU p94)
Tracheoesophageal fistula (FA15 p342) (FA16 p338) (SU p94)
Trachea (FA15 p600) (FA16 p608) (SU p94) )>
z
Diaphragm (FA15 p601) (FA16 p609) (SU p95) (COA p306)
Respiratory muscles ~
0
Airway zones (FA15 p600) (FA16 p608) :s
Lung anatomy (FA15 p601) (FA16 p609) -<
)>
Pneumocytes (FA15 p600) (FA16 p607) (SU p96) z
0
""0
I
-<
{J)

War -Up Rev.1ew 0


,-
0
1. To which lymph nodes would cancer in each of the following areas metastasize?(FA15p198) Cl
-<
(PA16 p190)
· · Stomach
• Duodenum, jejunum
• Sigrrmid colon
• Scrotum

2. . What is the classic triad of symptoms in multiple sclerosis? (FA15 p488) (FA16 p492)

3. WhichJysosomal.storage disease matches each of the following statements?


(FA15 p111) (FA16 p100)
• Accumulation of GM2 ganglioside
• Associated with renal fail.ure

r 4 i9 1
of ion Qu 1z
4. What structures traverse the diaphragm,.and at what vertebral levels do they pass through?

>-
CJ
0_ j
0
VI
>-
:r::
o_
0
z What histological change takes place in the trachea of a smoker?
<t
>-
:?:
0
~
z
<t

6~ What cell type proliferates during lung damage?

7. . What amniotic fluid measurement is indicative of fetal lung maturity?

8. A young woman has infertility, recurrent URis and dextrocardia. Which of her proteins is
defective?

RA PI D- FIRE FACTS
Chronic sinusitis, infertility and situs inversus

r 420:
LUNG VOLUMES AND PULMONARY CIRCULATION
Phys: Chapter 37
COA: Chapter 1
H:Chapter250
Lung volumes (FA15 p602) (FA16 p610) (SU p95) (Phys p469)
Physiologic dead space (FA15 p602) (FA16 p610) (SU p98-99)
c'
Chest wall dynamics (FA15 p603) (FA16 p611) (SU p95) z
Pulmonary circulation (FA15 p605) (FA16 p613) (COA p116) Gl
Pulmonary vascular resistance (PVR) (FA15 p606) (FA16 p614) <
0
Pulmonary hypertension (FA15 p614) (FA16 p622) (SU p103) (H p2076) c'
Treatment for pulmonary hypertension (H p2078) :s:
rn
- Bosentan (FA15 p620) (FA16 p627) (GG p110) (H p2079) (Jl
)>
z
0
u
c
Warm - p .Rev1ew '
:s:
0
z
1. Which nerve is most at risk of injury with each of the following types of injury? )>
:::0
(FA15 p418) (FA16 p419) -<
n
• Fracture of the .shaft of the humerus :::0
n
• Fracture of the surgical neck of the humerus c
• Supracondylar humerus fracture '~
0
z

2. A high school athlete falls on his arm during practice. In the ER, a radiograph ~hows
a midshaft break ofthe humerus. Which nerve and artery have the highest risk of
damage? What muscular actions would be affected? (FA15 p418) (FA16 p419)

3. What is the female homologue to each of the following male structures?


(FA15 p568) (FA16 p572)
• Scrotum
• Prostate gland
• Glans penis

[ 421 J
4. label the following diagram of lung volumes.

H
z A E F
0
VI
~ <Ll
_j

:::J E
u :::l
0:::
u
>-
0:::
<(
z
0
~
Cl
c
:::l
....I
I
2
_j

:::J
o_
0
Jc G
:z:
<(
lf)
w
:?:
:::J
Jo Time
_j

0
> A
l'J
:z:
:::J
_j B.

c.
D.

E.

F.

G.

H.

5. Chest wall dynamics

6 TLC Chest wall


Lung-chest wall
system

-
_J
........
Q)
4
E
:::l
g I FRC
2 I
I
I
I
Lung -·--
·~
0
-20 -10 0 10 20 30 40
Static pressure (cmH 20)

r 422 1
6. Primary pulmonary hypertension
• AKA idiopathic pu lmonary arterial hypertension
• Associated with abnormalities in BMPR2: Bone Morphogenetic Protein Receptor type II
BMPR2 protein prevents proliferation of vascular smooth muscle
Mutation leads to excessive vascu lar smooth muscle proliferation, leading to reduced vessel
radius, increased resistance, and increased pulmonary arteria l pressure r-
c
• Associated with HIV and Kaposi sarcoma (HHV-8) z
Cl
• More common in women , average age is 36 <
0
r-
c
:s:
m
Ul
)>
z
0

End/o f \Sess1 on CQ urz """0


c
'
:s:
7. The following lung volumes are obtained from an elderly smoker: FRC5.0l, IRV 1.51, 0
z
IC2.0L, VC 3.5L What is his total lung capacity? )>
;;o
-<
n
;;o
n
c
'~
8. If a lung collapses, what happens to the intrathoracic volume? 0
z

9. What genemutation can cause primary ptdmonary hypertension?

10, What are some of the secondary causes of pulmonary hypertension?

11. What are some of the treatment options available for pulmonary hypertension?

12. What is.the mechanism of action of bosentan?

r 423 1
OXYGEN AND HEMOGLOBIN
Phys: Chapter 40
Hemoglobin (FA15 p603) (FA16 p611)
Oxygen-hemoglobin dissociation curve (FA15 p604) (FA16 p611) (Phys p498)
Methemoglobin (FA15 p604) (FA16 p612)
:z: Carboxyhemoglobin (FA15 p604) (FA16 p604)
(I]
0
__)
~
0
:2:
Ll.J
I
Warm - Up Rev1ew
0
:z: 1. The presence of which antibodies is specific for SLE? (FA15 p213) (FA16 p207)
<{
:z:
Ll.J
~
>-
X
0

2. What is the mechanism of type IV hypersensitivity? (FA15 p212) (FA16 p205)

3. Which structures arise from the paramesonephric ducts? (FA15 p567) (FA16 p571) (SU p196)

4. Oxygen-hemoglobin dissociation curve

r 4241
5. What substances are known for causing methemoglobinemia?

0
X
-<
Cl
m
z
)>
z
0
::r:
m
$
0
Cl
,--
End of Session Ou1z 0
OJ
z
6. Which form ofhemoglobinAhas a high affinity for oi<ygeri!Which has a lowatfmity
for oxygen?

What substances tend to shift the oxygen-hemoglobin dissociation curve to the


right?Does this favor oxygen loading or unloading?

What is the treatment for methemoglobinemia?

r 425 1
OXYGENATION
Phys: Chapters 39, 40
Oxygenation (FA15 p605) (FA16 p613)
Alveolar gas equation (FA15 p606) (FA16 p6?4) (SU p?O?)
Oxygen deprivation (FA?S p606) (FA16 p614)
z
0
~
z
w
\.C)
Warm -Up Rev1ew
>-
X
0
1. Which immunodeficiency matches each of the following statements? (FA15 p237-238)
(FA16 p208-209)
• .Neutrophils fail to respond to chemotactic stimuli
• Adenosine deaminase deficiency
• Failure of endodermal development
• • Defective tyrosine kinase gene
• Associated with high levels of lgE and eosinophils

2. Which bacterial structure fits each of the following descriptions? (FA15 pl18) (FA16 plOB)
• Mediates adherence of bacteria to the surface of a cell
• Protects against phagocytosis
• Space between the inner and outer cellular membranes in gram-negative bacteria
• Forms attachment between two bacteria during transfer of DNA material (i.e., conjugation)
• Genetic material within bacteria that contains genes for antibiotic resistance

3. Mechanisms of oxygen deprivation


• Inadequate Pa0 2 (hypoxemia)
High altitude
Hypoventilation
Pulmonary fibrosis
Right-to-left cardiac shunt (high A-a gradient)
- V/Q mismatch
• Inadequate 0 2 delivery to organs/tissues (hypoxia)
Hypoxemia
- Anemia
Carbon monoxide poisoning
Low cardiac output
• Inadequate perfusion (ischemia)
Obstruction of arteria l flow (MI or stroke)
Reduced venous drainage

r 426 1
Sess1 on Quiz
4. What is the normal value for the A-a gradient? What might an elevated A-a gradient '
indicate?
0
X
-<
Gl
m
:z:
::;
0
5. A42-year-old woman with fibroids is chronically tired. What is the most likely c:Jiagnosis, and :z:
what changes have occurred in the oxygen content and saturation?

A patient is shown to have hypoxia,·and chest x-ray ~veals an enlarged heart. What is the
most likely cause of hypoxia?

[ 427 J
VENTILATION AND PERFUSION
Phys: Chapters 39, 40

V/Q mismatch (FA15 p606) (FA16 p614) (SU p98-100)


C0 2 transport (FA15 p607) (FA16 p615)
Response to exercise (FA15 p607) (FA16 p615)
:z:
0
U)
=:J
LL
t:r
w
()__

0
Warm -Up Rev1ew
:z:
<I: 1. Which neurodegenerative disease is associated with each of the following
:z:
0 statements?
~
__J
• Senile plaques, neurofibrillary tangles
f- • Presents at birth as "floppy bab/'
:z:
w • Lewy bodies
>

2. What is the mechanism of action of infliximab? (FA15 p446) (FA16 p44B)

3. Apatient with multiple sclerosis is determined to have internuclear ophthalmoplegia.


What physical exam findings would be expected in this patient? (FA15 p486) (FA16 p490)

4. C02 transport

r 4281
End of Sess1on 1z
What is theV/Q ratio at the apex of the lung? At the base of the lung? [)uring airway
obstruction? Duringblood flow obstruction? ·

<
rn
z
-I
r-
~
0
z
)>
z
0
-o
rn
;o
"c
(J)

0
z

6. How is C02transpo.rted from the tissues to the lungs?

7. How do C02 levels in circulation change during exercise?

[ 429 J
EXTREME ENVIRONMENTS
Phys: Chapters 43, 44
High altitude
- Response to high altitude (FA15 p607) (FA16 p615)
- Effects of low oxygen pressure on the body (Phys p527)
(/")
f- Aviation
z Effects of acceleratory forces on the body in aviation and space physiology (Phys p531)
w
~ Space
z - Artificial climate in the sealed spacecraft (Phys p533)
0
0::: - Weightlessness in space (Phys p533)
>
z
Effect of high partial pressures of individual gases on the body (Phys p535)
w Scuba diving (Phys p539)
w Special physiologic problems in submarines (Phys p540)
:2: Hyperbaric oxygen therapy (Phys p540)
w
0:::
f-
xw

Warm-Up Rev1ew
1. Which exotoxin has each of the following characteristics? (FA15 p124-125) (FA16 p116-117)
• Inhibits ACh release --+ flaccid paralysis
• Stimulates adenylate cyclase--+ Cl- and water into gut-+ diarrhea
• Causes scarlet fever
• Inactivates EF-2 --+ pseudomembranous pharyngitis
• Blocks the release of the inhibitory neurotransmitter glycine

2. Which CD surface marker fits each of the following statements? (FA15 p209) (FA16 p202) ~-
• Displayed only by helper T cells
• Displayed mainly by cytotoxic T cells (but also NK cells)
• Found on all T cells
• Used to identify B cells

3. What is the underlying cause of achalasia? How is the diagnosis made? (FA15 p357)
{FA16 p354)

4. What are the clinical derangements that come about in a patient with acute mountain sickness?

r 430 1
5. What physiological changes occur during chronic mountain sickness?
• Increased------ - -- - - - - - - - - - - - - - - - -
• Increased blood viscosity _ _ _ _ _ __ _ _ _ _ _ _ _ _ _ _ __
• Elevated pulmonary artery pressure
• heart enlargement
• Peripheral artery pressure falls rTl
X
---i
• ;;o
rTl
:s
rTl
rTl
z
<
6. At what positive G-force does a visual"black-out" occur? ;;o
0
z
:s
rTl
z
---i
(J)

7. What G-force is achieved during a spacecraft liftoff?

8. How is the body affected at zero gravity?


• Decreased _ _ __ __ _ _ _ _ _ _ _ _ __ __ _ _ _ __ _
• Decreased RBC mass
• Decreased muscle strength/work capacity
• Decreased maximum _ _ _ _ _ _ _ _ _ _ _ _ _ _ _ _ _ __ __
• Loss of ca lcium and phosphate resulting in _ _ _ __ _ _ _ _ _ _ _ __

9. What is nitrogen narcosis?


• Nitrogen dissolves into neural membrane, wh ich causes reduced neuronal excita bility
• Diver becomes jovial/careless
• Lossof _ _ _ _ _ __ _ _ _ _ _ _ _ _ _ _ __ _ _ __ _ _ __

10. What are the symptoms of decompression sickness?

r 411 1
End of Sessto n. Qu1z
11. How does the body compensate for hypoxia at high altitude?

l/)
f-
z
w
~
z
0
0::
>
~ 12. How much do the hemoglobin and hematocrit (H&H) change in a person that has
~ acclimatized to a hypoxic environment for weeks?
w
0:::
f-
xLLJ

13. At what positive G-force does visual 11black out" occur? Why does this occur?

14. What physiologically is taking place in decompression sickness?

15. For what conditions is hyperbaric oxygen therapy particularly helpful?

16. By what physiological mechanism does acute mountain sickness cause acute cerebral
edema and acute pulmonary edema?

r 4i? l
PE, DVT AND PNEUMOTHORAX
H: Chapter 262
R: Chapter 15

Virchow's triad (FA15 p608) (FA16 p616) (SU p103)


Deep venous thrombosis (FA15 p608) (FA16 p616) (SU p103)
Pulmonary embolism (FA15 p608-609) (FA16 p617) (SU p103) --o
rn
Embolus types (FA15 p608-609) (FA16 p617) (SU p103)
0
Pneumothorax (FA15 p615) (FA16 p623) <
---1
)>
z
0

\N..arm -Up 8evi evv --o


z
rn
c
1. How does the mechanism of type II hypersensitivity differ from the mechanism of s
0
type Ill hypersensitivity? (FA15 p211)(FA16 p204-205! ---1
I
0
:;u
)>
X

2. After the loss of his job; a 35-year-old man has diarrhea and hematochezia. An
intestinal biopsy shows transmural inflammation. What is the mostlikelydiagnosis?

3. Which area of the colon is most susceptible to ischemic damage? (FA15 p224) (FA16 p21BJ

4. Virchow's triad
• Stasis: severely ill, para lyzed, long trips, cast
• Hypercoagulability: sickle ce ll , polycythemia , estrogen excess, cancer, pregnancy, OCPs, smoking
• Endothelial damage: vessel trauma, fracture, foreign body, bacterial infection

5. Deep venous thrombosis (DVT)


• Symptoms: foot/ankle (unilatera l) edema, ± pain, ± Homan's sign (pain with ankle dorsif1exion), ±
palpable cord
• Diagnosis: compression ultrasound (high probability); D-dimer then US (low probability)
• Prevent: heparin, enoxaparin, compression stockings or sequentia l compression device (SCDs)
• Treatment: heparin until warfari n is therape utic

[ 433 J
6. Pulmonary embolism (PE)
• Symptoms: pleuritic chest pain, shortness of breath, cough, hemoptysis (rare), fever, tachypnea,
tachycardia, altered mental status/ confusion
• Studies: elevated D-dimer, ± DVT on lower extremity ultrasound, usually normal chest x-ray, large
A-a gradient on arterial blood gas,± ECG changes (S1Q3T3 - deepS in lead I, large Q and inverted
>< Tin lead Ill), CT scan with contrast is most often used, V/Q scan, pulmonary angiogram
<(
0:::: • "Saddle embolus" = death
0
I
f--
• Treatment: if massive PE then consider thrombolysis, but usually just heparin / warfarin
0
:2:
=>
L1.J
z
0...
0
z
End of Sessron Quiz
<(
f-- 7. A patient suffers a stroke after incurring multiple long bone fractures in a skiing accident.
>
0 What caused the infarct?
w'
0...

8. A patient with a recent tibia fracture and no history of COPD or asthma is shown to have
hypoxia. Chest x-ray is normal. What is the cause of the hypoxia, and what disease process
does it mimic?

9. A patient presents to the ER after a motor vehicle collision with shortness of breath. A chest
x-ray reveals mediastinal deviation to the right. What is the most appropriate next step?

RAP! o-Ft RE FACTS


Elevated D-dimer

Hypercoagulability, endothelial damage, stasis of blood

r 434 1
COPD AID ASTHMA
H: Chapters 254, 260
R: Chapter 15
Obstructive vs. restrictive pulmonary disease (FA15 p610-611) (FA16 p618-619) (SU p97-98)
Asthma (FA15 p610) (FA16 p618) (SU p104)
Chronic bronchitis (FA15 p610) (FA16 p618) (SU p104) n
Emphysema (FA15 p610) (FA16 p618) (SU p105) 0
-o
Panacinar emphysema (FA15 p610) (FA16 p618) (SU p105) 0
)>
Bronchiectasis (FA15 p610) (FA16 p618) (SU p105)
z
0
)>
\.n
-I
I
Warm-Up Rev1ew :s:
)>

1. Whi.ch enzyme is inhibited by proton pump inhibitors {PPis)? (FA15p37BJ

2. What are the C's of Huntington disease? (FA16 p462) (SU p47)

[ 435 l
End of Sess 1on Qui z
3. What are the hallmark characteristics of obstructive lung disease on pulmonary function
tests?
<(
:;:;;:
I
f-
(/)
<(
4. What are the hallmark characteristics of restrictive lung disease on pulmonary function
0 tests?
z
<(
0
[)__
0
u

5. What is the Reid index?

6. How does the emphysema caused by smoking differ from the emphysema caused by «t-
antitrypsin deficiency?

RAP! o - F! RE FACTS
Blue bloater

Pink puffer

Curschmann spirals

f 436 l
PULMONARY MEDICATIONS
GG: Chapter 36

COPD and asthma medications (FA15 p621) (FA16 p628)


Expectorants (FA15 p620) (FA16 p627)
H, blockers (FA15 p620) (FA16 p627)
-u
Cough suppressants (FA15 p620) (FA16 p627) c
r-
Nasal decongestants (FA15 p620) (FA16 p627) :s
0
z
)>
:::0
-<
Warm~Up :s
m
CJ
1. What disease is associated with each of the following antibodies? (FA15 p213) (FA16 p207) n
• Anti-mitochondrial ~
0
• Anti-TSH receptor z
(/)

• Anti-centromere
• Anti-basement membrane
• Anti~neutrophiJ cytoplasmic

2. What is the female homologue to each ofthe following male structures? (FA15 p56B)
(FA16 p572)
• Corpus spongiosum
• Bulbourethral glands
• Ventral shaft of the penis

3. Antihistamines
Drug Use
Cyproheptadine Appetite stimulant
Promethazine Nausea, vomiting
Hydroxyzine Sedation, itching
Meclizine Vertigo

r 4371
End of Session Qui z
4. What asthma medication fits each of the following statements?
• Preventative inhaled treatment of choice for mild persistent asthma
• Inhaled treatment of choice for acute exacerbations
(f)
z • Narrow therapeutic index, drug of last resort
0
• Blocks conversion of arachidonic acid to leukotriene
~ • Inhibits mast cell release of mediators, used for prophylaxis only
u
0
w • Inhaled treatment that blocks muscarinic receptors
~
• Inhaled long . . acting ~ 2 agonist
>-
cr:
<(
• Blocks leukotriene receptors
z
0
~
_j

=>
o_
5. • Which medications, if taken long term, can result in rebound nasal congestion?

r 4"18 1
RESTRIOIVE LUNG DISEASE
H: Chapters 267, 268
R: Chapter 15
Restrictive lung diseases (FA15 p611) (FA16 p619)
Interstitial lung disease (FA15 p611) (FA16 p619) (SU p107-108)
;:>:J
Acute respiratory distress syndrome (ARDS) (FA15 p613) (FA16 p621) (SU p102) m
(J)
Neonatal respiratory distress syndrome (FA15 p613) (FA16 p607) (SU p102) -j
Pneumoconiosis (FA15 p612) (FA16 p620) (SU p108) ;:>:J
()
Coal workers' pneumoconiosis (FA15 p612) (FA16 p620) (SU p108) -j
- Asbestosis (FA15 p612) (FA16 p620) (SU p108) <
m
- Silicosis (FA15 p612) (FA16 p620) (SU p108)
- Berylliosis (FA15 p612) (FA16 p620) (SU p108) 'c
z:
Sarcoidosis (FA15 p434) (FA16 p435) (SU p108) Gl
Idiopathic pulmonary fibrosis (FA15 p611) (FA16 p619) (SU p108) 0
Langerhans cell histiocytosis (eosinophilic granuloma) (FA15 p611) (FA16 p619) (SU p107) (J)
m
Goodpasture syndrome (FA15 p541, 611) (FA16 p546, 619) (SU p107) )>
(J)
Granulomatosis with polyangiitis (FA15 p302, 611) (FA16 p296, 619) rn
Hypersensitivity pneumonitis (FA15 p611) (FA16 p619) (SU p108)

Warm-Up Review
Which portion of the brachial plexusjs injured in Erb-Duchenne palsy? What are the
symptoms? (FA15 p419/ (FA16 p42D)

2. What is the 111ost common salivary gland tumor? (FA15 p357) (fA16 p3S3) (SUp119)

3. What are the different sinuses that can become infected and cause sinusitis?

4. ARDS
Shock, infection, toxic gas inhalation, aspiration. high [0 2], pancreatitis, heroin OD, sepsis, trauma.
uremia , and amniotic fluid embolism

Inflammatory cells/ mediators and oxygen free radicals

Damage to endothelial or alveolar epithelial (type I) cells

Diffuse alveolar damage (DAD) and hyaline membrane disease (HMO)

1439 l
5. Coal workers' pneumoconiosis (CWP)
• Anthracosis (mild): black pigment in lung
• Simple CWP: small fibrotic lung nodules
• Complicated CWP: progressive massive fibrosis

w
(/)
<(
w
(/) 6. Sarcoidosis
0
\.:)
"A GRUELING Disease"
z • ACE enzyme increase
::l
-'
w • Granulomas
>
r- • Rheumatoid arthritis (RA)
u
0::: • Uveitis (eye)
r-
t/)
w • Erythema nodosum (tibial)
0:::
• Lymphadenopathy (bilateral , hilar)
• Idiopathic
• Noncaseating granuloma
• Gammaglobulinemia
• Vitamin D increase

7. Idiopathic pulmonary fibrosis (cryptogenic fibrosing alveolitis)


• Risk factors: age, smoking, genetics
• Genetic mutations: telomerase, mucin MUCSB
• Patchy interstitial fibrosis
• Median survival after diagnosis: 3 years
• Honeycomb lung on CT
• Usual interstitial pneumonia

8. What medications may cause restrictive lung disease?


• Bleomycin
Pneumonitis with infiltrates
• Busulfan
- Acute lung injury
Chronic interstitial fibrosis
- Alveolar hemorrhage
• Amiodarone
Pulmonary fibrosis (1%)
• Methotrexate
Hypersensitivity-like lung reaction

r 440:
9. A preterm infant has difficulty breathing. An x-ray reveals diffuse air space and
interstitial opacities, with air bronchograms. What is the diagnosis, and what could
haye prevented this condition?
;;o
rn
(f)
--1
;;o
n
--1
<
rn
10. What are common causes of ARDS?
'c
z
GJ
0
(f)
rn
)>
(./)
rn

A lung biopsy from a plumber shows elongated structures with clubbed ends in
tissue. What is the most likely diagnosis? What type of cancer is this patient most at
risk for acquiring?

RAP! O-Ff RE FA CTS


Bilateral hilar adenopathy, uveitis

Anti-glomerular basement membrane antibodies

Honeycomb lung on x-ray

"Tenn is racket" -shaped cytoplasmic organelles

Iron-containing nodules in the alveolar septum

r 44l 1
LUNG CANCER
H: Chapters 89, 257, 258
R: Chapter 15
Lung cancer (FA15 p619) (FA16 p626) (SU p114-115)
Adenocarcinoma (FA15 p619) (FA16 p626) (SU p114)
cr: Large cell carcinoma (FA15 p619) (FA16 p626) (SU p114)
w
u Squamous cell carcinoma (FA15 p619) (FA16 p626) (SU p114)
z
<( Small cell carcinoma (FA15 p619) (FA16 p626) (SU p114)
u
Carcinoid tumor (FA15 p619) (FA16 p626) (SU p114)
<..:>
z Pancoast tumor (FA15 p617) (FA16 p625)
::::J
_j Mesothelioma (FA15 p617) (FA16 p624) (SU p108)

Warm-Up Review
1. Which neurodegenerative disease is associated with each of the following
statements?
• Dementia often associated with frequent falls and/ or syncope
• Atrophy of caudate nucleus ~ chorea
• Depigmentation of substantia nigra
• Both upper and lower motor neuron signs (spasticity and weakness)
• Dementia +visual hallucinations
• Dementia + personality changes and/or aphasia

2. Which CD surface marker fits each of the following statements? (FA15 p206) (FA16 p202)
• Found on all NK cells and binds the constant region of lgG
• Inhibits complement C9 binding
• Endotoxin receptor found on macrophages

r 442 1
En Sess1on QUIZ
3. A patient develops bronchogenic lung cancer but has never smoked. He is a coal
miner. Exposure to what substances has put him at risk for developing lung
,-
c
z
Cl
n
)>

4. What complications can arise from lung cancer? z


n
m
;;D

5. ·what are the four ·mOSt corrilllonloeations of lung cancerllletastasis.

RAP/ O-F! RE FACTS


Lung cancer associated with SIADH

Tumor associated with Horner syndrome

Lung cancer associated with hypercalcemia

Lung cancer associated with Cushing syndrom e

Lung cancer associated with weakness

Substances associated with lung cancer

r 443 1
LUNG INFECTIONS
H: Chapters 89, 257, 258
R: Chapter 15

Pneumonia (FA15 p616) (FA16 p624) (SU p109)


Vl
z Lung abscess (FA15 p617) (FA16 p624)
0 Pleural effusions (FA15 p615) (FA16 p623)
f-- Sleep apnea (FA15 p613) (FA16 p621) (SU p101)
u
l.J..J
LL
z
\.?
z
::::>
_j
Wa rm-Up Rev1ew
1. Which nerve is most at risk of injury with each of the following types of injury? (FA 15 p418)
(FA16 p419)
•• Fracture of the medial epicondyle
• ··Anterior shoulder dislocation
• Injury to the carpal tunnel

2. Which lysosomal storage disease matches each of the following statements?


(FA15 p111) (FA16 p100)
• Accumulation of dermatan sulfate
• Deficiency in hexosaminidase

3. - What is the difference between direct bilirubin and indirect bilirubin? (FA15 p357)
(FA16p353)

[ 444]
4. Which infectious agent fits each of the following descriptions?
• Common cause of pneumonia in immunocompromised patients
• Most common cause of atypical/walking pneumonia
• Common causative agent for pneumonia in alcoholics
• Can cause an interstitial pneumonia in bird handlers
'cz
• Often the cause of pneumonia in a patient with a history of exposure to bats and bat droppings Cl

• Often the cause of pneumonia in a patient who has recently vis ited South California, New Mexico, ,z
m
or West Texas n
---j

• Pneumonia associated with "currant jelly" sputum 0


z
Ul
• Q fever

• Associated with pneumonia acquired from air conditioners


• Most common cause of pneumonia in chi ldren 1 year old or younger
• Most common cause of pneumonia in the neonate (birth-28 days)
• Most common cause of pneumonia in children and young adults (including co llege students, military
recruits, and prison inmates)

• Most common cause of viral pneumonia


• Causes wool -sorter's disease (a life-threatening pneumonia)
• Common pneumonia in ventilator patients and those with cystic fibrosis
• Pontiac fever

5. Pleural effusion
• Transudate • Exudate
CHF Cancer
Cirrhosis Pneumonia, infections, TB
Nephrotic syndrome Uremia
Fluid overload Connective tissue disease

r 445 1
End of Sess1 on Ou1 z
6. A 30-year-old comatose man on ventilatory support in the ICU develops an infection and
dies. Autopsy reveals a pus-filled cavity in his right lung. What is the likely etiology?
(f)
z
0
f-
u
w
LL
z
(_:J
z
:::::>
_j

7. What infectious agent is the cause of pneumonia based on each of the following
characteristics?
• Most common cause of viral pneumonia
• "Currant jelly" sputum
• Most common cause of atypical walking pneumonia
• Gram (+) coccLin neonate
• Gram (-) rods in neonate

8. A 52-year-old obese man complains of daytime fatigue and his wife complains that
the patient snores very loudly. What is the best diagnostic test, and what is the
treatment?

! 4461
E M EDICINE IN
~
MEDICINE IN SOCIETY SOC! CHARTER
COVERS HIGH~YIE L D
1 Research Study Des igns TOPICS THAT MANY
D
2 Diagnostic Test Evaluation PREFER TO IGNORE
BI OSTATISTICS A
3 Interpretation of Study Data EPIDEMIOLOGY, RESEARCH
DY DES! PUBLIC
4 Study Val1d1ty HEALTH, MEDICAL
ECONOMICS, AND
5 Statis tical D1 str ib ut1o n and Error ICS ALTHOUGH
MEDICAL SCHOOLS
6 81 ostat1 st1 cs OFTEN GUILTY
UNDERTEACHING IN
7 Disease Prevention and Publ ic Health ESE AREAS, TH ERE IS
NO REASON WHY YOU
8 Healthcare System DN'T GO IN
UR EXAM FEELING
9 Ethics
CONFIDENT AND WELL
PREPARED.
RESEARCH STUDY DESIGNS
H: Chapter 3
Research study designs (FA15 p48) (FA16 p32) (SU p17)
Drug development (FA15 p48) (FA16 p32)
Meta-analysis (SU p17)

Wa -Up ew
What are the arterial.branches off of the common hepatic artery? (FA15 p347) (FA16 :::0
m
Vl
m
)>
:::0
n
:r:
Vl
---i
c
0
-<
0
m
Vl
Cl
z
{J)

2. Which cell type secretes the histamine that stimulates the histamine receptor on parietal
cells, thereby increasing gastric acid production? (FA15 p378) (FA16 p374)

r 4491
End of Sess1on Quiz
3. A physician is looking for risk factors for pancreatitis. He interviews 100 hospitalized
patients with pancreatitis and 100 hospitalized patients without pancreatitis. What
type of study is this?

(J)
z
l.9
(J)
LlJ 4. A group of smokers and a group of nonsmokers are followed over 20 years. Every
0
>-
two years, it is determined who develops cancer and who does not. What type of
0
::::> study is this?
1-
(J)

::r:
u
0::
<t
w
(J)
w
0::

5. Can a cross-sectional study help determine.the incidence or prevalence of a disease?

r 4c;n '
DIAGNOSTIC TEST EVALUATION
H: Chapter 3
Sensitivity and specificity (FA15 p49) (FA16 p33) (SU p16-17)
Positive predictive value (PPV) and negative predictive value (NPV) (FA15 p49) (FA16 p33) (SU p16-17)

Warm --u p Rev1ew


1. What dinical features suggest.congenital pyloric stenosis?(fA15 p342) (FA16 p339)
g
)>
Cl
z
0
U1
---i
n
---i
m
U1
---j
m
<
)>
2. What are the features of Plummer-Vinson syndrome? (FA1Sp35B) (FA16 p354) r-
c
~
0
z

3. What is Dressler syndrome? (FA15 p295) (FA16 p290)

I 451 l
4. What are the sensitivity, specificity, positive predictive value and negative predictive value using
antibodies to X to detect disease X?

Autoimmune Disease X
Present Absent

X 800 200
....0
>-
"'0
0
z ..c
0 ....!:':
f-
<(
::J
< 100 1400
_j
<(
>
w
f-
(/)
w
f-
u
f-
(/)
0
z
lC)
<(
0

r 4::;1 1
End of Session Quiz
.5. What are the equations for sensitivity, specificity, positive predic
negative predictive value?

CJ
)>
Cl
z
0
Ul
---j

n
---j
rn
Vl
---j
rn
<
)>
,-
c
~
0
z

6. What happens to the po~;ithre predictive value of a diagnostic test when the disease
prevalence increases?

7. A certain screening test has a 1% false-negative rate. What is the sensitivity of .the
test?

r 1s3 1
INTERPRETATION OF STUDY DATA
H: Chapter 3
Incidence and prevalence (FA15 p49) (FA16 p33) (SU p16)
Relative risk and odds ratio (FA15 p50) (FA16 p34) (SU p17-18)
Attributable risk and absolute risk reduction (FA15 p50) (FA16 p34)
Number needed to treat (FA15 p50) (FA16 p34)
Precision (reliability) and accuracy (validity) (FA15 p51) (FA16 p35) (SU p19)

Up
~ 1. What histological findings of the distal duodenum are most likely seen ln a patient w ith
0
celiacsprue? (FA15 (FA16p358)
>-
0
:=J
f-
Vl
LL
0
z
0

~
w
a:
g;: 2. What arterial branches come off of the celiac trunk?(FA15 p347) {FA16 p343)
w
f-
z

3. A woman with a history of hypertension presents with sudden onset of sharp, tearing chest
pain that radiates to the back. What .is the most likely diagnosis? What would be seen on a
c;hestx-ray? (FA15 p293) (FA16p287)

4. Attributable risk (AR)


• AR = incidence of disease (exposed group) - incidence of disease (unexposed group)
• Example:
In a population of sexually active people, 30% have HPV infection.
In a population of people who are not sexually active, only 5% have HPV infection.
- The attributable risk of sexual activity to HPV is 25%.

r 4541
5. Number needed to treat (NNT)
• NNT = 1/ absolute risk reduction
• Number of patients you would need to treat in order to save/affect one life
• Important number to help determine if a drug should be used or is cost effective
• Example:
100 acute stroke patients are given tPA, and 40 of them have complete or near complete
recovery.
- 400 acute stroke patients are not given tPA, and 80 of them have complete or near complete
recovery.
- Therefore, you would need to treat five patients with acute stroke to see the beneficial effect of
tPA in one patient. :z
---i
rn
:::0
"D
:::0
rn

E
0
6. What are the absolute risk reduction and number needed to treat in the following example? z
In a study where 100 patients received medication Z to prevent the development of diabetes, and 200 0
,.,
patients did not receive the medication, 10 patients in the experimental group developed diabetes, (J)
--l
and 40 patients in the control group developed diabetes. c
0
-<
0
)>

End Sess1on Qu1z


7. What are the equations for odds ratio, relative risk, attributable risk, and number
needed to treat?

(continued)

r 4r:;r:; 1
End of Sess1on Qui z (contmu ed)
8. The small town of Mickey City (pop. 8,000) is immediately adjacent to factories
where asbestos products are produced. During the past year, the prevalence of
mesothelioma has been 16 cases. In the town of Donaldville {pop. 6,000), 15 miles
upwind of (and theoretically safely distant from) Mickey City, there was a prevalence
of three cases during the same year. What is the relative risk of mesothelioma for the
population of Mickey City?

~
<(
0
>-
0
::::J
f-
(J)
LL
0
z
0
f-
<(
f-
L1.J
a::
o_ 9. A new glucose test arrives and you decide to see how well it works. There is
a::
L1.J
f-
a standard substance provided that has 90 mg/dl of glucose. Your repeated
z measurements of the substance reveal the following values: 54, 56, 55, 54, 53, 56, 55,
and 54. What can you say about the precision and accuracy of your new glucose test?

10. The prevalence of disease X in population A is two times the prevalence of disease
X in population B. The incidence is the same in poputations A and B. What can be
assumed aboutthe disease duration in population A versus population B?
STUDY VALIDITY
Bias (FA15 p52) (FA16 p36) (SU p18)
Confounding (FA15 p52) (FA16 p36)
Random error
Avoiding bias, confounding, and random error (FA15 p52) (FA16 p36)

Warrn-Up Re.v1ew
1. What are the risk factors fotesophagealcancer? (F/\15 p359) (FA16
Ul
---1
c
0
-<
<
)>
,-
0
---1
-<

2. What is the most common salivary glalldtumor? What is the second most con11'Tlon
salivary gland tumor? What is the most common location for a salivary gland tumor?
(FA15 p357) (FA16 p353)

3. Which calcium channel blockers can be to treat both hypertension and


tachyarrhythmias? (FA15 p304) (FA16 p29B) (SU p7B)

r 4"7 1
4. Examples of bias:
• Selection bias: The referral centers for a trial of a new anticancer drug have more patients with late- .~
stage disease than early-stage disease, so more patients with late-stage disease are referred for the
trial than early-stage disease.
• Berkson's bias: Women with symptoms of DVT were more likely to be hospitalized if they were
taking birth control pills because this was a known risk factor. When the relationship between birth
control pills and DVTs was studied using hospitalized patients, results were skewed due to the risk
factor history contributing to the hospital admission in the first place.
• Self-selection bias: In a study describing the association of a new drug with liver damage, a higher
rate of liver disease was found among the patients that self-referred to the study than those who
>- were solicited for the study through pharmacy records.
1-
0 • Healthy worker effect: The harmful effect of a toxin exposure among active employees is
_j
<( underestimated because the control group is filled with people from the general population.
>
>-
0
• Recall bias: Parents of autism patients may have a more detailed recall of events and illnesses in
::J their child's first two years of life compared to parents of healthy controls.
1-
U)
• Lead-time bias: While test PSA-xyz may detect prostate cancer before it is detected by a traditional
PSA, early detection using PSA-xyz does not increase cancer survival compared to traditional PSA.
• Late-look bias: Sending a survey out to people diagnosed with a fatal illness five years after
diagnosis will preferentially sample those with a low-grade disease (or few comorbidities).
• Procedure bias: The positive benefit of a new drug during a study simply may have been due to
the fact that study participants were required to attend clinic monthly where they received extra
disease education and counseling compared to the controls.
• Interviewer bias: An interviewer asked more questions about perceived symptom improvement to
participants in the experimental group than in the control group.
• Loss to follow up: More people were lost to follow-up from the experimenta l group of a lipid
treatment study than the control group, because of the side effects of the experimental treatment.
• Hawthorne effect: When studying the effects of infection control education on physicians, the
investigator notes that both the experimental and control groups improve their hand hygiene.
5. Example of confounding:
A very strong association was found between asbestos miners and lung cancer. It was then discovered
that the miners also smoked more cigarettes than the control population.

End SSIOn QUIZ


6. What is bias?
U1
-i
c
0
-<
:;;
r
0
-i
-<

7. What are some ways investigators can reduce bias in a study design?

8. What is confounding?

r 459 1
STATISTICAL DISTRIBUTION AND ERROR
Statistical d istribution (FA15 p53) (FA16 p37) (SU p19)
Null hypothesis (FA15 p53) (FA16 p37) (SU p19)
Error (FA15 p54) (FA16 p38) (SU p19-20)
Power (SU p19-20)

Warm -Up Review


1. What rectal diseases are found proximal to the pectinate line, and what diseases are found
~
0 distal to the pectinate line? (FA15 p349) (FA16 p345)
~
~
w
0
:z
<(

:z
0
f-
=:J
OJ
0:::
f-
(/) 2. Which antihypertensive drug has each of the following side effects? (FA15 p304-305)
0
_j
(FA16 p298-299)
<(
u • First-dose orthostatic hypotension
f-
(/)
• Hypertrichosis
!;: • Cyanide toxicity
f-
(/) • Dry mouth, sedationand severe rebound HTN

3. During a high school football game, a young athlete collapses and dies immediately. Which
type of cardiac disease should be suspected? (FA15 p296) (FA16 p291)

r 4 ?JI l
nd of Session Ou1z
StUdy X shows that vitamin C can prevent coronavirus infections, but 10 o~r studies show
no benefrt. What type of error is found in study X?

_,
Vl
_,
)>

5. Study Y shows that aspirin administration during an Mloffers no improvement in _,


Vl

patient morbidity ormortality. What type of error is present? n


)>

'0
_,
Vl
:;o
OJ
c_,
0
:z:
)>
z
0
m
:;o
6. What are the differences between mean, median, and mode? ;;o
0
;;o

7. What does it mean to say that a statistical distribution has a positive skew?

r 461 1
BIOSTATISTICS
Standard deviation (SO) and standard error of the mean (SEM) (FA15 p53) (FA16 p37)
Confidence interval (CI) (FA15 p54) (FA16 p38)
t-test (FA15 p55) (FA16 p39)
ANOVA (FA15 p55) (FA16 p39)
Chi-square('/!) (FA15 p55) (FA16 p39)
Correlation coefficient (FA15 p55) (FA16 p39)

-U p Revi ew
(J)
u What histological findings are seen in alcoholic hepatitis? (FA15 p369) (FA16 p367)
f-
(J)

~
(J)
0
co

Why is cough one of the common side effects of ACE inhibitors but notangiotensin
receptor blockers.(ARBs)? (FA15p55S)(FA16 p559/

3. What is the equation for determining the confidence interval?

r 462 1
4. In a study of USMLE scores at a particular medical school, the mean score from a random sample of
100 students is 230 and the standard deviation is 20. Calculate the 95% confidence interval.

5. What is the 99% confidence interval in this same study?

OJ
0

s
Vl

Vl
-1
n
Vl
6. In a study of USMLE scores at a particular medical school, the mean score from a random sample of
16 students is 230 and the standard deviation is 20. Calculate the 95% confidence interval.

End of Ses ion u1z


7. In a n~rmai .Gaussian.curve,what percentage.of the sample population fall~ one
standard deviation, two standard deviations, and three standard deviations from the
mean?

What is the equation for determining the confidence interval?

9. In a study of diabetic patients on drug Meca-leca-liver, the average patient HgbA1c


after three months is 8.0 and the standard deviation is 0.5. Knowing that the sample
size is 10,000, calculate the 99% confidence interval.

r 463 1
DISEASE PREVENTION AND PUBLIC HEALTH
Disease prevention (FA15 p55) (FA16 p44)
Vaccines
Reportable diseases
Injury prevention

Warm-Wp Rev iew


I
':::J 1. Which hormones stimulate pancreatic secretion? (FA15 p353) (FA16 p350)
<(
w
:r::
u
_j
CD
=>
(}__

0
z:
<(
z
0
f-
z 2. What are the equations for odds ratio; relative risk, attributable risk, and number needed to
w
>
LU treat? (FA15 p50) (FA16 p34)
0:::
(}__

LU
(/)
<(
LU
(/)

r 4641
3. Child routine vaccines
Vaccine Disease prevented

Rotavirus Rotavirus

DTaP, DT Diphtheria, tetanus, +/- pertussis

Tdap, Td Tetanus, diphtheria , +/ - pertussis

Haemophilus influenzae type b infections


Hib (meningitis, epiglottitis, osteomyelitis, pneumonia , 0
V>
bacteremia, septic arthritis) m
)>
V>
Streptococcus pneumoniae infections (bacteremia, m
Pneumococcal conjugate (PCV13) v
pneumonia, sinusitis, otitis media, meningitis) ;o
m
<
m
Streptococcus pneumoniae infections (bacteremia, z
Pneumococcal polysaccharide (PPSV23) --;
pneumonia, sinusitis, otitis media, meningitis)
0
Neisseria meningitidis infections z
)>
Meningococcus (bacteria l meningitis, bacteremia, pn eumonia, septic z
arthritis) 0
v
c
co
IPV Poliomyelitis r-
n
I
m
Influenza (inactivated) Seasonal influenza )>
~
:r:
Influenza (live) Seasonal influenza

MMR Measles, mumps, rubella

Varicel la Varice lla (chicken pox)

Hepatitis A Hepatitis A

Hepatitis B Hepatitis B, cirrhosis, liver cancer

HPV HPV, cervical cancer, other genital / ana l cancers

DTaP, Diphtheria-Tetanus-acellular Pertussis; DT, Diphtheria-Tetanus; Tdap, Tetanus-diphtheria-acellular pertussis;


Td, Tetanus-diphtheria; Hib, Haemophilus influenzae type b; /PV, Inactivated poliovirus; MMR, Measles-Mumps-Rubella; HPV,
Human papillomavirus

f 465 l
4. Adult routine vaccines
• Tdap or Td
• PPSV23
• +/- PCV13
• Influenza (annually)
• HPV (male and female)
• MMR
• Varicella
• Zoster
:r:
~
<(
l.J..J
:r: 5. Reportable diseases
u
_j
CD
• Sexually transmitted infections: H IV, chlamydia, gonorrhea, syphilis
::J
0... • Hepatitis: ------------- - - - - ---------------
0
z • Vaccine-preventable infections: mumps, measles, rubella, diphtheria, and pertussis
<(
z • Diarrheal illnesses:
0 ------------------------
f- • Tubercul osis
z
l.J..J
>
l.J..J
0:::
0...
l.J..J
(./) 6. Car seat and seat belt guidelines
<(
l.J..J
(./) • < 2 years:
0 Rear-facing car seat unless the child outgrows the weight or height limit for the car seat
Preferred placement: middle of the back seat
• 2-4 years:
Forward-facing car seat w ith a harness for as long as possible
• 4-12 years:
Continue with forward-facing car seat until the weight or height limit is reached for the car seat, then
switch to a belt-positioning booster seat until the vehicle seat belt fits properly (typically when they
have reached 4 feet 9 inches in height and are between 8-12 years of age)
• Older children:
- Always use lap and shoulder seat belts
- All children younger than 13 should be in the rear seats of the vehicle

r 4661
7. What is the.current recommendation for the meningococcal vaccine?

8. Which adult patient populations should receive the pneumococcal polysaccharide


(PPSV23) vaccine?

0
Vl
rn
)>
Vl
rn
-o
:::0
rn
<
rn
z:
9. What diarrheal illnesses are reportable diseases? --1
0
z:
)>
z:
0
-o
c:
OJ
r-
n
I
rn
)>
~
I
10. Which vaccine could prevent each of the following diseases?
• Seasonal influenza
• Hepatitis A
• Hepatitis B ·
• Hepatitis C

• Anal cancer
• Genital warts
• {\/eisseria meningitidis sepsis
• Epiglottitis

• Whooping cough
• Septic arthritiS
• Clostridium tetani
• Clostridium.botulinum

• Cervical .cancer
• Shingles
• Rubeola
• Lockjaw

f 4671
HEALTHCARE SYSTEM

Warm -Up Revi ew


1. What are the equations for sensitivity, specificity, positive predictive value, and negative
predictive value? (FA15 p49) {FA16 p33)

:2:
Ll.J
I-
(/)
>-
(/)
Ll.J
0:::
<C
u
:r:
~
<C
Ll.J
:r:

"Medicine is a science. Healthcare is a business. Healing is an art.,,

2. What was the practice of medicine like before insurance?


• Physicians and hospitals could charge patients whatever they wanted .
• Patients trusted physicians and hospitals to charge them a fair rate for their services.
• Patients sometimes incurred excessive unforeseen medical expenses.
• Patients were often unable to compensate healthcare providers for the services rendered .
• Physicians would often write off debts that patients were unable to pay.

3. Healthcare terms you need to know- Table #1


Premium The amount the insured person has to pay the insurance company (usually paid
monthly)
Co-pay The amount the insured person pays at the time of service (e.g., $30 for a clinic
visit or $15 for a particular drug)
Deductible The amount an insured person must pay "out-of-pocket" before the hea lth
insurance begins to pay ~-

r 4f:.R 1
4. Health insurance - private pay insurance - third-party payer
Party Financial Duty Risk Reward
Patient - Monthly premiums Paying more to (1) Financial benefit if the cost of
-Co-pays insurance than what medical care exceeds what is paid
- Deductible is received in medical to insurance
care (2) Peace of mind allows the
individual to focus on other things
Insurance Healthcare expenses of Paying more for a Financial benefit if the patient pays
patient beyond what the patient's medical care more than the cost of medical care
patient is required to pay than what is received
financially from the
patient :r:
m
)>
r::::j
:r:
n
)>
;;o
5. What problems came about when health insurance was introduced? m
Vl
• In short, everyone tries to get the biggest slice of the financial pie now in the hands of the health -<
Vl
--j
insurance companies. m
s:
• Health insurance companies leverage their volume of subscribers to drive down the reimbursement
to medical care providers. Providers still provide care because they agreed to do so.
• Healthcare providers and hospitals counter by increasing the cost of medical care. Insurers pay
because they agreed to do so.
• Patients demand more and more care in order to "get what they have already paid for."
• Insurance companies end up receiving not enough funds from patients to pay for the rising costs of
healthcare.

6. Healthcare terms you need to know - Table #2

The group of healthcare providers that has agreed to a reduced payment in


Network
order to have access to a larger number of patients

High-deductible plan that covers the patient in the event of excessive


Major medical
medical expenses

7. Health maintenance organization (HMO)


• PCP is the gatekeeper to more specialized care.
• In order for medical expenses to be covered, the provider has to be (fin-network."

8. Preferred provider organization (PPO)


• There is no gatekeeper to the specialist.
• Patient can see whomever they want; however, the cost is higher for "out-of-network" medical care.

i 4(.9 l
9. What problems came about when HMOs and PPOs were introduced?
• The HMO and PPO drive down the payments to physicians and hospitals.
• As physicians receive less compensation, the quality of visits is decreased in order to generate a
higher quantity of visits.
• The patient pays more and more for less and less medical care.
• In short, the third-party payers drive down the cost of care. The physicians counter by decreasing
the quality of care. The patient loses.
• Physician s who are unsatisfied with the new arrangement exit the system and establish new
systems.

2:
w
I-
V)
>-
l/)
10. New systems established by physicians
w
a::
<(
Minor emergency clinic • Heavily utilized by patients without any third -party coverage
u • Usually offers no primary or secondary prevention
::c
!::::J Cash-only clinic • Physician ref uses to take any third-party payment, essentially
<(
LU
::c returning back to the old patient-physician system
• Patient may or may not carry additional "major medical-on ly"
coverage
Sliding scale clinic A patient pays a nominal fee based on income. This is usually run by a
charity with a volunteer physician in order to minimize costs.
Concierge medicine Physician charges an annual fee (e.g., $1200/ year) for a specifical ly
designated set of services (e.g., unlimited visits, or four visits a year).
Boutique medicine Physician offers unique services not covered by insurance but desired
by affluent patients (usually cosmetic in nature) in order to increase
clinic revenue (e.g., Botox injections).
Group physician networks In an effort to exclude third -party systems and regain control of their
- Clinic systems own payments, physician groups join clinics together, build their own
- Physician hospitals hospitals, and run their own surgery centers.
- Day surgery centers

11. Healthcare terms you need to know - Table #3


Formulary The medications for which insurance companies will pay. Generics are
preferred.
Utilization management Eva luation of the appropriateness, necessity, and efficiency of
healthcare services.
Resource-based A scale that determines what a physician should be paid for a
Relative value scale procedure (CPT code) or service in a specific region of the country,
based on physician work, regional practice expense, and regional
malpractice expense.
E/M Evaluation and management codes
ICD-10 International classification of diseases
CPT Current procedure terminology

r 4701
12. Health plan - managed care organizations (MCO) - healthcare organizations
• Organization that attempts to maximize the quality of care and minimize the cost of care.
• Uses techniques such as encouraging patients and physicians to choose less costly care, controlling
inpatient admissions and lengths of stay, and emphasizing preventive medicine.
• Accomplished through a designated utilization management person or department.

13. What about those without the financial resources for healthcare?
• Medicare I
m
)>
• Medicaid
r::::j
• Children's Health Insurance Program (CHIP) I
n
)>
• City-, county-, or state-funded health networks ;;o
m
• Federally-funded teaching hospitals {J)
-<
• Federally-funded VA hospitals __,
{J)

m
• Affordable Care Act s

14. How are physicians compensated?


Fee-for-service Payment is provided for a • Surgical procedure
specified service • Clinic visit (e.g., 99211-99215)
• Inpatient visit (e.g., 99231-99233)
Capitation Fixed payment for a period of • ER shift/minor emergency shift
time or number of patients • Concierge practice
• HMO
Salary Specified amount regardless of • Universities
work performed • Hospital administration
• Base salary
Pay for Payment is increased if a Increased pay by the healthcare
performance physician meets pre-established organization for meeting certain criteria
targets such as preventive medicine targets
(vaccines, colonoscopy, HgbA1c)
Nonpayment for No payment is made for a A funding entity may agree NOT to pay a
performance complication that could have physician for complications such as leaving
been avoided a catheter in place, nosocomial infections,
and surgical site infections

f -471 l
15. Current problems
• Young invincibles: Th ese are people (usually under 30) that tend not to have many health problems and
tend not to purchase insurance. In order for a third-party payer to afford to care for an ai ling patient at a
low-premium cost to that patient, healthy patients must be present in the system to bear the burden of
the additional expense.
• Freedom of choice: Patients that need others to carry the burden of their healthcare expenses are at odds
w ith those that want to be free of carrying that burden.
• Excessive complexity: All this complexity leaves patients at a loss to understand where they need to go for
healthcare. Additionally, this complexity requires hiring individuals to manage it which drives up the cost of
healthcare.
2: • Self-interest: Everyone still wants the biggest possible piece of the pie. Patients want the highest-costing
LLJ
f-
ifl
care. Physicians want the highest reimbursements. Insurance providers and drug companies want the
>-
if) largest profits.
LLJ
0::::
<(
• Physician shortage: Many physicians now see better opportunities elsewhere and are leaving full-time
u medicine. The population continues to increase, and the percentage of geriatrics in the population is also
I
t:::J increasing.
<(
LLJ
I
• Insufficient government funds: It may be very difficult for the government to pay for Medicare for the
baby boom generation.

16. Developing solutions


• Government involvement in the healthcare system
• Patients taking responsibility for their own health via the Internet, OTC medication options, and
complementary and alternative medicin e
• Patients seeking U.S.-train ed physicians practicing in developing co untries where the cost of ca re is
low
• New generic medications that help lower the cost of healthcare
• Cooperative healthcare teams that help reduce the need for costly hospitalization
• More mid-l evel practitioners to meet the rising demand
• Increased reliance on protocols established by evidence-based medicine rather than physician
individualization of care
• Reducin g the privacy of medical records among medical professionals and insurance companies for
increased efficiency in the coo rdination of care
• EHRs/ EMRs
• Quality measures

r 4721
End Ses Quiz
17. What are the differences between premium, co-pay, and deductibie?

I
m
18. Which usually offers patients a greater variety of physicians to choose from: HMO or )>
~
PPO? I
n
)>
:;;o
m
(/)
-<
Ul
---1
m
s
~-···~

19. What is the difference between being paid by capitation and fee-for-service?

20. What must a hospital submit to a third-party payer in order to receive payment for
services?

21. Which department in a hospital oversees the maximization of thequality of


while minimizing the cost of care?

r 4T~ 1
ETHICS
H: Chapter 9
Ethical principles (FA15 p56) (FA16 p39) (SU p21)
Confidentiality (FA15 p57) (FA16 p41)
Informed consent (FA15 p56) (FA16 p40) (SU p21)
Decision -making capacity (FA15 p57) (FA16 p40) (SU p21)
Surrogate decision-makers (FA15 p57) (FA16 p41) (SU p22)
Directives (FA15 p57) (FA16 p41) (SU p21-22)
Consent for minors (FA15 p56) (FA16 p40) (SU p22)
Ethical situations (FA15 p58) (FA16 p42)
Quality improvement (FA16 p45-46)
VJ Malpractice (SU p22)
u
:r
f-
w

Warm -U p Rev1ew
1. What is the classic presentation of a patient with giant cell arteritis? (FA15 p302) (FA16 p296)

2. In a patient with an elevated alkaline phosphatase, elevations in which other serum


lab markers would point toward a diagnosis of liver disease? (FA15 p368) (FA16 p366)

3. Failure Modes and Effects Analysis (FMEA)


A proactive. systematic analysis of how a process might fail and the impact of these failures in order to
identify the parts of the process that are most in need of change.

I) List steps in the process


2) Identify failure modes ("What could go wrong?")
3) Identify failure causes ("Why would the failure happen?")
4) Identify failure effects ("What are the consequences of each failure?")

f 4741
nd of Session Quiz
4. A 68-year;.old man is diagnosed with incurable glioblastoma. His family asks you, the
doctor, not to tell the patient. How do you handle the situation?

5. Apatienthad made itd~ar.to you on previous visits that, ifsomething should happen
rn
to him, he would not want surgery. The patient now comesto you with a condition ---1
I
that requires surgery, but he is incapable of providing for you his present feelings on ()
Ul
the matter because of his condition. His friend relays to you that the man has told him
nu.111er?us ti.J1leSthat he does not want surg~ry, His wife shows up later a~d tells you
to do everything you can to save her husJ:>and'slife, including surgery. What should
you do?

6. How should yol.rhandle a situation where you smell alcohol on a physician's breath
is
while that physician practicing medicine?

7. A 36-year-old woman.has a first-degree relative who had breast cancer at age 40.
You recommend a mammogram, but she refuses to have one because she says it is
too painful. What do you do?

8. You find yourself attracted to.your 24-year-oldpatient. How do you handle the
situation?

9. A72-year-old m::m in the hospital with an Ml refuses to take his aspirin on the
grounds that it makes him feel «tunny." What do you do?

r 475 1
SO ..MANY PATHWAYS'
BlOCH EM ENZYMES AND
SUBSTRATES ... · •· so
LITTLE TIME FROM DNA
1 DNA BaSICS
TRAN SCRIPTION AND
PROTEIN SYNTHESIS TO
2 DNA Repl1cat1on, Mutation and Repair
ENERGY METABOLISM
AND VITAMINS, A GREAT
3 RNA
DEAL OF BIOCHEMISTRY
IS TESTED ON TH
4 Protem Syn th esis
BOARDS, MUCH OF
WHICH YOU PROBABLY
5 Inheritance
TRIED FORGET
6 GenetiC Lab Techn iques THE MOMENT YOU
CO/VIPL YOUR .
7 Glyco lys is BJOCHEM CLASS. B.UT
DON'T WORRY~wf'LL
8 Gluconeogenesis HELP YOU FIGURE OU T
WHAT'S WORTH KNOWING
9 Glycogen FO R YOUR EXAM

10 En ergy Metabolism

11 HMP Shunt an d Othe r Sugars

12 Sta rvation and Malnutr1t1on

13 Lipids

14 Ammo Acids and Nitrogen

15 Ammo Acid Disorders

16 Mmerals

17 Fat-Soluble Vitam 1ns and Ant iOXIdants

18 Water-Soluble V1tamms
DNA BASICS
Phys: Chapter 3
Chromatin (FA15 p62) (FA16 p48) (SU p281)
Nucleotides (FA15 p63) (FA16 p49) (SU p281)
Pyrimidine synthesis (FA15 p63) (FA16 p49) (SU p279-280)
Orotic aciduria (FA15 p392) (FA16 p390) (SU p280)
Purine synthesis (FA15 p63) (FA16 p49) (SU p279)
Purine salvage (FA15 p64) (FA16 p50) (SU p279)
Lesch-Nyhan syndrome (FA15 p64) (FA16 p50) (SU p279)
Adenosine deaminase deficiency (FA15 p64) (FA16 p50) (SU p280)

rm-Up Revi
1. Which antibiotic has each of the following side effects? (FA15 plB0-190) (FA16 p170-1BO)
(SU p337-342)
• Teeth discoloration
• Tendonitis
• Red ma.n syndrome 0
z
• Gray baby syndrome )>
OJ
• Cartilage damage in children )>
(J)
• Nephrotoxicity, ototoxicity n
(J)
• Pseudomembranous colitis

2 . . Which structures form the Hesselbach triangle? (FA1S p352} (FA16p349)

3. Carbamoyl phosphate synthetase (CPS-1 and CPS-2)

CPS-1 CPS-2

Pathway

Location

Source of nitrogen

r 4791
4. Pyrimidine and pu nne
. synthesis

ATP ~02 +glutamine


ADP~.®
Carbam I Ribose-5-P
Aspartate~l oy phosphate jPRPP synthetase

®J------------
Orotic acid
£>-®
~
PRPP
UMP

~
J IMP

CTP
/
UDP
~d @.,;2>-@
UDP AMP
/
CD'\.
GMP

(/)
u
(/)
<(
!
dUMP

cl
a:J
<(
@~ethyleneTHF----...
z
0 dTMP DHF _____...-THF
@
w<D
A
B.

c.
D.

E.

F.
G.

H.

J.
K.
L. - - -

M.
5. Purine salvage
GMP IMP AMP

Guanosine Inosine © Adenosine


® ®
Guanine Hypoxanthine

~_A
Xanthine

Uric acid 0
z
)>
o:J
)>
Vl
A. ----------------------------------------------- ()
Vl

8.

C. - -- -- -- -- - - -- -- -- -- -- -- - - - - - - -- -- -- - - -- -- -- -- --

[ 481 J
En d of Session Qu 1
6. What is the rate-limiting enzyme in purine synthesis? In pyrimidine synthesis?

7. What are the sources of carbon in the synthesis of purines? In pyrimidine synthesis?

8. Which medication inhibits each of the following enzymes?


• Ribonucleotide reductase
(/) • Dihydrofolate reductase
u
(/)
• Thymidylate synthase
<(
m • Inosine monophosphate dehydrogenase
<(
z
• PRPP amidotransferase
0

9. What accounts forthe positive charge of histones? What accounts for the negative
charge of DNA?

10. How many adenine residues are found in a molecule of DNA if one strand contains
A=2000, G=SOO, C=1500, and T=1000?

RAP /O- F! RE FACTS


A boy with self-mutilating behavior, intellectual
disability and gout

Orotic acid in the urine+ elevated serum ammonia

Orotic acid in the urine + normal serum ammonia

Megaloblastic anemia that does not improve with


folate and B12

r 4821
DNA REPLICATION, MUTATION AND REPAIR
Phys: Chapter 3
DNA replication (FA15 p65) (FA16 p51) (SU p281-283)
DNA mutations (FA15 p66) (FA16 p52) (SU p283)
DNA repair (FA15 p67) (FA16 p53) (SU p283)

Warm-Up Review
1. Which antibiotic has each of the following mechanisms of action? (FA1Sp180-190)
(FA16p170-18D)
• Inhibits prokaryoticRNA polymerase
• Inhibits prokaryotic topoisomerase
• Inhibits prokaryotic qihydrofolate reductase

2. Macrolides are effective against which organisms? (FA1Sp186)(FA16p176)(SU p339) 0


z
)>
;:u
rn
"U
,-
n
~
0
:z:
:s:
c

3. Apatient has pain in the right upper quadrant that he .can point to with one finger.
~
0
The area is tender to light touch, and pain is worsened when the patient is asked to :z:
)>
raise his arms above his head. What is the most likely diagnosis? :z:
0
;:u
rn
"U
)>
;:u

r 4Ri 1
4. Label the elements on this simplified diagram of prokaryotic DNA replication.

3'

cr:
<(
o_ 5'
w
cr:
0
z
<(
z
0
~
f--- A.
=:J
z:
z B.
0
~
c.
u
_j
o_ D.
LLJ
cr:
<(
E.
z
0
F.

G.
H.
I.

r 4x4 1
End of Sess1on Ou 1z
5. What strand of DNAnucleotides opposes this DNA strand: S'..ATTGCGTA-3'?

6. How does UV radiation damage DNA?

. - ~ .

Which eukal"yotic DNA polymerase performs each ofthe following functions?


• R(';plicates lagging strand
• Synthesizes RNAprirner 0
z
• Repairs DNA )>

• Replicates mitochondrial DNA :;o


rn
"D
• Replicates leading strand DNA ,-
n
~
0
z
:s:
c::
g
0
z
)>
z
0
:;o
rn
"D
)>
:;o

r 4R'1 1
RNA
Phys: Chapter 3

RNA subtypes
Codons (FA15 p67) (FA16 p53) (SU p285)
Regulation of transcription (FA15 p68) (FA16 p54) (SU p284)
Operon control mechanisms (FA15 p66) (FA16 p52) (SU p279)
RNA polymerases (FA15 p68) (FA16 p54) (SU p284-285)
Termination of transcription (SU p285)

Warm -Up Rev1ew


1. Tetracyclines are effective against which organisms? (FA15p185! (FA16 p17S)

<t
z
0:::
2. A 25-year-old.pregnant woman in her third trimester has a normal blood pressure
when standing and sitting. When supine, her blood pressure drops to 90/50 mm Hg.
What is the most likely underlying cause of these findings?

3. What drugs should sulfa-allergic patients avoid? (FA1S p214) (FA16 p254)

4. Regulation of transcription
• Operon: structural genes that are transcribed + promoter region +all regulatory regions
• Transcription factors: must bind to the promoter region (-75 CCAAT box, -25 Hogness/ TATA box,
and -10 Pribnow/ TATAAT box) in order for transcription to take place
• Operator region: binds repressor (stops transcription, see lac operon) or indu ce r (starts
transcription), located between the promoter region and sta rt site
• Response elements: enhancer region and represso r region - increase or decrease the rate of
transcription when bound by protein factors; location may be close to, far from , or within the
promoter region
• Common structural motifs: helix-loop-helix, helix-turn-helix, zinc finger, leucine zipper

r 486 l
5. DNA-binding motifs

.~ Helix-loop-helix

Helix-turn-helix

:::0
z
)>

Zinc finger

Leucine zipper protein

Ftec;ognltion
helix

[ 487]
6. lac operon

CAP-binding RNA polymerase h


site binding site (Promoter) RNA sy nt esis

Excess lactose + Absent glucose ---+ Stimulate lac operon to make r3-galactosidase

lac repressor
Operon OFF both
Excess glucose
Absent lactose
t=BBimllllllllili!l b/c lac repressor bound
and b/c CAP not bound

<(
z
0:::
The prokaryotic cell derives its energy f rom glucose metabolism. In environments of low glucose
concentration, the cell has the capability to make ~-galactosidase , an enzyme that cleaves lactose into
glucose and galactose, thereby making more energy available. Of course, if no lactose is present, then
the costly activity of making ~-galactosidase is useless. The cell uses CAP (catabolite activating protein)
and the lac repressor to determine when to begin manufacturing the costly ~ - galactosidase. CAP
facilitates RNA polymerase binding and promotes ~-galactosidase production but is inhibited by excess
glucose levels. The lac repressor cannot bind to the operator site and inhibit ~-galactosidase
production in the presence of lactose.

7. Termination of prokaryotic RNA transcription


Rho (p) factor of E. coli (an RNA-dependent ATPase)
• Recognition of termination region in DNA (p-independent mechanism)
GC-rich DNA --7 GC same-strand binding forms stem-loop (hairpin) in RNA~ pause in
RNA polymerase ~ subsequent weak RNA bonds (uracil-rich region) ~ separation of RNA
polymerase

i 4881
n of Session Quiz
8. What amino acid is encoded by the most common start codon?

9. What is the difference between an intron and an exon?

~ ~ ··· ·~ . . -
Production of what enzyme is regulated by the lac operon?

AJ
:z
)>

11. What two proteins regulate the lac operon?

12. What two substrate conditions must be met for the lac genes to be transcribed?

[ 489 J
PROTEIN SYNTHESIS
Phys: Chapter 3
tRNA (FA15 p70) (FA16 p56) (SU p285)
Protein translation (FA15 p71) (FA16 p57) (SU p285-2BB)
Post-translational modification (FA15 p71) (FA16 p57)

Warm -Up Review


1. What are the most common causes of meningitis from birth to 6 months of age?
(FA15 p173) (FA16p163) (SU p49)

(/)
(/)

~ 2. What is the mechanism of action of dantrolene? (FA15 p499) (FA16 p503) (SU p417)
f-
z
>-
(/)

z
w
f-
0
0:::
o_

3. What substances inhibit the reuptake of norepinephrine? (FA15 p249) (FA16 p243) (SU p26)

[ 490]
End of Sessio Qu iz
4. What enzyme matches amino acids to tRNA?

5. What antibiotics are.inhibitors of prokaryotic protein synthesis atthe 305 ribosome?

6. Whata~fibiotitsare inhibitorsof prokaryotic protein synthesis atthe 50S ribosomal


subunit? ·
-o
;;u
0
---1
rn
z
Vl
-<
z
---1
:r:
rn
Vl
Vl

r 49, 1
INHERITANCE
R: Chapter 5
Pedigrees (FA15 p82) (FA16 plO) (SU p288)
Anticipation (FA15 pBO) (FA16 p68) (SU p289)
Genetic terminology (FA15 pBO) (FA16 p68) (SU p288-289)
Prader-Willi syndrome (FA15 p81) (FA16 p69) (SU p289)
Angelman syndrome (FA15 p81) (FA16 p69) (SU p289)
Hardy-Weinberg population genetics (FA15 p81) (FA16 p69) (SU p289-290)

Wa rm -Up Rev1 ew
1: WhafprobJem/abnormality is associated with each of the following buzzwords?
• •Boot-shaped heart
• Continuous machine-like murmur
• Tendon xanthomas
• Cafe-au-lait spots
• Tuft of hair on lower back
w
u
z
~
er::
w
::r: 2. What is the classic triad of tuberous sclerosis? (fA15 p83) (FA16 p71)
z

3. Which cell type proliferates during lung damage? (FA15 p600) (FA16 p607)

4. Mitochondrial inheritance defects


• Mitochondrial myopathies (ragged-red muscle fibers seen on biopsy)
• Leber hereditary optic neuropathy
• Leigh syndrome (subacute sclerosing encephalopathy)

5. What is the likelihood that child X will have the genetic mutation?

r 4921
6. The numbers in this diagram indicate the age of disease presentation. What is the name of this
phenomenon?

62

28

7. If the shaded boxes indicate a phenotypic expression of a genetic mutation, what is the name given to
this phenomenon?
:z
:r
m
;:>;J

~
:z
nm

8. Prader-Willi syndrome
• Deletion of proximal portion of chromosome 15q11-q13 from paternal origin
• Presents in infancy: hypotonia, poor feeding, characteristic facial features (almond-shaped eyes,
downward turned mouth)
• Symptoms: hyperphagia, obesity, short stature (partial GH deficiency), intellectual disability, behavior
disorders (tantrums, skin picking, OCD), hypogonadotropic hypogonadism ~ genital hypoplasia,
osteoporosis, delayed menarche
• Diagnosis: confirmed with FISH (fluorescence in situ hybridization)
• Treatment: limit access to food, GH if short stature

9. What is the frequency of the BB phenotype and the Bb phenotype if the frequency of allele B is 70%?

[ 493 J
End of Sess1on Qui z
10. What is the frequency of the Aa genotype and the AA genotype if the frequency of
allele A is 0.95?

11. lf49% ofa particular population is homozygous for a curly hair gene that is dominant
to a straight hairgene, what percentage of the population has curly hair?

LU 12. A male infant is born to a woman that is heterozygous for an x:.linked disease. The
u
z father is normal. What is the probability that the son will be affected?
~
a:
LU
I
z

13. Afemale infant is born to a woman that.is heterozygous for an X-linked disease. The
father is normal. What is the probability that the daughter is a carrier?

14. What is the probability that a female carrier of an X-linked recessive disease will have
a child with that disease assuming she mates with a normal male?

15. If aa symbolizes a recessive disease, what is the likelihood that parents Aa and Aa will
have a phenotypically normal child?

16. Cystic fibrosis is an autosomal recessive disorder. Two parents that are heterozygous
for cystic fibrosis have a normal, non-affected child. What is the probability that the
child is homozygous normal?

[ 494]
GENETIC LAB TECHNIQUES
R: Chapter 5
PCR (FA15 p77) (FA16 p64) (SU p290)
Gel electrophoresis (SU p290)
Blots (FA15 p78) (FA16 p65) (SU p290)
ELISA (FA15 p78) (FA16 p67) (SU p291)
FISH (FA15 p79) (FA16 p67) (SU p291)
Cloning (FA15 p79) (FA16 p67) (SU p291)
Modifying gene expression (FA15 p79) (FA16 p68) (SU p291)
Karyotyping (FA15 p79) (FA16 p67) (SU p292)

Warm -Up Rey1ew


·1. What is the most common cause of each of the following?
• Hypoparathyrcildism
• Metastatic disease to brain
• Lysosomal storage disea?e
• Myocarditis Cl
m
z
m
-I
()
r-
)>
2. What amniotic fluid measurement is indicative of fetal lung maturity? (FA15 p600) co
(FA16 p607} (SU p96) -I
m
()
I
z
0
c:
m
(f)

. 3. What substances stimulate the release of norepinephrine from neurons?


(FA15 p249) (FA16 p243)

r 495 1
Gel electrophoresis
4. If we111 contains DNA sample A, well 2 contains DNA sample B, and well 3 contains DNA sample C,
what can you say about wells 4, 5, 6, 7, and 8?

High

Molecular
weight

(J)
LU
:=:>
a
z:
:r:
u Low
LU
1- {+)
(l) Anode
<(
-'
u
1-
LU
z:
LU
\..C)

5. DNA sequences
(-)
Cathode
High

DNA sequence patient A

A B
Molecular
weight
DNA sequence patient B

D E

Low
(+)
Anode

i 496 l
End of Sess1 on Quiz
6. What are.the differences between Southern blot, Northern blot, and.Western blot?

7. What type of test uses a known.antigen.tQ discern the presence of an antibody?

8. ·· What type oftest is performed in order to diagnose chromosomal imbalances?


Gl
m
z
m
-I
n
,-
)>
OJ
-I
m
n
:::r:
z
0
c
m
Ul

r 4971
GLYCOLYSIS
Phys: Chapter 67
Overview of energy metabolism
ATP yield (FA15 p98) (FA16 p86)
Glucose transport (FA15 p314) (FA16 p308) (SU p298)
Hexokinase and glucokinase (FA15 p98) (FA16 p86) (SU p299)
Glycolytic pathway (FA15 p97) (FA16 pBS) (SU p299-300)
Glycolytic enzyme deficiencies (SU p299)
Regulation of glycolysis (FA15 p99) (FA16 p87) (SU p300)

War m-U p Rev1ew


1. Which zoonotic bacterium causes each of the following conditions? (FA15 p142)
(FA16 p132)
• Cat scratch fever
• Lyme disease
• Recurrent fever from variable surface antigens
• Bloody diarrhea
• Q fever
• Tularemia
• .·Leptospirosis
• Cellulitis and osteomyelitis from cat or dog bites

2. Hexokinase and glucokinase


glucose

hexokinase
or
glucokinase

__________ ,..,
glucose-6-P

r 49R 1
3. Label the following enzymes and substrates of glycolysis.

0C---- -GI;~~I;;i~- -:~:~~.


-------- ---------w ~

--------------- c
CDeill,@
o®,w
--------------- D

glyc;st~
aldehlyde-3-P ~dihydroxyacetone-P

t
NAD+
NADH+H+

1,3-bisphosphoglycerate
Gluconeogenesis
~~:::!l
G)

'n=<
0
3-phosphoglycerate '=<
(/)

+t
2-phosphoglycerate
Vl

G:: -------- -i~~CD ~ill


~........:;-- - - - - - - - - - - . pyruvate

A. - - - - - - - - - - - - - - - - - - - - - - - - - - - - - - - - - - - - - - - - - - -
B.

C. - - - - - - - - - - - - - - - - - -- ------------------------

D. - -- -- - - - - - - - - - - - - - - - - - - - - - - - - - - - - - -- -- -- -
E.
F.

G. - - - - - - - - - - - - - - - - - - - - - - - - - -- -- -- -- -- -- -- - - - -

H. ------------------------- -- -- -- -- -- -- -- ----
1.

J.
K.

L.
M. _________________

N. ___________________________________________
NOTE: Items 0 through X are discussed in Biochem B.

r 499 1
4. Glycolytic enzyme deficiency
• Clinical presentation: hemolytic anemia, due to inability to maintain Na+-K+ ATPase ~ RBC swelling/
lysis
• Common cause: pyruvate kinase deficiency

5. A muscle biopsy on a patient of yours reveals elevated glycogen levels, elevated fructose 6-phosphate,
and decreased pyruvate. What enzyme deficiency do you suspect most?

6. Regulation of glycolysis

Insulin
PFK-2 (fed)
Fructose-6-P ~========::::::::::::::::======~ Frudose-2,6-B P
FBPase-2 (fasting)

Fructose-1 ,6-B P

r soo 1
7. How does a low insulin/high glucagon state inhibit glycolysis and lead to conversion of energy?

low insulin: glucagon ratio high insulin: glucagon ratio (after a meal)

~ ~
(PFK-2 is phosphofructokinase-2}
Adenylate Cyclase (FBP-2 is fructose bisphosphatase-2)

~
ATP cAMP

active
!
Protein Kinase A
Phosphorylated PFK-2/FBP-2 complex <11141---7'---:::~-y..:---- PFK-2/FBP-2 complex (dephosphorylated)
• active FBP-2 fJ ' • active PFK-2
• inactive PFK-2 ADP ATP • inactive FBP-2

Cl

fructose-2,6-bisphosphate
8 fructose-6-phosphate
@
fructose-6-phosphate fructose-2,6-bisphosphate
~
n
0
~
(less activation of PFK-1 by fru-2,6-bisP) Vl
Vl

In short:
• High glucagon -+ more active protein kinase A-+ active FBP-2 -+ less fru-2,6-bisP-+
less active PFK-1-+ less glycolysis
• Low glucagon (high insu lin) ~ less active protein kinase A~ active PFK-2 -+ more fru-2,6-bi sP ~
active PFK-1-+ more glycolysis

! 501 l
En of SessJon Quiz
8. What enzymes convert glucose to glucose-6-phosphate?

9. .What is the clinical consequence of a glycolytic enzyme deficiency?

Vl 10. What enzymes are responsible for increasing and decreasing the intracellular levels of
Vl
:::J fructose-2,6-bisphosphate?
0
u
:::J
l'J

11. What is the rate-limiting enzyme in the glycolytic pathway?

r so21
GLUCONEOGENESIS
Phys: Chapter 67
Gluconeogenesis (FA15 p97) (FA16 pBS) (SU p300)
Gibbs free energy
Rate-limiting enzymes (FA15 p96) (FA16 p84)

Wa rm -Up Rev1ew
Damag~ to which part of the basal ganglia produces hemiballismu~? . (FA15 p459)
(FA16 p461)

What are the retroperitoneal abdominal structures? (FA15 p343) (FA16p339)


Gl
r-
c
n
0
:z
rn
0
Gl
rn
:z
rn
Vl
Vl

r soJ 1
CDC----------·---------u
3. Label the following enzymes and substrates of gluconeogenesis.

-GI;~~~;~i~- -:~:~~...

--------------- c
~@ill,©
~ 0®ffi
D '

glycp~~
aldehlyde-3-P ~dihydroxyacetone-P
I
NAD•

t
Gluconeogenesis
NADH+W

1,3-bisphosphoglycerate
Vl ADP--..j t
ATP ~ I
Vl
LU
zLU
l9
3-phosphoglycerate
0
LU
z ~t
0
u
:::J
_J

l9
:::--"'fG
-D+P- - - - - - - - - :-:h_o=*sp71ycerate
ADP @ CD
COz ATP ( [ ) Q@ ,ffi
~........:.- - - - - - - - - -pyruvate

NOTE: Items A through N are discussed in Biochem 7.

0. ____________________________________________

P.

Q. - - - - -- - - - - -- -- - ··-··-····---···-··-·. .··-- - - -- -
R. - - - -- -- - - - - - - -- - - - - - - -- - - -- -- - - - - -- - - - - - - -- -
S.

T.

u. --------------------------------------------
v. - - -- - ·---··- ·····-···· - - - -- -- - - - - -- ·

W . ____________________________________________

X. - - - - - - - -- -- -- - - -- - - - - -- - - -- - - -- -- -- - - - - - - -- -

r c;n4 1
4. What is the formula for Gibbs free energy?

End .of Sess1on Qu1z


5. What irreversible enzymes are involved in gluconeogenesis? Cl
,-
c
n
0
:z:
m
0
Cl
m
:z:
m
(J)
(J)

6.> What enzyme catalyzes the rate-limiting step in gluconeogenesis?

7. Order the following molecules by how much energy they contain that can be made
available to fuel endergonic reactions: pyruvate, adenosine monophosphate, glucose,
adenosine, adenosine triphosphate.

r c;o r:; 1
GLYCOGEN
Phys: Chapter 67
H: Chapter 362
Glycogenesis (FA15 p109) (FA16 p98) (SU p300-302)
Glycogenolysis (FA15 p109) (FA16 p98) (SU p302)
Glycogen regulation (FA15 p109) (FA16 p97)
Glycogen storage diseases (FA15 p110) (FA16 p99) (SU p302-304)

Warm-Up Review
1. Which cytokines inhibit Th1 cells? (FA15 p207) (FA16 p200) (SU p376-377)

z
w
(C) 2. Outline the flow of aqueous humor. (FA15 p479) (FA16p483)
0
u
:'::J
CJ

3. What is the mechanism of action of metoclopramide? (FA15 p380) (FA16 p346) (SU p141)
4. Glycogenesis
Glycogen

Uridine diphosphoglucose (UDP-glucose)

UDP-glucose pyrophosphorylase

Glucose-1-phosphate

!l
Glucose-6-phosphate

Gl
Glucokinase or Hexokinase S(
n
0
Glucose Gl
rn
z

5. Glycogenolysis
a-1,4 glycosidic linkage to glycogen chain
0ATP, glucose-6-P, glucose
.--------------------.
Glycogen phosphorylase
X (V/ McArdle)

r c;n7 1
ess1 Quiz
6. What is ..the rate-limiting .enzyme for glycogen synthesis? For glycogenolysis?

Which enzyme converts glucose-6-phosphate to glucose?

8. Which glycogen storage disease matches each of the foll.owing phrases?


" Glycqgen phosphorylase deficiency
• Glucose~6-'phosphatase deficiency
· •· Lactic acidosis, hyperlipidemia, hyperuricemia (gout)
z:
UJ
(.9
• a-1,6•glucosidase deficiency
0
u
:'::i
(.9

• Diaphragm weakness leading to respiratory failure


• glycogen in liver, severe fasting hypoglycemia

• Hepatomegaly, hypoglycemia, hyperlipidemia (normal kidneys, lactate, uric acid)


• Painful muscle cramps, myoglobinuria with strenuous exercise
• Severe hepatosplenom~galy, enlarged kidneys

r SOB l
ENERGY METABOLISM
Phys: Chapters 67-73
Fates of pyruvate (FA15 p100) (FA16 p88) (SU p307)
Pyruvate dehydrogenase (FA15 p99) (FA16 p87) (SU p306)
Pyruvate dehydrogenase deficiency (FA15 p100) (FA16 pBB) (SU p306)
TCA cycle (FA15 p100) (FA16 pBB) (SU p306)
Electron transport chain (FA15 p101) (FA16 p89) (SU p305-306)
Lactic acid cycle (Cori cycle) (FA15 p105) (FA16 p93) (SU p300)
Alanine cycle (FA15 p105) (FA16 p93) (SU p297-298)

Warm -Up Rev1ew


1. Wl'leit is the landmark for a pudendal nerve.block? (SUp209)

rn
z:
rn
2.> Which enzyme catalyzes the rate-limiting step in carbohydrate digestion? ;:o
Gl
-<
3:
rn
;r;
OJ
0
,-
Vl
3:
~'- 3. Which vitamin deficiency results from Hartnup disease? (FA15 p531) (FA16 p78) (SU p293)

r so9 1
4. Electron transport chain

Mitochondrial matrix

5. Cori cycle
ANAEROBIC GLYCOLYSIS GLUCONEOGENESIS
Muscle/RBCs Liver

glucose +-----+ +---- glucose


~2A~ 6ATP~
l ~.~~:... . .
2pyruvate 2 pyruvate

l -----+
lactate
dehydrogenase

21actate +----+ 21actate

r s1 ol
Alanine cycle
6. Outline the basic processes of transamination and oxidative deamination involved in the interorgan
transport of nitrogen.
excretion in kidneys

Muscle Serum Liver t


protein ;

ureaJ

URE~
1 ;

~YCLE

>-<
amino acids a - ketoacids .! NH +
4

NH/ ,, NH4 + ~
a- ketoglutarate glutamate S.... glutamine glutamine ..4 glutamate a- ketoglutarate

>-< >-<PY·r·
'

pyruvate alanine ,
.... alanine

l
rn
z
rn
::::0
Cl
...., -<
glucose glucose srn
:;!
OJ
0
r-
Ul
s
7. Why are alanine and glutamine found in such high concentrations in the blood?
They are the two major carriers of - -- -- -- -- - - - - -- -- -- -- from tissues.

8. What enzymes catalyze transamination reactions?


• - -- -- -- -- -- -- -- -- -- -- transfer the amino group from an amino acid to a-ketoglutarate to
form glutamate
• The remaining deaminated amino acid becomes a ketoacid (such as pyruvate) that is used in energy
metabolism

9. What are the two most important transaminases? What reactions do they catalyze?

10. What cofactor is required by all transaminases?


Pyridoxal phosphate (a derivative of vitamin - - - - - - -

r s 111
End 6fSess1on Qui
11. What are the possible products of pyruvate?

1.2. Whata.re the two main nitrogen transporters in the blood?

13. What are the major regulatory en~ymes ofthe citric acid (TCA) cycle?

2:
(f)
_j

0
co
;:;
w
2:
>-
~ 14. What subsJances are known to inhibitthe complexes of the electron transport chain?
w
z
w

15. What substances can increase the permeability of the .inner mitochondrial
membrane, thereby decreasingATP synthesis but increasing heat generation?

512]
HMP SHUNT AND OTHER SUGARS
Phys: Chapters 67, 84
HMP shunt (FA15 p102) (FA16 p90) (SU p306)
Oxidative burst (FA15 p208) (FA16 p201) (SU p307-308)
Glucose-6-phosphate dehydrogenase deficiency (FA15 p102) (FA16 p90) (SU p307)
Fructose metabolism (FA15 p103) (FA16 p91) (SU p304)
Galactose metabolism (FA15 p103) (FA16 p91) (SU p304)
Lactose metabolism (FA15 p104) (FA16 p92) (SU p304)

Warm-Up Rev1..ew
1. For what conditions is dantrolene useful? (FA15 p499) (FA16 p503)

2. What are the possible etiologies of acute pancreatitis? (FA1S p376) (FA16 p373)
I
:5:::
--o
{J)
I
c
:z
-j
)>
:z
CJ
0
-j
I
~--., m
;;o
{J)
c
Cl
)>
;;o
Vl

. . ..·· ...... ... .


_ ,._. . _.,

3. Which cell type releases renin? (FA15 p535) (FA16 p540) (SU p147)

r s13 1
4. Oxidative burst

Q2 NADPH
NADPH oxidase v--
~ NADP+
o2-(superoxide ion)
'
Superoxide dismutase ~

H 202
CI ~I~ H20+02
Myeloperoxidase ~
+
(/)
HOCI (hypochlorous acid)
0:::
<J.:
\.9
::::>
(/)
0:::
LU
:r:
r--
0

~
<!:
5. Glutathione and G6PD
r--

NADPH\ / 6PG
:z:
::::>
~ H20 GSSG
o__
:;E
:r:

Glucose--6-.
phosphate . 1
tydrogerJ?Se l
7\ ·=~

NADP
./ ~- ~ G6P

[ 514 J
6. In patients with G6PD deficiency, ingestion of what substances can induce oxidative
damage to RBCs?
"Spleen Purges Nasty Inclusions From Damaged Cells"

• Primaquine
• Nitrofurantoin


• Dapsone
• Chloroquine

7. Fructose metabolism Glycolysis

I I
:s:
--o
(J)

Aldolase B I
c
z
--1
)>
z
0
0
--1
I
rn
;:u
Glycerol (J)
c
Cl
)>
;:u
(J)

8. Galactose metabolism
Gaf-1-P
Galactokinase uridyltransferase
Galactose Galactose-1-P Glucose-1-P

Aldose reductase

Galactitol Glycolysis/
gluconeogenesis

r s1s l
End ..of Session Qu1z
9. What is the rate-limiting step of the pentose phosphate pathway?

10. Which tissues of the body use the pentose phosphate pathway?

11..Explain why a deficiency of the enzyme that is the rate limiter for the HMP shunt can
result in hemolytic anemia.
(f)
cr
<(
\.C)
::::J
(f)

cr
w
I
5 12. What are the symptoms of classic gal.actosemia?
0
z
<(
f-
z
::::J
I
(f)
o_
~
I

13. Whatdisorder is caused by a deficiency of each ofthe following enzymes?


• .·Galactokinase
. • Aldol.ase B
• Lactase
• Galactose-1-phosphate uridyltransferase
• Fructokinase

516 1
STARVATION AND MALNUTRITION
R: Chapters 9, 24
Phys: Chapter 84
H: Chapters 75, 392
Starvation (FA15 p113) (FA16 p103) (SU p310)
Ketogenesis (FA15 p112) (FA16 p102) (SU p310)
Ethanol metabolism (FA15 p95) (FA16 p83) (SU p304-305)
Malnutrition states (FA15 p94) (FA16 p82) (SU p310)
Re[eeding syndrome (SU p310)

War -Up Review


t; Wf1at is the typical presentation of a patient with pancreatic insufficiency?
(FA1S p361) (FA1pp3!)B)(SUp113)

(J)

:;;
:::0

2. What are the symptoms of serotonin syndrome? (FA1Sp523) (FA16p527)(SU p55) ~


0
z
)>
z
0
:s
)>
,-
z
c
-I
:::0
-I
3. Which artery prevents a hors~shoe kidney from ascending in the abdomen? (FA15 p527) 0
z
(FA16 p531) (SU p144)

4. What fuels are produced and used in the post-absorptive period?


• Produced-

• Used-
- Muscles, brain , and other tissues use predominantly _ _ _ _ _ _ _ _ _ _ _ __

5. When does gluconeogenesis begin in the post-absorptive period? When does it become fully active?
• Begins hours after the last meal
• Fully active when glycogen stores are depleted (_ _ _ _ _ _ hours after la st meal)

r si7 1
6. How does the pattern of fuel production and usage change in early starvation (24 hours after the last
meal)?
• Produced-

• Used -
Brain uses predominantly _ _ _ _ _ _ _ __
Muscles and other tissues use some but predominantly _ _ _ _ _ _ _ __

7. In intermediate starvation (48 hours after the last meal), how does the pattern of fuel production and
utilization change?
• Produ ced -

• Used-
z Brain uses predominantly _ _ _ _ _ _ _ _ but also some _ _ _ _ _ _ __
0
!:::: Muscles and other tissue use predominantly but also
0:::
I-
some _ _ __ __ _ _ ___
::>
z_J
<(
::2::
8. What metabolic scenario favors the synthesis of ketone bodies?
0
z
<(
z
0
~
>
0::: 9. True or false? Ketone bodies can be used by all body tissues including the brain.
~
(/)

10. What is the pattern of fuel production and utilization in prolonged starvation (5 days after last meal)?
• Produced-

• Used-
Brain uses predom inantly _ _ _ _ __ __ __ _ __ _ _
Muscles and other tissue use predom inantly _ _ _ _ _ _ _ _ _ _ __
but also some _ _ _ _ _ _ _ _ _ _ _ __

11. Comparing an overnight fast to a 3-day fast, what percentage of energy comes from glucose and from
ketone bodies?
• Overnight - _ _ _ %from glucose(% from glycogen breakdown, Y:J from gluconeogenesis)

_ _ _ % from ketone bodies

• 3 -day- _ _ _% from ketone bodies (%are 13-hydroxyb utyrate, %acetoacetate)

_ _ _% from glucose (most from gluconeogenesis)

r 518 1
End . of .S.ess1o Quiz
12. What is the primary energy source in a patient that has not eaten in two days?

13. What is the rate-limiting enzyme in ketone body synthesis?

A stressed physician comes home from work, consumes s~ven or eight shots of
tequila in rapid succession_beforedinner, and becomes hypoglycemic. Why did she
become hypoglycemic?
(J)

:;;
;;u

~
0
z
)>
z
0
15. What are sorne of the hallmark features of kwashiorkor? :s
)>

'cz
--i
;;u
--i
0
z

[ 5 !9 1
LIPIDS
Phys: Chapter 68
H: Chapter 356
Lipid transport (FA15 p114) (FA16 p104) (SU p308)
Abetalipoproteinemia (SU p129)
Apolipoproteins (FA15 p115) (FA16 p104)
Dyslipidemias (FA15 p115) (FA16 p105) (SU p73-74)
Signs of hypercholesterolemia (FA15 p291) (FA16 p285)
Cholesterol synthesis (FA15 p114) (FA16 p103)
Fatty acid metabolism (FA15 p112) (FA16 p101)
Omega-3 fatty acids

rm~np Rev1ew
What are the classic manifestations of Guillain-Barre syndrome? (FA15p4BB)(FA16 p493)

(f)
0
Q_
_j

What are the risk factors for the developmentof hepatocellular carcinoma? (FA15 p370)
(FA16 p135)

3. What is the presentationcWtricyclic. antidepressant (TCA)overdose? (FA15 p523)


(FA16p527)

r s2o 1
4. Lipid transport

Free- fatty-adds
'--o
~t-
0
Adipose-t issue,-musde l/)

HDL transport
Scavenger receptor 81 (SRB1)

Hepatic lipase
--~---'---=:------ HDL-3

Triglycerides
Cholesterol esters

Cholesterol ester
HDL-2or3 VLDL, IDL, LDL
transfer protei n (CETP)
lecithin-cholesterol
acyltransferase (lCAT)
Trig lycerides
NascentHDL

Cholesterol

i !))l l
ofSessron .Qurz
5. What deficiency causes familial hypercholesterol~mia?

6. Which apolipoprotein matches each of the following statements?


• Activates LCAT
• Mediates chylomicron secretion
• Mediates VLDL secretion
• Cofactor for lipoprotein lipase
• Mediates uptake of remnant particl es

Vl
0 What is the rate~limiting enzyme for each of the following metabolic pathways?
o_
_j
• Fatty acid synthesis
• ~-oxidation of fatty acids
• Ketone body synthesis
• Cholesterol synthesis

8. Which group of medications inhibits. the rate-limiting enzyme


synth~sis? · ·
AMINO ACIDS AND NITROGEN
Phys: Chapter 69
Amino acid metabolism (FA15 p105) (FA16 p93)
Derivatives of amino acids (FA15 p106) (FA16 p94)
Urea cycle (FA15 p105) (FA16 p93)
Ornithine transcarbamylase deficiency (FA15 p106) (FA16 p94)

Warm -Up Review


Describe which light reflexes would be seen in both eyes if the right optic nerve were
damaged prior to the pretectal nucleus. (FA15p483) (FA16 p487)

)>
:s
z
0
2. What are the major ECG changes seen with a myocardial infarction? (FA15 p293) )>
n
(FA16 p289) 0
(J)
)>
z
0
z
---1
;::o
0
Gl
rn
3. What are the clinical features of neuroleptic malignant syndrome? (FA15 p521) (FA16 p525) z

r s:n 1
4. Derivatives of amino acids
Phenylalanine
BH, ! Ph•nyl>l>nln• hyd•o,yl'>'

Homogentisic acid ~---Tyrosine


Homogentisate
oxidase
J BH4
~
I Tyrosine hydroxylase

Tyrosinase
Dopa
!
Maleylacetoacetic acid Melanin
J lntamln B, Dopa dewbo,ylase

e
Dopamine HVA
Vitamin C ~

NE VMA
SAM ~ (±)Cortisol
z
w
l:J Epi Metanephrine- VMA
0
00::
f-
z
0
z
<(
U)
0
End of Session Qu iz
u
<( 5. What are the essential amino acids?
0
z
~
<(

6. What amino acid is a precursor to each of the following molecules?


• Histamine
• Porphyrin, heme
• NO
• •GABA
• · S-adenosyl-methionine (SAM)
• Creatine

7. Compare carbamoyl phosphate synthetase I to carbamoyl phosphate synthetase II.

! 5241
AMINO ACID DISORDERS
H: Chapter 364
PKU (FA15 p107) (FA16 p95) (SU p292)
Alkaptonuria (FA15 p108) (FA16 p96) (SU p292)
Albinism (FA15 p107, 438) (FA16 p439) (SU p293)
Homocystinuria (FA15 p108) (FA16 p96) (SU p293)
5-adenosyl-methionine
Cystinuria (FA15 p108) (FA16 p96) (S U p293)
Maple syrup urine disease (FA15 p107) (FA16 p95) (SU p293)
Hartnup disease (FA15 p531) (FA16 p78) (SU p293)

Warm -Up Rev1ew


1. What dermatologic fmding(s)is se(:!n in each ofthe stages of syphilis? (FA15 p141)
(FA16 p131) (SU
• Primary
• Secondary
• Tertiary )>
s
:z:
0
)>
n
2. Which antihypertensives are commonly used durin·g pregnancy? {FA15 p581) (FA16pS90) 0
(5Up76 -77) 0
V1
0
AJ
0
m
AJ
V1

3. What does each of the following cells of the Gl tract secrete? (FA15p353-354)
(FA16 p350-351)
• Gcells
• I cells
• S cells
• D cells
• Parietal cells

r 525 1
End of Session Qu1z
4. A full-term neonate feeds poorly, is hyperactive and has a musty odor. What is the
diagnosis?

5. A patient with PKU should have a diet low in phenylalanine. What other dietary
modifications should a patient with PKU make?

6. A middle-aged man has dark spots on his sclera and has noted that his urine turns
black when left sitting for a period of time. What is the diagnosis?
Vl
0:::
LlJ
0
0:::
0
Vl
0
0
u
<! 7. What is the underlying cause of maple syrup urine disease?
0
z
:2:
<!

r 526 1
MINERALS
R: Chapter 9
H: Chapter 357
Iron
- Iron poiso ning (FA15 p397) (FA16 p396)
- Iron proteins (FA15 p395) (FA16 p394)
Zinc (FA15 p94) (FA16 p82)
Calcium (FA15 p319-321) (FA16 p313-315) (SU p174)
Toxic me tals
- Lead poisoning (FA15 p391) (FA16 p389) (SU p257, 259, 311)
- Mercury poisoning

Wa rm -Up Revi ew
1. What is ..the patho~enesi~ of ~lai.Jcoma?(FA15 p4BO) (FA16 p484)

5:
:z
rn
;;o
)>

'
l/)

2. Which infectious agent is the cause of pneumonia based on each of the following lab
tests?
• Gram-negative rods in a neoriate with pneumonia
• Gram-positive cocci in a neonate with pneumonia
• Gram-positive cocci in pairs
• Gram-positive cocci in clusters

3. What is the classic presentation of aortic dissection? What chest x-ray fmding~would
support the diagnosis? What antihypertensive class is the preferred treatment?
(FA15 p293) (FA16 p287) (SU p80)

4. Iron proteins
Ferritin Transferrin
• Iron-protein comp lex (ferric acid and apoferriti n) • Protein that binds ferric molecules
• Cell ular storage protein for iron and transport s them through plasma.
• Ac ute-phase reactant Synthesized in the liver.
• t* = 8 days
• Increased in iron defi ciency

I 5271
5. Mercury poisoning
• Accumulates in the kidney and brain
• Acrodynia : peeling of the fingertips
• Abdominal pain
• Common sources: shark, swordfish, tilefish , king mackerel , old thermometers, batteries

End of Session Quiz


6. What has been the most common clinical scenario of. excess iron ingesticm in the last
three decades?

7. What are some of the clinical effects of zinc deficiency?


(f)
_j

<(
0:::
w
z
:2:
R What are some of the signs of hypocalcemia?

9. What organs are primarily affected by an excess of mercury?

RAPID-FIRE FACTS
I Bluish-colored lines on the gingivae

r .S/8 1
FAT-SOLUBLE VITAMINS AND ANTIOXIDANTS
R: Chapter 9
Phys: Chapter 71
Vitamin D (FA15 p93) (FA16 p81) (SU p312)
Vitamin K (FA15 p94) (FA16 p81) (SU p313)
Vitamin A (FA15 p89) (FA16 pll) (SU p312)
Vitamin E (FA15 p93) (FA16 p81) (SU p313)
Vitamin C (FA15 p93) (FA16 pBO) (SU p313-314)

Wa -Up Rev1 ew
Which cytokine inhibits Th 2 cells? (FA15 p202! (FA16 p200)

,
2. What dermatologic fmding is associated with Bacillus .anthracis infection? (FA15pl32) ~
Vl
(FA16 p121) (5Up327) 0
,-
c
o:J
,-
rn
<
:;;
:s
z
Vl
3. What are common causes of acute respiratory distress syndrome (ARDS)? (FA15p613) )>

(FA16 p621).(SU p102) z


0
)>
z
-I
0
X
0
)>
z
-I
Vl

r s1g l
End of Sess1on Qui z
4. What vitamin in excess can cause hypercalcemia?

5. What are the symptoms of vitamin A toxicity?

(./)
f-
z
<(
0
X
o 6. What clinical features would lead you to suspect that a patient has scurvy?
f-
z
<(
0
z
<(
(./)
z
~
~
>
ll.J
_J
7. Vitamin C is necessary for the hydroxylation of which amino acids in collagen
(l)
::>
synthesis?
_J

0
(./)

~
LL

8. Which enzymes are used to convert cholecalciferol to calcitriol?

RAPID - FIRE FACTS


Treatment of choice for rickets or osteomalacia

Swollen gums, poor wo und hea ling, bleeding


mucous membranes, and spots on the skin
WATER-SOLUBLE VITAMINS
Phys: Chapter 71
Vitamin 8 1 (FA15 p89) (FA16 pll) (SU p315)
Vitamin 8 2 (FA15 p89) (FA16 p78) (SU p315)
Vitamin 8 3 (FA15 p90) (FA16 p78) (SU p315)
Vitamin 8 5 (FA15 p90) (FA16 p78) (SU p316)
Vitamin 8 6 (FA15 p90) (FA16 p78) (SU p314)
Vitamin 87 (FA15 p91) (FA16 p79) (SU p316)
Vitamin 8 9 (FA15 p91) (FA16 p79) (SU p314)
Vitamin 8,2 (FA15 p92) (FA16 pBO) (SU p314)

Warm -Up Rev1ew


1. Describe which light reflex~s.would ..be seen in both eyes if the right oculomotor
nerve wel'e damaged. (FA15 p483) (FA16 p487)

2. What is the differential diagnosis for eosinophilia? (FA15 p383! (FA16 p379)

3. What substances acton smooth muscle myosin light-chain kinase? How do these
affgct blood pressure?

r s~ 1 i
of Sess1on Quiz
A patient presents with convulsions and irritabil.ity. What vitamin deficiency is
causing these symptoms in this patient?

5. . What type of anemia can be caused by folate or 8 12 deficiency?

6. Where is B12 absorbed into the circulation?

<.n
z
~
7. Whi.ch vitamin deficiency matches each of the following descriptions?
<(
r- • Peripheral neuropathy, glossitis
> • Neurartube defects
w
---'
CD • Dermatitis, diarrhea,. dementia
=:J
---' • . Meg8loblastic anemia
0
l{l Pernicious anemia
cr:
w
r-
<(
s
Which vitamin matches each of the following statements?
• Used in oxidation/reduction reactions
• Used in carboxylation reactions
• Requires intrinsic factor for absorption
• Used by pyruvate dehydrogenase and a-ketoglutarate dehydrogenase
• Can be used to elevate HDL and lower LDL
• Deficiency can be caused by isoniazid use
• Cobalt is found within this vitamin
• Critical for

RAP/ 0-FI RE FACTS


Most common vitamin deficiency in the U.S.

Hypersegmented neutrophils

Dilated cardiomyopathy, edema, and polyneuropathy

r 53 2 1
UGH OFTEN

PSYCH VIEWED A 'so


SCIENCE, PSYCHIATRY
IS !MPORTANTBOTH IN
1 Psychosis CLINICAL PRACTICE AND
ON L!CENS/NGEXAM
2 Ant1psychot1cs
THEASTUTE CLI NICIAN
UNDERSTANDS THAT
3 Bipolar Disorder
PSYCHOPATHOLOGY
AND TH EM/NO - BODY
4 Dep ress 1on
CONNECTION
INFLUENCE HEALTH ON A
5 Ant idepressant s
FUNDAMENTAL LEVEL.
6 Anxiety and So matoform Diso rd ers

7 Eat1ng Disorders

8 Chi ld Psych

9 Ego Defe nse Mechanisms and


Personali ty Disorders

10 Alco hol Abuse

11 Su bstance Abuse
PSYCHOSIS
H: Chapter 391
Symptoms of psychosis (FA15 p508) (FA16 p513)
Types of hallucinations (FA15 p509) (FA16 p513)
Anatomy and physiology of schizophrenia
Schizophrenia (FA15 p509) (FA16 p514) (SU p53)
Schizophrenia-related disorders (FA15 p509) (FA16 p514) (SU p54)
"Schizo-" (FA15 p516) (SU p60)
Delusional disorder (FA15 p509) (FA16 p514) (SU p54)

Warm-Up Review
·1. What is the characteristic DNA sequence of the promoter region? What does a
mutation in the sequence cause? (FA15 p68) (FA16p54) (5Up284!

2. What is the treatment for methemoglobinemia? (FA15 p257! (FA16 p251) (SU p16)

3. What is the hallmark sign of COPD? (FA15 p610) (FA16 p618) (SUp104!

-u
Vl
-<
n
:r
0
Vl
Vl

r 5351
What are some ofthe positive symptoms of schizophrenia?

What are some of the negative symptoms of schizophrenia?

How long must symptoms last in order to meet the diagnostic criteria for each of the
foltowing?
• Schizophrenia
• Delusional disorder
(f) • Brief psychotic disorder
(f)
0
• Schizophreniform disorder
:r:
u
>-
(f)
[l_

r s36 1
ANTI PSYCHOTICS
H: Chapter 391
GG: Chapter 16

Antipsychotic classes (FA15 p521) (FA16 p525) (SU p54) (GG p428) (H p3544)
Neuroleptic side effects (FA15 p521) (FA16 p525) (SU p54-55)
Neuroleptic malignant syndrome (FA15 p521) (FA16 p525) (SU p54)
Atypical antipsychotics (FA15 p521) (FA16 p525) (SU p54) (GG p428) (H p3544)

Warm -Up ew
1. Which structural motifs allow proteins to bind to DNA?

2. What are the diJferent eukaryotic RNA polymerases? (FA15 p68) (FA16 p54)

3. What is the hallmark sign of a restrictive lung disease? (Fids p611) (FA16 p619)(SU p98)

)>
:z:
---1
--o
Vl
-<
n
4. / Antipsychotics ~ I
0
-1
n
Typical Anti psychotics Atypical Anti psychotics Vl

(Traditional Neuroleptics)

/
Low potency ""
High potency

r 537l
5. What are the common side effects of low-potency traditional neuroleptics?
• _ __ _ __ __ _ _ _ _ _ (delirium, drying of secretions, constipation , urinary retention,
mydriasis)
• Sedation
• Orthostatic hypotension

6. What movement disorders can be seen as side effects of high-potency traditional neuroleptics?
• (sustained muscle contraction)
Within several days
• Treat by adding an anticholinergic drug

Within the first month • (bradykinesia, akinesia)

• (restlessness, compulsion to
Within the first 2 months move)

• (choreoathetosis of the
Several months to years tongue, face, neck, trunk, or limbs)

7. What are the signs and symptoms of neuroleptic malignant syndrome?


• _ __ _ __ __ _ __ __ _ _ - initial symptom in most patients (agitated delirium with
confusion rather than psychosis)
• Muscle rigidity +/- tremor
• - -- - -- - - -- - - - (> 38-40°C)
V)
u • Rhabdomyolysis appearing over 1-3 days
f---
0 • _ __ _ _ __ __ _ __ _ _ _ - tachycardia, high/labile blood pressure, tachypnea,
:r::
u diaphoresis
>-
V)
0....
f-
z
<(

8. What drugs are used to treat neuroleptic malignant syndrome?

9. What are the common side effects of atypical anti psychotics



• Diabetes
• Sedation
• Clozapine can cause _ _ __ _ __ _ __ ___ (monitor CBC once a week at first)

r 538 1
d of Sess1. on Ou1z
10. Place each ofthe following antipsychotics in the appropriate category:
olanzapine, thioridazine, quetiapine, chlorpromazine, haloperidol, f1uphenazine, loxapine,
risperidone, thiothixene, trit1uoperazine, clozapine, aripiprazole

High-potencyneuroleptics

Low-potency neuroleptics

Atypicalantipsychotics

11. Which antipsychotic is most closely associated witban increased risk of diabetes?
)>
z
--j
\J
lfl
-<
n
:r:
0
--j
n
lfl
12. What are the clinical features of neuroleptic malignant syndrome?

13. What is the ftrst line pharmacotherapy for neuroleptic malignant syndrome?

r s39 1
BIPOLAR DISORDER
H: Chapter 391
GG: Chapter 16

Mania (FA15 p510) (FA16 p514)


Hypomania (FA15 p510) (FA16 p515)
Bipolar disorder (FA15 p510) (FA16 p515) (SU p58) (H p3539)
Lithium (FA15 p522) (FA16 p526) (SU p58, 434) (GG p445)
Dissociative disorders (FA15 p510) (FA16 p512)

Warm-Up Rev1ew
1. Which enzyme is responsible for tRNA charging? Which enzyme catalyzes peptide
bond formation? (FA15 p70) (FA16 p56)

2. How does the emphysema caused by smoking differ.from the emphysema caused by
«t-antitrypsin deficiency?.(FA15 p610) (FA16 p618) (SU p105)

et:::
w
0
et:::
0
VJ
0 3. What are the causes of a transudative pleural effusion? (FA15 p615) (FA16 p623)
et:::
<(
__.J

0
0...
co

4. Patient Profile: Lithium? I hardly know him!


A 38 -year-old man comes to the physician for routine follow-up. He has a 5-year history of bipolar II
disorder, which is treated with lithium. What side effect is most likely to be seen shortly after increasing
the lithium dose?

r 540 1
nd o f Session Quiz
5. Whatare thecriteriaforthediagnosis of mania?

6. What mood stabilizers are used in the treatment of bipolar disorder?

7. What are the potential side effects of lithium?

OJ
u
0
';;u
)>

0
Vl
0
;;u
0
m
;;u

r £AI 1
DEPRESSION
H: Chapter 391
Grief (FA15 p512) (FA16 p516)
Major depressive disorder (FA15 p511) (FA16 p515) (SU p55)
- with atypical features (FA15 p511) (FA16 p515) (SU p55)
- with peripartum onset (FA15 p511) (FA16 p516) (SU p55)
- with psychotic features
- with seasonal pattern (SU p55)
Persistent depressive disorder (FA15 p511) (FA16 p515) (SU p55)
Premenstrual dysphoric disorder
Suicide risk factors (FA15 p512) (FA16 p516)

Warm - Up Rev1 ew
1. What are the different prokaryotic RNA polymerases? (FA15 p68) (FA16 p54)

2. Which enzyme is deficient in lesch-Nyhan syndrome? What is the treatment?


(FA15 p64) (FA16 p50)

z
0
Ul
Ul
w 3. What are the causes of an exudative pleural effusion? (FA15 p615) (FA16 p623)
0:::
CL
w
0

i .542 l
4. What are the Kubler-Ross stages of grief?
• Denial
• Anger
• Bargaining
• Grieving (depression)
• Acceptance
Stages may occur in any order, and more than one stage may be present simultaneously.

5. What are the diagnostic symptoms of major depressive disorder?


At least 5 of the following 9 symptoms during a 2-week period:
• Depressed mood
• I (diminished) - "anhedonia"
• Guilt or worthlessness
• Sleep disturbance
• Concentration (impaired)
• A or weight changes
• Psychomotor agitation or retardation
• E loss (fatigue)
• Suicidal ideation (recurrent thoughts of death)

6. What are the major risk factors for suicide?



• Feelings of _ _ __ _ __ _ _ _ _ _ _ __
• Impulsivity
• 0
rn
-o
• Male sex :::0
rn
• Access to _ _ _ _ _ _ _ _ _ _ _ _ _ __ _ l/)
l/)

• History of _ _ _ _ _ _ _ _ _ _ _ _ _ __ 0
:z

[ 543 l
En d of Sess 1on Qui z
7. What are the criteria for the diagnosis of major depressive episode?

8. What is the presentation of major depressive disorder with atypical features?

:~

z 9. A 28-year-old woman has symptoms of mild depression for six years. What's the
0
tn
diagnosis?
tn
LU
a:
o_
LU
0

r 5441
ANTIDEPRESSAITS
GG: Chapter 15

SSRis (FA15 p523) (FA16 p527) (SU p56)


SNRis (FA15 p523) (FA16 p527) (SU p56)
Tricyclic antidepressants (TCAs) (FA15 p523) (FA16 p527) (SU p56)
Monoamine oxidase inhibitors (MAGis) (FA15 p523) (FA16 p528) (SU p56)
Other antidepressants (FA15 p524) (FA16 p528) (SU p56)
Electroconvulsive therapy (ECT) (FA15 p512) (FA16 p516) (SU p51)

Warm-Up 1ew
1. What are the mRNA stop codons? (FA15 p67) (FA16 p53)

2. What are the differences between carbamoyl phosphate synthetase (CPS) I and
CPS II? (FA15 p96) (FA16 p84)
··.

CPS I CPS II
·.·. "

Location

Pathway
}>
z
--j

Nitrogen source 0
:· rn
-o
;:o
rn
(})
(})
}>
z
--j
(})

r s1s 1
eSSlOn
3. What is the mechanism pf action of each of the following medication classes?
• SSRI
• SNRI
"TCA
• MAO!

4. CategOrize ~ach ofthe following antid~pressants:


amitriptyline, bupropion, citaloprarn, clomipramine, doxepin, duloxetine, fluoxetine,
imipramine, nefazodone, nortriptyline, paroxetine, phenelzine, sertraline, tranylcypromine,
trazodone, venlafaxine ·

SSRI

TCA

V1 SNRI
f-
z
-
<(
V1
V1
LLJ
0::: Serotonin· modulators
CL
LLJ
0
f-
z
<(

Which antidepressant matches each of the following statements?


• Side effect of priapism
• Lowers the seizure threshold
• Appetite stimulant that is likely to result in weight gain
• Can be used for smoking cessation
• Can be used for bedwetting in children

6. What are the symptoms of TCA overdose?

7. What are the symptoms of serotonin syndrome?


ANXIETY AND SOMATOFORM DISORDERS
H: Chapter 391
Panic disorder (FA15 p512) (FA16 p517) (SU p57) (H p3529)
Specific phobias (FA15 p513) (FA16 p517) (SU p57) (H p3533)
Obsessive-compulsive disorder (FA15 p513) (FA16 p517) (SU p57) (H p3535)
Post-traumatic stress disorder (FA15 p513) (FA16 p517) (SU p57) (H p3534)
Generalized anxiety disorder (FA15 p513) (FA16 p517) (SU p57) (H p3530)
- Buspirone (FA15 p522) (FA16 p526) (SU p410) (GG p349)
Malingering and factitious disorder (FA15 p514) (FA16 p518) (SU p58)
Somatoform symptoms and related disorders (FA15 p514) (FA16 p518) (SU p58) (H p3541)

Warm -Up Revi


V\(hat are the major criteria for acute rheumatic fever? {FA15 p299) (FA16 p294)

)>
z
X
rn
---i
-<
)>
In a patient with an elevated alkaline phosphatase, elevations in which other serum z
0
lab markers would point toward a diagnosis of liver disease? (FA15 p368) (FA16 p366) (/)
0
5:
)>
---i
0
";:u
0
5:
0
(/)
0
;:o
0
rn
;:u
(/)

[ 547 J
End of Sess1on Qu iz
3. A patient on whom you wish to obtain an MRI tells you that he cannot go through
with it because of claustrophobia. What can you do to help this patient?

4. Ayoung woman complains about daily anxiety thatis worsened by her stressful job.
She has occasional brief episodes of intense fear and palpitations, but even outside
of stressful situations she generally feels anxious. What is the most appropriate
treatment for this patient?

(f)
0:::
Ll.J
0
0:::
0 5. A nurse at a hospital asks off from work because of multiple episodes of
(f)

0 hypoglycemia. Blood analysis reveals no elevation in C-peptide. What is her


~ diagnosis?
0:::
0
LJ..._

0
~
~
0
(f)

0
z
<(
>-
6. What somatoform disorder matches each of the following descriptions?
f-
Ll.J • Unexplained pain
>< • Patient with normal anatomy is convinced a part of his/her appearance is abnormal
z
<(
• Unexplained loss of sensory or motor function (tests and PE are negative)
• Unwavering belief by the patient that she has a specificdisease (despite medical
reassurance)
• Unexplained complaints in multiple organ systems

[ 548]
EATING DISORDERS
H: Chapters 25, 371, 392

Anorexia nervosa (FA15 p516) (FA16 p520) (SU p52)


Bulimia nervosa (FA15 p516) (FA16 p520) (SU p52)
Binge-eating disorder (FA16 p520) (SU p52)
Electrolyte disturbances in purging

Warm-Up .Rev1ew
1. Which artery supplies each of the following regions? (FAts p346) (FA16 p342)
• Foregut
• Midgut
• Rectum and distal third oft he colon

2. What disorder is characterized by c:m excess of gastrin? (FA15 p336) (FA16p332)

3. In Langerhans cell histiocytosis, electron microscopy of a dendritic cell would show


what characteristic finding? (FA15 p403) (FA16 p403)

rn
~
z
Cl
0
(J)
0
;;u
4. Patient Profile: Hungry for Honors 0
rn
A 24-year-old female medical student comes to th e emergency department after vomiting blood. ;;u
(J)

She has been unable to find time to exercise and follow a healthy diet w hile studying intensely for
upcoming exams, and she has been inducing vomiting to prevent weight gain. The patient frequently
eats excessive amounts of food and fe els guilty about it afterward. She has successfully hidden these
eating binges from her roommate. The patient complains that "med school is making her fat," and she
worries that she'll be too fat to be the maid of honor at her sister's upcoming wedding. The patient's
height is 170 em (67 inches). Her weight is 68 kg (150 lbs). Her body mass index (BMI) is 23.5. She is
well-nourished, well-developed, and in no acute distress. Her oral mucosa is slightly dry. The parotid
glands are enlarged and tender. The remainder of her physica l examination is unremarkable. What
electrolyte abnormalities and acid -base disorder would most likely be present due to her recurrent
induced vomiting?

[ 549 l.1
End of Ses sio n Qui z
5. What are the diagnostic characteristics of anorexia nervosa?

6. .What are the diagnostic characteristics of binge-eating disorder?

7. What are the diagnostic characteristics of bulimia nervosa?

8. What electrolyte changes are seen in patients with excessive vomiting over time?

Vl
0:::
w
0
0:::
0
Vl
RAP! D- F! RE FACTS
0
lCJ
Most common class of med ication for bulimia nervosa
:z
~
w

i 550 l
CHILD PSYCH
H: Chapter 79
Autism spectrum disorder (FA15 p507) (FA16 p511)
Attention deficit hyperactivity disorder (ADHD) (FA15 p506) (FA16 p511) (SU pSB)
Separation anxiety disorder (FA15 p506) (FA16 p511)
Hair-pulling disorder
Tourette syndrome (FA15 p506) (FA16 p511) (SU pSB)
Conduct disorder (FA15 p506) (FA16 p511)
Oppositional defiant disorder (FA15 p506) (FA16 p511)
Child abuse (FA15 p506) (FA16 p510) (SU p220)
Sexual abuse (FA15 p506) (FA16 p510) (SU p220)
Child neglect (FA15 p506) (FA16 p510) (SU p220)
Results of neglect (FA15 p506) (FA16 p510)

Warry1-Up .Rev1
1. What would be seen on microscopic examination of joint aspirate from a patient with
gout? With pseudogout? (FA1Sp430~431) (FA16 p431) (SU p233)

2. Where is Virchow node located? (FA15 p360) (FA16 p356)(SU p120)

n
I
r-
0
3. Which three cell types present antigens toT cells? -o
(./)
-<
n
I

4. What are the features of autism spectrum disorder?


• Defi cits in social communication and social interaction
"Living in his own world"
Lack of responsiveness to others, poor eye contact, absence of social smile
• Restricted or repetitive patterns of behavior
Peculiar repetitive motor behaviors (e.g., rocking, spinning, hand f1apping)
Insistence on sameness, inf1exible adherence to routine
Fixation or fascination with specific interests or objects (e.g., vacuum cleaners, sprinklers)
Hyperreactivity or hyporeactivity to sensory input

i C:C:: I 1
5. ADHD medications
• Stimulants
Methylphenidate (Ritalin , Concerta)
Dexmethylphenidate (Focalin)
Mixed amph etamines (Adderall)
Dextroamphetamine (Dexedrine)
• Norepinephrine reuptake inhibitor
- Atomoxetine (Strattera)
• a 2 -agonists
- Clonidine
- Guanfacine

6. Hair-pulling disorder (trichotillomania)


• Compulsive, nervous hair pulling
• More common in young girls
• Examination reveals unusual patterns of broken hairs of varying length
• Treatment: education ~ cognitive behavioral therapy ~ fluoxetine or clomipramine

End of Session Qui z


7. . What name is given to chronic, compulsive hair pulling common in young girls?

I
8. What stimulant medications are commonly used in the treatment of ADHD?
u
>-
en
0...
0
_J

I
u

9. Which childhood psychiatric disorder matches each of the following statements?


• Impairments in social interactions/communication/ play; repetitive behaviors
• Ignoring the basic rights of others
• Characterized by hostility, annoyance, vindictiveness, disobedience, and resentfulness
• Multiple motor and vocal tics
• Impulsive and inattentive
• 7-year-old that avoids going to school to stay home with parent

RAP/ D- FI RE FACTS
I Most common medication for ADHD

r c; s? 1
EGO DEFENSE MECHANISMS AND PERSONALITY DISORDERS
H: Chapter 391
Ego defense mechanisms (FA15 p504-505) (FA16 pSOB-509) (SU p59)
Personality disorders (FA15 p515) (FA16 p519) (SU p60) (H p3542)

Warm -U p Rev1ew
1. How do cytotoxic T cells kill.virus;.infected cells and neoplastic cells? (FA15p202!
(FAl6p194)

rn
Cl
0
0
,rn
rn
2. A deficiency of which complement proteins may lead to.recurrent Neisseria :z
(/)
infections?.(F.A.1S {)206) {FA16 p199) (SU p371) rn
:s:
rn
n
I
)>
:z
(/)
5
(/)
)>
:z
3. What medications are used to treat an acute gout exacerbation? (FA15 p430) (FA16 p431) 0

(SU p233-234) -u
rn
;;o
(/)
0
:z
)>
,-
---1
-<
0
(/)
0
;;o
0
rn
;;o
(/)

r ss3 1
En of Sess1on Quiz
4. What defense mechanism fits each of the following statements?
• Voluntarily choosing not to think about a piece of bad news
• Not acknowledging a piece of bad news, as though it were never said
• Involuntary withholding of a feeling from conscious awareness
• A veteran who can describe horrific war details without any emotion
• A belief that a person or thing is either all good or all bad
• Man yells at his family when he has a bad day at work
• 8-year-old girl who is being bullied in school starts carrying around her security blanket
again

(/")
5. A 30-year-old woman tells you during one of her office visits that you are the best
a: doctor she's ever had but that your nurse is very disrespectful. On a subsequent
UJ
0
a: visit, she threatens to change doctors because you do not feel a particular lab test
0
(/") is necessary. Additionally, you notice several symmetrical cuts on her left forearm
0
)-
which she attributes to cat scratches. What type of personality disorder does this
f---
_j
person have?
<(
z
0
(/")
a:
UJ
o_

0
z
<(
(/")

~
6. A 55-year-old woman comes to your office wearing all black, including a black
(/)

z
miniskirt and a black feather boa. She is also wearing an excessive amount of lipstick,
<(
I
and you notice her having conversations with manyof the other patients in the
u waiting room. What type of personality disorder do you suspect in this patient?
UJ
~
UJ
(/")
z
UJ
LL
UJ
0
0
\.'J
UJ
7. A 32-year-old man is brought to the clinic by his mother who complains that he never
leaves the house. He has no friends and rarely interacts with anyone. He denies
any depression and is not bothered by his isolation. What is the likely personality
disorder?

! 5541
ALCOHOL ABUSE
Substance use disorder (FA15 p517) (FA16 p521)
Stages of change (FA15 p517) (FA16 p521)
Alcohol (FA15 p518) (FA16 p522)
Delirium tremens (FA15 p519) (FA16 p524)
Alcoholism (FA15 p519) (FA16 p523) (SU p52) (H p3549)
Wernicke-Korsakoff syndrome (FA15 p519) (FA16 p523)
Nlallory-Weiss syndrome (FA15 p519) (FA16 p523)
Long-term treatment of alcoholism

Warm -U Rev1 ew
1. Which type ofinfection is a patient with IL-12 receptor deficiencyat highest risk?
(FA1S p214) (FA16 p208)

2. What drugs can be used in the treatment of chronic gout? (FA15 p446)(FA16 p431) (SU p233)

3. What arethe classic symptoms of Sjogren syndrome? (FA15 p 430) (FA16 p432) (SUp236) )>
r-
n
0
I
0
r-
)>
ClJ
c
Ul
m

[ 555]
4. Patient Profile: The Confabulating Drunk
A 52-year-old man presents to the emergency department disheveled and confused. He has difficulty
answering questions and believes that he's been in Russia for the past several days. He is ataxic and
on exam has horizontal nystagmus, ophthalmoplegia and sluggish pupillary reflexes . His blood pressure
is 95/48 mm Hg and his heart rate is 110/min. Laboratory examination reveals: glucose 65 mg/dl,
creatinine 1.2 mg/dl, aspartate transaminase 420 IU/L, alanine transaminase 150 IU/L. His drug
screen is positive for opiates, alcohol and benzodiazepines. The patient is given intravenous fluids with
glucose, which promptly results in the patient becoming comatose. What treatment would most likely
lead to improvement of his symptoms?

Wernicke encephalopathy:
• Confusion
• Nystagmus
• Ophthalmoplegia
• Ataxia
• Sluggish pupillary reflexes
• Coma and death if untreated

Korsakoff syndrome:
• Anterograde amnesia (inability to form new memories)
• Retrograde amnesia (loss of existing memories)
• Confabulation (false perceptions or memories)
• Hallucinations

UJ Summary:
V)
:::::> • Wernicke-Korsakoff is often seen in alcoholics
co
<(
• Symptoms include: confusion, ataxia, eye symptoms, memory problems
---'
0
I • Due to thiamine deficiency
0
u • Replace thiamine before glucose
---'
<(

5. What medications are effective in helping to prevent relapse in recovering alcoholics?


• is the tried-and-true best relapse prevention



• Acamprosate (Campral)

r 5561
End of Sess1on QUIZ
6. What are the stages of behavioral change?

7. What is the mechanism of action of disulfiram?

8. When does delirium tremens occur?

)>
,--
()
0
I
0
,--
)>
CD
c
Ul
rn

RAPID - FIRE FACTS


Medical treatm ent for alcohol withdrawa l

Most effective treatment for alcohol abuse

Atrophy of the mammi llary bodies

[ 557]
SUBSTANCE ABUSE
H: Chapter 393
GG: Chapters 18, 24

Barbiturates and benzodiazepines (FA15 p518) (FA16 p522) (SU p52-53)


Opioids (FA15 p518) (FA16 p522) (SU p53) (H p3552) (GG p658)
Opioid analgesics (FA15 p494) (FA16 p499) (SU p244) (GG p481)
Amphetamines and cocaine (FA15 p518) (FA16 p522) (SU p53) (GG p661)
Caffeine and nicotine (FA15 p518) (FA16 p522) (SU p53) (GG p657)
PCP (FA15 p519) (FA16 p523) (SU p53) (GG p666)
LSD (FA15 p519) (FA16 p523) (SU p53) (GG p665)
Marijuana (FA15 p519) (FA16 p523) (SU p53) (GG p663)
MDMA (FA16 p519)

Warm - Up Rev1 ew
1. What are some common triggers of an acute gout exacerbation? (FA15 p430) (FA16 p431)
(SU p233)

2. What are the seronegative spondyloarthropathies? (FA15 p432) (FA16 p433)

UJ
Vl
=>
aJ
<(
UJ
u
z
;::
Vl
m
=>
<.n 3. Which receptors found on gastric parietal cells regulate acid secretion? (FA15 p347)
(FA16 p351)

[ 558 J
End Session Qu iz
4; What drug intoxication or withdrawal is causing the following symptoms in each of
the following patients?
• post-op constipation and/or respiratory depression
• Severe depression, headache, fatigue, insomnia/hypersomnia, hunger
• Pinpoint pupils, nausea/vomiting, seizures
• Belligerence, impulsiveness, nystagmus, homicidal ideation, psychosis
• Headache, anxiety/depression
• Anxiety/depression , delusions, hallucinations, flashbacks
• Euphoria, social withdrawal, impaired judgment, hallucinations
• Rebound anxiety, tremors, seizures, life threatening
• Anxiety, piloert:;ction, yawning,fever,rhinorrhea, nausea, diarrhea

5. What is the treatment for a patient that is overly intoxicated with each of .the
following substances?

Alcohol

Benzodiazepines

PCP
Vl
c
CD
Vl
):;!
Qpioids :z
n
m
)>
CD
c
Vl
m

r .ss9 1
STUDENTS OFTEN FIND

RENAL NEPHROLOGYAND RENAL


PHYSIOLOGY DAUNTING
AND MAY THEREFORE
1 Renal Embryology and Anatomy
NEGLECT THESE
TOPICS DURING EXAM
2 Glomerular Physiology
PREPARATION. BETWEEN
THE MANY GLOMERULAR
3 Nephron Physiology
DISEASES, ELECTROLYTE
DISTURBANCES AND
4 Druretics
ACID - BASE PHYSIOLOGY,

5 Electrolyte Drsorders MASTERY OF THE KIDNEY


CAN HELP DILIGENT

6 Acrd - Base Drsorders STUDENTS DISTINGUISH


THEMSELVES.
7 Nephrrtrs

8 Nephrosrs

9 Renal Fa ilure

10 Other Urrnary Pat hology


RENAL EMBRYOLOGY AID AIATOMY
R: Chapter 20
Phys: Chapters 25-26
GG: Chapter 25
Renal embryology (FA15 p526) (FA16 p530) (SU p143)
Potter sequence (FA15 p526) (FA16 p530) (SU p144)
Horseshoe kidney (FA15 p527) (FA16 p531) (SU p144)
Glomerulus (FA15 p528) (FA16 p532) (SU p145)
Collecting system and ureters (FA15 p528) (FA16 p532)

Warm ~ Up Review
1. · What are the manifestations of CREST scleroderma?~ (FA15 p436) (FA16 p437) (5Up236)

2. What is the mechanism of action of the penicillin drugs? (FA15 p1BO) (FA16 p171) (SU p337)

;;o
rn
z
)>
r-
rn

3. Which embryologic structure of the heart gives rise to each of the following adult s:
OJ
;;o
structures? (FA15 p26B) (FA16 psz62) -<
0
r---
• Smooth parts of the left and right ventriCles 0
• Smooth part ofthe right atrium G>
-<
)>
• Trabeculated left and right atria z
0
• Trabeculated parts of the left and right ventricles )>
:z
~
0
:s
-<

[ 563 J
End of Sess1on Ou1z
4. What artery prevents a horseshoe kidney from ascending in the abdomen?

5. What fundamental problem creates Potter sequence?

6. What cell type releases renin?

7. What are the three stimuli that induce renin release?

>-
:2:
0
~
::z:
<(
0
::z:
<(
>-
CJ
0
_j

0
>-
a::
ro
:2:
w
_j

<(
::z:
w
a::

r s64l
GLOMERULAR PHYSIOLOGY
R: Chapter 20
Phys: Chapter 25-26
GG: Chapter 25
Fluid compartments (FA15 p529) (FA16 p533) (SU p145)
Clearance and GFR (FA15 p529) (FA16 p533) (SU p146-147)
Renal plasma flow (FA15 p530) (FA16 p534) (SU p146)
Filtration fraction (FA15 p530) (FA16 p535) (SU p147)
Glomerular dynamics (FA15 p530) (FA16 p535) (SU p147)
Reabsorption and secretion (FA15 p531) (FA16 p535)
Clearance of glucose and amino acids (FA15 p531) (FA16 p536) (SU p147)

War -Up Rev1ew


1. Which penicillin drug would you use given each of the following infections?
• UTI
• -Neonatal infection
• Syphilis
• Pseudomonas

2. What are the different classes of antiarrhythmics? (FA15 p308 -310) (FA16 p302-304)

Cl
0'
:s
rn
AJ
c
')>
AJ
3. How will each of the following affect GFR, RPF, and filtration fraction? -u
I
GFR RPF FF (GFR/ RPF) --<
Vl
0
Constriction of the afferent arteriole
'0
Cl
Constriction of the efferent arteriole --<
Dilation of the afferent arteriole
Dilation of the efferent arteriole
Increase in serum protein
Ureter stone obstruction
ACE inhibitors
NSAIDs

[ 565 J
End of Session Qu1 z
4. A 40-year-old patient of yours weighs 100 kg. What is her estimated plasma volume?

5. What effect wm a renal stone that obstructs the ureter have on GFR and FF?

6. .At what glucose concentration is the tubular reabsorption of glucose maximized?

7. What vitamin deficiency results from Hartnup disease?

8. What substances can be used to estimate GFR? What substances can be used to
estimate renal plasma flow?

>-
~
0_ j
0
(f)
>-
6:: 9. What is the. equation for the renal clearance of any substance?
a::
<(
_j
:::>
a::
Ll.J
~
0
_j

r 5661
NEPHRON PHYSIOLOGY
Phys: Chapters 27-28
GG: Chapter 25
Proximal tubule (FA15 p532) (FA16 p537) (SU p149)
Relative concentrations along proximal tubule (FA15 p533) (FA16 p539)
Thin descending loop of Henle (FA15 p532) (FA16 p537) (SU p149)
Thick ascending limb (FA15 p532) (FA16 p537) (SU p149)
Early distal tubule (FA15 p532) (FA16 p537) (SU p149)
Collecting tubule (FA15 p532) (FA16 p537) (SU p149)

Warm -up \.Revi


1. What condition is suggestedby each of the following findings?
• Gout, intellectual disability, lipNting
• Anti-histone antibodies
• Renal failure + lytic bone lesions on x-ray

2. Streptococcus pyogenes produces which toxins? (FA15 p125) (FA16 p117)

3. What is HUS? (FA15 p138) (FA16p128)

z
m
"D
:r:
70
0
4. Proximal tubule (first half) z
"D
:r:
-<
(J)

0
HC~3- ~+ .----

a:~~~~::~
C02+H20
t 0
Cl
-<

Lumen lnten»titium

r r;r,71
5. Proximal tubule (second half)

c•----
Na+

Interstitium

c•----
Na+

6. Proximal tubule (anions and cations)


,--------------------------~--
organic anion

aIpha-ketoglutarate

I~

Interstitium

>-
(..:J
organic cation organic cation
0_J
0
Vl
>- 7. Thick ascending limb
:r:
a...
:z
0
cr:: - - - - - • K+ ~---...;......,~
:r: ~-----+ cl- --~
a...
w
:z

Interstitium

Mg++

r 568 1
8. What class of drugs inhibits the Na+/2CI-/K+ symporter in the thick ascending limb?

9. Early distal tubule

Interstitium

10. What two types of cells compose the collecting duct and the last segment of the distal tubule? What
do they do?
• Principa l ce lls

• Interca lated cel ls

11. What are the two types of intercalated cells? z


m
-a
I
;;o
0
z
-a
I
-<
(J)

0
'0
Cl
-<

r 569 1
12. Collecting tubule (principal cell)

Lumen Interstitium

13. Collecting tubule (intercalated cell)

Interstitium

>-
CJ
0_ j
0
(/)
>-
:r:
o_
z 14. What determines how much water is reabsorbed in the distal tubules and the collecting
0
0::: ducts?
:r:
o_
w
z

15. What class of diuretic directly affects principal cells?

16. What effect does aldosterone have on the principal cells and intercalated cells of the
collecting duct?
• Principal cells
• Intercalated ce ll s

r r:;7o l
17. Which segment of the renal tubule matches each ofthe following statements?
· • Reabsorbs 67% of the fluid and electrolytes filtered by the glomerulus
• .Site of secretion..of organicanions and cations
• Always impermeabl.e to water
• Permeable to water only in the presence ofADH
• Site of the Na+/2G / K+ co-transporter .
· · · Site of isotonicfluid reabswp~on
• SiteresponsibleJor diluting urine
• Only site where glucose and amino acids are reabsorbed
• Water reabsorption in the loop of Henle

18. What are the two main cell types of the collecting duct?

19. What drugs antagonize aldosterone's action on the principal cells of the collecting
duct, thereby promoting Na+ excretion and inhibiting K+ excretion?

20. What class of. drugs inhibitsthe Na+/2CI-tK+ symporter in the thick ascending limb?
z
rn
-o
:::r:
;;o
0
z
-o
:::r:
-<
Vl
0
'G)
0
-<

I 571 l
DIURETICS
GG: Chapter 25
Phys: Chapter 31
Acetazolamide (FA15 p553) (FA16 p557) (SU p165)
Mannitol (FA15 p553) (FA16 p557) (SU p165)
Loop diuretics (FA15 p553) (FA16 p557) (SU p165)
Thiazide diuretics (FA15 p554) (FA16 p558) (SU p165)
Potassium-sparing diuretics (FA15 p554) (FA16 p558) (SU p165)
Blood chemistry changes (FA15 p554) (FA16 p558) (SU p164)

Warm-Up Rev1ew
1. Which bacteria secrete enterotoxins?

2. What is Charcot's neurological triad? (FA15 p488) (FA16 p492)

3. Which organisms are most commonly implicated in subacute endocarditis?


(FA15 p298) (FA16 p293) (SU p87)

(f)
u
f=
w
0:::
::::l
0

r sn 1
nd of Session Ou1z
4. Which type ofdiuretic is each of the followingdrugs?
• Triamterene
• Acetazolamide
• Hydrochlorothiazide
• Bumetanide
• Spironolactone
• Ethacrynic acid
• Mannitol
• Metolazone
• Chlorthalidone
• Furosemide
• Amiloride .
• Torsemide

5. Which diuretic or class of diuretic would be most useful in each ofthe following
situations?
• Acute pulmonary edema
• . Idiopathic hypercalciuria (~ calcium stones)
• Glaucoma
• Mild tomoderate CHFvvith expanded ECV
• In conjunction with loop or thiazide diuretics to retain k+
• Edema associated with nephrotic syndrome
• Increased intratranialpfessure
• Mild to moderate hypertension
• Hypercalcemia
• Altitude sickness
• Hyperaldosteronism

What is the site of action of the thiazides?

CJ
c
;;o
_,
m

n
Ul
7. A patient with heart failure exacerbation needs medical diuresis but has a sulfa
allergy. Which diuretic can be used?

[ 573]
ELECIROLYTE DISORDERS
R: Chapter 20
Phys: Chapters 28-29
H: Chapters 45, 340

Endocrine function (FA15 p535) (FA16 p541) (SU p148)


5/ADH (FA15 p333) (FA16 p328) (SU p166, 184)
Diabetes insipidus (FA15 p333) (FA16 p328) (SU p184-185)
Clinical presentation of electrolyte disorders (FA15 p537) (FA16 p542)
Potassium shifts (FA15 p536) (FA16 p542)

Warm-Up Rev1ew
1. Which bacteria are obligate anaerobes and gram-positive, spore-forming rods?
p127) (FA16 p118)

2. Why are aminoglycosides ineffective against gram-positive anaerobes? (FA15 p184)


(FA16 p174) (SU p339)

3. Which part of the basal ganglia causes hypokinesia if lesioned?

(./)
0:::
w
0
0:::
0
(./)

0
l.J..J 4. Potassium shifts
f-
:::; K+ shift out of cells ~ hyperkalemia K+ shift into cells ~ hypokalemia
0
0::: • Low insulin • Insulin
f-
u
l.J..J • ~-blockers • ~-agonists
_j
w
• Acidosis • Alkalosis
• Digoxin • Cell creation/proliferation
• Cell lysis (e.g., leukemia)

r s74l
End of Sess ion ...Quiz
5. Which electrolyte disturbance fits each of the following presentations?
• Correcting too rapidly may result in central pontine myelinolysis
• Peaked Twaves
• Tetany
• Arrhythmias
• Oecreas@q deep tendon reflyxes
• Flattened T waves, lJ waves on ECG

6. What are the diagnostic features of diabetes insipidus?

7. What factors/substances cause hyperkalemia?

8. What factors/substances cause hypokalemia?

rn
,-
rn
n
---i
;;o
0
'---i
=<
rn
0
V>
0
:;;o
0
RAP/ O- F! RE FACTS rn
;JJ
V>

Treatment for central 01

Treatment for nephrogenic 01

Treatment for lithium-induced nephrogenic 01

r 575 1
ACID-BASE DISORDERS
R: Chapter 20
H: Chapter 47
Causes of acidosis and alkalosis (FA15 p538) (FA16 p543) (SU p150)
Renal tubular acidosis (FA15 p539) (FA16 p544)
Acid-base problems (FA15 p538) (FA16 p543) (SU p151)

Warm-Up Rev1ew
1. What are the toxic side effects of tricyclic antidepressant use? (FA15 p523) (FA16 p527)
(SU p56)

2. What are some of the characteristics of polymyositis that distinguish it from


polymyalgia rheumatica? (FA15 p434-435) (FA16 p435-436)

3. MPTP exposure causes depletion of which neurotransmitter? (SU p45)

(./)
a::
LLJ
0
a::
0
(./)

0
4. Normal gas values Normal ranges (use for the next question)
LLJ
(./) pH 7.35-7.45 pH 7.35-7.45
<!
OJ 35 -45 35 -45 mm Hg
0' > 90 (45 X 2 = 90) 75-105 mm Hg
u
<! 22 (45/2 = 22.5) 22-28 mEq/ L

r r:.7f. 1
5. For each set of ABG values, determine the acid-base disorder:
pH HC03 - pC0 2 Acid-Base Disorder
7.40 23 40
7.50 35 42
7.33 13 28
7.42 32 64
7.20 18 40
7.20 24 54
7.52 22 22
7.66 36 30
7.47 14 22
7.46 35 53
7.39 12 22
7.34 31 62
7.10 15 50
(Please note that the above values are only representative and may not accurately reflect valu es in
appropriate compensatory mechanisms.)

End of Sessi on QUiz


6. What are the. causes of acidosis with an elevated anion gap?

)>
n
CJ
7. For
. each set of ABG values, determine the acid-base disorder: '
o:J
)>
pH HC03 - pC02 Acid-Base Disorder (J)
m
CJ
7.24 24 53 (J)
0
7.58 34 28 :;u
CJ
m
7.23 19 38 :::0
(J)

7.33 30 58
...

8. A patient with a history of kidney stones presents with hypokalemia and metabolic
acidosis. The anion gap is normal. Urine pH is 5.7. What is the underlying defect?

f 5771
NEPHRITIS
R: Chapter 20
H: Chapter 283
Nephritis vs. nephrosis (FA15 p540) (FA16 p545) (SU p152)
Nephritis (FA15 p540-541) (FA16 p546-547) (SU p154)
Poststreptococcal glomerulonephritis
- lgA nephropathy
- A/port syndrome
Rapidly progressive glomerulonephritis
- Diffuse proliferative glomerulonephritis

Warm-Up Rev1ew
1. Which nerve ·runs with each of the following arteries?
• Anterior interosseous artery
• Posterior interosseous artery
• Posterior circumflex artery
• Suprascapular artery
• Thoracodorsal artery
• Deep brachial artery
• Dorsal scapular artery
• Lateral thoracic artery
• Ulnar artery
• Brachia l artery

2. What CSF changes are seen in Guillain-Barre syndrome? (FA15 p4BB) (FA16 p493) (SU p47)

(f) 3. What conditions are associated with arash that involves the palms and soles?
f-
a:
:r:
Q_
Ll.J
z:

r 5781
d of Session Qu1z
4. A teenager presents with episodes of gross hematuria and high-frequency hearing
loss. What is the most likely diagnosis?

5. An8-year""old girl presentswith abdominal pain, hip and knee pain, and purpura on
the backs of the arms and legs. Urinalysis shows microscopic hematuria and mild
proteinuria. What is the ..most likely diagnosis?

6. Which glomerular disease would you suspect most in a patient with each of the
following findings?
• Linear pattern of lgGdeposition on IF
• Lumpy7bumpy depQsits of lgG, lgM, and C3 .in the mesangium
• Deposits of lgA in the mesangium
• Anti -GBM antibodies, hematuria, hemoptysis
• Crescentformation in th e glomeru li
• Wire loop appearance on LM

z
rn
u
I
;;o
-i
(.f)

r t::79 l
NEPHROSIS
R: Chapter 20
H: Chapter 283
Nephrosis (FA15 p542-543) (FA16 p548-549) (SU p152)
Minimal change disease
Focal segmental glomerulosclerosis
- Membranous glomerulonephropathy
- Membranoproliferative glomerulonephritis (FA15 p541) (FA16 p547) (SU p154)
- Diabetic nephropathy
- Amyloidosis

Wa rm - Up Revi ew
1. Which nerve is most at risk of being damaged in a fibula neck fracture? (FA15 p421)
(FA16 p422)

2. Which two muscle receptors are responsible for opening the sarcoplasmic reticulum
in response to depolarization? (FA1S p423) (FA16 p424)

3. What are the features of congenital syphilis? (FA1S p41) (FA16 p131) (SU p33S)

(f)
(f)
0
a:
I
0...
l.U
z

r r.::An 1
End of Session Ou1z
4. What are the defining features of.nephrotic syndrome?

· 5. Glomer1.1lar histology reveals.multiple mesangial nodules. This lfesion is incl.icative ()f


what disease? ·

6~ Which glotr'f:!rula.r disease woulg you suspect most in .a patientwith each of the
following findings?
• Most common nephrotic syndrome in chi ldren
• Most common nephrotic syndrome in adults
• EM: effacement of.epithelial foot processes
• Nephrotic syndrome associated with hepatitis B
• Nephrotic syndrome associated with H IV
• EM: subendothelial humps and tram -track cjppearance
• LM: segmentaLsclerosis and hyalinosis
• Purpura on backs of arms and legs, abdominal pain, lgA nephropathy
• Apple-green birefringence with Congo red stain under polarized light
• EM: spiking of the GBM due to electroncdense subepithelial deposits

z:
rn
-u
:::r::
;;o
0
(J)
l/)

r ss1 1
RENAL FAILURE
R: Chapter 20

Acute kidney injury (FA15 p550) (FA16 p553) (SU p158-159)


Results of renal failure (FA15 psso) (FA16 p553) (SU p160-161)
- Renal osteodystrophy (FA15 p550) (FA16 p553)
Acute tubular necrosis (FA15 p549) (FA16 p554) (SU p159)
Acute interstitial nephritis (FA15 p548) (FA16 p554) (SU p159)
Renal papillary necrosis (FA15 p549) (FA16 p554)
Diffuse cortical necrosis (FA15 p548) (FA16 p552)
Urinary casts (FA15 p539) (FA16 p544)

Warm -Up Review


1. Which antibiotics inhibit prokaryotic protein synthesis at the 305 ribosomal subunit?
At the 505 ribosomal subunit? (FA15 p184) (FA16 p174) (SU p339)

2. What are the criteria for the diagnosis of a major depressive episode? (FA15 p511)
(FA16 p515) (SU p55)

ll.J
0:::
:::>
_J

<(
'-'--
-'
<(
z
ll.J
0:::

r 582 1
3. Classification of acute kidney injury (AKI)
Prerenal azotemia Intrinsic renal diseases Postrenal azotemia

4. Fractional excretion of sodium (FEN.)

UNa x SCr
FENa(Ofo) = 100 x SNa X UCr

Prerenal azotemia Intrinsic renal disease


Urine Na+ < 20 mEq/L > 40 mEq/L

FEN a < 1% >2%


BUN:creatinine ratio

5. Results of renal failure


Functions of healthy kidneys Kidney failure ;::o
rn
• Fluid balance • Volume overload :z:
)>
• Electrolyte balance • _ _ _ _ _ _ _ _ _ _ _, hyperphosphatemia r-

• pH balance • Metabolic acidosis >'


r-
c
• Vitamin D activation • Metabolic bone disease (renal osteodystrophy) ;;a
rn
• Erythropoietin production • Anem ia
• Nitrogen (urea) excretion •
- Anorexia
Nausea/vomiting
- Skin changes
Pericarditis
Platelet dysfunction
Encephalopathy
• Hypertriglyceridemia

r ss3 1
6. Causes of acute tubular necrosis (ATN)
• Ischemia
Prolonged hypotension
Sepsis
Blood loss
Surgery
• Nephrotoxins
_ _ _ _ _ _ __ (e.g., aminoglycosides, cisplatin)

Myoglobin(_ _ _ _ _ _ _ _ _ _ __
Synthetic cannabinoids

7. Acute interstitial nephritis (AIN)


• Classic presentation:
Fever
E_____________
- Azotemia
Rash
• Most common cause is drugs:

Penicillins and cephalosporins


Ciprofloxacin
Rifampin
Sulfonamides (e.g. , TMP-SMX)

Proton pump inhibit ors


Cimetidine
- Allopurinol

8. Renal papillary necrosis


• Ischemia ~ necrosis and sloughing of renal papillae
<( • Presentation:
LL
_j Gross hematuria +/ -flank pain
<(
z - A zotemia
w
~
Hypertension
• Causes:

- Analgesics (acetaminophen, possibly NSAIDs)


Diabetes
Sickle cell disease
9. In which diseases is each of the following types of urinary casts commonly seen?
• Hyaline casts
• RBC casts
• WBC casts
• Epith elial cell cast
• Granular casts
• Fatty casts
• Waxy casts

d Session 0
10. What is the classic presentation of acute interstitial nephritis (AIN)?

11. What drugs are known to causeAIN?

;:o
12. What changes will be seen in a basic metabolic panel in a patient with renal failure? rn
z
)>
,--
-,-,
)>
,--
c
;o
rn

r 585 1
OTHER URINARY PATHOLOGY
R: Chapter 20
Pyelonephritis (FA15 p548) (FA16 p552) (SU p157)
Renal cysts (FA15 p551) (FA16 p555) (SU p162-163)
Nephrolithiasis (FA15 p544) (FA16 p549) (SU p162)
Hydronephrosis (FA15 p545) (FA16 p550)
Urinary tract tumors
- Renal cell carcinoma (FA15 p545) (FA16 p549) (SU p163)
- Renal oncocytoma (FA15 p546) (FA16 p550)
- Wilms tumor (FA15 p546) (FA16 p551) (SU p163)
Transitional cell carcinoma (FA15 p547) (FA16 p551)
- Squamous cell bladder cancer (FA15 p547) (FA16 p551)

Warm-Up Review
1. What are the potential side effects of lithium? (FA15 p522) (FA16 p526) (SU p5B)

2. Which enzyme matches amino acids to tRNA? (FA15 p70) (FA16 p56)

>- 3. Which drug intoxication or withdrawal is the most likely cause of each of the
\.:)
0_.J following sets of findings? (FA15 p51B-519) (FA16 p522-523)
0 • Anxiety, depression, delusions, halluCinations, flashbacks
:r::
~ • Belligerence, impulsiveness, nystagmus, homicidal ideation, psychosis
o__

>- • .· Pinpoint pupils, nausea, vomiting, seizures


0:::
<r: • Severe depression, headache, fatigue, insomnia or hypersomnia, hunger
z
0:::
:=J
~
LLJ
:r::
f-
0 4. Pyelonephritis
• Inflammation/ infection of the renal cortex and interstitium
• Presentation:
Possible dysuria and frequency
Fever and chills , nausea/vomiting, flank pain , _ _ _ _ _ _ _ _ _ __ _ _
Urinalysis: pyuria , WBC casts
• Chronic pyelonephritis is associated with recurrent acute pyelonephritis, kidney stones or
vesicoureteral reflu x

[ 586 J
5. Renal cysts
Autosomal dominant polycystic kidney disease (ADPKD)
• Typically presents in adulthood
• Mutation in the gene _ _ _ _ _ _ _ _ _ (chromosome 16) or PKD2 (chromosome 4)
• Numerous bilateral renal cysts with massive enlargement of the kidneys
• Presentation :

Progressive rena l insufficiency ~ ESRD


Hemorrhage into th e cysts ~ flank pain +/- hematuria
• Increased risk of kidney stones and UTI
• Associated w ith hepatic cysts, _ _ _ _ __ _ _ _ _ _ _ _ , mitral valve prolapse

Autosomal recessive polycystic kidney disease (ARPKD)


• Mutation in the gene PKHDl (chromosome 6)
• Typically presents in _ _ _ _ _ _ _ _ _ _ _ (or diagnosed on prenatal ultrasound)
• Bilateral renal cysts, enlarged and echogenic kidneys
• Presentation
Oliguria ~ ~ Potter sequence
Hypertension, renal insufficiency, hepatomegaly

Autosomal dominant tubulointerstitial kidney disease (AKA medullary cystic kidney disease)
• Autosomal dominant inheritance
• _ _ _ _ _ _ _ _ _ _ _ __ _ _ and interstitial fibrosis
• Rarely produces cysts in the renal medulla
• Progressive rena l failure

6. Nephrolithiasis
0
---1
Stone content Associations X-ray Crystal shape Prevention :r:
m
;:u
• "envelope" c
;:u
• Hyperoxaluria
z

~
- ingestion )>
Calcium oxalate -
;;o
-<

.• Hypocitraturia
ingestion
-o
2:j
:::r::
0
,-
Urease-positive bacteria Radiopaque "coffin lid" Eradicate 0
Cl
• stag horn infection -<

~ ~
(NH 4 MgP0 4 ) • Klebsiella calculi
• S. saprophyticus

• Hereditary impairment of Radiolucent

0
cystine reabsorption ~ staghorn
Cystine cystinuria calculi
• Presents in chi ldhood
• Hyperuricemia and gout Radiolucent rhombus or rosette Allopurinol

Uric acid
[J

r 587 !
7. Renal cell carcinoma
• Most common primary renal malignancy
• Arises from PCT in the cortex
• Risk fact ors: men ages 50-70, smoking, obesity, hypertension
• Deletion on chromosome 3
May arise sporadically or as part of von Hippei-Lindau disease
• Presentation:
Often asymptomatic; usually found incidentally
Classic triad (flank pain , hematuria, palpable abdominal mass) seen in < 10% of cases
Paraneoplastic syndromes ( , anemia, hypercalcemia)
• Pathology: solid tumor with "clear cells" full of lipids/carbohydrates

8. Wilms tumor (nephroblastoma)


• Most common renal malignancy of early childhood (2-4 years)
• Presentation:
Palpable abdominal/flank mass
Possible abdominal pain or hematuria
• Mutation of tumor suppressor gene _ _____ or WT2 (chromosome 11)
• WAGR complex
- Wilms tumor

Genitourinary malformations
mental Retardation (intellectua l disability)

9. Transitional cell carcinoma (urothelial carcinoma)


>- • Most common malignancy of the urinary tract
CJ
0
_j
• Can occur in the renal calyces or pelvis, ureters, and urinary bladder
0
I • Presentation: _ _ _ _ _ _ _ _ _ _ __
~
o_ • Major risk factors:
>-
0:::
<(
z Aniline dyes
0:::
::::J Naphthylamine dyes
0::
w
I
f--
0

r sea 1
nd Se ss1 on Ou 1z
10. What is the WAGR complex?

11. What are the risk factors for urothelial cell carcinoma?

12. What genitourinary pathology fits each of the following descriptions?


• Most common malignancy of the urinary tract
• Most common renal malignancy of early chi ldhood (ages 2-4)
• Most common type of rena l stone
• Type ofrenal .stone associated with Proteus vulgaris
• Most common primary renal tumor in adults

0
--j
I
m
;:o
c
;:o
:z
)>
;:o
--<
-o
~
I
0
,--
0
Gl
-<

r 5891
0/SE.ASES AN

PEDIATRICS DISORDERS CHILDREN


ARE ADDRESSED IN EVERY
SECTION OF THE COURSE
1 Ped1atr1c Rev1ew THi5PfDIATRIC REVIEW
COVERS ISSUES OF THE
NEONATAL PERIOD AND
EARLY CHILDHOOD NOT
PRESENTED ELSEWHERE
PEDIATRIC REVIEW
Apgar score (FA15 p59) (FA16 p581) (SU p211)
Neonatal infections (FA15 p172-173) (FA16 p162-163)
Low birth weight (FA15 p59) (FA16 p581)
Sudden infant death syndrome
Developmental milestones (FA15 p59) (FA16 p43) (SU p219)
Tanner stages (FA15 p574) (FA16 p584)
Ethics- consent for minors (FA15 p56) (FA16 p40) (SU p22)
Neoplasms in children

Warm -Up Review


1. When a young monkey is separated from its mother, it becomes withdrawn, socially
isolated, and grooms poorly. It is thought that this behavior is the mcmkey equ~yalent
·of whathumcm problem? (FA15 p506)(FA16 p510)

2. What are the symptoms of anticholinergic toxicity? (FA15 p251) (FA16 p245)

3. Sudden infant death syndrome (SIDS)


• Usually occurs at 2-4 months old
• Usually occurs whi le infant is sleeping
• Maternal risk factors: low socioeconomic status, age < 20, drugs or cigarettes during pregnancy
"D
• Infant risk factors: low birth weight, male, prematurity, prone sleeping position, m
CJ
prior sibling with SIDS
~
• Preventive measures: "Back to Sleep" campaign, avoid co-sleeping and smoke exposure, :::0
n
breastfeeding, use a pacifier w hen sleeping, fan in the room :::0
m
<

r .'i9l 1
4. Complete the following chart of developmental milestones:
Verbal/ Social/
Gross motor Cognitive Fine motor Self-care

3m Rolls over Hands together

6m

9m

12m 1-3 words Drinks from cup

15m Walk backward, run

18m 4-cube tower

2y Jump up Half understandable Wash/ dry hands

3y

4y

Copies square and


Sy
triangle

sw
>
w
0::: 5. A mother presents with her 1-year-old child that can stand alone, has just learned to walk, and has a
u S-word vocabulary. She would like to know if her child is developmentally normal and when she can
cr::
1-
<( begin toilet training.
0
w
0...

r 5941
6. A 2-year-old child speaks in short sentences of 2-3 words but cannot identify colors or recite his
ABCs. The mother is concerned that he is not developing normally. What do you tell the mother?

7. A girl can speak in complete sentences, has an imaginary friend, and considers boys to be "yucky."
How old is she?

8. Tanner stages

Stage Boy genitalia development Girl breast development Pubic hair

1 Prepubertal Prepubertal Prepubertal

Bud with elevation of


Enlargement of scrotum Sparse, long, slightly
2 breast and papilla; areola
and testes pigmented hair
enlarges

Enlargement of penis Darker, coarser, and more


3 Further enlargement
(length first) curled

Penis - growth in breadth


Areola and papilla form a
and development of glans Adult hair in type but covering
4 secondary mound above
Testes - enlarge a smaller area
the level of the breast
Scrotum - larger and darker

Mature - only papilla


5 Adu lt Adult in type and quantity
projects as areola recedes

Female development: Breast development (10.5) ~ Growth spurt (12) ~ Menarche (13)
Male development: Stage 2 (12) ~Growth spurt (14-15)

-o
9. Neoplasms in children m
0
• ALL • Retinoblastoma
)>
---1
;;o
• Astrocytoma • Rhabdomyosarcoma n
:::0
• Neuroblastoma
• Ewing sarcoma
m
<
• Hemangioma • Osteogenic sarcoma
m
:2:
• Wilms tumor • Lymphoma (lymphoblastic)
• Hepatoblastoma • Teratoma
/~

I '19'i l
of Sess1 QUIZ
10. What factors are taken into consideration when giving a newborn an Apgar score?

11 .. What is the•definition of.low birth weight? What complications are associated with
tow birth weights?

12. Based on the milestones given, how old is each of the foltowing children?
• Jumps up, 6-cube tower, eats with spoon, 2-3 word sentences
• Regards face, responds to sound, not yet able to roll over
• Stands with support, 1-3 words, stranger anxiety, drinks from a cup
• Rides tricycle, understandable sentences, plays board games

s
w
>
w
0:::
u
0:::
~
0
w
o_

r S9h 1
THE ENDQCRINOLOGY

ENDOCRINE QUESTIONS ON
BOARD EXAMS
TEND TO BE FAIRLY
1 Endocrme Overv1ew STRAIGHTFORWARD I

BUT THERE IS A WI
2 Pituitary
AND .EDMPLEX VARIETY
OF SYSTEMS TO
3 Adrenal Steroid Synthesis
UNDERSTAND. PITUITARY,
ADRENA THYROID
4 Cushing Syndrome - ;~ I

RATHYR016S, AND THE


ENDOCRINE PANCREAS.
5 Hyperaldosteronism and
PAY SPECIAL ATTENTION
Adrenal Insufficiency
TO DIABETES MELLITUS,
6 Ad renal Medulla A DISEASE WITH A HIGH
PREVALENCE IN BQTH
7 Hypothyroid 1sm THE CLINIC AND ON THE
EXAM.
8 Hyperthyroid ism

9 Thyro1d Cancer

10 Diabetes

11 Diabetic Emergenc ies

12 Diabetes Drugs

13 Obes1ty

14 Calc1um Metabolism
ENDOCRINE OVERVIEW
Phys: Chapter 74
H: Chapter 338
Signaling pathways of endocrine hormones (FA15 p321) (FA16 p316) (Phys p886) (H p2869)
Signaling pathways of steroid hormones (FA15 p321) (FA16 p316) (Phys p891)
Review of hormone actions and origins (Phys p883)

Wa rm -Up Review
1. What are the acute phase cytokines? (FA15 p207) (FA16 p200)

2. What agents are used in the treatment of Parkinson disease? (FA15p500-501)


p504-505)

3. What are the tumor markers for pancreatic cancer? (FA15 p237) (FA16p373! .

4. What hormone has each of the following actions?


• Stimulates bone and muscle growth
• Stimulates milk production
• Stimulates milk secretion during lactation
• Responsible for female secondary sex characteristics
• Stimulates metabolic activity
• Increases blood glucose level and decreases protein synthesis
• Responsib le for male secondary sex characteristics

• Prepares endometrium for implantation/ maintenance of pregnancy


• Stimu lates adrenal cortex to synthesize and secrete cortisol
• Stimulates follicle maturation in females and spermatogenesis in males
• Increases plasma calcium, increases bone resorption m
z
0
• Decreases plasma calcium, increases bone formation 0
n
• Stimulates ovulation in females and testosterone synthesis in males ;J:J
z:
• Stimulates thyroid to produce TH and uptake iodine m
0
<
m
AJ
<

r c;qo 1
5. From where is each of the following hormones secreted?
• Growth hormone (GH)
• Thyroid hormone
• Glucocorticoids (cortisol)
• Progesterone
• Prolactin
• Oxytocin

• Atria l natriuretic hormone (ANH)


• Glucagon
• Testosterone
• Follicle-stimulating hormone (FSH)
• Vasopressin (ADH)
• Ca lcitonin

• Thyroid -sti mu lating hormone (TSH)


• Epinephrine and norepinephrine
• Insulin
• Estradiol
• Estriol
• Estrone

• Estrogen in males
• Parathyroid hormone (PTH)
• Somatostatin
• Luteinizing hormone (LH)
• Mineralocorticoids (aldosterone)
• Adrenocorticotropic hormone (ACTH)

En of Sess1on Qu1z
6. In what part of the cell would you find steroid hormone receptors?

7. What clinical finding would you expect to find in a man with high sex hormone
binding globulin?

s
w
>
0::
w 8. What clinical finding would you expect to find in a woman with low sex hormone
> binding globulin?
0
w
z
0::
u
0
0
z
w
PITUITARY
R: Chapter 24
Phys: Chapter 75
H: Chapter 339
Posterior pituitary (FA15 p313) (FA16 p307) (SU p171)
Antidiuretic hormone (ADH) (FA15 p317) (FA16 p311) (SU p166)
Oxytocin (SU p170)
Anterior pituitary (FA15 p313) (FA16 p307) (SU p171)
Luteinizing hormone (LH) (FA15 p315) (FA16 p309) (SU p169, 171)
Follicle-stimulating hormone (FSH) (FA15 p315) (FA16 p309) (SU p169, 171)
Adrenocorticotropic hormone (ACTH) (FA15 p315) (FA16 p309) (SU p169, 171)
Melanocyte -stimulating hormone (MSH) (SU p169, 171, 186)
Thyroid-stimulating hormone (TSH) (FA15 p315) (FA16 p309) (SU p169, 171)
Growth hormone (GH) (FA15 p317) (FA16 p311) (SU p169, 171)
Prolactin (FA15 p316) (FA16 p310) (SU p169, 171)
Hyperprolactinemia (FA15 p316) (FA16 p310)
Pituitary adenoma (FA15 p332) (FA16 p326) (SU p184)
Acromegaly (FA15 p333) (FA16 p327) (SU p184)
Somatostatin (FA15 p340) (FA16 p336) (SU p117)
Sheehan syndrome (FA15 p334) (FA16 p329) (SU p184)

Warm - Up Rev1ew
What are the two nervous tissue bundles that run through the Gl tract, and where are they
located? (FA15 p345) (FA16 p341)

What diseases specifically damage the anterior hom? (FA15 p471-472) (FA16 p474)

"""0
-1
c
~
;::o
-<

rw11
3. ACTH, MSH, and proopiomelanocortin (POMC)
• ACTH is synthesized as part of a large precursor called POrv1C
• POrv1C contains the sequences for other hormonal peptides including: lipotropin, rv1SH, and
~-endorphin
• In primary adrenal insufficiency:
Production of cortisol is low
Hypothalamus and pituitary make more ACTH, trying to stimulate the adrenals to make more
cortisol
- When ACTH concentration is excessive, rv1SH receptors in the skin are stimulated , resulting in
skin hyperpigmentation

Hypothalam~
··~.
.
fY
!

CRH \
\
Anterior Pituitary

,~ ACTH

Adrenal
Gland

4. Hyperprolactinemia
• Causes
Pregnancy/ nipple stimulation
Stress (physical or psychological)
Prolactinoma (associated w ith bitemporal hemianopia)
Dopamine antagonists: anti psychotics (haloperidol, risperidone), domperidone, metoclopramide
• Premenopausal female symptoms: hypogonadism ~ infertility, oligo/ amenorrhea, rarel y galactorrhea
• Postmenopausal female symptoms: none since already hypogonadal
• rv1ale symptoms: hypogonadism (low testosterone)~ decreased libido, impotence, infertility (low
sperm counts), gynecomastia, rarel y galactorrhea

?JYI 1
5. Somatostatin
~ • Produced in several places in the body: D cells in the Gl mucosa , pancreatic islet cells, and nervous
system
• Actions:
Reduces splanchnic blood flow, reduces Gl motility and gallbladder contraction, inhibits secretion
of most Gl hormones
Decreases exocrine secretion in the pancreas
Decreases hormone secretion in the CNS, PNS, and endocrine organs
• Clinical uses for somatostatin ana logs (octreotide, somatostatin LAR, and lanreotide-P):
Pituitary excesses: acromega ly, thyrotropinoma, ACTH -secreting tumors
Gl endocrine excess: Zollinger- Ellison syndrome, carcinoid syndrome, VIPoma, glucagonoma,
insulinoma
Certain diarrheal diseases
Need to reduce splanchnic circulation: portal hypertension (bleeding varices), bleeding peptic
ulcers

6. Sheehan syndrome
• Postpartum hemorrhage leading to underperfusion of the pituitary
• Pituitary necrosis and hypopituitarism
• Presentation:
Agalactorrhea due to a deficiency of prolactin
Amenorrhea after delivery
Secondary hypothyroidism leading to fatigue, cold intolerance, and weight gain
Hyponatremia (rare)
7. A patient's MRI reveals replacement of tissue in the sella turcica with CSF. What is the most
likely clinical presentation?

8. Which hormones share a common alpha subunit?

9. > Wttat is the most common presentation of hyperprolactinemia ina female patient?

of the possible clinical features >


of>acromegaly?

RA PID -FIRE FACTS


Inability to breastfeed, amenorrhea, cold
intolerance
Infertility, galactorrhea, and bitemporal
hemianopia

>-
0:::
~
:=J
f--
o_

'hfl4 l
ADRENAL STEROID SYNTHESIS
R: Chapter 24
Phys: Chapter 77
H: Chapter 342
Adrenal anatomy (FA15 p312) (FA16 p306) (R p1122) (Phys p921)
Fetal adrenal gland
Steroid synthesis (FA15 p318) (FA16 p312) (SU p186) (Phys p923) (H p2940)
Congenital adrenal hyperplasia (FA15 p318) (FA16 p312) (SU p185-186) (R p1127) (H p2959)

Warm -Up Rev1ew


1. An abdominal aortic aneurysm is most likely a consequence of what process?
(FAlS p292) (FA16 p286)

. 2. What is pulsus paradoxus, and what are the causes? (FAlS p299) (FA16 p294)

3. A study shows that high LDL does not increase the risk of CAD. Which type of error is this?
(FA15 p54) (FA16 p38)

)>
0
;:o
rn
z:
)>

'U1
-i
rn
;:o
0
0
U1
-<
z
--i
I
rn
(})
(})

r 60S i
4. Steroid synthesis pathway

5. What features characterize a deficiency in 3~-hydroxysteroid dehydrogenase (3~- HSD)?


• Inability to produce

• Excessive _ _ _ _ _ _ _ _ excretion in the urine


• Early death

6. What features characterize a deficiency in 17a-hydroxylase?


• Inability to produce sex hormones and cortisol ~ _ _ _ _ _ _ _ _ _ _ _ _ _ _ _ _ __
• Increased production ofmineralocorticoids ~sodium and fluid retention ~ _ _ _ _ _ _ _ _ _ __

7. What features characterize a deficiency in 21a-hydroxylase?


(J)
(J)
• Inability to produce cortisol ~ _ _ _ _ _ _ _ _ _ _ _ _ _ _ _ _ _ _ __
LLJ
:r:: • Inability to produce mineralocorticoids ~ _ _ _ _ _ _ _ _ _ _ _ _ _ __
f-
z • Increased production of sex hormones ~ _ _ _ _ _ _ _ _ _ _ _ _ _ __
>-
<J)

0
0
0::
LLJ
f-
(J)
_j

<r:
z
LLJ
0::
0
<r:

r 6061
8. What features characterize a deficiency in Up-hydroxylase?
• Inability to produce

• Increased production of deoxycorticosterone (a weak mineralocorticoid)~ _ _ _ _ _ _ __


• Increased production of sex hormones~-----------

End ..of Ses s1on Ou 1z


9.' What are the symptoms of 21a-hydroxylase deficiency?

10. What are the symptoms of 11(3:-hydroxylase deficiency?

11. What food substance is an essential starting point inthe synthesis of adrenal
steroids?

)>
0
;;o
m
z:
)>
r-
(J)
---1
m
;;o
0
0
(J)
-<
z
---1
:r:
m
(J)
(J)

[ 607 J
CUSHING SYNDROME
R: Chapter 24
Phys: Chapter 77
H: Chapter 342
Cortisol (FA15 p319) (FA16 p313)
Glucocorticoids (FA15 p340) (FA16 p336) (SU p189)
Cushing syndrome (FA15 p323) (FA16 p317) (SU p185) (R p1123) (H p2945)

Warm- Up Rev1ew
1. Which patient groups should avoid fluoroquinolones? (FA15 p187) (FA16 p178) (SU p341)

2. How is hnRNA processed before it leaves the nucleus? (FA15 p68) (FA16 p54)

3. A 4-year-old girl complains of pain in her genitalia. On exam, a discharge is noticed,


and a smear of the discharge shows Neisseria gonorrhoeae. How was she infected?
(FA15p506) (FA16 p510)

UJ
:?:
0
0:::
0
z
>-
c.n
l:J
z
I
c.n
::l
u

r 608 1
End of Session Quiz
4~ An adult man with elevated serum cortisol levels and signs of Cushing syndrome
undergoes a dexamethasone suppression test. 1 mg of dexamethasone does not
decrease cortisol levels, but 8 mg does. What is the diagnosis?

5. What eff~ctdoes cortisol have on bone formation and immune. system functioning?

6. What are the potential side effects of glucocorticoid use?

RAP/ o - F! RE FACTS

n
c
(J)
Most common causes of Cushing syndrome :r:
z
Cl
(J)
-<
z
CJ
;;o
0
:5:
m

f ()09 l
HYPERALDOSTERONISM AND ADRENAL INSUFFICIENCY
R: Chapter 24
Phys: Chapter 77
H: Chapters 342
Aldosterone (FA15 p534) (FA16 p540) (Phys p924)
Hyperaldosteronism (FA16 p318) (SU p185-186) (R p1125) (H p2949)
Adrenal insufficiency (R p1129) (H p2954)
- Primary: Addison disease (FA15 p324) (FA16 p318) (SU p186) (R p1130)
Primary: Waterhouse-Friderichsen syndrome (FA15 p324) (FA16 p318) (SU p186) (R p1129)
- Secondary (FA15 p324) (FA16 p318) (R p1131)
- Tertiary (FA15 p324) (FA16 p318)

Warm-U p Review
1. What is the difference between the following disorders? (FA15 p516) (FA16 p514)
• Schizotypal
• Schizophrenia
• Schizoaffective
• Schizoid
• Schizophreniform

2; What is the treatment for nephrogenic diabetes insipidus? (FA15 p333) (FA16 p328)

>-
u
z
LJ_J

u
l..L_
l..L_
~
lfl
z
_J
<(
z
w
0:::
0
<(
0
z
<(

z
lfl
z
0
0:::
w
f--
lfl
0
0
---'
<(
0:::
w
CL
>-
:r:
E Session Ou1z
3. What are the clinical manifestations of Addison disease? What is the cause of Addison
disease?

4. A30-year-old man is undergoing an evaluation for hypertension. He has a history of


multiple episodes of hypokalemia. Laboratory evaluation shows a potassium of3.0
mEq/l despite replacement, and metabolic alkalosis. What is the rnost likely cause of
hypertension in this patient?

5. A very tan child Vllith a pale mother comes to the clinic and is found to be hypotensive.
What is the most likely diagnosis?

~.

RAP/ O-F! RE FACTS


Most common cause of primary hyperaldosteronism

Medical treatment for hyperaldosteronism


I
-<
u
Adrenal disease associated with skin hyperpigmentation rn
;;o
,.
)>

0
HTN, hypokalemia, metabolic alkalosis 0
VJ
---1
rn
;;o
0
z:
VJ
s
)>
z:
0
)>
0
;;o
rn
z
)>

'
z:
VJ
c,
,
n
rn
:z:
n
-<

[ 61! J
ADRENAL MEDULLA
R: Chapter 24
H: Chapters 343, 351
Pheochromocytoma (FA15 p326) (FA16 p320) (SU p187) (R p1134) (H p2962)
Neuroblastoma (FA15 p325) (FA16 p319) (SU p187j
Multiple endocrine neoplasia (MEN) (FA15 p337) (FA16 p333) (SU p195) (R p1136) (H p3072)

rm - U Rev1ew
1. What are the stages of behavioral change? (FA15 p517) (FA16 p521)

Which causes of vaginal.discharge/vaginitis are associated with a high vaginaLpH?


Which are associated with a low vaginal pH? (FA15 p174) (FA16 p164)

<(
__j
__j

:J
0
LLJ
~
__j
<(
z
LLJ
CL
0
<(
End Sess 1on Qu 1z
3. · . What tumor locations are associated with the three different types of multiple
endocrine neoplasia?

4. A 34-year-old ·man comes to the physician for headaches with episodes of high
blood pressure. He says during the episodes he also feels like his heart is racing and
becomes sweaty. Evaluation for what neural crest tumor should be tested in this
patient?

What might a lab detect in the urine of a patient with pheochromocytoma?

RAPID - FIR E FACTS


Most common tumor of the adrenal

Most common tumor of the adrenal medu lla (in


adu lts)
Most common tumor of the adrena l medulla (in
children)

Medical treatment for pheochromocytoma

Pheochromocytoma, medullary thyroid cancer,


and hyperparathyroidism
Pheochromocytoma, medullary thyroid cancer,
and mucosal neuromas

)>
0
::0
rn
z)>
,-
s::
m
0
c
,-
,-
)>

r 613 1
HYPOTHYROIDISM
R: Chapter 24
Phys: Chapter 76
H: Chapter 347
Thyroid development (FA15 p312) (FA16 p306)
Formation of thyroid hormone (FA15 p322) (FA16 p315) (SU p190)
Hypothyroidism (FA15 p327-328) (FA16 p321-322) (SU p190)
Causes of hypothyroidism (FA15 p328) (FA16 p322) (SU p190)
- Congenital hypothyroidism
- Hashimoto thyroiditis
- Subacute thyroiditis
Riedel thyroiditis

Warm -Up Rev1ew


1. Which cell wall inhibitor matches each of the following statements? (FA15 plB0-183)
(FA16 p170~173)
• Next step in treatment of otitis media if amoxicillin resistant
• Prophylaxis against bacterial endocarditis
• Sufficientfor the treatment of syphilis
• Single-dose treatment for gonorrhea

2. What is the equation for determining the confidence interval? (FA15 p54) (FA16 p38)

3. What are the Kubler-Ross stages of grief? (SU p219)

:E
Ul
0
0
~
>-
I
1-
0
0..
>-
I

r 6141
End of Session Quiz
4. What is the most common location for ectopic thyroid tissue?

5. How would pregnancy affect serum thyroid hormone.levels?

6. What are the causes of congenital hypothyroidism? How can it be prevented?

7. A.~S-year-old woman has a diffuse painless goiter alld hypothyroidism. What is the
most likely diagnosis, and what are the most likely relative values of TSH and thyroid
hormones?

RAP/ O-F! RE FACTS :r:


-<
-u
I Cold intolerance
0
-I
I
-<
;;<J
0
0
Vl
:s:

r 615:
HYPERTHYROIDISM
R: Chapter 24
Phys: Chapter 76
H: Chapter 341
Hyperthyroidism (FA15 p327, 329) (FA16 p321, 323) (SU p191 -192)
Graves' disease (FA15 p329) (FA16 p323) (SU p191 -192)
Other causes of hyperthyroidism (FA15 p329) (FA16 p323)
Approach to thyrotoxicosis (FA15 p329) (FA16 p323) (SU p193)

Warm-Up Rev1ew
1. What questions are asked during the four clinical phases of drug development?
(FA15 p48) (FA16 p32)
• Phase I
• Phase II
• Phase Ill
• Phase IV

2. What are the common causes of restrictive cardiomyopathy? (FA15 p296) (FA16 p291)

2:
U)

0
0
0::
>-
:r:
f---
0::
LL.J
0...
>-
::r:::

[ 6! 6 J
3. Approach to thyrotoxicosis
Suspect thyrotoxicosis

~
Thyroid function tests
I

TSH low TSH normal


T/T4 high T/T4 normal

t
Iodine uptake
t
Consider menopause
thyroid scan panic attack
anxiety
I pheoch rom ocytom a

Low uptake Single hot Multiple Diffuse

~
nodule hot and uptake
cold nodules
· HxURI Excess
·Painful T/T4 dosing
tender
thyroid
gland
t
Subacute
l
Exogenous ··· Thyroid Multinodular Graves' disease
jhyroiditis thyrotoxicosis ' adenoma goiter . T$1
.•.Exophthalmos
. Thyroid bruit

End of Sess1on Quiz


4. A 35-year-old \NOm an has a diffuse goiter and hyperthyroidism. What are the most
likely relative laboratory values of TSH and thyroid hormones?

5. What is the most likely diagnosis in a patient with the symptoms of hyperthyroidism
in addition to each of the fol.lowing scenarios?
• Extremely tender thyroid gland
• Pretibial myxedema
• Pride in recent weight loss, medical professional
::r:
• Palpation of single thyroid nodule -<
-u
• Palpation of multiple thyroid nodules m
;:o
--i
• Recent study using IV contrast (iodine) ::r:
• Eye changes: proptosis, edema, injection -<
;:o
0
• History of thyroidectomy or radioablation of thyroid 0
(/)
:s

r Al71
THYROID CANCER
R: Chapter 24
H: Chapter 347
Thyroid cancer (FA15 p330) (FA16 p324) (SU p194)
- Papillary
- Follicular adenoma
- Medullary
- Anaplastic
Complications of thyroid surgery (SU p192)

Warm-U p Review
1. A tall, thin male teenager has abrupt onset dyspnea and left-sided chest pain.
Percussion on the affected side reveals hyperresonance; and breath sounds are
diminished. What is the diagnosis? (FA15 p615) (FA16 p623)

2; In a population of 100 workers that cleaned an oil spill on a beach, 10 develop


leukemia. In a population of 1,000 hotel beach umbrella monitors on a clean beach,
50 develop leukemia. What is the attributable risk? (FA15 p50)(FA16p34)

0:::
w
u
z
<(
u
0
0
0:::
>-
I
1-

I f-iR l
End of Sess1 n QUIZ
3. What type of cancer in the thyroid is associated with each of the following statements?
• Activation of receptor tyrosine kinases
• .Hashimoto thyroiditis is a riskfactor
• Arises from parafollicular C cells
• RAS mutation or a PAXS-PPARy-1 rearrangement
• Rearrangements in RET oncogene or NTRK1
• Most common mutation in the BRAF gene (serine/threonine kinase)

4. Wllat nerve canbe easily damaged in thyroid surgery and lead to hoarseness?

5. What is the most likely electrolyte abnormality in a patient with periorbital tingling,
periorbital paresthesias,and recent thyroidectomy?

RAPID-FIRE FACTS
Most common thyroid cancer

Enlarged thyroid cells with ground-glass nuclei

-i
:r::
-<
:::0
0
0
n
)>
z
n
m
;:o

r 6 19 1
DIABETES
Phys: Chapter 78
H: Chapter 344
Endocrine pancreas cell types (FA15 p313) (FA16 p307) (SU p174) (R p1105) (Phys p939)
Glucagon (FA15 p315) (FA16 p309) (SU p174) (R p1107) (Phys p947)
Insulin (FA15 p314) (FA16 p308) (SU p174) (Phys p939)
Diabetes mellitus (FA15 p334) (FA16 p330) (su p178) (R p1105)
Type 1 vs. Type 2OM (FA15 p335) (FA16 p331) (SU p179) (R p1109)
Sorbitol (FA15 p104) (FA16 p92)

W a rm ~
1.. What is the number needed to harm? (FA15 pSO) (FA16

2. Which class of antibiotics inhibits prokaryotic DNA topoisomerase? (FA15 p65)


(FA16 p178) (SU p281)

3. Insulin and glucose transport

ATP _____.:If'

Glucose
transporter
I Ca 2 + channel
Glucose +
+ (depolarization opens)
+

Insulin

(f)
LLJ
r-
LLJ
m
<(
0

r 620 1
n Sess1on Quiz
4. How is hemoglobin glycosylated in diabetes mellitus to form HgbA1c?

5~ Which type of diabetes mellitus fits the following descriptions?


• Associated with obesity
• May cause ketoacidosis
• Strong genetic predisposition
• Associated with HLA~DR3 and HLA~DR4

6. In which tissues will you find GLUJ-2 receptors?

Which tissues depend on insulin for glucose uptake?

CJ
)>
CD
m
-1
m
Ul

[ 62 1 l
DIABETIC EMERGENCIES
H: Chapter 344
GG: Chapter 43
Diabetic ketoacidosis (DKA) (FA15 p335) (FA16 p331) (SU p179) (H p2976)
hyperosmolar hyperglycemic state (HHS) (FA16 p332)

rm -U
What are some of.the charact(i!ristics of polymyositis that distingu is~ it from
polymyaigia rheumatica? (FA16 p435-436)

2. Which type of antipsychotic is often the first line of treatment for psychosis? Which
antipsychotic should be .reserved for severe,·refractory psychosis b.ecause of the risk
of agranulocytosis? (FA15 p521) (FA16 p525)

3. Common precipitating factors for DKA


• Undiagnosed diabetes
• Missed insulin doses
• Infection (pneumonia , gastroenteritis, UTI)
• Severe ischemic events (e.g., Ml , CVA)
• Trauma
• Dehydration
• Alcohol and drug (especially stimulants) abuse

(f)
w
u
z
w
(.9
[>"'
w
~
w
u
f---
w
m
<(
0

r 622
End ..of Sessron Quiz
4. What are three important components of OKA management? HHS management?

5. What are the common precipitating factors for OKA?

CJ
)>
OJ
m
---i
n
m
s
rn
;:o
Cl
m
z
n
rn
(f)

r 623 1
DIABETES DRUGS
H: Chapter 344
GG: Chapter 43
Diabetes treatment
Diabetes drugs (FA15 p338-339) (FA16 p334-335) (SU p181-182) (GG p11248) (H p2996)
Metformin
Sulfonylureas
Thiazolidinediones
- DPP-4 inhibitors
GLP-1 agonists
SGLT-2 inhibitors
Pramlintide

Warm~U p Rev1ew
1. How do acidosis and alkalosis affect extracellular K+ concentrations? (FA15p536)
(FA16 p542)

change takes place .in.the trachea .of a smoker?

3. ... What is Budd~Chiari syndrome? (FA15 p370) (FA16 p368)

4. Insulin formulations

.5
::s
VI
(/)
~
::J
0:::
-c

0
(/)
w
f-
w
m
<(
0
Hours

r 624
5. Diabetes drugs
Drug names Mechanism Side effects and risks
Biguanide Metformin • Decreases hepatic • Gl side effects common
gluconeogenesis • Increased risk of
• Improves insulin
sensitivity when used in rena l failure

Sulfonylureas • Glimepiride Stimulates continuous •


• Glipizide insulin re lease from • Weight gain
• Glyburide the~ cells

Thiazo lidinediones • Pioglitazone • Improves insulin • Weight gain


"-glitazone" • Rosiglitazone sensitivity • Fluid retention
• Decreases hepatic • Increased risk in CHF
gluconeogenesis • Possible hepatotoxicity

DPP-4 inhibitors • Sitagliptin Inhibit DPP-4, which Low risk of side effects
"-gliptin" • Alogliptin results in increased
• Saxagliptin endogenous GLP-1
• Linagliptin

GLP-1 agonists • Exenatide Mimic incretin Gl side effects (nausea)


"-glutide" • Liraglutide hormone GLP-1:
• Albiglutide • Decreases glucagon
• Dulaglutide • Increases insulin
secretion
• Delays gastric
emptying (which
reduces appetite)

SGLT-2 inhibitors • Dapaglif1ozin Inhibit Na+-glucose • Risk with kidney dysfunction


"-flozin" • Empagliflozin linked transporter • UTis, mycotic infections
• Canaglif1ozin in the kidney (more • Dehydration, polyuria
glucose loss in the • Possible DKA
urine)

0
)>
co
m
-i
m
(J)

0
::::0
c:
Gl
(J)

[ 625]
End ofSession Quiz
6. Which of the medications used in the treatment of type 2 diabetes has each of the
following characteristics?

Lactic acidosis is a rare but worrisome risk

'

LVI 1rv1 side effect is hypoglycemia

Recommended first-line treatment for most


patients

Not safe in patients with symptomatic CHF or


fluid balance problems

1 Should not be used in patients with abnormal


kidney function

Not associated with weight may help with


weight loss

Metabolized by the liver, could be used in


patients with renal dysfunction

closes K+ channel on !3 cells ~


depolarization ~ Ca 2+ influx ~ insulin release

MOA: agonist at PPARy receptors ~ improved


target cell response to insulin

MOA: decreases hepatic gluconeogenesis

MOA: decreases glucose reabsorption at the


renal tubules

Possible non-insulin treatment for patients with


(J)
(C)
organ failure (renal, fiver, heart)
::::J
0:::
0
(J)
w
f-
w
m
<(
0

r r,; A :
OBESITY
Phys: Chapter 71
R: Chapter 9
H: Chapters 77, 78, 242
BM/ (Phys p850) (H p622)
Adipocytes (Phys p821) (H p624)
Leptin (FA15 p454) (FA16 p456) (SU p170) (Phys p846) (H p625)
Hormonal regulation offat utilization (Phys p825)
Lipodystrophy (H p1993)
Obesity (SU p52) (Phys p850) (R p444) (H p629)
Metabolic syndrome (H p1992)
Medical complications of obesity (R p447) (H p627)
Nonalcoholic steatohepatitis (NASH) (FA15 p369) (FA16 p367)

~arpl - Up Rev1. ew
1. Which ECG leads will show evidence of ischemia in an inferior wall Ml? (FA15 p295) (FA16
p289)

2. What are the findings of Brown-Sequard syndrome? (FA15p472) (FA16 p475)

3. Metabolic syndrome diagnostic criteria


Diagnosis based on any three of the following:
• Abdominal obesity: waist circumference;;:: 40 in (102 em) in men , or;;:: 35 in (88 em) in women
IDF criteria: ;;:: 94 em in men, and ;;:: 80 em in women
Recognize that not all metabolic syndrome patients are overweight
• Triglycerides: ;::: 150 mg/dl
• HDL: < 40mg/dl in men, or< 50 mg/dl in women
• Blood pressure: ;;:: 130/ 85 mm Hg
• Fasting serum glucose: ;;:: 100 mg/ dl (or 2 hour post oral glucose ;;:: 140 mg/ dl)

0
OJ
en
(J)

----1
--<

[ 627 J
4. Nonalcoholic steatohepatitis (NASH)
• Most common causes: obesity, type 2 diabetes mellitus, hyperlipidemia, insulin resistance
• Due to insulin resistance at the liver ~ excess lipid accumu lation in the liver
• Can progress to cirrhosis, lead to hepatocellular carcinoma, worsen hepatitis C progression
• Suspect if chronically elevated liver function tests
• Diagnosis
Liver US, CT scan, or MRI
Magnetic resonance spectroscopy (gold standard)
Liver biopsy

of Sess1on Qu1z
5. Whatare the five categories of criteria for the diagnosis of metabolic syndrome?

What liver disease is associated with obesity?

7. At whatBMI is a patient considered obese?

>-
1---
(f)
UJ
ro
0

r 62Bl
CALCIUM METABOLISM
Phys: Chapter 79
H: Chapters 46, 352, 353
Parathyroid glands (SU p167)
Parathyroid hormone (FA15 p320) (FA16 p314) (SU p174, 228)
Vitamin D (FA15 p319) (FA16 p313) (SU p312-313)
Calcitonin (FA15 p321) (FA16 p315) (SU p228)
Hypercalcemia (FA15 p332) (FA16 p326) (SU p311)
Hyperparathyroidism (FA15 p332) (FA16 p326) (SU p194)
Hypocalcemia (SU p311)

Warm -Up Review


1. . . . What substances are known for causing methemoglobinemia? .

2. What are the positive symptoms of schizophrenia? Whatare the negative symptoms
of schizophrenia? (FA15 p509)(FA16 p514)

3. Hypercalcemia
90%
• Primary hyperparathyroidism
Solitary parathyroid adenoma (90-95%)
Parathyroid hyperplasia (5%)
• Malignancy
Squamous ce ll cancers (especially lung) (via PTH-related peptide)
Renal ce ll carcinoma
Brea st metastasis
Multiple myeloma (via local osteolytic factors)
()
10% )>

• Excess vitamin D ingestion 'n


c
• Excess ca lcium ingestion (milk-a lkali syndrome) :s:
:s:
• Granu lomatous disease (excess vitamin D due to sarcoid or TB) rn
);!
• Increased bone turnover (hyperthyroidism. vitamin A intoxication, immobilization) OJ
0
• Thiazide diuretics ~ decrease d renal excretion of calc ium '
V>
:s:

[ 629 J
4. Signs and symptoms of primary hyperparathyroidism
Stones Bones Abdominal groans
• • •
• • •
• • •
• •
Psychiatric overtones Other
• •
• •
• •
• •

5. Review: What are the causes of hypercalcemia?

6. Hypocalcemia
• Hypoparathyroidism
Parathyroidectomy (from damage during thyroidectomy)
- Autoimmune destruction of parathyroids
Pseudohypoparathyroidism (kidneys unresponsive to PTH)
DiGeorge syndrome
• Vitamin D deficiency
Nutritional deficiency and paucity of sunlight
Chronic renal failure
• Acute pancreatitis

1630 l
End ofSess1on Ou1z
7. What are the three functions of vitamin D?

8. > How does PTH affect calcium?.How does PTH affect phosphate?

9. What cell type produces PTH? What cell type pr()duces calcitonin?

10. What are two signs of hypocalcemia?

11. What are the two most common causes of primary hyperparathyroidism?

12. What are some possible causes of hypocalcemia?

RAP/ D- FI RE FACTS
Most common cause of hyperca lcemia

Most common cause of primary


hyperparathyroidism
Most common cause of secondary
hyperparathyroidism
n
}>
Most common cause of hypoparathyroidism ,-
n
c
:s:
Facial muscle spasm when tapping on the cheek :s:
rn
r :;;
Parathyroid, pancreatic, and pituitary tumors CD
0,-
Ul
:s:

[ 631 l
WHETHER J(S T HE

HEME DIFFERENTIAL .DIAGNOSIS


OF A PATIENT WITH
ANEMIA, TH E COMPL EX
1 Clottm g Factors
COAGULATION CASCADE
OR THE MYRIAD FORMS
2 Anticoagulant Drugs
OF LEUKEMIA AND
LYMPHOMA, THERE
3 RBC BaSICS
IS MUCH AB

4 Erythropoiesis

5 M1c rocyt1c Anem1 a


AND CONFUSING YET
6 Macrocytic and Normocytic Anem1a THESE TOPICS ARE
CERTAINLY WELL WITHIN
7 Hemo lytic Anem1as THE SCOPE OF THE EXAM
AND DESERVE YOUR
8 Platelet Fu nct1on ATTENTION.

9 Plate let Disorders

10 Lymphoma

11 Leukem1as

12 Multip le Myeloma
n
,--
CLOTTING FACTORS 0
---1
---1
z
R: Chapter 4 Cl
Phys: Chapter 36 ~
H: Chapter 116 n
---1
0
Coagulation (FA15 p386) (FA16 p383-384) (SU p266 -267) (R p89) ;:o
(J)
Coagulation lab tests (FA15 p397) (FA16 p396)
Coagulopathies (FA15 p397) (FA16 p396) (SU p269) (H p973)
Hypercoagulable states (FA15 p398) (FA16 p398) (R p123)

War -Up Review


1. Which personality disorder best fits each of the following statements? (FA15 p515)
(FA16 p519) (SU p60)
• Excessive need to
be takeri care .of, submissive and clinging behavior
• Low self-confidence, fears of separation and losing support
• Grandiosity, feels he is entitled to things, lack of empathy
• Suicide attempts ( -4 15% unstable mood and behavior

2. What is the WAGR complex? (FA15 p546) (FA16 p551) (SU p163)

[ 635 J
(/")
0::
0
f-
u
<r.:
End of Sess1on Qu1z
LL
l.:J
:z:
3. Which coagulation factor is deficient in hemophilia A?
f=
f--
0_J
u

4. Which coagulation factor is deficient in hemophilia B?

5. What is the clinical consequence of a deficiency in either protein C or protein 5?

r 636 1
)>
z:
ANTICOAGULANT DRUGS ---1
n
0
)>
GG: Chapter 30 Cl
H: Chapter 118 c:
,-
)>
Heparin (FA15 p405) (FA16 p405) (SU p270) (GG p853) (H p992) z:
---1
Direct thrombin inhibitors (FA15 p405) (FA16 p405) (SU p270) (H p997)
0
Low molecular weight heparins (FA15 p405) (FA16 p405) (SU p270) ;;o
c:
Direct factor Xa inhibitors (FA15 p406) (FA16 p407) Cl
Warfarin (FA15 p405) (FA16 p406) (SU p270) (GG p860) (J)

Heparin vs. warfarin (FA15 p406) (FA16 p406) (SU p271)


Thrombolytics (FA15 p406) (FA16 p407) (SU p270)

Warm -Up Rev1ew


1. . Which glomerular disease should be suspected most irfa patient wittre~ch of the
foUowingfmdings? (FA15 p540-543) (FA16 p545-548) (SU p152"154)
• IF: granular pattern of immune complex deposition; LM: diffuse capillary thickening
• IF: granular pattern of immune complex deposition; LM: hypercellular glomeruli
• IF: linear pattemof immune complex deposition
• IF: deposition of lgG, lgM, lgA, and C3 in the mesangium
• EM : subendothelial humps and "tram track)) appearance
• Nephritis, deafness, cataracts
• LM: crescent formation in the glomeruli
• Ll\11: segmental sclerosis and hyalinosis
• IF: Anti-GBM antibodies
• LM: Kimmelstiel-Wilson lesions
• Purpura on backs of arms and legs, abdominal pain, lgA nephropathy
• spiking GBM due to electron-dense subepithelial deposits

[ 637]
U)
~
=>
0:::
0 End of Sess1on Qu1z
f-
z
<( 2. What lab test is used to monitor adequate anticoagulation in a patient taking
_.J

=> heparin? In a patient taking warfarin?


(.9
<(
0
u
i=
z
<(

3. What is the treatment for an overdose of heparin? Of warfarin?

4. What is the treatment forheparin-induced thrombocytopenia?

RAP! o - Ft RE FACTS
Preferred anticoagulant for immediate anticoagu lation

Preferred anticoagulant for long-term anticoagulation

Preferred anticoagulant during pregnancy

r 638 l
;:;o

RIC BASICS OJ
n
OJ
)>
Vl
R: Chapters 10, 14 n
Vl
Phys: Chapters 32, 35, 83
H: Chapter 104
RBC basics (FA15 p382) (FA16 p378)
Pathologic RBC forms (FA15 p388-389) (FA16 p386-387) (SU p255-257)
Blood groups (FA15 p385) (FA16 p382) (SU p254)
Erythroblastosis [eta lis (FA15 p385) (FA16 p382) (SU p254)

Warm ~ Up Rev1ew
1. Which genetic syndrome is caused by each of the following underlying mecharlisms?
• Absence of HGPRTase
• Deficiency of aldolase 8
• Deficiency of cystathionine synthase
• Galactose-1-phosphate uridyltransferase deficiency~ intelleCtual disability,
hepatosplenomegaly, and
• Deficiency of tyrosinase

2. What is Potter sequence? (FA15 p526) (FA16 p530) (SU p144)

3. Erythroblastosis fetalis
• Maternal antibodies to fetal RBC antigen
• The most common antibody is anti-Rh-D
• In Rh(-) moms, dose anti-Rh-O immunoglobulin at 28 weeks, at any traumatic event (e.g., MVA), and
within three days of delivery
• Clinical features in the neonate:
- Anemia due to hemolysis of RBCs by maternal antibodies
- Jaundice ~ possible kernicterus
Hydrops fetalis (generalized fetal edema)
Intrauterine death

[ 639]
lf)
u

~ En of Session Qui z
u
~ 4. What allows RBCs to change shape as they pass through vessels?

5. What name is given to immature erythrocytes in circulation?

6. Which pathologic form of RBC would you see in each of the following diseases?
• Lead poisoning
• G6PD deficiency
• DIC
• Abetalipoproteinemia
• Asplenia

r 640 1
rn
ERYTHROPOIESIS :::0
-<
-i
:r:
:::0
R: Chapter 14 0
"""0
Phys: Chapters 32, 35 0
H: Chapters 57, 104 rn
lJ)
lJ)
Fetal erythropoiesis (FA15 p270) (FA16 p381) (SU p257-258)
Hemoglobin variants
Heme synthesis (FA15 p396) (FA16 p395)
Porphyria (FA15 p396) (FA16 p395)
Lead poisoning (FA15 p391, 396) (FA16 p389, 395) (SU p259-260)
Polycythemia (FA15 p404) (FA16 p404) (SU p257)

difference between kwashiorkor and marasmus? (FA15 p94)(FA16

2. Which vitamins should vegetarians take to supplement their diets? (FA15 p92) (FA16 pBO)

3. What are the different forms of hemoglobin (Hb)?


Hemoglobin Where Found Structure
HbA 97% of normal a2 ~2
hemoglobin
HbA2 2% of normal hemoglobin a2 82
HbA1c Poorly-controlled diabetes a2 ~2-glucose
HbF Fetal hemoglobin a2 y2 (gamma chains replace beta chains)
Hb Gower Embryonic hemoglobin ~2E2

HbS Sickle cell hemoglobin a2 ~ 5 2 (glu ~val in~ chain)


HbC Hemoglobin C disease a2 ~c2 (glu ~ lys in S chain)
Hb Bart's Severe a-thalassemia y4 (no alpha chains)
HbH Severe a-thalassemia ~4 (no alpha chains)

r 641 1
lf)
lf)
4. Heme synthesis
w
0
o_ Glycine+ succinyi-CoA
0
0:::
I
Vitamin 86 ~
f-
>-
0::: 6-ALA
w

~
Porphobilinogen

! - -- - - -
Hydroxymethylbilane
"""'"''"'"'""'"lir!50@!1
(Causes acute inte rmittent porphyria)

!
Uroporphyrinogen Ill

~
Coproporphyrinogen Ill

~
Protoporphyrin
Fe'• l -------------- Fenochelatase (Causes lead poisoning)
Heme

d Sess1on Quiz
5. Where does fetal erythropoiesis take place? In which adult bones does erythropoiesis
take place?

6. What are some of the different causes of polycythemia?

What are .the hematologic and non-hematologic.n.ndings in .a patient with lead


poisoning?

r 6421
s:
MICROCYTIC ANEMIA n
;;o
0
n
-<
___,
H: Chapters 57, 103, 704
R: Chapter 14 n
)>
Anemias (FA15 p390) (FA16 p388) (SU p260-265) z
fTl
Iron deficiency (FA15 p390) (FA16 p388) (SU p260) (R p630) s:
Alpha thalassemia (FA15 p390) (FA16 p388) (SU p260) (R p630) )>

Beta thalassemia (FA15 p391) (FA16 p389) (SU p260) (R p630)


Lead poisoning (FA15 p391) (FA16 p389) (SU p259)
Sideroblastic anemia (FA15 p390) (FA16 p389) (SU p261)
Iron studies (FA15 p395) (FA16 p394) (SU p262) (R p630)

Warm-Up Review
1. How does hexokinase differ from glucokinase? (FA15 p98) (FA16 p86) (SU p299)

2. Which substrate and cofactor are required for the generation of GABA? (FA15 p106)
(FA16 p94)

3. What medication inhibits alcohol dehydrogenase? What medications inhibit


acetaldehyde dehydrogenase? (FA15 p95) (FA16 p83) (SU p305)

4. Iron studies
%transferrin
Diagnosis Serum iron TIBC Ferritin
saturation

Iron deficiency anemia i


Anemia of chronic disease l
Hemochromatosis i i
Sideroblastic anemia i Nl or i

r 643 1
<(

::?:
LU
z
<( End of Session Qu1z
u
f-- 5. What test can be used to diagnose beta-thalassemia minor?
>-
u
0
a::
u
2:

6. A patient is found to have microcytic, hypochromic anemia. What lab findings allow
you to distinguish iron deficiency anemia from anemia of chronic disease?

7. What should you rule out in a man over 50 with new-onset iron deficiency anemia?

RAP! O-Ff RE FACTS

Causes of hypochromic, microcytic anemia

Skull x-ray shows a "hair-on-end" appearance

Ba sophilic stippling of RBCs

r 6441
:s
MACROCYTIC AID NORMOCYTIC AIEMIA )>
()
AJ
0
()

H: Chapters 105, 107 -<


-i
R: Chapter 14 ()
)>
Macrocytic anemia (FA15 p392) (FA16 p390) (SU p262-263) z:
0
- Folate deficiency
z:
- 8 12 deficiency 0
- Orotic aciduria AJ

Normocytic anemia (FA15 p393) (FA16 p391) (SU p263-264) :s


0
- Anemia of chronic disease (R p652) ()

- Aplastic anemia (R p653)


-<
-i
- Renal failure ()
)>
z:
m
:s
)>
Wa rrn - Up Rev1ew
What is the rate-:: limiting enzyme of beta,;,oxidation offatty acids? (FA15 p96) (FA16pB4)

2. Which cofactors are required for the function of pyruvate dehydrogenase? Which
other enzyme requires the same C()factors? (FA15 p99) (FA16 p87)

3. What is the result of a glycolytic enzyme deficiency? What is the result of a


deficiency in pyruvate dehydrogenase? (FA15 p100) (FA16pBB! (SU p299, 306)

r 645 1
<(

2:
w
z
<(
ofSeSSIOn QUIZ
u
f- 4. What are the causes of aplastic anemia?
>-
u
0
2:
a::
0
z
0
z
<(
u
f-
G 5. Apatient is.diagnosed with a macrocytic, megaloblastic anemia. What is the .danger
~
u
of giving folate alone?
<(
2:

What isthe cause of anemia given each of the following statements?


• Microcyticanemia +swallowing difficulty +glossitis
• Microcytic anemia with > 3.5%. . HbA2
• Macrocytic anemia+ hypersegmented neutrophils
• Megaloblastic anemia not correctable by 8 12 or folatesupplements
• Megaloblastic anemia + peripheraL neuropathy
• Micrqc;ytic anemia+basophilicstippling
• Microcytic anemiareversiblewith B 6 supplements
patientwith rhqctocytic anemia
elevated creatinine

r 646
:r:
HEMOLYTIC ANEMIA rn
.$
0
~
H: Chapter 106 ---i
R: Chapter 14 n
)>
Types of hemolysis (FA15 p393) (FA16 p391) (SU p264-265) :z:
rn
Intrinsic hemolysis (FA15 p394) (FA16 p392) (SU p265) (H p879) .$
Hereditary spherocytosis )>
- G6PD deficiency
- Pyruvate kinase deficiency
- Paroxysmal nocturnal hemoglobinuria
Sickle cell disease
Hemoglobin C disease
Extrinsic hemolysis (FA15 p395) (FA16 p393) (SU p264-265) (R p643-644)
- Microangiopathic anemia
- Macroangiopathic anemia
Infections
Autoimmune hemolytic anemia

Wa rm-Up Revtew
1. A deficiency of which vitaminresults in gum bleeding, bruising, anemia, and poor
WQtJnd healing? (FA15 p93) (FA16 /)BO)(SU p314)

2. What disorder is associated with each ofthe following findings?


• Hypertension + hypokalemia +metabolic alkalosis
• Fever + night sweats + weight loss
• Adrenal hemorrhage due to meningococcemia
• Blue sclerae
• Hyperphagia+ hypersexuality+ hyperorality + hyperdocility
• Nystagmus + intention tremor + scanning speech
• Lower extremity purpura + arthralgias + rena l disease

r t.47 1
<(

:2:
3. Cold agglutinins
UJ
z • Antibodies against RBCs that interact more strongly at low temps (4°C) than at body temperature
<(
u • Nearly always _ _ _ __ _
r-
~ • Occur regularly in infections with _ _ _ _ _ _ _ _ _ _ _ _ _ and with malignancies
0
:2: • Problems/disease occur when there is circulation to a cold extremity ~ lgM binds RBC antigen ~
UJ
:r: complement fixation ~ MAC lysis (and opsonization ~ phagocytosis)

4. Warm agglutinins
• Antibodies that react against RBC protein antigens at body temperature
• Nearly always _ _ _ _ __
• Seen in:
1)
2)
3)
4)

5. Coombs test (+)


RBC agglutination with the addition of antihuman antibody because RBCs are coated with
immunoglobulin or complement proteins
• Direct Coombs (DAT)
Prepared antibodies are added to a patient's washed RBCs to detect the presence of
immunoglobulins already present on the RBCs (using an antibody to detect an antibody)
Positive in: hemolytic disease of the newborn, drug-induced autoimmune hemolytic anemia,
hemolytic transfusion reactions
• Indirect Coombs
Patient's serum is incubated with normal RBCs to detect the presence of antibodies
Positive when: antibodies to foreign RBCs are present (used to test blood prior to transfusion,
and to screen for maternal antibodies to a fetus's blood)

r 6481
I
fTl

:s
End of Sess1on Quiz 0
~
-j
6. A child anemic since birth has now been cured with spleneCtomy. What is the n
·disease? )>
z
m
:s
)>

Whatfindings are associated with hereditary spherocytosis?

What is the difference between the hemoglobin S defect andthe hemoglobin C


defect?

9. What is the difference between a warm agglutinin and a cold agglutinin?

10. What are schistocytes?

11. In which hematologic disorder would you find each of the following abnormal tests/
findings?
• .Ham's test
• Heinz bodies
• Basophilicstippling
• Osmotic fragilitytest

RAPID-FIR E FACTS
Painful cyanosis of the fingers and toes with hemolytic anemia

Red urine in the morning and fragile RBCs

Basophilic nuclear remnants in RBCs

~ Autosplenectomy

Drug used to treat sickle cell disease

[ 649 J
z
0
f-
u
PLATELET FUNCTION
z
:::::l
LL R: Chapters 4, 14
f- H: Chapters 115, 118
w
_J
w
GG: Chapter 30
~
_J
Platelets (FA15 p382) (FA16 p378) (SU p266) (R p117)
0... Platelet plug formation (FA15 p387) (FA16 p385) (SU p266) (R p116)
Platelet stimulation (FA15 p387) (FA16 p385) (SU p266) (R p122)
Antiplatelet drugs
- Aspirin (FA15 p407) (FA16 p447) (SU p270) (R p85)
- ADP receptor inhibitors (FA15 p407) (FA16 p407) (SU p270) (GG p869)
- Glycoprotein lib/lila inhibitors (FA15 p407) (FA16 p407) (SU p270) (GG p870)

Wa r m~Up Review
1; A woman develops intense muscle cramps and darkening of her urine after exercise.
What is the most likely diagnosis? (FA15 p110) (FA16 p99) (SU p302)

2. Which personality disorder best fits each of the following statements? (FA15 p515)
(FA16p519) (SU p60)
• ·Sense of emptiness and loneliness; impulsiveness
• Odd appearance, thoughts, and behavior; no psychosis; social awkwardness
• Controlling, perfectionistic, orderly, stubborn, indecisive
• Criminality, inability to conform to social norms, disregard for others' rights

3. What substances are known for causing hemolytic anemia in patients with G6PD
deficiency? (FA15 p102) (FA16 p90) (SU p307)

4. Von Willebrand factor (vWF)


• Several subunits linked by disulfide bonds
• Synthesized by endothelial cells and megakaryocytes
• Major functions:
- Complexes with and stabilizes factor VIII (deficiency --t j PTT)
Platelet adhesion to vessel wall and other platelets (deficiency --t i bleeding time)

r 6so J
5. Platelet stimulation "lJ
r-

• Adhesion (endothelial damage, vWF, gplb) ~


rn
r-
m
• Activation ___,
,
- Secretion of ADP. PDGF. serotonin, fibrinogen , lysosomal enzymes, thromboxane A 2 , calcium, c
z
thrombin n
___,
Thromboxane A 2 --+ vasoconstriction and platelet aggregation 0
z
- Thrombin: fibrinogen --+ fibrin
• Aggregation of platelets via gpllb/llla

End of Session QU IZ
6. Wh~t is-the life span pf a platelet (also the maximum life span of platelets after a
transfusion)?

7. What molecule is expressed on the surface of a platelet after it becomes activated?

8. NSAI.Ds inhibit the productiortofwhich substance important in platelet aggregation?

9. What is the mechanism of action of each of the following drugs?


• Streptokinase
• Aspirin
• Clopidogrel
Abciximab
Tirofiban
• Ticlopidine
• Enoxaparin
• Eptifibatide

r 6sJ 1
VJ
a:
UJ
0
a:
PLATELET DISORDERS
0
VJ R: Chapter 14
0
H: Chapters 115, 118
f---
UJ
_j Clinical presentation (FA15 p397) (FA16 p397) (SU p268)
UJ
Immune thrombocytopenia (FA15 p397) (FA16 p397) (SU p268)
~
_j TTP-HUS (FA15 p397) (FA16 p397) (SU p268)
o_
Bemard-Soulier syndrome (FA15 p397) (FA16 p397) (SU p268)
Glanzmann thrombasthenia (FA15 p397) (FA16 p397) (SU p268)
von Willebrand disease (FA15 p398) (FA16 p398) (SU p269)
Disseminated intravascular coagulation (FA15 p398) (FA16 p398) (SU p269)

Warrn -lJ.p
1. What disorder is associated with each of the following findings?
• Thyroid cells with optically clear nuclei
• Anemia with hypersegmented nel.ltrophils
• Branchingrods on oral infection
• Eczema + recurrent infections + t hrombocytopenia
• Hemosiderinuria +thrombosis

2. Which stage of sleep is associated with each of the following EEG findings?
• Alpha waves
• Beta waves
• Delta waves
• Theta waves

3. What is the underlying pathophysiology in thrombotic thrombocytopenic purpura (TTP)?


• Deficiency of the metalloprotease ADAMTS13
Normally, vWF multimers are released from injured endothelial cells
- ADAMTS13 cleaves the multimers into smaller, active vWF units
• ADAMTS13 deficiency~ vWF multimers are not broken down
• Unregulated platelet aggregation ~ widespread thrombosis
• Consumption of platelets~ thrombocytopenia
• Activation of coagulation cascade ~ excessive fibrin mesh ~ microangiopathic hemolytic anemia

r 652 1
4. What is the classic triad of findings in HUS? What is the classic pentad of findings in TTP? -o
.--
~
rn
.--
rn
-i
CJ
(J)
0
;::o
CJ
rn
;::o
(J)

End of Sess1on Quiz


What is the cause ()fiTP?

6. What is the defect in Bernard-Soulier disease?

After a normal spontaneous. vaginal delivery, the new mom bleeds profusely from her
vagina and later from her gllms. What abnormal lab values would you suspect?

RA P! O-F! RE FACTS
Antiplatelet antibodies

Bleeding disorder with gp lb deficiency

Most common inherited bleeding disorder

[ 653]
<(

~
:r:
LYMPHOMA
[l_

2:
~ R:Chapter13
H: Chapters 110
Leukemia vs. lymphoma (FA15 p399) (FA16 p399) (SU p272-275) (R p588)
Hodgkin lymphoma (FA15 p399) (FA16 p399) (SU p271)
Non-Hodgkin lymphoma (FA15 p399, 400) (FA16 p399, 400) (SU p271-272)

Wa·rm -U p Hev1 ew
1. What pathoJogy matches ea.ch of the following statements?
• Focal myocardial inflammation with multinucleate giant cells
• Eosinophilic cytoplasmic globules in liver near nucleus
.; Desquamated epithelium casts Tn sputum
• Onion skin /periosteal reaction
• <Pseudopalisading tumor cell arrangement
• Elevated serum uric acid

2. What is the equation for Gibbs free energy?

3. What is the name of the genetic syndrome that fits each of the following
descriptions?
• Decreased NADPH due to lack of HMP enzyme
• Inherited defect in tubular amino acid transporter
Deficiency in.homogentisic acid oxidase
Blocked degradation of branched-chain amino adds

r 654l
Endof Sess1on Qu1 z
4. Compare the age distribution of those affected by Hodgkin lymphoma to those
affected by non-Hodgkin lymphoma.

5. What is the mostcommontype of non-Hodgkin lyiJlphoma in adults? In children?

6. Which form of lymphoma matches each of the following statements?


• Most common lymphoma in the US ..
• Reed-Sternberg cells
• Particularly associated with EBV
• Associated with long-term celiac disease
• Lymphoma equivalent of CLL
• "Starry sky" pattern due to phagocytosis of apoptotic tumor cells
• Associated with Sjogren syndrome, Hashimoto thyroiditis, and H. pylori

RAPID - FI RE FACTS
Cancer most commonly associated with a noninfectious fever

Sheets of lymphoid cells, "starry sky" appearance

Large B cells with bilobed nuclei and prominent "owl's eye inclusions"

r 655 1
LEUKEMIAS
R: Chapter 13
H: Chapters 109, 110
Leukemias (FA15 p402) (FA16 p402) (SU p273-275)
Chromosomal translocations (FA15 p403) (FA16 p403)
Leukemoid reaction (FA15 p399) (R p584)
Myelodysplastic syndromes (FA15 p401) (FA16 p401)

Warm-Up Rev1ew
1. A 15-year-old girl of normal height and weight for her age has enlarged parotid
glands but no other complaints. Her mother confides that she found .laxatives in her
daughter's closet. What is the most likely diagnosis? (FA15 p516) (FA16p520)

2. Which defense ·mechanism best fits each of the. following descriptions? (FA15.p504-S05)
(FA16 p508-509) (5UpS9)
• Involuntary withholding of a feeling from conscious awareness
• A veteran can describe horrific war details without any emotion
• A child abuser was himself abused as a child
• Underlies all other defensemechanisms
• May lead to multiple personalities
• Adult whining, bedwetting, crying

r 656 1
3. Acute leukemia 'rn
c
?<:
• Rap id onset and rapid ly progressive rn
:s:
• Over 20% myeloblasts (AM L) or lymphoblasts (ALL) in the bone marrow )>
(J)
• Numerous blast (immature) ce ll s(> 20% blasts)
• Often associated with pancytopenia (anemia, bleeding tende ncy, infection)

ALL AML
• Philadelphia chromosome may be seen (poor • Philadelphia chromosome
prognosis) rarely seen
• Most common in chi ldren and yo ung adu lts • Characteristic Auer rods
• Ma les > females, whites > blacks • 8 different morphological
• B ce ll types more common than T ce ll classifi cations
• 3 morphologic variants and 5 phenotypic • Most are CD13/33 (+)
variants • Usual ly nonspecific esterase
• Bone pa in is common (+) myeloid ce ll s
• Very good prognosis in chi ldren (up to 90% • Median age of onset is 65
remission) • PAS(-)
• Most have the enzyme termina l • Associated w ith numerous
deoxynucleotidyl transferase (TdT) risk factors
• PAS(+)
• Difficult to diagnose on blood smear (others
can be diagnosed with smear)

4. AML risk factors


• Rad iation, benzene, or alkylating agents (such as in Hodgkin lymphoma treatment)
• Myeloproliferative disease, myelodysplastic syndrome, or aplastic anemia
• Down syndrome, Fanconi anemia, or Bloom syndrome

5. Chronic leukemia
• Insidious onset and gradual progression (months to years)
• Mat ure ce lls (rather than blasts) (< 5% blasts)
• Can be either myeloid (CML) or lympho id (CLL)
• Associated with hepatosplenomegaly and lymphadenopathy
• Prominent infiltration of bone marrow by leukemic ce lls; peripheral WBC count may be high

CLL CML
• Most common adult leukemia •May progress to AM L (80%) or ALL (20%)
seen in western countries •Numerous basophils and PMNs are LAP(-)
• Males > fema les, whites > •Adults ages 25 -60
blacks •Hyperp lasia of all 3 ce ll lines (granulocytic,
• Elderly adu lts (median age of erythroid, and megakaryocytic) but
diagnosis is 70) granulocyte precursors predom inate
• 95% have B cel l markers • Ph iladelphia chromosome (t 9;22) is always
(rather than T cel l) present
• 10% progress to AL L • Fatigue, abdominal pain, splenomega ly,
• Characteristic smudge ce lls bleeding tendency
• Autoimmune hemolytic
anemia
• Tends to be indolent

r 6571
6. Which .form of leukemia matches each of the following statements?
• Mostcommon leukemia in children
•• Most common leukemia in adults in the U.S.
• Characteristic Auer rods
• Myelodysplastic syndromes have a tendency to ·progressto _ ___:______ __ _ __

•• More than 20% blasts in marrow


• Leukemia with more mature cells and ·< 5% blasts
• TdT(+) acute leukemia
• Commonly presents with bone pain
• Numerous basophils, splenomegaly, and negative for leukocyte alkaline phosphatase (LAP)

• Always positive for the Philadelphia chromosome (t 9;22)


• Acute leukemia positive forperoxidase
• Solid sheets oflymphoblasts in marrow
• Always associated with the BCR-ABL gene

RA P! O- F! RE FACTS
Smudge cells

Reddish-pink rods in the cytoplasm of leukemic blasts

6581
MULTIPLE MYELOMA
R: Chapter 13
H: Chapter 111 s:
-<
rn
Myeloproliferative disorders (FA15 p404) (FA16 p404) (SU p275) r-
0
Multiple myeloma (FA15 p401) (FA16 p401) (SU p276) s:
)>
MGUS (FA15 p401) (FA16 p401) (SU p278)

Warm -Up Rev1ew


Which enzyme is deficient in each ofthe following diseases? (FA15 p103)(FA16 p91)
(SU p310)
• Fructose intolerance
• Essential Jructosuria
• Classic galactosemia

2. What is the result of blocking each of the following dopaminergic pathways?


• Mesocortical
• Mesolimbic
• Nigrostriatal
• Tuberoinfundibular

r 659 1
<J::
2:
0_ j
LLJ
>-
Session Quiz
2:
LLJ
_j
3. What pathology.best fits each. ofthe .following descriptions?
o_
•• "Punched out" Jyticbqne lesions
f::::j
:::J
• RBCs clumped together like a stack of coins
2: • Plethora +pruritus + headache
• Back pain+ anemia+renal insufficiency hypercalcemia
• Teardrop-shaped.RBCs
• . Monoclonal antibodyspike

A patient with anemia, hypercalcemia, and bone pain undergoe$ a bone marrow
biopsy which reveals plasma cells. What is the diagnosis,.andwhatmay be found on
urinalysis?

r 66o 1
CANCER IS COVEREQ

ONCOLOGY PRIMARILY IN THE


CHAPTERS ON TH
VARIOUS ORGAN
1 Cancer Bas1cs
SYSTEMS. THIS SECTION
OF THE COURSE,
2 Genetics of Cancer
HOWEVER, FOCUSES
ON THE MOLECULAR
3 Cancer R1sk Factors
BIOLOGY OF NEOPLASIA
AND MALIGNANT
4 Cancer Screening and Prevention
TRANS FOR MAT/ ON,
CANCER EPIDEMIOLOGY
5 Cancer Drugs part 1
AND CH EMOTHERAPEUTICS

6 Cancer Drugs part 2

I~
CANCER BASICS
R: Chapter 7
Histologic nomenclature (FA15 p233) (FA16 p226) (R p268)
Tumor nomenclature (FA15 p234) (FA16 p228) (R p266, 268)
Tumor grading and staging (FA15 p233) (FA16 p228) (R p332) n
Overview of neoplastic progression (FA15 p232) (FA16 p227) (R p268-274) )>
z
Sites of metastasis (FA15 p240) (FA16 p233) n
m
Paraneoplastic effects of tumors (FA15 p238) (FA16 p229) (SU p115) (R p330) ;;o
Cachexia (FA15 p234) (FA16 p232) (R p330) o:J
)>
Psammoma bodies (FA15 p239) (FA16 p232) (SU p50) Ul
n
Ul

Warm -Up Review


1. A w()man presents with headache, visual disturbance, and amenorrhea. Whatis the
diagnosis? (FA15 p332) (FA16 p326) (SU p184)

2. A patient presents with hypertension, hypokalemia, metabolic alkalosis, and low plasma
renin. What is the diagnosis, and how do you treat it? (FA15 p554) (FA16 p558) (5Up185)

3. How does the brain utilize ketone bodies? (FA15 p112) (FA16 p102) (SU p310)

[ 663 J
End of Sess1on Quiz
4. What type of tumor matches each of the following?
• Benign tumor of epithelium
• Malignant tumor of blood vessels
(/)
u • Benign tumor of bone
(/)
<( • Malignant tumor of smooth muscle
m
0:::
Ll.J
u
z
<(
u ' 5. What is the most common cause of hypercalcemia? What cancers may cause
hypercalcemia?

6. What neoplasm is most commonly responsible for each of the following


paraneoplastic syndromes?
• ACTH ~ Cushing syndrome
• Erythropoietin ~ polycythemia
• ADH ~ . SIADH

7. Which cancers metastasize to bone? To brain? To liver?

r 664 l
GENETICS OF CANCER
R: Chapter 7
Molecular basis of cancer (R p280)
Tumor suppressor genes (FA15 p236) (FA16 p230) (R p290)
Oncogenes (FA15 p236) (FA16 p230) (R p283)
Cl
m
z
m
---\
n
Warm-Up Revrew (J)

0
-n
1. What are the most common causes of hypocalcemia? n
)>
z
n
m
;:o

2~ Which cancer is associated with Hashimoto thyroiditis? (FA15 p32B) (FA16 p322) (SU p272)

3. To which main two molecules is iron bound in human cells? To which main two molecules is
iron bound in blood?

4. Molecular basis of cancer


• Malignant transformation (carcinogenesis) is fundamentally due to nonlethal genetic damage.
• Malignant tumors are derived from clonal expansion of a single precursor cell.
• The four main targets of genetic damage are:
- Proto-oncogenes
- Tumor suppressor genes
- Genes that regulate apoptosis
- DNA repair genes
• Malignant transformation is a multistep process resulting from multiple mutations.

r 665 1
5. Retinoblastoma (Rb)
• 'l4 of cases are bilateral (both eyes). and all bilateral cases are inherited point mutations.
• The other% of cases are unilateral (one eye); most of these are sporadic mutations.
• Overall, sporadic mutations account for about 60% of the mutations in the Rb gene, and these
are always unilateral. In other words, 40% of retinoblastomas are inherited (which results in either
0::: bilateral or unilateral retinoblastoma).
w
u • In order for the cancer to occur, there must be mutations to both alleles (AKA "two hits").
:z
<t
u In the heritable form, one of those hits comes from the parent gene and the other hit arises
LL
0
sporadically.
~ In the somatic/sporadic form, both hits arise sporadica lly.
f-
w
:z
~ 6. p53
• Acts through p21 to cause cell cycle arrest
• Involved at the G/S checkpoint and G/M checkpoint
• Causes apoptosis by inducing the transcription of pro-apoptotic genes (such as BAX)
• Mutations in this gene allow the cell to progress through the checkpoint despite the presence of
DNA damage/mutations

7. Oncogenes
• Proto-oncogenes: normal cellular genes that regulate cell proliferation and differentiation, can
become oncogenes
• Oncogenes: genes that promote autonomous cell growth in cancer cells by promoting cell growth in
the absence of normal mitotic signals
• Oncoproteins produced from these genes are missing important regulatory elements

8. RAS oncogene
• Mutation in RAS is the most common oncogene abnormality in human tumors
• 15-20% of all human tumors contain mutated versions of RAS proteins
• K-RAS mutation ~ colon, lung, and pancreatic tumors (Kolon, panKreatic)
• H-RAS mutation ~ bladder and kidney tumors (Hematuria)
• N-RAS mut ation ~ melanomas, hematologic malignancies, follicular thyroid cancer

r 666 1
d of Sess1on Qu iz
9. What type of cancer ls associated with each of the following tumor suppressor
genes?
• Rb
Cl
• DPC m
z
• p53 m
-I
• APC n
(J)
• WT1 0
--,-,
• BRCA1 and BRCA2 n
)>
z
n
m
;;o

10. What cancers are associated with a mutation of the K-RAS oncogene?

11. What transcription factor is normally bound by the protein product of the Rb tumor
suppressor gene?

r r,t,71
CANCER RISK FACTORS
R: Chapter 7

Chemical carcinogens (FA15 p238) (FA16 p231) (R p278, 321 -324)


Vl Radiation carcinogenesis (FA15 p238) (FA16 p231) (R p324)
0::: Oncogenic infections (FA15 p237) (FA16 p231) (R p325-329)
0
r- Diseases associated with neoplasms (FA15 p235) (FA16 p229) (R p278)
u
<( Cancer epidemiology (FA15 p239) (FA16 p228) (R p275)
l.J....
~
Vl
0:::
0:::
LLJ
u
z: Warm - Up Rev1ew
<(
u
1. Which oral agent used in the control of type II diabetes has each of the following
characteristics? (FA15 p338) (FA16 p334) (SU p181-182)
• Lactic acidosis is a rare but worrisome side effect
• Most common side effect is hypoglycemia
• MOA: closes ~ channel on~ cells -7 depolarization -7 Ca 2+ inf1ux -7 insulin release
• MOA: agonist at PPARy receptor

2. What is the most common urea cyde disorder? (FA15 p106) (FA16 p94) (SU p31B)

3. What are the major regulatory enzymes of the citric acid cyde? (FA15 p100) (FA16 pBBJ (SU p306)

r 6681
4. What are the most common cancers associated with ionizing radiation?

()
5. Which cancers are associated with UV radiation exposure? Which type of UV radiation is most )>
z
problematic? ()
rn
;;;o
;;;o
U1
?"':
-n
)>
()
--1
0
;;;o
U1

End of Session Quiz


6. What cancers are associated with each of the following?
• Nitrosamines
• ·Asbestos
• 2-naphthylamine
• Schistosoma haematobium
• EBV
• HPV

7. What is the most common cancer among women? Which cancer causes the highest
mortality among women?

8. What is the most common cancer among men? Which cancer causes the highest
mortality among men?

r r,r,9 l
CANCER SCREENING AND PREVENTION
H: Chapter 82
R: Chapter 7
Risk factor reduction (primary prevention) (H p655)
Cancer screening (secondary prevention) (H p659)
z - Breast cancer screening
0
f- Cervical cancer screening
z - Prostate cancer screening
w
>
w
- Colon cancer screening
0:::: - Lung cancer screening
CL
Serum tumor markers (FA15 p237) (FA16 p232)
0
z
<t:
1..9
z
z
w
w
0::::
u
Warm- Up Revi ew
lf)
0:::: 1. Which hormones use steroid receptors? Which hormones use tyrosine kinase receptors?
w
u (FA15 p321) (FA16 p316)
z
<t:
u

2. What is the most likely scenario in which a person would receive toxic exposure to arsenic?

3. What is the rate-limiting enzyme in purine synthesis? In pyrimidine synthesis? (FA15 p63)
(FA16p49) (SU p279)

r 670 1
4. What are the screening recommendations for the following cancers?
USPSTF Recommended Screening

Breast cancer

Cervical cancer
n
);>
z:
n
Prostate cancer rn
:;o
Vl
n
Colon cancer :;o
rn
rn
z
Lung cancer z
Cl
);>
z
0
u
:;o
rn
<
rn
z:
___,
0
z:
5. Which tumor marker would you use to monitor each of the follow·
• Hepatocellular carcinoma
• Ovarian cancer
• Pancreatic cancer
• Melanoma
• Colon cancer

6. What is the USPSTF recommended screening test for colon cancer, and what is the
recommended screening frequency? At what age should average-risk individuals start
being screened?

7. According to USPSTF guidelines, at what age should women start having Pap smears
to screen for cervical cancer? What is the recommended frequency?

8. According to USPSTF guidelines, what patients should have an annual chest x-ray to
screen for lung cancer?

i f17f l
CANCER DRUGS PART 1
GG: Chapter 60
Cell cycle (FA15 p408) (FA16 p408)
Antimetabolites (FA15 p409) (FA16 p409)
Methotrexate (SU p437)
r-
0:::
- 5-fluorouracil (SU p425)
<t - 6-mercaptopurine (SU p436)
o_
- 6-thioguanine (SU p457)
lfl
lCJ - Cytarabine
:=J
0::: Alkylating agents (FA15 p410) (FA16 p410)
0
Cyclophosphamide (SU p416)
0:::
UJ - Ifosfamide (SU p429)
u - Nitrosoureas (SU p441)
:z:
<t - Busulfan
u
Antitumor antibiotics (FA15 p410) (FA16 p410)
- Doxorubicin and daunorubicin (SU p420, 417)
- Dactinomycin (SU p417)
- Bleomycin (SU p409)

-u Rev1
Which hormone has each of the following actions?
Stimulates milk secretion during lactation
• Stimulates metabolic activity
• Increases blood glucose level and decreases protein synthesis
ovulation in females and testosterone synthesis in males

What is the rate-limiting enzyme for each of the following metabolic pathways?
(FA16p84)
Urea cycle
• Hexose monophosphatepathvvay
• Fatty acid synthesis
• ~-oxidation of fatty acids
• Ketone body synthesis
• Cholesterolsynthesis
acid synthesis

r 672 l
,r"- nd of Se ssio n Ou1z
3. How does the mechanism of action of methotrexate differ from 5-fluorouracil?

n
)>
z
n
rn
;;u
0
;;u
c
Cl
4. < How does the body metabolize 6-mercaptopurine? (J)

""D
)>
;;u
-1

5. Which antic.ancer drug is also often used in rheumatoid diseases as well as ectopic
pregnancies?

6. Which anticancer drug fits each of the following descriptions?


• Alkylates DNA, toxicity~ pulmonary fibrosis
• Fragments DNA, toxicity ~pulmonarynbrosis
• Blocks purine synthesis, metabolized by xanthine oxidase
• Nitrogen mustard, alkylates DNA (electrophile that binds DNA)
• Folic acid analog that inhibits dihydrofolate reductase

• Intercalates DNA, produces oxygen free radicals, cardiotoxic


• DNA alkylating agents used in brain cancer
• Applied topically for actinic keratoses and basal cell cancers
• Treatment for childhood tumors (Ewing sarcoma, Wilms tumor, rhabdomyosarcoma)
• Side effectof hemorrhagic cystitis

r 6Ti 1
CANCER DRUGS PART 2
GG: Chapter 60
Cisplatin, carboplatin, oxaliplatin (FA15 p411) (FA16 p411) (SU p414, 411)
Topoisomerase inhibitors
N
- Etoposide, teniposide (FA15 p411) (FA16 p411) (SU p424)
f- - Jrinotecan, topotecan (FA15 p411) (FA16 p411)
er:
<( Microtubule inhibitors (FA15 p411) (FA16 p411)
Q_
- Vincristine (SU p461)
lf)
\.'J - Vinblastine (SU p460)
=:> - Paclitaxel (SU p444)
rr:
0 Hydroxyurea (FA15 p411) (FA16 p411) (SU p429)
rr: Prednisone (FA15 p412) (FA16 p412) (SU p447)
UJ
u SERMs
z
<( - Tamoxifen (FA15 p412) (FA16 p413) (SU p455)
u - Raloxifene (FA15 p412) (FA16 p413) (SU p451)
Trastuzumab (FA15 p413) (FA16 p413)
Rituximab (FA15 p412) (FA16 p412) (SU p452)
Bevacizumab (FA15 p412) (FA16 p412)
Jmatinib (FA15 p412) (FA16 p412)

Whatare the irreversible enzymes involved in gluconeogenesis? (FA15 p101) (FA16 p89)

r 674 1
/-",. End.of Session Ou1z
3. Whictl anticancer drug fits each oUhe fo11owing descriptions?
• Cross-links DNA, nephrotoxic, ototoxic
• Prevents tubulin disassembly
n
• Inhibits ribonucleotide reductase )>
z
• Inhibitor of the mutated tyrosine kinase produced by the Philadelphia chromosome n
m
• Mc:moclonal antibody against HER2 :::0
CJ
• SERMthat c]oes not increase the risk of endometrial cancer :::0
c
G1
IJl
"""1J
What are the possible side effects of glucocorticoids? )>
:::0
-1
N

j 67.5 l
TEST DAY TIPS
What should I do five days prior to the test?
• Split your Step 1 review book (such as First Aid) into 4-5 equal sections. Since FA contains some
600 pages of material, each section will have about 120 pages. Over 4-5 days, review your text
entirely by going through about 120 pages per day. If you go through 10 pages an hour, it should
take you 9-12 hours per day to do this. As you come across a detail that you do not recognize,
write it down. At the end of each day and the beginning of the next day, review those details you
have written. The small details in FA are often overlooked and may be on your test. If you decide
to continue with a question bank during these five days, do no more than 25-50 questions a day.
Concentrate on your knowledge base.
(./)
Q_
• At the end of Day 5:
f- Review all of the unfamiliar details written from the previous five days.
~ Review all the questions in the DIT study guide, the "definitely know" topics highlighted during
0
f--- the course, and the 3-5 star topics highlighted during the course.
(.1)
LU
f- Decide what information is going to be put on your markerboard right before the test and
memorize that information.
Consider taking a 150-question NBME test online (free or $45). Since these tests are slightly
easier than the real thing, it will boost your confidence a little. There are no answers given, so
you won't spend time going through them. You can expect 3-4 of those questions to be on your
actual test. This test will allow you to regain familiarity with the actual testing format and make
sure you've gone through the tutorial, so you can skip it on your test day and add that time to
your break time.
Skim through FA again at your leisure to review random topics, but pay particular attention to
the "definitely know" and 3-5 star topics.
- Stop studying at least two hours before bedtime to make sure you get a good night's sleep.
• By going through this 5-day program, you will have reviewed everything in the five days before the
test.

What should I put on my markerboard prior to the start of the test?


• Don't write on your markerboard for longer than five minutes before you start your test.
• Put whatever you want, but you may want to consider the following:
Developmental milestones
- The four pharmacokinetic equations
Error square
Sensitivity, specificity, PPV, NPV, OR, RR equations and square
Lung volume diagram

Test Day Tips


• Bring a cooler with ice, water, Gatorade, or juice. Pack a lunch. Bring some fruits and snacks. (You
may not be able to predict what you're going to want to eat, so it's better to bring too much than
too little.) Eat light, not heavy.
• Consider getting out in the sun and/or stretching during your breaks.
• Bring a light sweater or sweatshirt in case it's cold.
• Don't forget your ID and USMLE pass.
• Take your breaks when you need them/however you want them (for example: 2 sections ~ break
~ 2 sections ~ break ~ 1 section ~ break ~ 2 sections). Some breaks may need to be longer
than others. Don't be afraid to take a small 5-minute bathroom break.
• Expect 5-10 questions in each section that you have never seen before. If you expect this, then it
won't freak you out when it happens (and it will happen).
• Consider answering 10 practice questions prior to going into the test center for "warm-up" (but
don't look at the answers in case you are incorrect).

r f>7f> 1

You might also like